Integral Kokeboken

April 7, 2017 | Author: 최진석 | Category: N/A
Share Embed Donate


Short Description

Download Integral Kokeboken...

Description

Integral Kokeboken 4 3 2 1

2

Z



0

Z

4

6

2

8



sin(πx ) 2− 2 dx = 4 sinh2(πx)

1 + log(log x) dx = x log(log x) + C log x Z



cos(x2) + sin(x2) dx =





−∞

Z

∞ X xs−1 1 dx = Γ(s) = Γ(s)ζ(s) s ex − 1 n n=1



0

Z



log 0



(1 + sin x)1+cos x 1 + cos x



dx = 0

i

Del I . . . . . .

. . . . . .

. . . . . .

. . . . . .

. . . . . .

. . . . . .

. . . . . .

. . . . . .

. . . . . .

. . . . . .

. . . . . .

. . . . . .

. . . . . .

. . . . . .

. . . . . .

. . . . . .

1 2 4 6 6 6 7

2.1 Introduksjon II . . . . . . . . . . . . . 2.2 Bevis . . . . . . . . . . . . . . . . . . . 2.2.1 Den antideriverte . . . . . . . . 2.2.2 Delvis integrasjon . . . . . . . . 2.2.3 Substitusjon . . . . . . . . . . . 2.3 Symmetri og nyttige sammenhenger . 2.4 Periodiske funksjoner . . . . . . . . . . 2.5 Diverse substitusjoner . . . . . . . . . 2.5.1 Weierstrass substitusjon . . . . 2.5.2 Euler substitusjon . . . . . . . . 2.6 Brøker og kvadratrøtter . . . . . . . . 2.6.1 Brøker . . . . . . . . . . . . . . 2.6.2 Kvadratrøtter . . . . . . . . . . 2.7 Delvis integrasjon . . . . . . . . . . . . 2.7.1 Delvis kanselering . . . . . . . 2.7.2 Eksponentialfunksjonen . . . . 2.7.3 Tabell og reduksjonsformler . . 2.8 Trigonometrske funksjoner . . . . . . . 2.9 Logaritmiske funksjoner . . . . . . . . 2.10 Ulike tips og knep . . . . . . . . . . . . 2.10.1 Rekursjoner og funksjonsfølger 2.10.2 Nyttig funksjonallikning . . . . 2.10.3 Integral par . . . . . . . . . . . 2.11 Oppgavesammling II . . . . . . . . . . 2.11.1 Integral . . . . . . . . . . . . . 2.11.2 Oppgaver . . . . . . . . . . . .

. . . . . . . . . . . . . . . . . . . . . . . . . .

. . . . . . . . . . . . . . . . . . . . . . . . . .

. . . . . . . . . . . . . . . . . . . . . . . . . .

. . . . . . . . . . . . . . . . . . . . . . . . . .

. . . . . . . . . . . . . . . . . . . . . . . . . .

. . . . . . . . . . . . . . . . . . . . . . . . . .

. . . . . . . . . . . . . . . . . . . . . . . . . .

. . . . . . . . . . . . . . . . . . . . . . . . . .

. . . . . . . . . . . . . . . . . . . . . . . . . .

. . . . . . . . . . . . . . . . . . . . . . . . . .

. . . . . . . . . . . . . . . . . . . . . . . . . .

. . . . . . . . . . . . . . . . . . . . . . . . . .

. . . . . . . . . . . . . . . . . . . . . . . . . .

. . . . . . . . . . . . . . . . . . . . . . . . . .

. . . . . . . . . . . . . . . . . . . . . . . . . .

11 12 13 13 15 17 18 30 40 47 50 53 53 56 60 61 63 65 69 76 80 80 84 88 90 90 91

. . . . . . . . . . . . .

. . . . . . . . . . . . .

. . . . . . . . . . . . .

. . . . . . . . . . . . .

. . . . . . . . . . . . .

. . . . . . . . . . . . .

. . . . . . . . . . . . .

. . . . . . . . . . . . .

. . . . . . . . . . . . .

. . . . . . . . . . . . .

. . . . . . . . . . . . .

. . . . . . . . . . . . .

. . . . . . . . . . . . .

. . . . . . . . . . . . .

. . . . . . . . . . . . .

103 104 105 105 106 107 108 108 111 121 125 129 129 131

1.1 Brøk . . . . . . . . . . . . . 1.2 Trigonometriske funksjoner 1.3 Oppgavesammling I . . . . 1.3.1 Integral . . . . . . . 1.3.2 Integral . . . . . . . 1.3.3 Oppgaver . . . . . .

. . . . . .

. . . . . .

. . . . . .

. . . . . .

. . . . . .

Del II

Del III 3.1 Introduksjon III . . . . . . . . . . . . 3.2 Viktige konstanter . . . . . . . . . . 3.2.1 Euler–Mascheroni konstanten 3.2.2 Catalan’s Konstant . . . . . . 3.2.3 Glaisher–Kinkelin konstanten 3.3 Spesialfunksjoner . . . . . . . . . . . 3.3.1 Gulv og tak-funksjonene . . . 3.3.2 Gammafunksjonen . . . . . . 3.3.3 Betafunksjonen . . . . . . . . 3.3.4 Digamma-funksjonen . . . . . 3.3.5 Polygamma-funksjonen . . . 3.3.6 Riemann zeta funksjonen . . 3.3.7 Hurwitz zeta function . . . .

. . . . . . . . . . . . .

ii

3.4

3.5

3.6 3.7 3.8 3.9 3.10

3.3.8 Polylogaritmen . . . . . . . . . . . . 3.3.9 Dilogaritmen . . . . . . . . . . . . . 3.3.10 Dilogaritmen . . . . . . . . . . . . . 3.3.11 Elliptiske Integral . . . . . . . . . . . Transformasjoner . . . . . . . . . . . . . . . 3.4.1 Laplace transformasjonen . . . . . . 3.4.2 Fourier-transformasjon . . . . . . . . 3.4.3 Mellin transformasjonen . . . . . . . 3.4.4 Landen’s transformasjon . . . . . . . 3.4.5 Cauchy-Schl¨ omilch transformasjonen 3.4.6 Diverse transformasjoner . . . . . . Diverse applikasjoner . . . . . . . . . . . . . 3.5.1 Gulv og tak-funksjoner . . . . . . . . 3.5.2 Itererte integral . . . . . . . . . . . . Derivasjon under integraltegnet . . . . . . . Uendelige rekker . . . . . . . . . . . . . . . Dobbel Integraler . . . . . . . . . . . . . . . Kompleks Integrasjon . . . . . . . . . . . . . 3.9.1 Typer integraler . . . . . . . . . . . . Oppgavesammling III . . . . . . . . . . . . . 3.10.1 Integral . . . . . . . . . . . . . . . . 3.10.2 Oppgaver . . . . . . . . . . . . . . .

. . . . . . . . . . . . . . . . . . . . . .

. . . . . . . . . . . . . . . . . . . . . .

. . . . . . . . . . . . . . . . . . . . . .

. . . . . . . . . . . . . . . . . . . . . .

. . . . . . . . . . . . . . . . . . . . . .

. . . . . . . . . . . . . . . . . . . . . .

. . . . . . . . . . . . . . . . . . . . . .

. . . . . . . . . . . . . . . . . . . . . .

. . . . . . . . . . . . . . . . . . . . . .

. . . . . . . . . . . . . . . . . . . . . .

. . . . . . . . . . . . . . . . . . . . . .

. . . . . . . . . . . . . . . . . . . . . .

131 132 135 136 140 140 146 147 147 149 152 155 156 156 157 164 167 176 177 194 194 194

Konvergens . . . . . . . . . . . analysens fundamentaltheorem. Konvergens . . . . . . . . . . . Funksjonalanalyse . . . . . . . Bohr-Mullerup Theoremet . . .

. . . . .

. . . . .

. . . . .

. . . . .

. . . . .

. . . . .

. . . . .

. . . . .

. . . . .

. . . . .

. . . . .

. . . . .

201 203 211 215 223 226

Del IV A.1 B.1 C.1 C.2 C.3

Del V

. . . . .

. . . . .

. . . . .

. . . . .

. . . . .

. . . . .

. . . . .

229 5.1 Kortsvar . . . . . . . . . . . . . . . . . . . . . . . . . . . . . . . . 230 5.30 Langsvar . . . . . . . . . . . . . . . . . . . . . . . . . . . . . . . . 238

I

2

1.1

1 Brøk

I

BRØK

˚ trekke sammen eller forTidligere har du møtt oppgaver som ber deg om a enkle brøk-uttrykk 1−x x 1−x x−1 x x+1 3x − 1 + = + = 2 1+x x−1 1+x x−1 x−1 x+1 x −1 ˚ g˚ ˚ dele eller spalte brøken? Som ett Hva om vi ønsker a a andre veien? Alts˚ aa ˚ vise at eksempel kan du se om du klarer a x2

2 1 1 = − −1 x−1 x+1

˚ trekke sammen høyresiden. 1 Det finnes mange m˚ ˚ trekke sammen uten a ater a brøker p˚ a og for en enda grunndigere gjennomgang kan kalkulus anbefales eller

BESTEMTE

INTEGRAL KAN VÆRE LUNEFULLE

Ett gjennomg˚ aende tema i dette heftet er at integrasjon er en kunst og i del II skal vi se at bestemte integral kan bli beregnet p˚ a forunderlige m˚ ater. ˚ prøve ut flere metoder for a ˚ bestemme ett integral. Oppfordringen før des blir a Selv om du ser en løsning som du vet vil virke, ta en pause. Tenk. Er det noen andre metoder som kan virke, er de lettere, mer elegant enn min? Ett eksempel er følgende integral Z I= 1

2

4x dx x2 − 1

˚ ha lest forrig˚ Flere vil etter a aende seksjon begynne med delbrøksoppspalting. Flott! Men hva om teller hadde vært x2 + 1, hva kunne en gjort da? Svaret er jo substitusjon, og detaljene skal leser f˚ a kose seg med alene. Ett litt tøffere eksempel – som vi kommer tilbake til senere – er følgende Eksempel 1.1.1. Bestem verdien av følgende integral Z ∞ 1 I= dx x(1 + x2 ) 1 En ser ikke noen umidelbar smart substitusjon s˚ a en naturlig vei er delbrøksoppspalting. Det finnes mange m˚ ater, men for eksempel 1 1 + x2 − x2 1 x = = − 2 x(1 + x ) x(1 + x2 ) x 1 + x2 ˚ bestemme integralet av siste del, bruker vi Integrasjonen blir n˚ a enklere. For a substitusjon Z Z 1 x 1 2x dx 1 I= − dx = log x − = log x − log(1 + x2 ) + C 2 2 x 1+x 2 1+x 2 1 Som

liten førsteklasse tass p˚ a gymnaset var jeg dypt interessert i temaet og holdt p˚ a i flere uker ˚ dele brøker og lage regler. Det var først det p˚ ˚ret at skolen begynte med teorien. med a afølgende a

I

1 Brøk

3

˚ gjøre noen sm˚ Dette uttrykket er rett. Men vi velgere a a algebraiske omskrivninger ved hjelp av logaritmereglene log a + log b = log ab. Grunnen til dette vil snart bli klar2     Z 1 1 1 x2 1 = +C dx = log log 1 − x(1 + x2 ) 2 1 + x2 2 1 + x2 Hvor det blant annet ble brukt at log x = 21 log x2 og 1 ˚ sette inn grensene v˚ 1 − 1+x are f˚ ar vi endelig 2 . Ved a Z



1

x2 1+x2

=

(x2 +1)−1 1+x2

=

∞   1 1 1 dx = log 1 − x(1 + x2 ) 2 1 + x2 1     1 1 1 1 − log 1 − = lim log 1 − 2 x→∞ 1 + x2 2 1+1

Siden 1/x → 0 n˚ ar x → ∞ s˚ a vil 1/(1 + x2 ) → 0 n˚ ar x → ∞. Alts˚ a g˚ ar første del mot log 1 = 0. Z ∞ 1 1 1 1 1 I= dx = log(1 − 0) − log = log 2 2) x(1 + x 2 2 2 2 1 og vi er ferdige. I siste overgang brukte vi at log(1/2) = log 1 − log 2 = − log 2. Finnes det en enklere metode? Ja, heldigvis. Men den krever et bedre falkeøye ˚ se. Vi begynner i stedet med den noe uvante substitusjonen y 7→ 1/x. Dett for a er det samme som at x 7→ 1/y slik at dx = − dy/y 2 . N˚ ar x → ∞ s˚ a vil y → 0 siden y = 1/x. Og tilsvarende x > 1 gir y = 1/x = 1. Dermed s˚ a f˚ as Z 1



1 dx = − x(1 + x2 )

Z 1

0

dy 1 1 = 1/y(1 + (1/y)2 ) y 2 2

Z 0

1

2y dy 1 + y2

˚ bruke en a ˚penbar substitusjon. Tanken er at selv om Problemet løses s˚ a ved a det ikke ser ut som en metode vil føre frem, kan det være lurt a˚ gi den en sjangse like vell. Substitusjonen x 7→ 1/y kalles gjerne den inverse substitusjonen, eller en bijectiv substitusjon. Den er mye brukt p˚ a bestemte integral med grenser 1 og ∞.

2 Prøv

˚ fortsette a ˚ beregne integralene uten disse algebraiske krumspringene. Hva skjer? a

4

1.2

I

2 Trigonometriske funksjoner

TRIGONOMETRISKE FUNKSJONER

For en diskusjon om Rieman-integrerbarhet og ulike typer integraler henvises leses til Appendix 1. Her blir og temaet om konvergens av integraler studert gjennomg˚ aende. Proposisjon 1.2.1. Vi har at (arctan x)0 = t

Z 0

1 1+x2

og

1 dx = arctan t 1 + x2

(1.1)

for alle t ≥ 0. Videre s˚ a er Z 0

t

1 dx = 1 + x2

Z



1/t

1 dx = arctan t 1 + x2

(1.2)

Bevis. Vi beviser første del av proposisjonen mens siste del overlates til leser, se oppgave. Derivasjonen av arctan x kan vises ved implisitt derivasjon. La y = arctan x, da er tan y = x. Derivasjon gir d d tan y = x dx dx d dy (tan y) =1 dy dx  dy 1 + (tan y)2 =1 dx 1 dy = dx 1 + (tan y)2 d 1 arctan x = dx 1 + x2 d d dy = . I siste linje kan dx dy dx ˚ f˚ vi benytte oss av at y = arctan x og tan y = x for a a det ønskede resultatet. Derivasjonen av tan x kan for eksempel føres I andre linje ble kjerneregelen benyttet, alts˚ a at

dy sin x (sin x)0 · cos x − sin x · (cos x)0 (sin x)2 (cos x)2 dy tan x = = = + dx dx cos x (cos x)2 (cos x)2 (cos x)2 Som viser at (tan x)0 = 1 + (tan x)2 , siden sin x/ cos x = tan x per definisjon. ˚ integrere (arctan x)0 = 1/(1 + x2 ) fra null til t f˚ Ved a as t

Z 0

dx = 1 + x2

Z 0

t

h it (arctan x)0 dx = arctan x = arctan t 0

Som følger fra at siden tan 0 = 0 s˚ a er arctan 0 = 0. Alternativt kunne vi integrert uttryket via substitusjonen x = tan y. Da blir  d = (tan y)0 ⇒ dx = 1 + tan2 (y) dy = (1 + x2 ) dy dx

I

2 Trigonometriske funksjoner

5

Alts˚ a er dx/(1 + x2 ) = dy. Grensene blir arctan 0 = 0 og arctan t. Slik at integralet kan skrives som t

Z 0

1 dx = 1 + x2

Z

arctan t

dy = arctan t 0

Dette fullfører første del av beviset. Siste delen av beviset overlates til leser, se oppgave 4. Korollar 1.2.1. Z Z

Z ∞ 1 π 1 dx = dx = 2 2 1+x 1+x 4 Z1 ∞ 1 1 π dx = dx = 1 + x2 1 + x2 2 0

1

0 ∞

2 −∞

Bevis. Første overgang faller direkte ut av likning (1.2), med t = 1. Da har vi Z



1/1

1 dx = 1 + x2

Z

1

0

π 1 dx = arctan 1 = 2 1+x 4

Andre overgang følger fra likning (1.1). Verdien av arctan 1 vet vi, siden tan π/4 = 1 s˚ a er arctan 1 = π/4.

Oppgaver 1. a) Bevis siste halvdel av proposisjon (1.2.1), alts˚ a likning (1.2). b) Vis videre at integralene under t

Z 0

log x dx = − 1 + x2

Z



1/t

log x dx , 1 + x2

(1.3)

holder for alle t > 0. Bestem følgende integral Z ∞ log x dx , 1 + x2 0 ˚ benytte deg av likning (1.3). ved a Z 2. Bruk substitusjonen u 7→ tan x til a˚ bestemme I =

tan x)n+2 + tann x dx

0

Bestem integralet Z

π

π/4

tan9 x + 5 tan7 x + 5 tan5 x + tan3 x dx

0

˚ bruke resultatet ovenfor. ved a

6

I

3 Oppgavesammling I

1.3

O P P G AV E S A M M L I N G I

1.3.1

INTEGRAL

1.3.2

INTEGRAL x2 + 3x dx x2

Z 1.

Z

sin(x) dx Z

Z

e

Z

π dx dx −x2

18. 1

1

x dx x2 + 1

3. 0

Z

Z

4. 3

Z

−3



Z 6.

Z

20. Z 21.

4 − x dx

Z

1/2

Z 7.

(2x − 1)

50

dx

0

Z 23.

√ n

Z 8.

π

x dx

Z 24.

Z

1 dx x ln x

9.

Z 25.

Z 10.

π

sin x cos x dx



Z

e−1

1

Z 12.

Z x dx ex

13. 14.

en

ln x dx n ∈ R

15.

x

x · a dx

4

−3/4

x √ dx + 1 − x2

 ln 1 + x2 x dx

1

Z

x2 dx x2 + 4x + 8

Z

1 dx x ln(x)n

42.



x3 e−x dx

0

Z

x

ex+e dx

2

Z

ln(x)3 dx

46.

30.

x2

e−1

41.

45.

1

Z

1− √

x + 2x + 2 dx x−1

0

Z

Z

Z

e−y dx

29.

dx

x2

Z

1

x

x2 dx −1

Z

44.

2

sin x cos x dx Z

e

39.

43.

0

Z

Z

dx

sin 2x dx sin x

27.

28.

Z

x2

x−1 dx 40. x2 + 2x + 7

π/4

loge (a)2 √

1

ex + 1 dx ex − 1

 ln 1 + x2 x dx

11.

1 √ dx x (1 + x )



38.

1+e √ x dx e +x

26.

Z

Z

x

Z √

sin(1/x)e √ x!

dx

 x+1 dx x

37.

ln x √ dx x

22.



Z  36.

1 √ dx 1+ x

1 x

2x ex dx

35.

xex dx (x + 1)2

Z

1 dx 1−x

5.

 x ln

x ln(x + 1) dx 0

x dx x+1

2

Z

19.

dx

sin(2x)esin(x) dx

33. 34.

ln 2



1 √ x

ln

π/6

sin(x) dx Z



Z 32.

2

17.

Z 2.

π/3

x+1 dx (x + 2)4

Z

m

47.

(m − x)(x − n) dx n

Z

Z 16.

log10 (x) dx

31.

p x3 1 − x2 dx

Z 48.



e2

π dx − 16

I

3.3 Oppgaver Z √

Z 50. Z

Z

x −2 dx x−4

57.

sin(2x) dx sin x

58.

49.

Z

π/3n

51. 0

4a dx x2 − a2

52. Z

2

54. Z

Z

π/2

55. π/6

Z 56.

1.3.3

1 dx tan x

x2 + 1 dx x(x2 + 3)

Z

5

Z 67.

2

1

xn ln x dx

0

Z 68.

log5 (x) dx log25 (x)

Z 69.

2−4 dx x4 − x2

62.

1 √ dx 2x ln x

66.

2ln x dx

Z

xn ln x dx

0

x2 sin(x) dx π−2 0 Z q √ √ 60. x + x dx 61.

1

65.

π

2

eln(x +1) dx x+1

Z

Z

59.

2

esin(x) ecos(x) dx

53.

1 dx x +x

1 dx x ln x − x

Z

tan(nx) dx Z

√ 3

7

x2 Z



70. −∞

Z 63.

cos(sin(x)) cos x dx

Z 71.

Z 64.

x+x

 2

x2 − x dx − 2x + 1 ex/2 dx 1 + ex

 1 ln x 2

ln(x2 )



dx

e3(x+ln 3) dx Z 5 x −x+1 dx 72. x3 + 1

O P P G AV E R

1. Vis at integralet Z

a+2π

K=

sin x + 1 dx , a

er konstant dersom a er et reellt tall. Kan resultatet forklares geometrisk? 2. La D betgne omr˚ adet avgrenset av linjene y = sin x, x = sin y og y = 2π+x. Bestem arealet av D Hint: En god tegning gjør ofte susen 3. Vi har følgende integral ea

Z I= 1

1 dx , x log x + ax

hvor a er et reellt tall. Drøft integralet for tilfellene a < 0, a = 0 og a > 0, 4. Bestem følgende integral r

Z 1+

x−1 x+1

!2 dx

˚ gange ut parentesen. Hint: Integralet kan løses b˚ ade med og uten a

8

I

3 Oppgavesammling I 5. Vis at Z

ϕ

 22 x2 − 1 e2x dx = e2ϕ − e2ψ

ψ

hvor ψ og ϕ henholdsvis er minste og største løsning av x(x − 1) = 1. R a x −2 dx = 12 6. Finn ett reelt tall a som tilfredstiller 0 e +e 2 5 Z x log t 7. For x > 0, la f (x) = dt. Finn funksjonen f (x) + f (1/x) og vis at 1 1+t f (e) + f (1/e) = 1/2. Z 2π Z π Z ∞ r2 8. Bestem følgende integral dφ sin θ dθ 2 dr (1 + r2 ) 0 0 0 9. 10. La f (x) være en funksjon slik at 1) f (x) er integrerbar for x ∈ [0, 3] Z 1 2) f (x) dx = 1. 0

3) f (x) =

1 2

f (x) for alle x ∈ [0, 3]. Bestem integralet 3

Z

f (x) dx 0

11. Det oppgis at Z



−∞

π dx = og x2 + a2 a

Z



−∞

dx (x2

+

2 a2 )

=

π 2a3

˚ bestemme hvor a > 0 er en positiv konstant. Bruk dette til a Z ∞ x2 2 dx . 2 −∞ (x + 1) Integralene ovenfor kan fritt benyttes, og det oppgis og at integralet ovenfor konvergerer. 12. Vis at integralet Z 0



f (x) x

er invariant (er det samme) om f byttes ut med f (cx), hvor c er en konstant. ˚ se p˚ 13. Ved a a integralet mellom f (x) = sin x og g(x) = cos x kan vi f˚ a et omr˚ adet som likner p˚ a en bart (se 345). Bestem arealet av barten. √ 14. M˚ alet med denne oppgaven er a˚ bevise hva som er størst av log 2 og 1/ 2 . Og de er nærmere enn hva enn skulle tro ved første øyenkast.

I

3.3 Oppgaver

9

a) Vis at ulikheten √ 2 1+ 2 1 √ 2 > 1 + t t+1+ 2 holder for alle 0 < t < 1. b) Den geometriske aritmetriske ulikheten er definert som x−y √ > xy log x − log y ˚ studere fider x > 0. Bevis ulikheten, Her kan det være fordelaktig a gur (1.1).

1/x

a

√ ab

x b

Figur 1.1: Den aritmetriske geometriske ulikheten √ ˚ benytte c) Bestem hva som er størst av log 2 og 1/ 2 . En st˚ ar fritt til a tidligere oppgaver.

˚ gi leser en pustepause og for a ˚ la Denne siden er med hensikt blank, for a forfatter sl˚ ass mot dinosaurer.

II

12

II

1 Introduksjon II

2.1

INTRODUKSJON II

I denne delen vil vi hovedsaklig dykke dypere ned i tidligere besøkte temaer, men ogs˚ a lære noen nye eksotiske teknikker for a˚ beregne spesielt h˚ arete integral. Det som forventes av leseren p˚ a dette tidspunktet er en grundig forst˚ aelse for de elementære integrasjonsteknikkene som ble gjennomg˚ att i del I. For a˚ kunne løse mange av integralene som kommer videre m˚ a disse elementære teknikkene sitte helt ut til fingerspissene. Hvor i forrige del fleste integralene kunne bli løst via et steg − En smart substitusjon, en delbrøkoppspalting, en frekk faktorisering osv − vil vi se frem˚ bli løst. Mange av løsningene vil mover at integralene krever flere steg for a virke merkelige og noen vil virke som de er tatt rett ut av det bl˚ a. Men husk at bak hvert skritt som fører mot en løsning ligger et stort maskineri. Ofte er grunnen til at vi bruker en s˚ a merkelig substitusjon eller delvis inte˚ ha sett grasjon enkel. Det fungerer. Fremmgangsm˚ aten blir en blanding av a liknende problem før, og prøve en rekke velkjente triks og knep. Og det er ogs˚ a m˚ alet med denne delen; og lære bort alle disse merkelige substitusjonene og ˚ beregne integral, slik at du selv kan anvende dem. For etter a ˚ metodene for a ha sett det samme smarte trikset nok ganger, blir det ikke lengre et triks men en nyttig teknikk du selv kan bruke. ˚ se studere n˚ Et nøkkelkonsept i denne delen er a ar en kan gjøre følgende omskrivning Z Z Z f (x) dx = g(x) dx − a f (x) dx . S

D

S

Metoden for a˚ komme fra venstre side til høyre side av likningen er ikke spesielt viktig. Det kan være en lur substitusjon, en brutal delvis integrasjon eller noe helt annet. Poenget er at likningen ovenfor kan løses med hensyn p˚ a integralet over S 1 , en beregner alts˚ a integralet indirekte. I mange tilfeller vil dette og være eneste metoden som fungerer da det eksisterer funksjoner som ikke har ˚ løse integraelementære antideriverte2 . I denne delen blir ogs˚ a andre m˚ ater a ˚ anvende symmetri, til smart bruk av delvis ler inderekte p˚ a studert, alt fra a integrasjon.

1 S˚ a 2 En

fremt a 6= −1, da ender en opp med at integralet over D er null. R sier gjerne at en funksjon har en elementær antiderivert dersom f dx kan uttrykkes ved elemen-

Rb

tære funksjoner. Selv om en funksjon ikke har en elementær antiderivert kan likevel f (x) dx a eksistere og være elementær. Dette er noe som skaper hodebry for mange matematikkere.

II

2.1 Den antideriverte

2.2

13

BEVIS

I denne delen skal vi se nærmere p˚ a noen geometriske bevis for b˚ ade delvis integrasjon, og hvorfor den antideriverte gir oss arealet under en funksjon. Tilsutt bevises ogs˚ a substitusjon mer formelt. Det formelle beviset for analysens fundamentaltheorem spares til appendixet, da det er noe langt. For den ivrige leser se theorem (B.1.1).

2.2.1

DEN

ANTIDERIVERTE

a F (x) + C Lemma 2.2.1. Anta at F 0 (x) = f (x) da vil ogs˚ en vilk˚ arlig konstant.

0

= f (x) hvor C er

a [a, b], Theorem 2.2.1. La f være en kontinuerlig funksjon som tar reelle verdier p˚ og la F være den antideriverte til f for x ∈ [a, b]. Da er Z

b

f (t) dt = F (b) − F (a) . a

Vi venter som sagt med et formelt bevis for FTC, men i korte trekk kan det formelle beviset kokes ned til følgende fire linjer F (b) − F (a) = = =

n X k=1 n X k=1 n X k=1 Z b



[F (xk ) − F (xk−1 )]

Teleskoperende sum

(2.1)

F 0 (x∗k )[xk − xk−1 ]

Middel verdi theoremet

(2.2)

Riemann sum

(2.3)

F 0 (x∗k )∆xk F 0 (x) dx

(2.4)

a

Merk at theorem (2.2.1) krever at f er begrenset og kontinuerlig p˚ a intervallet. Dette er den klassiske definisjonen av en Rieman-integrerbar funksjon. En kan utvide integralbegrepet til langt mer ville funksjoner, men da holder ikke lengre det p˚ afølgende beviset vann. Beviset under med andre ord langt mer en intuisjon enn et formelt bevis. Bevis. Dette beviset tar utgangspunkt i figur (2.2) og baserer seg p˚ a visuell fremstilling. Anta at det eksisterer en eller annen funksjon A(y) som beskriver arealet under f (x) fra et punkt a til ett eller annet punkt y > a. Det røde omr˚ adet i figur(2.1) er eksempelvis A(b). N˚ a utvider vi omr˚ adet litt med en faktor h. Arealet av det bl˚ a omr˚ adet kan da skrives som ∆bl˚ a = A(b + h) − A(b) . Ut i fra figur (2.1) s˚ a vil det bl˚ a omr˚ adet alltid ligge mellom det grønne rektangelet og det oransje. Dette kan beskrives ved følgende ulikhet h · f (b) ≤ A(b + h) − A(b) ≤ h · f (h + b) .

14

II

2 Bevis y

f (h + b) f (b)

A(b) x a

b

b+h

Figur 2.1: Grafen til en vilk˚ arlig funksjon f (x) Siden arealet av et rektangel er lik grunnlinjen ganget med høyden. Grunnlinjen her blir G = (h + b) − b = h, og høyden er f (x). ’Trikset’ er n˚ a at siden h > 0 s˚ a kan en fritt dele ulikheten p˚ ah f (b) ≤

A(b + h) − A(b) ≤ f (h + b) . h

(2.5)

Videre lar en h → 0. Visuelt kan en se p˚ a dette som at vi gjør det bl˚ a omr˚ adet mindre og mindre. Helt tilsvarende som n˚ ar en utledet formelen for den deriverte. lim f (b) ≤ lim

h→0

h→0

A(b + h) − A(b) ≤ lim f (h + b) . h→0 h

˚ se pent ut. Det midterste leddet kan en kjenne igjen som Dette begynner a definisjonen av den deriverte f 0 (x) = lim

h→0

f (x + h) − f (x) . h

Første leddet blir f (b), og siste leddet blir f (b). Da har en alts˚ a skvist A0 (x) f (b) ≤ A0 (b) ≤ f (b) . Dette medfører at A(x) er en antiderivert til f (x)! Siden A0 (x) = f (x), s˚ a betyr det at A(x) = F (x) hvor F 0 (x) = f (x). Funksjonen A kan dermed skrives som A(b) = F (b) + C , hvor C er en eller annen konstant lemma (2.2.1). V ønsker n˚ a a˚ bestemme denne ukjente konstanten. Merk at per definisjon s˚ a er A(a) = 0, slik at 0 = F (a) + C ⇒ C = −F (a) .

II

2.2 Delvis integrasjon

15

Arealet under funksjonen f (x) fra x = a til x = b kan da skrives som A(b) = F (b) − F (a) , ˚ skrive dette som og som kjent er det mer vanlig a Z

b

f (x) dx = F (b) − F (a) . a

Merk at beviset tar utgangspunkt i at f (x) er stigende p˚ a intervalet, hadde funksjonen vært konstant kunne en brukt akkuratt samme argument. Hadde funksjonen i stedet vært synkende ville eneste forskjell vært at ulikhetstegnene i likning (2.5) vært snudd. Beviset her krever at f er en kontinuerlig, men at funksjonen er strengt positiv er ikke nødvendig. Med litt mer spissfindighet kan en føre et liknende argument for en funksjon som er negativ. Integrasjon holder i langt mer generelle former ˚ dekke alle spesialtilfeller, men gi a ˚ en viss enn dette, men poenget er her ikke a intuisjon om hvordan derivasjon og integrasjon henger sammen.

2.2.2

D E LV I S

INTEGRASJON

Formelen for delvis integrasjon er gitt under Z Z u dv = uv − u du ˚ integrere produktregelen Tidligere har vi utledet denne formelen ved a (uv)0 = u0 v + uv 0 ˚ utlede formelen ovenfor p˚ I denne delen skal vi se p˚ a en mer visuell m˚ ate a a, uten bruk av derivasjon. Forh˚ apentligvis vil denne metoden være mer intuitiv, i ˚ være en alternativ fremgangsm˚ tillegg til a ate. ˚ uttrykke arealet av det Beviset tar utgangspunkt i figur (2.2) og ide`en er a gr˚ a omr˚ adet p˚ a to ulike m˚ ater. P˚ a den ene siden kan arealet under u(v), fra A til bli beskrevet ved følgende integral Z v2 ∆ABB 0 A0 = u dv v1

Tilsvarende s˚ a kan ogs˚ a arealet begrenset av punktene ABB 0 A0 bli beskrevet av den inverse funksjonen v(u) langs u-aksen som Z u2 ∆CBB 0 C 0 = v du u1

Arealet av det gr˚ a omr˚ adet er likt summen av ∆CBB 0 C 0 og ∆ABB 0 A0 . P˚ a den andre siden kan det gr˚ a omr˚ adet - mer formelt polygonet avgrenset av CBAA0 B 0 C 0 - uttrykkes som differansen mellom OA0 B 0 C 0 og OABC. ∆OABC = u1 v1 og ∆OA0 B 0 C 0 = u2 v2

16

II

2 Bevis v u2

u1

u(v)

C

0

B0

C B

O

A0

A v1

u

v2

Figur 2.2: En vilk˚ arlig funksjon i uv-koordinatsystemet

˚ ta differansen s˚ Ved a a kan det gr˚ a omr˚ adet uttrykkes som h iu2 ,v2 ∆CBAA0 B 0 C 0 = ∆OA0 B 0 C 0 − ∆OABC = u2 v2 − u1 v1 = uv u1 ,v1

˚ sammenlikne de to uttrykkene vi har for det gr˚ Ved a a omr˚ adet s˚ a er h

uv

iu2 ,v2 u1 ,v1

Z

v2

Z

u2

u dv +

=

v du

v1

(2.6)

u1

som ikke er noe annet en den kjente formelen for delvis integrasjon med leddene stokket om. Noen vil kanskje stusse p˚ a grensene ovenfor, men husk at b˚ ade u og v begge er funksjoner av en underliggende variabel si x. Spesifikt s˚ a er u(x1 ) = u1 , v(x1 ) = v1 og u(x2 ) = v2 , v(x2 ) = v2 . S˚ a likning (2.6) kan skrives Z

x=x2

x=x1

h ix=x2 Z u dv = uv − x=x1

x=x2

v du .

(2.7)

x=x1

som er akkuratt formen for delvis integrasjon med grenser. Merk at beviset tok utgangspunkt i at funksjonen u(v) var en-til-en eller bijektiv. Dette er ikke noe nødvendig krav, som vi s˚ a n˚ ar vi utledet formelen tidligere. Uten for mye bevis s˚ a kan beviset ovenfor utvides til kurver som ikke er en-tilen, s˚ a lenge u og v er funksjoner kun avhengig av en underliggende variabel. Integralene vil fortsatt kunne beregnes, men n˚ a vil noen av omr˚ adene være negative.

II

2.2.3

2.3 Substitusjon

17

SUBSTITUSJON

I starten n˚ ar en lærer substitusjon vil en ofte missbruke notasjonen. Vi skriver du gjerne u = f (x) ⇒ = f 0 (x) ⇒ du = f 0 (x) dx. Men hva skjer egentlig i dx den siste overgangen? Vi arbeider jo tross alt med differensialer og ikke brøker. Husk p˚ a at substitusjon egentlig bare er kjerneregelen baklengs. Akkuratt som delvis integrasjon essensielt er produktregelen baklengs. Essensen av kjerneregelen er dy dy du = , dx du dx Grunnen til at vi foretrekker Leibniz notasjonen og skriver deriverte som forhold er at notasjonen er veldig hintende (suggestive). Ofte n˚ ar ting ser ut som de kanselerer s˚ a gjør de virkelig det. Ofte oppfører disse differensialene seg som brøker for eksempel dy/ dx = 1/ dx/dy. Men a˚ rettferdigjøre at en kan behandle differensialer som brøker er en egen gren innenfor matematikk som kalles for infinitesmal kalkulus. La du/ dx = u0 (x), substitusjon kan da skrives direkte som Z Z Z Z du g(u(x))u0 (x)dx = g(u(x)) dx = g(u) du = g(t) dt dx Hvor det er den andre overgangen om m˚ a rettferdigjøres. Det som ser ut som en kanselering av dx i andre overgang er direkte knyttet til kjerneregelen. Nota˚ understreke kanseleringen. sjonen er med andre ord valgt for a Theorem 2.2.2. La I = [a, b] ⊂ R være et intervall og la u(x) være en til en funksjon samt kontinuerlig deriverbar. Anta at g er kontinuerlig p˚ a I, da er Z b Z u(b) g(u(x))u0 (x) dx = g(t) dt . a

u(a)

Bevis. Vi definer funksjonen F som følger Z x F (x) = g(t) dt 0

Vi studerer s˚ a funksjonen F (u(x)) og dens deriverte. dF du F (u(x))0 = F 0 (u(x))u0 (x) = du dx Z u(x) d = g(u(t)) dt · u0 (x) = g(u(x))u0 (x) du 0 Hvor siste overgang følger fra første del av analysens fundamentaltheorem ˚ integerere F (u(x))0 = g(u(x))u0 (x) fra a til b s˚ (B.1.1). Ved a a har vi Z b Z b du dx. F (u(b)) − F (u(a)) = g(u(x))u0 (x) dx = g(u(t)) dx a a Selvsagt vet vi ogs˚ a at Z

u(b)

F (u(b)) − F (u(a)) = u(a)

og dette fullfører beviset.

Z

u(b)

g(t) dt =

g(u) du u(a)

18

II

3 Symmetri og nyttige sammenhenger

2.3

SYMMETRI OG NYTTIGE SAMMENHENGER

˚ se p˚ M˚ alet med denne delen er a a hvordan integral og funksjoner kan forenkles ved hjelp av symmetri. Det første vi skal se p˚ a er definisjonen av en symmetrisk funksjon. Definisjon 2.3.1. En funksjon f kalles symmetrisk omkring c dersom  f (x − c) = f −(x − c) En funksjon f kalles anti-symmetrisk omkring c dersom  f (x − c) = −f −(x − c) Dette m˚ a selvsagt holde for alle x. Dersom det bare holder for x ∈ C hvor C = [a, b] er et endelig interval, s˚ a sier en at f er henholdsvis odde eller like over C Definisjon 2.3.2. En funksjon f kalles for en likefunksjon eller jevnfunksjon dersom funksjonen er symmetrisk omkring origo. f (x) = f (−x) En funksjon f kalles odde eller for en oddefunksjon dersom funksjonen er antisymmetrisk omkring origo. f (x) = −f (−x) Som igjen m˚ a holde for alle x. En grafisk fremstilling av odde og like funksjoner er vist i figur (2.3). Ek2 sempelvis s˚ a er x2 , cos x, e−x og |x| like funksjoner. Tilsvarende s˚ a er x3 , sin x, 2 xe−x og x eksempel p˚ a odde funksjoner. En sammlebetegnelse for odde og like funksjoner er paritet. S˚ a pariteten 2 2 til e−x er like, partiteten til xe−x er odde, og ex har ingen paritet. Odde og

y

y

x

(a) Et plot av to odde funksjoner

x

(b) Et plot av to like funksjoner

Figur 2.3: Pictures of animals like funksjoner har mange nyttige egenskaper hvor de viktigste egenskapene er

II

3 Symmetri og nyttige sammenhenger

19

sammlet i tabell (2.1). Her er O en forkortelse for en odde funksjon, og L er en forkortelse for en likefunksjon. Videre s˚ a er E enten odde eller like, og E ∗ har motsatt paritet av E. S˚ a dersom E er like s˚ a er E ∗ odde. Uttrykket E · E ∗ = O betyr det samme a er  som at O · L = O og L · O = O. Gitt to funksjoner f og g s˚ f ◦ g = f g(x) . Tabell 2.1: Noen egenskaper til odde og like funksjoner. E+E E/E E · E∗ E◦L

= = = =

E·E E/E ∗ O◦O L◦E

E L O L

= = = =

L O O L

Fra figur (2.3) virker det som arealet av en likefunksjon er likt p˚ a høyre og venstre side av origo. Tilsvarende for en oddefunksjon, bare at omr˚ adene n˚ a har motsatt fortegn. Dette stemmer faktisk, men før vi viser det la oss ta et litt annet eksempel med hensyn p˚ a symmetri. Eksempel 2.3.1. Se p˚ a funksjonen f (x) = x ˚ vises at arealet under f (x) og f (a − x) grafisk. Alts˚ Det ønskes a a Z a Z a f (x) dx = f (a − x) dx 0

0

Ved a˚ se p˚ a figur figur(2.4) s˚ a er det rimelig a˚penbart at omr˚ adet under funksjo-

y

x Figur 2.4: f (x) og f (a − x) i samme figur. nene er like. Arealet under x fra 0 til a er en trekant med grunnlinje a og høyde a dermed s˚ a er Z a a2 x dx = 2 0 Tilsvarende for a − x er dette og en trekant med grunnlinje a og høyde a s˚ a Z a 2 a x dx = 2 0

20

3 Symmetri og nyttige sammenhenger

II

Eksemepelet ovenfor var veldig konstruert og pedantisk. Men det stiller et viktig spørsm˚ al, gjelder dette for alle funksjoner? Svaret er heldigvis ja, og kan generaliseres til følgende proposisjon Proposisjon 2.3.1. Anta at f er en vilk˚ arlig funksjon, og a,b er to reelle tall da er Z b Z b f (a + b − x) dx = f (x) dx . a

a

Bevis. Igjen kan dette visualiserer med at f blir rotert omkring linjen x = (a + ˚ se p˚ a b)/2 og det er derfor logisk at arealet er uforandret. Begynner med a venstre side av likheten Z b f (x) dx a

˚ bruke substitusjonen x 7→ a + b − u s˚ Ved a a er du = −dx. Videre er x = a → u = b, og x = d → u = a s˚ a Z b Z a Z b f (x) dx = f (a + b − u) − dx = f (a + b − u) du a

b

a

˚ bytte integrasjonsvariabelen tilbake til x. og beviset fullføres ved a Dette enkle beviset fører direkte til at Korollar 2.3.1. Gitt at f er en symmetrisk funksjon omkring c, g er en antisymmetrisk funksjon omkring c da er Z c+a Z c+a Z c+a f (x) dx = 2 f (x) dx og g(x) dx = 0 dx , (2.8) c−a

c−a

0

der at a er en positiv, reell konstant. Spesielt dersom c = 0 s˚ a er Z a Z a Z a f (x) dx = 2 f (x) dx og g(x) dx = 0 dx , −a

0

(2.9)

−a

gitt at f er en likefunksjon, og g er odde. Som det overlates leser a˚ vise. Her er det nok a˚ putte inn i proposisjon (2.3.1) og benytte seg av definisjonen av en odde og like-funksjon. Dette er og svært logisk om en betrakter figur (2.3). Langt i fra alle funksjoner har symmetriske egenskaper, for eksempel s˚ a er ex hverken like eller odde. Men ex =

ex + e−x ex − e−x + = sinh x + cosh x 2 2

Slik at ex kan skrives som en odde og en likefunksjon. Faktisk s˚ a kan enhver funksjon f skrives som en en odde og en likefunksjon f (x) =

f (x) − f (−x) f (x) + f (−x) + 2 2

(2.10)

Hvor første del er en like, og andre del en odde. Fra denne observasjonen kan en vise

II

3 Symmetri og nyttige sammenhenger

21

Proposisjon 2.3.2. La f være en vilk˚ arlig funksjon da er Z a Z a f (x) + f (−x) dx f (x) dx = −a

0

Bevis. Legg merke til at Z a Z a Z a f (x) + f (−x) f (x) − f (−x) = f (x) + f (−x) og =0 2 2 −a 0 −a Dette følger fra korollar (2.3.1) siden første første funksjon er like og andre ˚ integrere likning (2.10) fra −a til a f˚ funksjon er odde. Ved a as da Z a Z a f (x) dx = f (x) + f (−x) dx , −a

0

som ønsket. Alternativt kan en og dele integralet p˚ a midten og benytte substitusjonen x 7→ −y i integralet fra −a til 0. Proposisjon 2.3.3. La f være en integrerbar funksjon p˚ a intervalet [0, 2a], hvor a ∈ R er en positiv konstant. Da holder Z 2a Z a Z a f (x) dx = f (x) dx + f (2a − x) dx 0

0

0

for alle funksjoner f . Korollar 2.3.2. La f være en vilk˚ arlig funksjon da er  Z a Z 2a  2 f (x) dx hvis f (2a − x) = f (x) f (x) dx = 0  0 0 hvis f (2a − x) = −f (x) hvor a er reell konstant. Bevis. Beviset følger direkte fra proposisjon (2.3.3) og nesten rett fra korollar (2.3.1) med et snedig skifte av koordinater. Geometrisk beskriver integralet en funksjon som er odde eller like omkring x = a. Vi deler først integralet p˚ a midten og bruker deretter substitusjonen x 7→ 2a − u p˚ a det siste integralet. Z 2a Z a Z 2a f (x) dx = f (x) dx + f (x) dx 0

a

0

Z

a

Z

2a−2a

−f (2a − u) du ,

f (x) dx +

= 0

2a−a

hvor grensene blir u = 2a − 2a = 0 og u = 2a − a = a. Bruker vi n˚ a at Ra Rb f (x) dx = − f (x) dx kan integralet skrives som b a Z

2a

Z

0

a

f (x) + f (2a − x) dx .

f (x) dx =

(2.11)

0

˚ sette inn henholdsvis f (2a − x) = f (x) og Dette beviser proposisjon (2.3.3). A f (2a − x) = −f (x) fullfører beviset for korollar (2.3.2). Som vi skal se snart følger likning (2.11) ogs˚ a rett fra (2.13) i lemma (2.3.1).

22

II

3 Symmetri og nyttige sammenhenger ˚ se hvor nyttig symmetriegenskaper kan være La oss ta et eksempel for a

Eksempel 2.3.2. Putnam 1987: B1 p Z 4 log(9 − x) dx p p J= log(9 − x) + log(3 + x) 2 N˚ a er a + b − x = 6 − x s˚ a ved a˚ benytte seg av proposisjon (2.3.1) f˚ ar en direkte p p Z 4 Z 4 log(9 − (6 − x)) dx log(3 + x) dx p p p p = log(9 − (6 − x)) + log(3 + (6 − x)) log(3 + x) + log(9 − x) 2 2 ˚ ta gjennomsnittet av uttrykkene f˚ Meget pent. Ved a ar en at p p Z log(9 − x) log(3 + x) 1 4 p p p +p dx J= 2 2 log(9 − x) + log(x + 3) log(x + 3) + log(9 − x) Dermed forenkler integranden dramatisk og en st˚ ar igjen med p Z 4p Z log(9 − x) + log(x + 3) 1 1 4 (4 − 2) p p J= dx = =1 1 dx = 2 2 2 2 log(9 − x) + log(x + 3) 2 Merk at teknikken som ble brukt her kan enkelt generaliseres til følgende to korollar Korollar 2.3.3. La f (x) være en vilkr˚ alig funksjon p˚ a (a, b) da er Z b f (x) b−a dx = , f (a + b − x) + f (x) 2 a gitt at a og b er to reelle konstanter og at integralet konvergerer. For a˚ sikre seg konvergens holder det at f (a + b − x) + f (x) 6= 0 p˚ a intervalet. Beviset er i seg selv enkelt, og en kan bruke nøyaktig samme fremgangsm˚ ate ˚ vise, men i praksis er som i eksempelet ovenfor. I seg selv er korollaret enkelt a ˚ bruke. For eksempel s˚ det vanskeligere a a er Z 1 Z 1 x3 + x + 1 x3 + x + 1 1 dx = dx = . 2 3 3 2 0 4 − 3x + 3x 0 [(x − 1) + (x − 1) + 1] + x + x + 1 Men a˚ se denne overgangen krever at en har en liten algebra-trollmann i magen. ˚ tippe at denne teknikken fungerer og se om theoSom regel er det enklere a ˚ løse rem (2.3.1) kan benyttes. P˚ a enkle integrander kan korollaret brukes til a integraler ved inspeksjon. Lemma 2.3.1. Z

b

f (x) dx = a

1 2

Z

b

f (x) + f (a + b − x) dx

(2.12)

f (x) + f (a + b − x) dx

(2.13)

f (x) + f (a + b − x) dx

(2.14)

a

Z

c

= a

Z =

b

c

Gitt at a,b er to reelle tall og c = (a + b)/2 er gjennomsnittsverdien av a og b.

II

3 Symmetri og nyttige sammenhenger

23

Bevis. Første likning faller rett ut fra (2.3.1), siden integralet over f (x) og f (a + ˚ se neste overgang kan en snitte integralet i to b − x) er like store. For a b

Z

c

Z f (x) dx =

Z f (x) dx +

a

a

b

(2.15)

f (x) dx , c

med c = (a + b)/2. Ved a˚ bruke substitusjonen x 7→ a + b − u p˚ a siste integral f˚ as b

Z

Z

c

Z

a

f (x) dx −

f (x) dx = a

Zac

f (a + b − u) du c

f (x) + f (a + b − x) dx .

= a

Grensene blir u = (a + b) − c = c og u = (a + b) − b = a, og en byttet tilbake til x som integrasjonsvariabel i siste linje. Dersom en i stedet bruker substitusjonen x 7→ a + b − u p˚ a første integral i (2.15) f˚ ar en Z

b

Z

c

f (x) dx = − a

b

Z f (a + b − x) dx +

b

Z

f (x) dx c

b

f (a + b − x) + f (x) dx .

= c

som var den siste likningen som skulle vises. ˚ kjære trigonometriske funksjonene v˚ La oss rette fokus mot de kjente a are. Vi vet allerede at  2π Z 2π Z 2π 2 − sin(2x) 1 − cos(2x) dx = =π sin2 (x) dx = 2 4 0 0 0 Der det ble benyttet at sin2 x − cos2 x = cos 2x, og enhetsformelen cos2 x + sin2 x = 1. Dette er et integral som dukker mye opp b˚ ade i fysikk og flervariabel analyse, men hva med integralet av cos2 (x), kan vi bruke resultatet p˚ a noen ˚ se p˚ m˚ ate? Ved a a figur (2.5) s˚ a er cos2 x og sin2 x samme funksjon, bare at cosinus er forskjøvet med en halv periode alts˚ a π  sin − x = cos x 2 Der virker da logisk at arealet under sinus og cosinus er det samme, gitt at en integrerer over en hel periode. Dette viste du vel allerede fra før, men ved a˚ sette inn i proposisjon (2.3.1) f˚ as direkte at Z

π/2

Z

2

2

Z

π/2

cos(x)2 dx

sin(π/2 − x) dx =

sin(x) dx = 0

π/2

0

0

Interessant, siden arealet under av funksjonene er like s˚ a er Z 0

π/2

sin(x)2 =

1 2

Z 0

π/2

sin(x)2 + cos(x)2 dx =

1 2

Z

π/2

1 dx = 0

π 2

24

II

3 Symmetri og nyttige sammenhenger

1

y

x π/2

π

3π/2



Figur 2.5: Her er cos x den stipplede funksjonen, og sin x er den heltrukne.

1 y x 0

π

π 2

3π 2



Figur 2.6: f (x) og f (a − x) i samme figur. Stilig! Teknikken fungerer ikke bare p˚ a alle intervall, men alle multipler av π/2. Z 0

nπ/2

sin(x)2 =

Z 0

nπ/2

cos(x)2 =

1 2

Z

nπ/2

1 dx = 0

nπ 4

Men dette er kanskje kjent fra før? La oss angripe problemet p˚ a en mer grafisk m˚ ate. Ved a˚ se nærmere p˚ a intervalet 0 til π. Legg merke til symmetrien fra x = 0 til x = π/2 i figur (2.6). Funksjonen f (x) = sin(x)2 deler kvadratet avgrenset av y = 0, y = 1 og x = 0, x = π/2 i nøyaktig to p˚ a figuren. Slik at arealet under f er nøyaktig halvparten av arealet til kvadratet. Alts˚ a Z π/2 1 π π sin2 x dx = = 2 2 4 0 Dette er helt tilsvarende som a˚ bestemme arealet av en trekant, den er nøyaktig halvparten av kvadratet. ˚ f˚ For a a samme resultat som før kan en dele opp integralet i n kvadrat hver med areal π/2, og nøyaktig det samme argumentet kan og benyttes p˚ a cos(x)2 . Hittil har det bare vært h˚ andvifting, hvordan kan en være sikker p˚ a at fremgangsm˚ aten er rett? Svaret finner vi i følgende theorem Theorem 2.3.1. Anta f er kontinuerlig p˚ a [a, b] og at f (x) + f (a + b − x) er konstant for alle x ∈ [a, b] da er   Z b  b−a a+b = f (a) + f (b) f (x) = (b − a)f 2 2 a

II

3 Symmetri og nyttige sammenhenger

25

Bevis. Beviset her tilegnes Roger Nelsen [? , s39-41], og er mildt sagt pent. Merk at teoremet kan vises utelukkende ved a˚ betrakte figur(2.7). Men litt algebratre-

f (a) + f (b)

f

y

f (a)  a+b

f (x)

2

f (b)

x a

(a + b)/2

b

Figur 2.7: Bevis for teoremet uten ord. ˚ beregne integralet benyttes lemma (2.3.1) og ning har aldri skadet noen. For a likning (2.13) Da er   Z b Z c b+a −a K f (x) dx = f (x) + f (a + b − x) dx = 2 a a Siden f (x)+f (a+b−x) = K, hvor K er en konstant. For a˚ bestemme konstanten kan hvilken som helst x ∈ [a, b] benyttes. Velges x = a eller x = b f˚ as K = f (a) + f (b) Derimot om en velger f (x) = c f˚ as  K = 2f

a+b 2



Integralet kan dermed skrives som   Z b  b−a a+b f (x) = (b − a)f = f (a) + f (b) 2 2 a som var det som skulle vises. En ser at om f (x) = sin2 x eller f (x) = cos2 x s˚ a er f (x) + f (π/2 − x) = 1 konstant s˚ a Z π/2 Z π/2  π/2 − 0  π f (0) + f (π/2) = sin2 x dx = cos2 x dx = 2 4 0 0 som stemmer med tidligere resultater. La oss ta et noe vanskeligere eksempel avslutningsvis

26

II

3 Symmetri og nyttige sammenhenger

Eksempel 2.3.3. Putnam 1987: B1 π/2

Z

dx



1 + tan(x)

0

2

.

Dette er en av de vanskeligste oppgavene som har vært gitt p˚ a en Putnam eksa˚ tegne funksjonen f˚ men3 . Ved a ar en noe som likner p˚ a figur (2.8). Allerde n˚ a

y 1 0.8 0.6 0.4 0.2 0

x 0

π/4

π/2 √

Figur 2.8: Grafen til funksjonen 1/ tan(x)

2

.

burde en kjenne igjenn teknikken. Omr˚ adet under funksjonen er halvparten av rektangelet. Slik at det forventes at Z

π/2

dx



1 + tan(x)

0

2

=

1 π π = . 2 2 4

˚ være sikre p˚ For a a dette sjekkes det at f (x) − f (a + b − x) er konstant. N˚ a er tan(π/2 − x) =

sin(π/2 − x) cos x = = 1/ tan(x) cos(π/2 − x) sin x

slik at √

f (x) + f (π/2 − a) = = 3 Putnam

1



1 + tan(x) 1



1 + tan(x)

2

2

+ +

1

1 1

√ + 1/ tan(x) 2 √ tan(x) 2 √ + tan(x) 2

·

tan(x)

2

tan(x)

2



er en den mest prestisjefylte matematikkonkuransen for studenter i USA og Kanada. Konkurransen best˚ ar av 12 spørsm˚ al, med maks poengsum 120. Oppgaven som er vist her klarte bare ˚ f˚ 23 av 2043 deltakare a a 3 eller mer poeng p˚ a

II

3 Symmetri og nyttige sammenhenger

27

Dermed s˚ a er f (x) + f (a + b − x) konstant for alle x ∈ [0, π/2]. Arealet under f (x) er dermed nøyaktig halvparten av kvadratet med høyde 1 og bredde π/2. Fra theorem (2.3.1) har en alts˚ a at   Z π/2 dx π/2 − 0 π √ f (0) + lim f (x) = . = 2 2 4 x→π/2 1 + tan(x) 0 Der f (0) = 1 og den siste grensen blir 0, siden tan(x) g˚ ar mot uendelig n˚ ar x → π/2 dermed vil 1/ tan x → 0. I oppgaven ovenfor var figuren en god indeksjon p˚ a symmetrien. La oss avslutningsvis ta et eksempel som i utgangspunktet virker motstriende mot tidligere resultater. Eksempel 2.3.4. Her skal vi studere følgende integral Z A=

π/2 p

1 + cos2 x dx

0

La som definere funksjonen f som f (x) =

p

1 + cos2 x

Ved a˚ se p˚ a figur(2.9) ser det ut som at integralet fyller opp nøyaktig halvparten av det stiplede omr˚ adet. S˚ a antakelsen om at arealet er

1.5

y

1

0.5

0

x 0

π/4

π/2

√ Figur 2.9: Den røde kurven viser funksjonen f (x) = 1 + cos2 x .

π f (0) + f (π/2) A˜ = 2



√  π 1+ 2 = 4

virker ikke urimelig. Dessverre s˚ a er g(x) = f (x) + f (π − x) =

p

1 + cos2 x +

p

1 + sin2 x

28

3 Symmetri og nyttige sammenhenger

II

˚ se p˚ ikke konstant p˚ a intervallet. Ved a a den deriverte kan en se at funksjonen √ duver sakte i intervalet, eksempelvis s˚ a er g(0) = g(π/2) = 1 + 2 , mens √ ˚ være konstant, men det g(π/4) = 6 . S˚ a f (x) − f (π − x) er akk s˚ a nære a hjelper dessverre ingenting. Ved direkte utregning s˚ a er faktisk √  Z π/2 p π 1+ 2 2 1 + cos x dx ≈ 1.9100 > ≈ 1.8961 4 0 S˚ a selv om det ser ut som funksjonen har pen symmetri, m˚ a en teste om f (x) + 2 f (a+b−x) er konstant. Problemet i denne oppgaven er at 1+cos (x) tilfredstiller √ f (x) = f (a + b − x), mens x ikke gjør det. En konvolusjon av en symmetrisk og ikke symmetrisk funksjon er dessverre ikke symmetrisk. ˚ Addendum: Integralet som ble studert i denne oppgaven ikke er mulig a ˚ beregne analytisk og betegnes som et elliptisk integral av andre grad. Men a vise dette overlates som en artig oppgave til leser. Oppgaver 1. La α være en reell konstant, og f (x) en kontinuerlig funksjon. Hvilke egenskaper m˚ a f ha for at Z α Z α f (x) dx = f (x) dx x −α 1 + β 0 skal gjelde? Bestem ogs˚ a Z



−∞

dx (1 +

x2 )(1

+ ex )

.

2. Vis at Z



 log

0

(1 + sin x)1+cos x 1 + cos x

 dx = 0 ,

3. Bestem konstantene A,B,C og D slik at Z ∞ Z ∞ Z 1 Z ∞ dx dx dx dx = A = B = C . 2 2 2 1 + x 1 + x 1 + x 1 + x2 −1 0 0 −∞

4.

Z 2p p x2 − x + 1 − x2 − 3x + 3 dx 0

5. Følgende transformasjon Z π Z π π xR(sin x, cos2 x) dx = R(sin x, cos2 x) dx 2 0 0 er nyttig i flere sammenhenger. Vis at den stemmer der R(x) er en rasjonell funksjon uten singulariter p˚ a intervalet. Gjelder identiteten for R(cos x, sin2 x) ?

II

3 Symmetri og nyttige sammenhenger

29

6. Et komplett elliptisk integral av andre grad kan skrives p˚ a fomen Z E(k) =

π/2 p

1 − k 2 sin2 x dx

0

a) Vis at integralet Z

π/2 p

1 + cos2 x dx

0

kan uttrykkes via E(x). ˚ beregne et La oss ta en liten digresjon til numerisk integrasjon. For a integral kan en dele opp intervalet i n deler og konstruere et trapes p˚ a hver del. Da ender en opp med at Z T (n) =

b

f (x) dx = a

n−1 X  h f (a) + f (b) + h f (xk ) 2 k=1

hvor n er antall interval, h = (b − a)/n, og xk = a + hk. En øvre grense for feilen i metoden er gitt som 0 ≤ Error ≤

(b − a)3 M 2n2

Hvor M er den største verdien |f 00 (ξ)| har p˚ a intervallet. ˚ være helt sikker p˚ b) Bestem hvor mange interval n som trengs for a a at √ Z π/2 p π(1 + 2 ) 1 + cos2 x dx > 4 0

30

2.4

II

4 Periodiske funksjoner

PERIODISKE FUNKSJONER

I forrige seksjon ble det sett nærmere p˚ a integralene av sin x, og cos x og begge to er eksempler p˚ a periodiske funksjoner. En enkel definisjon p˚ a en periodisk funksjon er følgende Definisjon 2.4.1. La f (x) være en funksjon, og la T være en konstant. Dersom f (x + T ) = f (x) holder for alle x, s˚ a kalles f for periodisk. En sier ogs˚ a at f har en periode p˚ a T. Alts˚ a dersom funksjonen gjentar seg selv, ved jevne interval s˚ a er funksjonen periodisk. Noen ganger kalles periodiske funksjoner ogs˚ a for sykliske funksjoner. merk at periodiske funksjoner ikke trenger a˚ oppføre seg like pent som sin x eller

y t

Figur 2.10: Illustrasjon av en periodisk funksjon cos x. Fra definisjonen ovenfor følger det at 1 har periode π. Siden f (x) = 1, s˚ a er f (x + π) = 1 for alle x. Faktisk s˚ a har konstante funksjoner, alle perioder4 . Proposisjon 2.4.1. Dersom f (x) = f (x + p) og g(x) = g(x + p) for alle x, hvor p 6= 0, og F (x) = f (x) + g(x) og

G(x) = f (x)g(x)

(2.16)

da er b˚ ade har b˚ ade F og G periode p. ˚ beregne F (x + p) og G(x + p), men merk at p ikke Dette er rett frem med a ˚ være den minste perioden funksjonen har. trenger a Eksempel 2.4.1. Definer f (x) = 1 − sin x og g(x) = sin x, da har b˚ ade f og g periode 2π siden sin(x + 2π) = sin x cos 2π + cos x sin 2π = sin x. Merk at F (x) = f (x) + g(x) = 1 Sik at F (x) har alle perioder, ikke bare 2π. 4

˚ definere periodiske funksjoner som f (x + T ) = f (T ), hvor En kan unng˚ a dette problemet ved a ˚ betrakte konstanter som periodiske T er ulik null. Men i mange situasjoner er det mer naturlig a funksjoner.

II

4 Periodiske funksjoner

31

Eksempel 2.4.2. Definer f (x) = cos x − i sin x og g(x) = cos x + i sin x, da har igjen b˚ ade f og g en minste periode p˚ a 2π. Merk at G(x) = f (x) · g(x) = cos2 x − i2 sin2 x = cos2 x + sin2 x = 1 Sik at F (x) igjen har alle perioder5 , ikke bare 2π. Theorem 2.4.1. La f (x), være en funksjon med periode T hvor T 6= 0, og la k være en vilk˚ arlig konstant da er a+T

Z

b+T

Z f (x) dx =

Z

b kT

T

Z f (x) dx = k

Z

(2.17)

f (x) dx

a

0 b+nT

0 b

Z

f (x) dx + (n − m)

f (x) dx = a+mT

(2.18)

f (x) dx

Z a

T

f (x) dx .

x

2T a

T

(2.19)

0

T +a

Figur 2.11: Illustrasjon av likning (2.17). Omr˚ adet fra a til a + T , er like stort som arealet over en periode − alts˚ a fra T til 2T .

Korollar 2.4.1. Dersom R(x, y) er en rasjonell funksjon s˚ a er Z



Z

π

R(sin x, cos x) dx =

R(sin x, cos x) dx −π

0

Bevis. Dette følger direkte fra likning (2.17) med a = 0 og b = −π og T = 2π. ˚ sette m, a, b = 0, og Merk at likning (2.18) følger direkte fra (2.19) ved a ˚ la b = a, m = 0, n = 1 s˚ n = k. Tilsvarende kan første likning vises ved a a Z

a+1T

Z

Z f (x) dx + (1 − 0)

f (x) dx = a+0T

a

a

T

Z f (x) dx =

0

T

f (x) dx 0

Ved a˚ heller sette b = a f˚ ar vi samme resultat, og integralene m˚ a derfor være like. ˚ bevise siste likning om en først har vist de to foreg˚ Men det er enklere a aende. Her bevises bare (2.17) og (2.18) mens siste likning overlates som en øvelse til leser. 5 Her

˚ innføre denne vil kanskje noe se p˚ er i2 = −1 den s˚ akalte imaginære enheten. A a som juks. Droppes derimot i blir G(x) = cos 2x som b˚ ade har periode 2π og π.

32

II

4 Periodiske funksjoner

˚ vise at begge integralene er likt integralet over 0 til Bevis. Det er noe enklere a T . Uten tap av generalitet la c ≡ a ≡ b, da er Z c+T Z T Z T +c f (x) dx = f (x) dx + f (x) dx c

c

T c

T

Z

Z f (x) dx +

= c

Z f (x + T ) dx =

0

T

f (x) dx 0

Hvor substitusjonen u = x − T ble brukt i andre overgang og f (x + T ) = f (x) siden f har periode T . Dette medfører at Z a+T Z T Z b+T f (x) dx = f (x) dx = f (x) dx a

b

0

som ønsket. Intuitivt beskriver dette en horisontal forflyttning, og om funksjonen er periodisk vil denne forlyttningen bevare arealet. Siden k er et naturlig tall kan en dele opp integralet kT

Z

f (x) dx = 0

k−1 X Z (n+1)T n=0

f (x) dx =

nT

k−1 XZ T n=0

f (u + nT ) du

(2.20)

0

der substitusjonen x 7→ u + nT ble benyttet i siste overgang. Via induksjon er ˚ vise at f (x + nT ) = f (x) holder for alle n ∈ Z. Argumentet holder det enkelt a for n = 1, og anta s˚ a at f (x + pT ) = f (x) for en eller annen p. N˚ a er  f x + (p + 1)T = f (x + T ) + pT ) = f (x + T ) = f (x) Her ble induksjonshypotesen brukt f (x0 + pT ) = f (x0 ) med x0 = x + T , og i siste overgang at f er periodisk. Resultatet følger ved induksjon og ved innseting f˚ as Z

kT

f (x) dx = 0

k−1 XZ T n=0

f (x + nT ) dx =

0

k−1 XZ T n=0

Z f (x) dx = k

0

T

f (x) dx , 0

som var det som skulle vises. Dette fullfører beviset. Likning (2.18) beskriver en samling av k like integral av typen i likning (2.17), siden integralet over hver hele periode er like stort. En kan og kombinere resultatene, for eksempel s˚ a f˚ as Z a+kT Z b+T f (x) dx = k f (x) dx a

b

˚ bruke likning (2.17) og (2.18). Setter enn n = m inn i (2.19) f˚ ved a ar en Z b Z b+nT = f (x) dx = f (x) dx (2.21) a

a+nT

hvor n selvsagt kan være negativ. Igjen gir dette mening siden om en integrerer over (2T, 3T ) eller (T, 2T ) spiller ingen rolle da funksjonen er periodisk, en st˚ ar ˚ skifte integralet slik det passer seg. alts˚ a fritt til a ˚ ta en rekke sm˚ I stedet for a a eksempler, sammler vi alt i et magnum opus eksempel nedfor. Merk en del av teorien fra avsnitt (2.3) vil og bli brukt

II

4 Periodiske funksjoner

33

Eksempel 2.4.3. Z

53π/4

dx (1 +

25π/4

2sin x ) (1

+

2cos x )

=

7π 4

Legg merke til at om en ansl˚ ar integralet numerisk, f˚ ar en I ≈ 5.4978, s˚ aI ≈ a at verdien av integralet 1.75π = 7π/4. Dette gir oss i hvertfall en god gjetning p˚ er rett. La f (x) betegne integranden n˚ a er f (x + 2π) = f (x) slik at integranden y 1 0.8 0.6 0.4 0.2 x k

k + 2π

k + 4π

k + 6π k + 7π

Figur 2.12: Grafen til f , hvor k = 25π/4. er periodisk. Dette kan en enten se ved innsettning eller at integranden best˚ ar ˚ skrive om grensene kan av sin x og cos x ledd som begge har periode 2π. Ved a integralet skrives som Z

5π/4+6·2π

I=

Z

5π/4

f (x) dx = π/4+3·2π

Z



f (x) dx + 3 π/4

f (x) dx 0

hvor (2.19) ble brukt, og f (x) igjen betegner integranden. I ’praksis’ er omskrivningen at integralet deles opp i to deler, delen som best˚ ar av hele perioder, og delen som best˚ ar av halve perioder. Eg fra k til k + 6, og k + 6 til k + 7 i figur (2.12). Kall integralene for henholdsvis A og B, merk at Z



π

dx = sin x (1 + 2 ) (1 + 2cos x )

Z

π

0

Z = 0

π

du 2sin u 2cos u   sin u cos u 2 1 + 2sin(u+π) 1 + 2cos(u+π) 2 2sin x 2cos x dx (1 + 2sin x ) (1 + 2cos x )

via substitusjonen x 7→ u + π, her ble ogs˚ a sin(u + π) = − sin u og cos(u + π) = − cos u benyttet. Dette resultatet f˚ ar en bruk for om en deler B p˚ a midten Z B=3

π

Z



f (x) dx + 3 0

Z f (x) dx = 3

π

0

π

1 + 2sin x 2cos x dx , (1 + 2sin x ) (1 + 2cos x )

34

4 Periodiske funksjoner

II

Det kan vises at f (x) + f (a + b − x) er konstant [Oppgave] slik at theorem (2.3.1) kan brukes. Da er   a+b 1 + 2sin(π/2) 2cos(π/2) 3π   = B = 3(b − a)f = 3π sin(π/2) cos(π/2) 2 2 1+2 1+2 Helt tilsvarende kan A beregnes s˚ a Z 3π/4 Z 5π/4 dx dx A= + sin x cos x sin x (1 + 2 ) (1 + 2 ) ) (1 + 2cos x ) π/4 3π/4 (1 + 2 ˚ bruke substitusjonen u = x − π/2 s˚ Ved a a kan siste integral skrives som Z 5π/4−π/2 Z 3π/4 du 2sin x dx   = (1 + 2sin x ) (1 + 2cos x ) 1 + 2sin(u+π/2) 1 + 2cos(u+π/2) 3π/4−π/2 π/4 ˚ bruke dette kan A Siden sin(u + π/2) = − sin u og cos(u + π/2) = cos u. Ved a skrives som Z 3π/4 Z 3π/4 dx 1 + 2sin x dx = A= sin x cos x (1 + 2 ) (1 + 2 ) 1 + 2cos x π/4 π/4 ˚ beregne integralet kan igjen benytte seg av theorem (2.3.1) siden g(x) + For a g(a + b − x) er konstant. Z 3π/4 π 1 π dx = = A= cos x cos(π/2) 1+2 2 1+2 4 π/4 som før. Her betegner da selvsagt g = 1/(1 + 2cos(x) ). Oppsumert s˚ a er Z 53π/4 3π π 7π dx = + = sin x ) (1 + 2cos x ) (1 + 2 2 4 4 25π/4 ˚ vise. som var det en ønsket a En noe mer intuitiv fremgangsm˚ ate a˚ vise A p˚ a er a˚ heller bruke likning (2.13) fra lemma (2.3.1). Z 3π/4 Z π/2 Z π/2 dx 1 1 π A= = + = dx = cos x cos x − cos x 1+2 1+2 4 π/4 π/4 1 + 2 π/4 Hvor cos(π/4 + 2π/4 − x) = − cos x. Om en ikke er s˚ a glad i benytte slike knep heller, kan integralet ogs˚ a beregnes enda mer elementært. Z 3π/4 Z π/2 Z 3π/2 dx dx dx = + cos x cos x 1+2 1 + 2cos x π/4 π/4 1 + 2 π/2 Z π/2 Z π/4 dx 2cos(π−x) = − dx cos x cos(π−x) π/4 1 + 2 π/2 1 + 2 Z π/2 Z π/2 cos x 2 dx dx + , = cos x cos x 1 + 2− cos x 1 + 2 2 π/4 π/4 der substitusjonen u 7→ π/2 − x ble brukt. Helt tilsvarende regning kan a˚ brukes ˚ beregne B elementært. for a ˚ se p˚ I dette eksempelet studeres en del spesialtillfeller av theoreme ved a a funksjoner med ulike egenskaper

II

4 Periodiske funksjoner

35

Proposisjon 2.4.2. La n ∈ Z/{0} være et heltall ulikt null og la f være en funksjon. Dersom 1) at f er en funksjon med periode T , f (x + T ) = f (x) s˚ a er Z T Z T f (nx) dx = f (x) dx 0

2) at f er en vilk˚ arlig funskjon slik at integralene konvergerer s˚ a er Z π/2 Z π/2 f (sin 2x) dx = f (sin x) dx 0 Z π/2

(2.22)

0

(2.23)

0

f (cos 2x) dx = 0

1 2

Z

π/2

f (cos x) + f (− cos x) dx

(2.24)

0

3) at f er en likefunksjon, f (−x) = f (x) s˚ a er Z

π/2

Z

π/2

f (sin nx) dx = 0 Z π/2

f (sin x) dx

(2.25)

f (cos x) dx

(2.26)

0 Z π/2

f (cos nx) dx = 0

0

I tilfellet hvor f er odde, f (−x) = −f (x) holder likning (2.25) og (2.26) generellt bare for n = ±1, ±2. ˚ heller studere Beviset overlates til leser se oppgave (7). Vi kommer til a symmetriegenskapene mellom sin x og cos x. Proposisjon 2.4.3. La m, p, q ∈ Z, da er Z π(s−k)/2 sin(mx)n dx = 0 og

Z

π(r−k)/2

π(q+k/2)

cos(mx)n dx = 0

π(p−k/2)

n˚ ar n ∈ 2N − 1 og r, s ∈ 2Z + 16 . Alts˚ a er r ∈ 2j + 1, s ∈ 2i + 1 hvor j, i ∈ N. Det kreves i tillegg at dersom i og j har samme paritet m˚ a k være like og og tilsvarende for p og q. Dersom i og j eller p og q har ulik paritet s˚ a kan k ∈ Z velges fritt. Beviset her uttelates men er logisk om en betrakter figuren. Beviset ville vært a˚ dele opp integralene halveis, og drøftet de ulike tilfellene for i, j, k, m, n, p, q, r, s men for leserens mentale helse utellates dette. Derimot fører proposisjonen direkte til Korollar 2.4.2. Z

2nπ 2k+1

cos(mx)

Z



dx =

0

Z

cos(mx)2k+1 dx = 0

(2.27)

sin(mx)2k+1 dx = 0

(2.28)

0 2nπ

0

Hvr n, m, k ∈ N. 6 Notasjonen

her betyr at n er odde. Dette kan og skrives som n ∈ {2u − 1 : u ∈ N}, denne notasjonen tar dog opp noe mer plass. Tilsvarende for 2Z ± 1, merk disse mengdene er identiske.

36

II

4 Periodiske funksjoner

Lemma 2.4.1. La R være en rasjonell funksjon da er Z

K

K

Z

R(cos x, sin x) dx

R(sin x, cos x) dx = 0

0

Spesielt s˚ a er Z

K

K

Z

f (cos x) dx .

f (sin x) dx = 0

0

Hvor K = π/2 eller K = 2π. ˚ vise likheten for K = π/2 er trivielt. Ved a ˚ bruke den kjære substitusjoBevis. A nen y 7→ π/2 − x s˚ a er π/2

Z

Z

0

 R cos(π/2 − y), sin(π/2 − y) dy

R(sin x, cos x) dx = − π/2

0

Z

π/2

R(cos x, sin x) dx ,

= 0

som ønsket. La oss n˚ a studere tilfellet K = 2π som er hakket mer spennende. ˚ først bruke den samme substitusjonen y 7→ π/2 − x s˚ Ved a a er Z



Z

−3π/2

 R sin(π/2 − x), sin(π/2 − y) dy .

R(sin x, cos x) dx = − π/2

0

Integralet kan forenkles ved a˚ snu grensene og benytte som før at sin(π/2 − y) = ˚ dele integralet ved x = 0. cos y og cos(π/2 − y) = cos y. Neste steg blir a Z

0

Z

π/2

R(cos y, sin y) dy +

R(cos y, sin y) dy

−3π/2

0

En naturlig fortsettelse er a˚ benytte substitusjonen y 7→ 2π+u eller likning (2.19) med n = m = 1,a = −3π/2,b = π/2 og T = 2π p˚ a første integralet. Uansett blir Z



Z R(cos u, sin u) du +

π/2

π/2

Z R(cos y, sin y) dy =

0



R(cos x, sin x) dx 0

Som var det som skulle vises. Nøyaktig samme bevis som ovenfor kan brukes p˚ a f (sin x), noe enklere er bare se at R(cos x, sin x) = f (cos x) om R ikke inneholder noen sinus ledd. Dette vil si at R(x, y) er en symmetrisk funksjon. Som betyr at i ethvert ˚ bytte om disse integral som kun best˚ ar av cos x og sin x ledd, st˚ ar en fritt til a Z 0



3 cos x dx = 2 sin x − cos x + 3

Z 0



3 sin x dx , 2 cos x − sin x + 3

s˚ a fremt en integrerer over en kvart, eller hel periode. Lemmaet her kan brukes ˚ vise et noe mer generelt resultat til a

II

4 Periodiske funksjoner

37

Proposisjon 2.4.4. La k være en konstant da er Z πk/2 Z πk/2 R(cos x, sin x) dx = R(sin x, cos x) dx , −πm/2

−πm/2

gitt at k, m ≡ 0 (mod 4) eller k, m ≡ 1 (mod 4). Siste del betyr at k og m alltid er p˚ a formen 4n eller 4n + 1 for et heltall n ˚ vise og ikke nødvendigvis like. Slik at vi kan godt velge k = 4 + 1 og m = −4. A dette tar liten plass n˚ ar vi allerede har vist lemma (2.4.1) Bevis. Vi kan uten tap av generalitet sette m = 0 igjennom hele beviset. Siden ˚ vise proposisjonen for de negative verdiene integralet kan deles ved x = 0 og a er akkuratt likt som for de positive. Tar først tilfellet for k = 4n, da er Z 2nπ Z 2π R(cos x, sin x) dx = n R(cos x, sin x) dx 0

0

Z



=n

Z

2nπ

R(sin x, cos x) dx = 0

R(sin x, cos x) dx , 0

der første og siste overgang følger fra likning (2.18) og andre overgnag følger fra lemma (2.4.1). Tilsvarende for tilfellet n˚ ar k = 4n + 1 s˚ a er Z 2nπ+π/2 Z 2π Z π/2 R(cos x, sin x) dx = n R(cos x, sin x) dx + R(cos x, sin x) dx , 0

0

0

hvor likning (2.18) ble brukt. En har vist symmetrien til begge integralene p˚ a ˚ bruke dette har en dermed høyresiden i lemma (2.4.1). Ved a Z 2nπ+π/2 Z 2π Z π/2 R(cos x, sin x) dx = n R(sin x, cos x) dx + R(sin x, cos x) dx 0

Z

0 2nπ+π/2

0

R(sin x, cos x) dx ,

= 0

som var det som skulle vises. Dette fullfører beviset. ˚ bruke denne symmetrien til a ˚ løse integral er ikke rett frem. Men den er A ˚ vise at ulike trigonometriske integral er like. Vi kan etablere svært nyttig for a liknende identiteter for Proposisjon 2.4.5. . La f (x) være en begrenset funksjon p˚ a intervalet og la m,n være heltall. Da er Z nπ Z 2πn 2 f (cos x) dx = f (cos x) dx (2.29) Z

−mπ nπ/2

−2mπ nπ

Z f (sin x) dx =

2 −mπ/2

f (sin x) dx

(2.30)

−mπ

gitt at n, m er p˚ a formen n, m ≡ 0 (mod 4) eller n, m ≡ 1 (mod 4). Beviset for dette overlates til leser, og kan vises p˚ a samme m˚ ate som de to tidligere proposisjonene. Merk at proposisjonen holder selv om m = 4p og n = 4l + 1. La oss ta en svak generalisering av Riemann-Lebesgue lemmaet

38

II

4 Periodiske funksjoner

Lemma 2.4.2. (Riemann-Lebesgue lemma) La f (x) være en kontinuerlig deriverbar7 funksjon p˚ a (a, b). La g(x) være en funksjon med periode T . Da er Z lim

n→∞

a

b

1 f (x)g(nx) dx = T

Z

T

Z g(x) dx

b

f (x) dx . a

0

˚ gjøre beviset enklere Lemmaet holder under svakere betingelser, men for a holder dette for oss Bevis. La oss først se p˚ a integralet over g(x), dette kan skrives som ! Z T Z x 1 g(x) dx + h(x) = xA + h(x) . g(t) = x T 0 0 Her betegner A integralet, og h er en eller annen kontinuerlig funksjon med periode T . Første overgang kommer fra likning (2.18), hvor k = x/T . For at dette skal gjelde m˚ a x/T være et heltall , dette ordnes via funksjonen h(x) som ˚ derivere begge sider s˚ er g(x) integrert over et passelig interval. Ved a a er g(x) = A + h0 (x) , slik at g(nx) = A + h0 (nx). Her trengtes ikke produktregelen og benyttes p˚ a først del, da A er en konstant og ikke avhengig av x. Settes dette inn i den opprinnelige likningen f˚ as Z

b

Z f (x)g(nx) dx = A

a

b

Z

b

f (x) dx + a

f (x)h0 (nx) dx .

a

˚ vise at det siste integralet g˚ ˚ M˚ alet er n˚ aa ar mot null, n˚ ar n g˚ ar mot ∞. Ved a bruke delvis integrasjon kan integralet skrives som b

Z

0



f (x)h(nx) n

f (x)h (nx) dx = a

b

1 − n a

b

Z

f 0 (x)h(nx) dx .

a

Siden f , f 0 og h(x) er kontinuerlige p˚ a (a, b) medfører dette og at de er øvre begrenset, som igjen betyr at integralet konvergerer. Høyresiden er dermed et reelt tall s˚ a høyresiden g˚ ar mot null n˚ ar n vokser. Dette medfører Z n→∞

b

Z f (x)g(nx) dx = A

lim

a

b

f (x) dx + 0 = a

1 T

Z

T

Z g(x) dx

0

b

f (x) dx , a

som var det som skulle vises.

7 kontinuerlig

deriverbar er en faglig betegnelse som betyr at f er kontinuerlig, og at dens deriverte ogs˚ a er kontinuerlig

II

4 Periodiske funksjoner

39

Oppgaver 7. a) Lett oppvarming. Vis at dersom f har periode T , alts˚ a f (x + T ) = f (x) s˚ a er Z T Z T f (nx) dx = f (x) dx , 0

0

gitt at n = Z/{0} − alts˚ a at n er et vilk˚ arlig heltall ulikt 0. Bevis ogs˚ a resten av proposisjon (2.4.2). 8. Bevis at det kun eksisterer to n ∈ N slik at Z π/2 Z π/2 dθ dθ I(n) = = 2 + sin nθ 2 + sin θ 0 0 π √ 2 3

Tilsvarende bestem n˚ ar I(n) =

.

9. Vis at Z

b+nT

Z

b

f (x) dx + (n − m)

f (x) dx = a+mT

Z

a

T

f (x) dx 0

hvor igjen a, b, T er reelle tall, f (x + T ) = f (x) for alle x, og n, m ∈ Z. 10. Bestem integralet Z

71π/2

I=

 log

23π

(1 + sin x)1+cos x 1 + cos x

 dx .

a periodiske funksjoner, og i denne oppgaven ser en 11. Tidligere har en sett p˚ nærmere p˚ a spesialtilfellet der integranden i tillegg er odde eller like. La f (x) være en likefunksjon, og la g(x) være odde. B˚ ade f og g har periode T = 2L, hvor L er en naturlig konstant. I oppgaven skal integralene Z x Z x G(x) = f (t) dt og F (x) = g(t) dt , 0

0

studeres nærmere8 . a) Bestem pariteten til F og G. Alts˚ a om F og G er odde eller like. b) Bestem G(2nL) og F (2nL), hvor n ∈ Z. c) Vis at F ogs˚ a har periode 2L, hvilke betingelser kreves for at G ogs˚ a skal ha periode 2L? 8 Merk

at egenskapene vi viser her minner svært om egenskapene til sin x, og cos x og blant elementære funksjoner er disse de eneste periodisk, odde og like funksjonene.

Tillater en derimot delte funksjonsuttrykk, finnes det mange flere eksempler, disse mye brukt innen signalbehandling og fourier-analyse.

40

5 Diverse substitusjoner

2.5

II

DIVERSE SUBSTITUSJONER

Vi betegner en substitusjon som følgende overgang Z ψ Z β f [g(t)]g 0 (t) dt f (x) dx = ϕ

α

og formelt sett er dette en avbildning fra et omr˚ ade X til et omr˚ ade Y via funksjonen x 7→ g(t). Hva dette i praksis vil si er at vi g˚ ar fra et koordinatsystem x,y til et nytt koordinatsystem x0 ,y 0 og det eneste som er konstant mellom systemene er at arealet under f (t) ∈ (a, b) og f (g(t)) ∈ (ϕ, ψ) er like store. Vi begynner med a˚ se p˚ a noen f˚ aq eksempler p˚ a nyttige susbtitusjoner. Inneholp der integralet (x − a)(b − x eller x−a s˚ a kan substitusjonen x 7→ a cos2 θ + b−x b sin2 θ Eksempel 2.5.1. Z I= a

b

dx p

(x − a)(b − x)



Vi benytter den anbefalte substitusjonen x 7→ a cos2 θ + b sin2 θ. Da er   (x − a)(b − x) = a(cos2 θ − 1) + b sin2 θ b(1 − sin2 θ) − a cos2 θ = (−a sin2 θ + b sin2 θ)(b cos2 θ − a cos2 θ) = (b − a)2 sin2 θ cos2 θ Mens den deriverte kan skrives som dx = −2a cos θ sin θ + 2b sin θ cos θ = 2(b − a) sin θ cos θ dθ Herfra m˚ a vi arbeide med grensene v˚ are. N˚ ar x = a f˚ ar vi likningen a = a cos2 θ + b sin2 θ som medfører (a − b) sin2 θ = 0. Her kan vi se bort i fra det trivielle tilfellet a = b. S˚ a vi m˚ a ha sin2 θ = 0 eller θ = nπ, n ∈ Z. Vi velger n = 0 for enkelhetens skyld. Tilsvarende for x = β finner vi at θ = π/2 + nπ. Denne gangen kan vi ˚ gjette at vi m˚ ikke velge n fritt. Det er relativt trygt a a velge θ = π/2. Vi kan rettferdiggjøre dette ved a˚ se at vi ønsker at x øker fra a til b, og dersom vi velger en annen verdi for den øvre grensen vil ikke dette skje. Substitusjonen gir alts˚ a Z π/2 Z π/2 Z b dx (b − a) sin θ cos θ dθ p p =2 = 2 dθ = π (x − a)(b − x) (b − a)2 sin2 θ cos2 θ a 0 0 som ønsket. Noen a er cos x og sin x √ tillegskommentarer: For x ∈ [0, π/2] s˚ 2 2 positive slik at sin θ cos θ = | cos x|| sin x| = cos x sin x. Vi kan og se raskt p˚ a konvergens, eneste farememomenter er x = a og x = b. Integralet oppfører R1 √ seg som 0 dx/ x nære disse punktene og konvergerer dermed. En har fra taylorutvikling at Z Z dx p ∼ (x − a)−1/2 (b − a)−1/2 dx = 2(x − a)1/2 (b − a)−1/2 (x − a)(b − x) som er konvergent. Helt tilsvarende omkring x = b.

II

5 Diverse substitusjoner

41

˚ evaluere bestemte integral ved bruk av substitusjon krever ømfintlighet. A Følgende theorem gir oss tilstrekkelige betingelser for en gyldig substitusjon Theorem 2.5.1. Dersom funksjonen t = φ(x) tilfredstiller følgende 1) φ(x) er en kontinuerlig en-til-en funksjon definert p˚ a intervalet [ϕ, ψ] og har en kontinuerlig derivert der. 2) Verdiene til φ(t) ligger i intervalet [a, b] 3) φ(ϕ) = a og φ(ψ) = b. da vil følgende formel holde Z b

Z

ψ

f (g(x))g 0 (x) dx

f (t) dt = a

ϕ

for alle f stykkevis kontinuerlig funksjoner derfinert p˚ a [a, b]. Gjennom nonen eksempler vil vi vise visse fallgruver om en ikke passer p˚ a betingelsenene ovenfor. Det enkleste er alts˚ a benytte seg av monotone9 substitusjoner. Ellers m˚ a en dele opp intervallet til substitusjonen blir monoton. Eksempel 2.5.2. Z

2

I= −2

dx 4 + x2

Variabelskifte x 7→ 1/t vil her lede til et galt svar Z 2 Z 1/2 Z 1/2 dx dt dt  I= = = − 2 2+1 2 1+ 1 4 + x 4t t −2 −1/2 −1/2 t2  −1/2 π 1 =− = arctan 2t 2 4 1/2 Som er umulig siden funksjonen er strengt positivt. Dette skjer fordi substitusjonen 1/t ikke er deriverbar i origo10 . Rett svar er selvsagt Z 2 h  π i dx x i2 1 hπ π I= = arctan = − − = 2 4 + x 2 2 4 4 4 −2 −2 En annen som kan komme er n˚ ar en bruker den iverse substitusjonen x = ψ(t) og x = φ(t) tar flere verdier11 . 9 En

monoton funksjon er en funksjon som er avtagende eller synkende p˚ a ett gitt omr˚ adet. er jo selvsagt ikke helt rett. Problemet ligget i at vi m˚ a dele opp intervallet v˚ art ved origo før vi kan derivere. Dette handler om at x 7→ 1/x mapper [−2, 2] til (−∞, −1/2] ∪ [1/2, ∞) og ikke [−1/2, 1/2] som en kanskje skulle trodd i utgangspunktet.

10 Dette

11 Innen

kompleks analyse klaller vi slike funksjoner gjerne for branch cuts.

42

5 Diverse substitusjoner

II

Eksempel 2.5.3. Z I=

3

(x − 2)2 dx

0

˚ bruke substitusjonen u 7→ (x − 2)2 vil resultere i et galt Ved a √ resultat. Dette er fordi den√ inverse funksjonen ikke er unikt definert x = 2 ± t . en√ene grenenen x1 = 2 − t kan ikke ta verdier for x > 2, og den andre x2 = 2 − t kan ikke ta verdier for x < 2. Direkte (men feil) utregning via u 7→ (x − 2)2 gir  1 Z 3 Z 1 1√ 2 3/2 8 7 1 2 (x − 2) dx = u du = u − =− = 2 3 3 3 3 0 4 4 ˚ penbart feil, da integranden er positiv p˚ ˚ f˚ A a hele omr˚ adet. For a a det korrekte ˚ dele integralet i to deler svaret er det nødvending a Z 2 Z 3 I= (x − 2)2 dx + (x − 2)2 dx 0

2



√ S˚ a kan vi la x 7→ 2 − t i første integrealet og x 7→ 2 + t i det andre. Dette gir Z 2 Z 0 Z dt 1 4√ 8 √ I1 = (x − 2)2 = − t = t dt = . 2 0 3 2 t 0 4 Z 3 Z 1√ Z 1 1 1 dt I2 = (x − 2)2 = t dt = t √ = . 2 0 3 2 t 2 0 ˚ beregne Derfor s˚ a er I = I1 + I2 = 38 + 13 = 3. Resultatet kan bekreftes ved a integralet direkte 3  Z 3 1 8 (x − 2) 2 = + =3 (x − 2) dx = 2 3 3 0 0 En kan og betrakte12 en substitusjon som at en omformer et materialet eller væske som en ikke kan trykkes sammen13 . En substitusjon kan da sees p˚ a som an en drar materialet i en eller annen retning, merk at transformasjonen m˚ a være kontinuerlig, og kan heller ikke innføre ”hull”i omr˚ adet. ˚ En avbildning fra X til Y ikke er entydig og det kan finnes mange m˚ ater a komme seg fra X til Y p˚ a. En kan for eksempel g˚ a fra X til Z ogs˚ a fra Z til Y , eller en rekke andre m˚ ater. Omr˚ adene vi vil arbeide med vil i hovedsak være interval p˚ a den reelle tallinjen slik at en kan skrive X = [a, b] og Y = [ϕ, ψ]. Men det er ikke noe mot at X er et interval og at Y er en sirkel for eksempel. Ofte n˚ ar vi bruker substitusjon er m˚ alet a˚ forenkle integranden, men hva om ˚ forenkle grensene i stedet. Hvordan skal vi finne en substitusjon slik vi ønsker a at grensene g˚ ar fra [a, b] til [ϕ, ψ]? Substitusjonen kan skrives som Z b Z ψ f (t) dt = f (g(x))g 0 (x) dx , a

ϕ

hvordan m˚ a da t = g(x) velges? Følgende propsisjon gir oss heldigvis svaret 12 En

alternativ synsvinkel er en lineær transformasjon mellom to topologiske rom. kalles et slikt materialet med konstant volum for inkompressibelt. N˚ ar det gjelder gasser, kalles en prosses med konstant volum for en isokor prosess.

13 Formelt

II

5 Diverse substitusjoner y

cos x

2

2

1.5

1.5

1

1

0.5

0.5

0 0.5

1

1.5

y0

1−t2 2 1+t2 1+t2

x0

0

x 0

43

0

2

(a) Funksjon med areal 1.

0.5

1

1.5

2

(b) Funksjon med areal 1.

Figur 2.13: Arealet under begge funksjonene er like og t 7→ tan(x/2) mapper x, y til x0 ,y 0 . Rβ Theorem 2.5.2. Integralet α f (t) dt kan bli transformert til et annet integral med grenser ϕ og ψ via den lineære transformasjonen αψ − βϕ β−α x+ ψ−ϕ ψ−ϕ

T (x) 7→

(2.31)

Da kan integralet skrives som Z

β

f (t) dt = α

β−α ψ−ϕ

Z

ψ

 f

ϕ

αψ − βϕ β−α x+ ψ−ϕ ψ−ϕ

 dx

(2.32)

˚ anta at x(t) = Beviset overlates til leser. Det er ikke vanskeligere enn a Ax + B. Kravene om at x(α) = ψ, x(β) = ϕ bestemmer konstantene A og B. Fra dette følger Korollar 2.5.1. Z

b

Z

a

1

 f (b − a)t + a dt  Z0 ∞  a + bx dt = (b − a) f 1 + x (1 + t)2 0

f (x) dx = (b − a)

(2.33) (2.34)

for alle a, b ∈ R. Dette følger fra og sette inn henholdsvis ϕ = 0, ψ = 1 og ϕ = 0, ψ → ∞ i likning (2.32). ˚ se likning (2.32) p˚ ˚ vise korollaret først. En noe mer intuitiv m˚ ate a a er a ˚pent interval (a, b) kan ’krympes’ ned til enhetsintervalet Dette viser at ethvert a (0, 1) via en eller annen transformasjon U . Dette betyr ogs˚ a at en kan krympe ˚ ta inversen av intervalet (ϕ, ψ) ned p˚ a (0, 1) via en transformasjon V . Ved a V , g˚ ar vi motsatt vei fra (0, 1) til (ϕ, ψ) alts˚ a ’forstørres’ omr˚ adet. Dette er ˚ først bruke transformasjonen U , og deretter V −1 illustrert i figur (2.14). Ved a

44

5 Diverse substitusjoner

a

b

U 0

V

II

U (V −1 )

1

V

ϕ

−1

ψ

Figur 2.14: Mapping fra (a, b) til (ϕ, ψ). ˚ vise at U (V −1 ) er samme transformasjon som i g˚ ar vi alts˚ a fra (a, b) til (ψ, ϕ). A likning (2.31) overlates til leser. Som vist ovenfor er ikke avbildninger unike, det finnes mange veier en kan ta fra X til Y - I tabell (2.2) er et utvalg transformasjoner fra X til Y vist. Her ˚ betgne omr˚ blir notasjonen (a, b) for a adet en integrerer over. Tabell 2.2: Et utvalg av sentrale substitusjoner X

Y

t = g(x)

(a, b)

(−1, 1)

(0, a) (0, 1) (0, 1) (−1, 1) (0, ∞) (1, ∞) (0, ∞) (0, ∞) (0, ∞) (0, ∞) (0, ∞) (0, ∞) (−∞, ∞)

(0, 1) (a, b) (−∞, ∞) (−∞, ∞) (a, b) (0, 1) (−1, 1) (−1, 1) (0, 1) (0, 1) (1, 0) (1, 0) (−∞, ∞)

b+a 2x + b−a b−a x/a a + (b − a)x (1 − 2x)/4(x2 − x) x/(1 − x2 ) (a − x)b/(b − x) 1/x (x − 1)/(x + 1) π arctan(x)/4 − 1 x/(1 + x) tanh x 1/(1 + x2 ) exp(−x) x + 1/x −

En transformasjon som er utelatt fra tabellen er hvordan en kan mappe (a, b) til R = (−∞, ∞), og motsatt. Grunnen til dette er plassmangel, og resultatet er gitt under Proposisjon 2.5.1. La a, b være to reelle tall med b > a, da er T (x) =

1 (b − a)(b + a − 2x) 4 x2 − (a + b)x + ab

En funksjon slik at T

X −→ Y Hvor X = (a, b) og Y = R.

(2.35)

II

5 Diverse substitusjoner

45

˚ vise, men det holder a ˚ sjekke at at x → a fører til Dette overlates til leser a at T → −∞, x → b fører til at T → ∞, og at T er kontinuerlig p˚ a (a, b). For a˚ utlede resultatet kan en for eksempel mappe (a, b) p˚ a (−1, 1) og deretter ˚pent (−1, 1) p˚ a R. Det eksisterer heldigvis enklere transformasjoner fra et a interval til den reelle tallinjen om en tillater seg bruk av hyperbolske funksjoner. ˚ finne en slik transformasjon er gitt i oppgaveteksten under. A Merk avbildningene ovenfor kan b˚ ade snus (0, 1) = −(1, 0) og deles opp (0, ∞) = (0, 1) ∩ (1, ∞). Dette vil komme til nytte i følgende omskrivning Proposisjon 2.5.2. Z



1

Z g(x) dx = S

0

0

1 g(Sx) + 2 g x



S x

 dx

(2.36)

Hvor S > 0 er en vilk˚ arlig konstant. ˚ benytte substitusjonen x 7→ St ⇒ dx = S dt f˚ Bevis. Ved a as Z ∞ Z ∞ g(x) dx = S g(St) dt 0

0

Grensene blir uforandret siden S > 0. Merk at Z ∞ Z 0   dt S , g(Sx) dx = − g t t2 1 1

(2.37)

˚ dele opp intervalet i via substitusjonen x 7→ 1/t ⇒ dx = − dt/t2 . Ved a (0, 1) ∩ (1, ∞) kan n˚ a integralet skrives som   Z 1 Z ∞ Z 1 1 S S g(Sx) dx + S f (Sx) dx = S g(Sx) + 2 f dx x x 0 1 0 Der likning (2.37) ble brukt. Dette fullfører beviset. Spesielt s˚ a vil tilfellet n˚ ar S = 1 Z ∞ Z g(x) dx = 0

0

1

1 g(x) + 2 g x



1 x

 dx ,

bli mye brukt fremmover. Oppgaver ˚ vise at 12. Benytt integralet fra eksempel (2.5.1) til a Z b dt π p J= = √ . ab a t (t − a)(b − t) Bestem ogs˚ a integralet Z c

hvor a, b, c, d ∈ R.

d

du p , 2 u (u − c2 )(d2 − u2 )

(2.38)

46

II

5 Diverse substitusjoner

13. En annen transformasjon er a

Z

 f

1

a2 t + 2 t



2

dt = t

a

Z

 f

1

a2 t+ t



dt , t

noen ganger kjent som Wolstenholme transformasjonen. Vis at denne stemmer 14. La f (x) være en kontinuerlig funksjon og Z



Z



f (x) dx =

f

−∞

−∞

R∞ −∞

f (x) dx eksisterer. Vis at

  1 x− dx x

Denne overgangen kalles ofte for Slobin transformasjonen. Z



15. Beregn integralet 0

dx a2 + x −

1 x

2 , hvor a ≥ 2.

16. a) Benytt transformasjonene

U (x) =

b−a a+b x x+ og V (x) = 2 2 1 − x2

˚ bevise proposisjon (2.5.1). til a b)

Bestem funksjonene v(a, b), og u(a, b) slik at Z

b

f (t) dt = a

1 2

Z



  f arctanh v(a, b, x) u(a, b, x) dx

−∞

Denne transformasjonen har en entydig invers p˚ a R − i motsetning til likning (2.35) − gitt ved Z



Z f (t) dt = Q

−∞

b

f (Q tanh x + P ) a

dx cosh2 x

Bestem konstantene Q og P slik at likheten stemmer. Her er P = P (a, b) og Q = Q(a, b) funksjoner av a og b.

II

2.5.1

5.1 Weierstrass substitusjon

WEIERSTRASS

47

SUBSTITUSJON

I denne delen skal vi se nærmere p˚ a en substitusjon kjent som weierstrass substitusjon14 . Teknikken er oppkalt etter den tyske matematikeren Karl Weierstrass ˚ sette (1815-1897) og best˚ ar i a x t 7→ tan . 2 Meningen med substiusjonen er a˚ transformere trigonometriske integral over til ˚ gjøre dette trengs uttrykk for b˚ rasjonale funksjoner. Men for a ade dx, sin x og cos x Theorem 2.5.3. (Weierstrass) Anta at a, b ∈ [−π, π] da er Z

b

Z

ψ

R(sin x, cos x) dx = a

 R

ϕ

1 − t2 2t , 2 1 + t 1 + t2



2 dt 1 + t2

 x

via substitusjonen t = tan 2 . Hvor R er en rasjonal funksjon, ϕ = tan(a/2), 2t ψ = tan(b/2) og tan(x) = 1−t 2 . Bevis. Dett beviset tar utgangpunkt i figur (2.15), hvor sidene ble valgt p˚ a en

C √ 1 + t2

t

x/2 A

B

1

Figur 2.15: Figur med t = tan(x/2). slik m˚ ate at t = tan(x/2). Dessverre f˚ ar en bare uttrykk for sin(x/2) og cos(x/2) fra figuren cos

x 2

= √

1 1 + t2

og sin

x 2

= √

t 1 + t2

(2.39)

˚ f˚ For a a de rette verdiene benyttes dobbelformelene

14 Flere

cos(A + B) = cos A cos B − sin A sin B

(2.40)

sin(A + B) = cos A sin B + sin A cos B

(2.41)

˚ gi noen refferanser til hvorfor. Tekbøker kaller denne substitusjonen for Weierstrass uten a ˚ paramterisere nikken var kjent lenge før Weierstrass var født, blant annet fra Euler og ideen om a ˚r. Se http://math.stackexchange.com/questions/ enhetssirkelen har vært kjent i flere tusen a 461527/on-the-origins-of-the-weierstrass-tangent-half-angle-substitution for flere detaljer. I de fleste verk s˚ a er teknikken kjent som weierstass-substitusjonen og derfor er det navnet som vil bli benyttet her og.

48

5 Diverse substitusjoner

II

˚ sette inn A = B = x/2 i likning (2.40) kan cos x skrives som Ved a 2  2 x x  t 1 − t2 1 − √ = cos(x) = cos2 − sin2 = √ . 2 2 2 2 1 + t2 1+t 1+t Tilsvarende s˚ a kan sin x skrives som    x x t 1 2t √ √ cos =2 , sin x = 2 sin = 2 2 2 2 1 + t2 1+t 1+t ˚ sette inn A = B = x/2 i likning (2.41). For a ˚ bestemme dt deriveres ved a t = tan(x/2) med kvotientregelen dt =

1 cos2 (x/2) + sin2 (x/2) 1 dx = (1 + t2 ) dx 2 cos2 (x/2) 2

Dette gir som ønsket at dx = 2 dt/(1+t2 ), hvor enhetsformelen og likning (2.39) ble benyttet i siste overgang. Til slutt har en sin x 2t . 1 − t2 2t tan x = = = cos x 1 + t2 1 + t2 1 − t2 og dette fullfører beviset. Eksempel 2.5.4. Vis at Z

dx = log |tan x + csc x| + C cos x

˚ f˚ ˚penbaring og deretter se at en kan gange med Standardtrikset her er a a en a sec x + tan x sec x + tan x ˚ benytte seg av substitusjonen u = sec x + tan x. Dette er dog svart magi fors˚ aa og overlates til leser. Her vises heller frem Weierstrass i sin prakt Z Z Z 2 dx 1 + t2 (1 + t) − (t − 1) = dt = dt cos x 1 − t2 1 + t2 (1 + t)(1 − t) ˚ bestemme integralet kan en n˚ For a a dele opp integralet relativt enkelt Z Z 1+t 1 1 dx + C = log 1 + tan(x/2) + C = − dt = log cos x 1+t t−1 1−t 1 − tan(x/2) ˚ ha en Men dette likner ikke helt p˚ a det som skulle vises. Heldigvis hjelper det a liten algebratrollmann i magen. Legg merke til at 1+t 1+t 1+t 2t 1 + t2 = · = + 2 1−t 1−t 1+t 1−t 1 − t2 ˚ sette inn har en alts˚ Ved a a at Z 2t dx 1 + t2 = log + 2 cos x 1−t 1 − t2

= log |tan x + sec x| + C

som var det som skulle vises. I siste overgang ble det ikke brukt mer enn theorem (2.5.3).

II

5.1 Weierstrass substitusjon

49

Eksempel 2.5.5. Som et siste eksempel skal vi se p˚ a det bestemte integralet Z 2π dx 2π = √ 2 + cos x 3 0 Merk at det ikke fungerer og bruke Weierstrass substitusjonen med en gang. Siden tan(0) = 0 og tan(2π/2) = 0. Grunnen er at for at tan(x/2) skal være unikt definert, m˚ a en begrense intervalet til x ∈ (−π, π]. Problemet unng˚ as dog ˚ dele integralet inn i perioder eller benytte korollar (2.4.1). ved a Z 2π Z π Z 2π Z π dx dx dx dx = + = 2 + cos x 2 + cos x 0 0 2 + cos x π −π 2 + cos x Her ble substitusjonen t 7→ 2π−x ble benyttet p˚ a siste integralet. Dette forandrer grensene (π, 2π) → (−π, 0). Ved n˚ a og benytte seg av Weierstrass, f˚ as  Z 2π Z Z ∞ −1 ∞ 2 dt 2 dt dx 1 − t2 = = 2+ 2 2 2 2 + cos x 1+t 1+t 0 −∞ 3 + t −∞ √ Siste integralet kan bli løst via substitusjonen t 7→ 3 u. S˚ a Z 2π Z ∞ dx 2 2π du = √ = √ 2 2 + cos x 1 + u 3 −∞ 3 0 Siste integralet er bare den deriverte av arctan u. Hittil har integralene v˚ are vært relativt hyggelige og pene, og i slike tilfeller er Weierstrass svært nyttig. Beklagigvis bryter metoden helt sammen for de aller fleste kompliserte integrander. −3 Z  Z Z Z 1 − t2 1 + 2t2 + t4 dx 2 dt 3 = = 2 dt sec x dx = cos3 x 1 + t2 1 + t2 (1 − t)3 (1 + t)3 Integralet kan videre bli løst via delbrøksoppspalting, men denne blir lang. Konklusjonen blir at Weiestrass-substitusjon blir en siste utvei for a˚ beregne trigonometriske integral. Den omskriver trigonometriske funksjoner til rasjonale, men ˚ inteuttrykkene en ender opp med kan være like vanskelig eller vanskeligere a ˚ beregne slike integral i det grere. Som vi skal se senere kan det være enklere a komplekse planet. Oppgaver 1. La oss se p˚ a to klassiske integral som kan løses rimelig enkelt via Weierstrass. Vis at Z 2π dθ 2π = √ (2.42) 2 a + b sin θ a − b2 0 r Z π/2 dθ 2 a−b = √ arctan (2.43) 2 2 a + b sin θ a+b a −b 0 hvor |a| ≥ |b| er reelle konstanter. Vis spesielt at Z 2π Z π/2 dθ dθ 6 = 2 + sin θ 2 + sin θ 0 0

50

II

5 Diverse substitusjoner

2.5.2

EULER

SUBSTITUSJON

Studiet av rasjonalle funksjoner med radikale nevnere har vært studert lenge Z A(x) p dx B(x) og har gitt oppspring til elliptiske kurver, elliptiske integraler, differensial geometri og mer. H˚ apet er at om A og B er ’pene’ nok s˚ a kan integraler skrives p˚ a formen P (x)/Q(x) hvor P og Q er polynomer. Studiet av disse integralene er ferdig i den forstand at alle slike integral kan bli løst ved hjelp av samme oppskrift. ˚ gjøre denne omformingen p˚ ˚ bruke hyperbolske eller inverse En m˚ ate a a er a ˚ benytte seg av en rekke trigonometriske substitusjoner. En annen m˚ ate er a substitusjoner kalt euler-substitusjonene. I nyere tid har disse knepene mistet mye av sin popularitet av uviss grunn. Eksempel 2.5.6. Z



p dx = log x + x2 + a , 2 x +a

a∈R

˚ arbeide seg gjennom. Dette er et standard integral, men det er likevell nyttig a Vi bruker substitusjonen p z 7→ x + x2 + a (2.44) Dermed s˚ a kan dx uttrykkes som √   x x + x2 + a z dz = 1 + √ dx = √ dx = √ dx 2 2 2 x +a x +a x +a Dette medfører at Z I=

√ dx x2 +a

=

dx √ = x2 + a

dz z

Z

˚ sette inn kan alts˚ . Med a a integralet skrives p dz = log |z| = log x + x2 + a + C z

Variabelskifte so ble brukt i dette eksempelet leder til en svært elegang løsning, men det ikke like klart hvordan substitusjonen kom frem. ?? kan skrives p˚ a formen p x2 + a 7→ z − x , og det skal vise seg at dette er nøyaktig en av euler substitusjonene. Proposisjon 2.5.3. Euler substitusjonene er brukt for a a for˚ beregne integraler p˚ men Z p R(x, ax2 + bx + c ) dx ved a ˚ fjerne kvadratroten. Anta att ax2 + bx + c ikke er ulelukkende negativt og at a 6= 0. Dersom

II

5.2 Euler substitusjon 1) a > 0 benytt variabelskiftet p √ ax2 + bx + c 7→ x a + t

51

(2.45)

2) polynomet ax2 +bx+c har to distinkte reelle røtter α, β da kan substitusjonen p ax2 + bx + c 7→ (x − α)t (2.46) benyttes. 3) c > 0 kan substitusjonen p

√ ax2 + bx + c 7→ xt + c

(2.47)

bli brukt. Eksempel 2.5.7. Z I=

dx √ (1 + x) 1 + x − x2

Siden c > 1 s˚ a kan likning (2.47) brukes p 1 − 2t 1 + x − x2 7→ tx + 1 ⇒ x = 1 + t2 Differensialene v˚ are blir dx = −2

dx t2 − t − 1 dt dt ⇒ √ = −2 2 2 2 (1 + t ) 1 + t2 1+x−x

√ 2 −t−1 Siden 1 + x − x2 = tx + 1 = − t 1+t Dette fører og til at 1 + x = 2 Setter vi alt inn f˚ as Z Z Z dx dt 1 1 + t2 −2 dt √ = −2 I= = 1 + x 1 + x − x2 t2 − 2t 1 + t2 t2 − 2t Z Z t (t − 2) − t dt (t − 2)0 +C = dt = − dt = log t(t − 2) t t−2 t−2

t2 −2t 1+t2

.

˚ substituere tilbake for x f˚ Ved a as √ Z dx 1 + x − x2 − 1 1 √ = log √ +C 1 + x − x2 − 2x − 1 1 + x 1 + x − x2 som fullfører eksempelet. Abels substitusjon ˚ kvitte seg med rottegnet i integralet er a ˚ benytte seg av Abel’s En annen m˚ ate a substitusjon. Den virker p˚ a integraler p˚ a følgende former Z Z dx dx √ √ , 2 m 2 2 m (ax + bx + c) ax + bx + c (x + p) kx2 + l ˚ la den deriverte Trikset er a av rottegnet bli satt som den nye variablen. Teknik√ √ ken baserer seg p˚ a at ( A ) = A0 / A . La alts˚ a henholdsvis p 0 p 0 t= ax2 + bx + c , t= kx2 + l

52

II

5 Diverse substitusjoner

Eksempel 2.5.8. Z I=

(x2

√ La som i teksten t = ( x2 + t )0 = begge sider med hensyn x f˚ as

dx √ + 2) x2 + 1

√ x x2 +1

√ . Slik at t x2 + 1 = x. Deriverer vi

p 0 dt p 2 x +1 +t x2 + 1 = 1 dx dt p 2 ˚ Siste er jo bare hvordan vi definerte t. S˚ a vi har x + 1 + t2 = 1. Ved a dx 2 trekke fra t p˚ a begge sider og dele kan uttrykket skrives som √

dt dx = 1 − t2 x2 + 1

Tilslutt kan x2 + 2 uttrykkes p˚ a følgende m˚ ate x2 = Innsatt kan integralet skrives som Z

1 dx √ = 2 x + 2 x2 + 1

Z

1 − t2 dt 1 = 2 2 2−t 1−t 2

Her kan en igjen benytte at arctanh x = Z

1 2

log

Z

t2 1−t2

⇒ x2 + 2 =

dt = 1 − t2 /2

1+x 1−x

2−t2 1−t2



2 t arctanh √ 2 2

. S˚ a

√ √ 1 2 +t 2x2 + 2 + x dx 1 √ = √ log √ +C = √ log √ 2 2 2 2 2 −t 2 2 (x + 2) x + 1 2x2 + 2 − x

.

II

6.1 Brøker

2.6

53

B R Ø K E R O G K V A D R AT R Ø T T E R

˚ beregne I denne delen skal vi se nærmere p˚ a noen spesielle teknikker for a brøker, kvadratrøtter, og kombinasjoner av disse. Tidligere har en sett p˚ a en rekke generelle substitusjoner, mens her blir noen substitusjoner som er spesielt effektive p˚ a kvadratrøtter og brøker nevt. √ Merk at de kvadratrøttene som stort sett nevnes i denne delen er p˚ a formen ax2 + bx + c , med rasjonelle konstanter.

2.6.1

BRØKER

˚ se litt p˚ ˚ være forsiktig. La oss begynne med a a fortegn og viktigheten av a Eksempel 2.6.1. 0

Z



I= −1/2

2x3

dx − 3x2 + 1

˚ faktorisere polynomet under rottegnet. Ser raskt at x = 1 er Vi begynner med a en løsning s˚ a 2x3 − 3x2 + 1 = (2x3 − 2x) − (x2 − 1) = 2x2 (x − 1) − (x − 1)(x − 1) = (x − 1)(2x2 − x − 1) ˚ bruke andregradsformelen, eller mer faktorisering kan polynomet skrives Ved a ˚ sette inn f˚ som 2x3 − 3x2 + 1 = (x − 1)2 (2x + 1). Ved a as Z 0 Z 0 dx dx √ p I= = 2 (x − 1) (2x + 1) −1/2 −1/2 |x − 1| 2x + 1 For −1/2 < x < 0 s˚ a har vi −3/2 a er |x − 1| √ < x − 1 < −1 og 0 < 2x + 1 < 1. S˚ ˚ sette inn f˚ negativ p˚ a intervallet, mens 2x + 1 beholder fortegnet sitt. Ved a as Z 0 1 dx √ I= 1 − x 2x +1 −1/2 √ dx Tanken er n˚ a a˚ la t 7→ 2x + 1 slik at x = (t2 −1)/2. Derivasjon gir dt = 2√2x+1 . Z I= 0

1

dt 1−

t2 −1 2

=

2 3

Z 0

1

dt 2 1 √ = √ arctanh √ 1 − (t/ 3 )2 3 3

˚ bruke definisjonen Hvor vi brukte . Vi kan skrive svaret en del penere ved a 1+x arctanh x = 12 log 1−x . Siden √ √ √ √ √ 1 + 1/ 3 ( 3 + 1)2 3 +1 3 +1 √ = log √ √ log = log = log(2 + 3 ) 2 1 − 1/ 3 3 −1 3 +1 S˚ a kan det endelige svaret skrives som Z 0 √ dx 2 1 1 √ = √ arctanh √ = √ log(2 + 3 ) 3 2 3 3 3 2x − 3x + 1 −1/2

sett inn refferanse til standard arctanh integral

54

II

6 Brøker og kvadratrøtter

˚ uttrykke integralet som en rasjonell funksjon hvor teller Ofte er det mulig a ax+b og nevner er lineære, ax+b cx+d . I disse tilfellene er følgende susbstitusjon t = cx+c ˚ elliminere rottegnet. eller t2 = ax+b cx+c nyttig for a Eksempel 2.6.2. Z

dx √ (1 − x) 1 − x2

I=

Først m˚ a vi skrive om integralet slik at vi f˚ ar frem det rasjonale uttrykket v˚ art Z Z Z dx dx dx q √ q I= = = 2 1+x 1+x (1 − x) 1 − x (1 − x) 1−x (1 − x)2 (1 − x)2 1−x ˚ la t2 7→ Ved a

1+x 1−x

s˚ a f˚ ar vi at x =

t2 −1 t2 +1

dx = Mens 1 − x = Z I=

2 1+t2

. Derivasjon gir oss at

4t dt (1 + t2 )2

. Dette er alt vi trenger. Innsetning gir n˚ a

dx q = 1+x (1 − x)2 1−x

Z 

1 + t2 2

2

4t dt = 2 2 (1 + t ) t

Z dt = t + C

Alts˚ a er Z

dx √ = (1 − x) 1 − x2

r

1+x +C 1−x

som var det som skulle vises. √ ˚ finne igjen Uttrykk p˚ a formen lineær · kvadratisk kan forenkles ved a den deriverte. Eksempel 2.6.3. Z I=

p (x + 1) 2x2 + 3x + 1

˚ finne koeffisienter α og β slik at Tanken er n˚ a at vi ønsker a x+1=a

d (2x2 + 3x + 1) + β = a(4x + 3) + β dx

Dette gir α = 1/4 og β = 1/4 via innsetning eller falkeblikk. Integralet kan alts˚ a skrives som  Z  1 1 p 2 I= (4x + 3) + 2x + 3x + 1 dx 4 4 Z 1 2 1 p 2 2 2/3 = · (2x + 3x + 1) + 2x + 3x + 1 dx 4 3 4

II

6.1 Brøker

55

p R R√ Siden f 0 (x) f (x) dx = v dv = 23 f (x)3/2 + C. Dermed blir uttrykket noe enklere og integrere og vi kan fokusere p˚ a siste delen. Fokuset i denne delen er p˚ a selve omskrivningen og ikke løsningen av det siste itnegralet. I korte trekk brukes z 7→ 4x + 3 slik at √ Z q Z p Z √ p 2 2 2 2 J= 2x + 3x + 1 dx = z − 1 dx = cosh2 y − 1 sinh y dy 16 16 Hvor vi benyttet oss av substitusjonen z √ 7→ cosh y i siste overgang. Siden p cosh2 x − sinh2 x = 1 s˚ a er cosh2 x − 1 = sin2 x . Alts˚ a f˚ as Z Z 1 1 cosh(2y) − 1 dy = sinh y cosh y − y J = sinh2 y = 2 2 2 1 Svaret blir alts˚ a 12 sinh(arccosh arccosh  y)y−  y. Vi har videre at sinh(arccosh y) = p p 2 y 2 − 1 og arccosh y = log y + y 2 − 1 . Dette gir

Z p



2x2

√ p 2 2 2 + 3x + 1 dx = (4x + 3) (4x + 3) − 1 − arccosh(4x + 3) 32 32 √ p 1 2 = (4x + 3) 2x2 + 3x + 1 − arccosh(4x + 3) 8 32

Det hele og fulle svaret blir alts˚ a Z p I = (x + 1) 2x2 + 3x + 1 √ p 1 1 2 2 3/2 2 = (2x + 3x + 1) + (4x + 3) 2x + 3x + 1 − arccosh(4x + 3) 6 32 128 En kan ogs˚ a bruke metoden med uttrykk p˚ a formen

√ lineær kvadratisk

.

Eksempel 2.6.4. Z I=



x+3 dx + 4x − 3

4x2

Igjen s˚ a leter vi etter a˚ skrive om teller, ved a˚ finne den deriverte av 4x2 + 4x − 3. Legg merke til at 4x2 + 4x − 3 = (2x + 1)2 − 4. S˚ a x+3=α

 d  (2x + 1)2 − 4 + β = 4α(2x + 1) + β dx

Dette gir α = 1/8 og β = 5/2. Alts˚ a kan vi skrive 1

5 2

Z Z 1 (4x2 + 4x − 3)0 dx 5 (2x + 1)0 dx √ + 8 4 (2x + 1)2 − 4 4x2 + 4x − 3 4x2 + 4x − 3 Z Z 1 du 5 dy √ + p = 4 4 2 u y2 − 4 R √ √ Det første integralet er et standard integral dx/2 x = x + C og det siste integralet kan bli løst ved en valgfri euler-substitusjon. Z

8 √

(8x + 4) +

dx =

56

6 Brøker og kvadratrøtter

2.6.2

II

K V A D R AT R Ø T T E R

Vi har sett mange eksempler p˚ a integraler med kvadratrøtter tidligere. Her kan en bruke euler-substitusjoner, hyperbolske eller trigonometriske substitusjo˚ forenkle ner ogs˚ a videre. Her ser vi p˚ a noen f˚ a mindre kjente teknikker for a integraler med kvadratrøtter i seg. Eksempel 2.6.5. Z I=



x2

dx √ + 1 − x2 − 1

˚ bruke v˚ Her kan vi begynne a are magiske trigononometriske identiteter. Men ˚ gange med den konjugerte det er langt enklere a √ √ √ √ 1 a+ b a+ b 1 √ √ √ = = √ √ √ a−b a− b a− b a+ b Hvor tredje kvadratsetning ble brukt, (a − b)(a + b) = a2 − b2 . Bruker vi det samme p˚ a integrlet v˚ art f˚ ar vi √ Z Z √ 2 dx x + 1 − x2 − 1 √ √ dx = (x2 + 1) − (x2 − 1) x2 + 1 − x2 − 1 Z p p 1 = x2 + 1 − x2 − 1 dx 2 Hvor begge integralene kan bli løst via delvis integrasjon eller en luddig substitusjon. Forsec:II-Delvis-integrasjon det første integralet kan substitusjonen x 7→ tan x benyttes d˚ a f˚ ar en Z p Z p Z dt J= x2 + 1 dx = (tan t)2 + 1 = (sec t)3 dt (cos t)2 ˚ bruog dette integralet vil bli studert nærmere senere15 , fremgangsm˚ aten blir a ke delvis integrasjon samt integralet av sec(x) som har blitt studert tidligere, eksempel (2.5.4). Alternativt gir delvis integrasjon Z Z p p x2 2 2 √ x + 1 dx = x · x + 1 − dx J= x2 + 1 R 2 R√ R Herfra legger vi merke til at x√x+1−1 dx = x2 + 1 dx − √x21+1 dx. Hvor 2 +1 det første integralet kjenner vi igjen som J. Alts˚ a er   Z p Z p 1 2 2 √ J= x + 1 dx = x · x + 1 − J − dx x2 + 1 Z p dx √ 2J = x x2 + 1 + x2 + 1 1 p 1 J = x x2 + 1 + arcsinh x + C 2 2 Hvor det siste integralet har R √blitt behandlet i seksjon 4 om hyperbolske integraler og funksjoner. Integralet x2 − 1 dx kan løses tilsvarende og overlates til leser. 15 Se

eksempel (2.8.1) fra avsnitt (2.8) om trigonometriske integraler.

II

6.2 Kvadratrøtter

57

Eksempel 2.6.6. Andre integraler kan skrives om før de integreres Z √ 2 x +2 I= dx x2 + 1 ˚ derasjonalisere uttrykket slik at vi kan skrive integranden som Her velger vi a √ √ x2 + 2 x2 + 2 x2 + 1 + 1 1 1 √ √ √ = = √ + 2 2 2 2 2 2 x +1 x +2 (x + 1) 2 + x 1+x (x + 1) 2 + x2 Dermed kan integralet skrives som Z √ 2 Z p x +2 dx 2 + 2 + √ + dx = log x x 2 2 x +1 (x + 1) 2 + x2 √ 2 hvor vi benyttet oss av eksempel (2.5.6) og euler-substitusjonen z 7→ √ x+ x + a . Merk at integralet kunne blitt løst like elegant ved bruk av x 7→ 2 sinh t. Det siste integralet samt generaliseringer overlates til leser. En nyttig egenskap n˚ ar det kommer til brøker er følgende Lemma 2.6.1. Z Z ∞ Z ∞ n−2 1 ∞ 1 + xn−2 dx x dx = = dx n n 2 0 1+x 1+x 1 + xn 0 0

(2.48)

Gitt at n er et reelt tall større enn 1. ˚penbart vann. Ved a ˚ benytte seg av Bevis. For n = 2 holder proposisjonen a substitusjonen 1/x s˚ a har en Z 0



dx =− 1 + xn

Z

0



1 xn dx = 1 + x−n xn x2

Z



0

xn−2 dx 1 + xn

(2.49)

Disse integralene m˚ a da være like store. Addisjon gir da Z



0

dx + 1 + xn

Z 0



xn−2 dx = 1 + xn

Z



0

1 + xn−2 dx 1 + xn

˚ dele p˚ a a 2 og bruke likning (2.49) fullfører beviset. Proposisjon 2.6.1. Gitt at n er positiv og a, b ∈ R, da er Z 0



a + bxn−2 a+b dx = 1 + xn 2

Z



0

1 + xn−2 dx 1 + xn

(2.50)

Dette er en generalisering av lemma (2.6.1), og følger nesten direkte fra ˚ se p˚ lemmaet. I stedet for a a beviset studeres heller integralene i likning (2.48) ˚ kunne veksle mellom disse ulike formene er nyttig og senere skal vi for n = 4. A R ˚ regne med16 . se at integraler p˚ a formen dx/(1 + xn ) er noe enklere a 16 Dette

vil komme godt frem i delen om kompleks integrasjon

58

II

6 Brøker og kvadratrøtter

˚ bestemme integralene Eksempel 2.6.7. Vi ønsker a ∞

Z

Z

dx , 1 + x4

0



0

x2 dx 1 + x4

Z



og 0

1 + x2 dx 1 + x4

merk først at fra lemma (2.6.1) s˚ a er de to første like og vi har sammenhengen ∞

Z

dx = 1 + x4

I= 0

Z 0



x2 dx 1 = 4 1+x 2

Z



1 + x2 dx 1 + x4

0

˚ notere seg er at ved a ˚ dele p˚ en annen nyttig ting a a siste integralet p˚ a x2 s˚ a er 1 I= 2



Z

1 + 1/x2 1 dx = x2 + 1/x2 2

0



Z 0

1 + 1/x2 dx (x − x−1 )2 + 2

˚ bruke substitusjonen u 7→ x − x−1 s˚ Ved a a er du = (1 + 1/x2 ) dx og grensene blir (−∞, ∞). Dette er ofte en nyttig omskrivning. I=



Z

1 2

du 1 = √ u2 + 2 2 2

−∞

Z



−∞

π dy = √ y2 + 1 2 2



hvor substitusjonen u 7→ 2 y ble brukt. Oppsumert har en alts˚ a at Z



0

Z



0

π 1 + x2 dx = √ 1 + x4 2 Z ∞ 2 x dx π dx = = √ 4 1 + x4 1 + x 2 2 0

(2.51) (2.52)

˚ bruke samme fremgangsm˚ Ved a ate kan en vise at følgende korollar Korollar 2.6.1. Z



xn−1 + x3n−1 1 π √ dx = 1 + x4n n 2 2

0

for alle n ∈ R+ . Oppgaver Z



1. Bestem integralene 0

dx og 1 + x3

Z



0

x dx . 1 + x3

Z

dx √ + 1) x2 + 2 b) Integralet ovenfor kan generaliseres til

2. a) Bestem integralet

Z Ia =

(x2

dx 1 √ = √ arctan 2 2 a−1 (x + 1) x + a

Bevis generaliseringen ovenfor. Hva blir I1 ?

√  a − 1x √ +C. x2 + a

II

6.2 Kvadratrøtter

59



2 + x2 uten bruk av (2.50). 1 + x4 0 b) Bevis at likning (2.48) følger fra Z ∞ Z a + bxn−2 a + b ∞ 1 + xn−2 dx = dx , 1 + xn 2 1 + xn 0 0 Z

3. a) Bestem integralet

Bevis s˚ a det motsatte alts˚ a at likningen over (2.6.1) følger fra likning (2.48). ?? 15

60

7 Delvis integrasjon

2.7

II

D E LV I S I N T E G R A S J O N

I denne delen skal vi igjen besøke delvis integrasjon Z Z uv 0 = uv − u0 v , med tilhørende smarte knep og metoder. Som oppfriskning tas et lite eksempel som vil komme til nytte senere Proposisjon 2.7.1. Z eαx eαx sin βx dx = 2 α + β2 Z eαx eαx cos βx dx = 2 α + β2

  α sin βx − β cos βx + C   β sin βx + α cos βx + C

Bevis. Resultatet ovenfor kan vises noe enklere ved hjelp av kompleks analyse. Men dette blir først sett p˚ a litt senere. Tar først for oss første integral, og kaller det for I. En delvis integrasjon gir da Z αx 1 e β cos βx dx I = eαx sin βx = eαx sin βx − α α ˚ bruke delvis integrasjon atter en gang blir Der u = sin βx og v = eαx /α. Ved a n˚ a integralet   Z β eαx β 1 cos βx + eαx sin βx dx I = eαx sin βx − α α α α Legg merke til at en ender opp med det originalet integralet I p˚ a høyre side av ˚ skrive ut og forenkle likningen. Ved a I=

1 αx β β2 e sin βx − 2 eαx cos βx − 2 I α α α

˚ gange begge sider med α2 og legge til β 2 I p˚ Ved a a begge sider f˚ as (α2 + β 2 )I = eαx α sin βx − eαx β cos βx Deles begge sider p˚ a α2 + β 2 f˚ ar en som ønsket at Z  eαx  α sin βx − β cos βx + C eαx sin βx dx = 2 α + β2 ˚ gjenns˚ vise det andre integralet. A helt tilsvarende regning kan benyttes for a kjenne integralet p˚ a høyre side er et velkjent indianertriks, og kommer ofte til nytte.

II

2.7.1

7.1 Delvis kanselering

D E LV I S

61

KANSELERING

Anta at en skal integrere en funksjon som p˚ a kan deles opp til f = g + h. Tanken er n˚ a a˚ bruke delvis integrasjon p˚ a h slik at integralet av g blir kanselert.  Z Z Z  g+h= g+ r− g =r Det spiller selvsagt ingen rolle om det er g eller h som blir utsatt for den delvise integrasjonen. Eneste forutsetningen at h kan skrives som et produkt av to funksjoner u og v 0 , p˚ a en slik m˚ ate at g = u0 v. Dette blir forh˚ apentligvis klarere i det neste eksempelet Eksempel 2.7.1. La oss se p˚ a det ubestemte integralet av funksjonen f (x) = (1 + 2x2 )ex

2

Integralet best˚ ar av summen av to funksjoner hvor det ikke er mulig a˚ integrere ˚ prøve delvis integrasjon, ulike noen av delene alene. Leser oppfordres til a substitusjoner eller andre metoder. Resultatet vil dessverre være nedsl˚ aende, ingen av metodene biter p˚ a integralet. Løsningen blir heldigvis a˚ dele integralet Z Z 2 2 2 x2 J = (1 + 2x )e dx = 2x2 ex + ex dx , og benytte delvis integrasjon p˚ a siste leddet med u = exp(x2 ) og v 0 = 1. Da er 0 2 u = 2x exp(x ) og v = x. Ikke minst s˚ a er u0 v = 2x2 exp(x2 ).   Z Z 2 2 2 2 J = 2x2 ex dx + xex − x · 2xex dx = xex + C Hvilken delvis integrasjon som fungerer er noe som kommer med erfaring. La oss se p˚ a et noe vanskeligere eksempel Lemma 2.7.1. Z

y x2 − y 2 dy = 2 2 2 2 (x + y ) x + y2

(2.53)

˚ dele integralet opp i to deler Bevis. La oss begynne a Z 2 Z Z x + y 2 − 2y 2 1 −2y 2 I= dy = dy + dy (x2 + y 2 )2 x2 + y 2 (x2 + y 2 )2   Z Z 1 d 1 = dy + y · dy 2 2 2 x +y dy x + y 2 En kan n˚ a rimelig enkelt benytte seg av delvis integrasjon p˚ a siste leddet. Her er u = y og v = 1/(x2 + y 2 ).   Z Z 1 y 1 y I= dy + − dy = 2 + C. x2 + y 2 x2 + y 2 x2 + y 2 x + y2 ˚ se omskrivningen av integralet kan en se at For a Z 1 2x dx = − 2 +C (x2 + a2 )2 x + a2

62

II

7 Delvis integrasjon

Via eksempelvis u = x2 + a2 , og at −1/u2 = (1/u)0 . Slik at Z Z 1 −2y d 1 dy = dy = 2 2 2 2 2 2 (x + y ) dy x + y x + y2 og dette fullfører beviset. Kanseleringen var ikke strengt talt nødvendig, siden 1/(x2 + y 2 ) ikke er spesielt vanskelig a˚ integrere, men poenget er at den delvise kanseleringen gjorde bereg˚ bruke denne teknikken er aldri eneste utvei, teknikken ningene noe enklere. A ˚ bruke produktregelen og kjerneregelen baklengs. Som er helt tilsvarende som a eksempel kunne en løst oppgavene ovenfor p˚ a følgende m˚ ate 2

2

2

2

f (x) = (1 + 2x2 )ex = x0 ex + x(ex )0 = (xex )0 Der produktregelen ble benyttet i siste overgang. Tilsvarende 2

2

x −y = (x2 + y 2 )2

(x2 + y 2 )

d d   y−y (x2 + y 2 ) d y dy dy = . (x2 + y 2 )2 dy x2 + y 2

˚ legge Har man ikke langt mosegrodd skjegg kan disse tingene være vanskelig a merke til. Det er derfor delvis kanselering i mange tilfeller er vell s˚ a nyttig. Oppgaver Z 1. I = ex sin x + ex cos x dx √ 2. Gitt at17 π =

Z



2

e−x dx beregn følgende integral18

−∞

Z



2

x2 e−x dx .

−∞

3. Bestem konstanten c og k slik at Z √ √ I = (x + c) ex x dx = kx ex x + C .

Z 4. I =

xex dx (1 + x)2

Z 5. I =

17 Integralet

log(log x) +

2 1 − dx log x (log x)2

betegnes ofte som det gaussiske integralet og vil bli vist p˚ a ulike m˚ ater i Del III. 2 ˚ betegnes ofte som forventingsverdien til den gaussiske funksjonen e−x og et hint for a løse det er selvsagt delvis integrasjon. Men nok fotnoter n˚ a.

18 Integralet

II

7.2 Eksponentialfunksjonen

2.7.2

63

EKSPONENTIALFUNKSJONEN

Anta at du er blitt tatt til fange av en ond verdensleder og f˚ ar ikke dra hjem før du har bestemt følgende integral Z (x4 − 6x2 + 8x − 3)ex dx (2.54) Er du noenlunde stø p˚ a de grunnleggende integrasjonsteknikkene begynner du vel allerede n˚ a og bite negler og skjelve. For den normale fremgangsm˚ aten er delvis integrasjon, hvor i hver delvise integrasjon derives polynomdelen slik at den tilslutt forenkles til en konstant. Metoden fungerer, men kan og føre til svært mange delvise integrasjoner og er svært langtrekkelig. I oppgaven ovenfor ˚ beregne integralet. Etter trengs det eksempelvis 4 delvise integrasjoner for a omstendig og monoton regning, kan svaret skrives som Z (x4 − 6x2 + 8x − 3)e−x dx = (x − 1)4 ex + C Legg merke til at polynomet p˚ a venstre og høyre side er av lik grad. Dette gjør ˚ gjøre følgende antakelse det rimelig a Proposisjon 2.7.2. Integralet av p(x)ex kan alltid skrives p˚ a formen Z p(x)ex dx = q(x)ex ,

(2.55)

der p og q har samme grad. arlig polynom. Bevis. La oss starte med funksjonen f (x) = q(x)ex , hvor p er et vilk˚ Derivasjon gir oss da at   f 0 (x) = q 0 (x)ex + q(x)ex = q(x) + q 0 (x) ex = p(x)ex Der p(x) = q(x) + q 0 (x). Legg merke til at p og q begge er polynomer av samme ˚ integrere p(x) har en n˚ grad. Ved a a at Z Z p(x)ex dx = f 0 (x) dx = q(x)ex + C Hvor p og q selvsagt har samme grad. La oss benytte oss av proposisjonen p˚ a et litt enklere eksempel Eksempel 2.7.2. Z I=

(x2 − 1)ex dx

Polynomet av grad 2, og det er derfor m˚ a ogs˚ a svaret inneholde et polynom av grad to. Vi gjør derfor følgende ansatz Z (x2 − 1)ex dx = (ax2 + bx + c)ex + C

64

7 Delvis integrasjon

II

˚ derivere begge sider av likningen f˚ Ved a as   (x2 − 1)ex = (ax2 + bx + c) + (ax2 + bx + c)0 ex  = ax2 + [b + 2a]x + [b + c] ex Ved n˚ a og anta at høyre og venstre side er like m˚ a alle koeffisientene være like. Dette gir følgende likninger 1=a 0 = b + 2a −1 = b + c Første likning gir at a = 1. Neste likning gir at b = −2a = −2, og siste likning gir at c = −b − 1 = 1. Dermed s˚ a er Z (x2 − 1)ex dx = (x2 − 2x + 1)ex = (x − 1)2 ex + C ˚ løse n-delvise integrasjoner med a ˚ løse n + 1 likningsett. En bytter alts˚ a ut a ˚ løse disse p˚ Siden det finnes mange svært raske og nøyaktige m˚ ater a a er det ofte a˚ foretrekke. Ved hjelp av gauss-elliminiasjon kan likninsettet ovenfor løses som følger       1 0 0 1 1 0 0 1 1 0 0 1 2 1 0 0 ⇒ 0 1 0 −2 ⇒ 0 1 0 −2 0 1 1 −1 0 1 1 −1 0 0 1 1 hvor en kan lese mer om dette i enhver lineær algebrabok. Oppgaver Z 1. ex (x4 − 6x2 + 8x − 3) dx Z 2.

eax+b (x2 − 1) dx

3. La oss se p˚ a en svak generalisering av oppgaven. Ved a˚ sette inn generelle uttrykk for polynomene kan likning (2.55) skrives som ! Z n n X X x k x k e ak x dx = e ck x +C, k=0

k=0

hvor ak er kjente konstanter og ak 6= 0 for alle k og tilsvarende for ck . ˚ uttrykke koeffisientene cn ved hjelp av an . Vis at ved a ˚ M˚ alet blir n˚ aa følge samme fremgangsm˚ ate som i eksempel (2.7.2) ender en opp med n X k=0

ak xk = cn xn +

n−1 X k=0

ck−1 + kck .

II

7.3 Tabell og reduksjonsformler

65

Dette er et likningsett som kan løses, vis at koeffisientene cn kan defineres rekursivt som cn−k−1 = an−k − (n − k)cn−k

k = 0, 1, · · · , n − 1 .

Hva er initialbetingelsen? Test ut iterasjonen p˚ a likning (2.54). Dersom ex ax+b i stedet hadde vært p˚ a formen e er det mulig a˚ n˚ a definere cn rekursivt, hva blir i s˚ afall rekursjonen? Z ∞ 4. e−x xn dx , n∈N 0

2.7.3

TA B E L L

OG REDUKSJONSFORMLER

˚ slippe a ˚ gjennomføre delvis integraI forrige del ble det sett p˚ a ulike m˚ ater a sjon p˚ a. Eksempelvis takket være at ex men hva med integraler p˚ a formen Z π (x3 − 1) cos x dx (2.56) 0

Her kan en ikke lengre anta hva svaret skal bli da cos x og sin x deriverte har en periode p˚ a fire og ikke en. Det en st˚ ar igjen med er alts˚ a vanlig delvis integrasjon. Metoden vi skal bruke her ble visstnok først tatt i bruk p˚ a 1960 tallet [? ] og er ikke spesielt kjent. Noe av grunnen nok at metoden ikke revolusjonerer delvis integrasjon; men heller automatiserer arbeidet. La oss integrere funksjonen f g, og lar f (k) betegne den k’te deriverte og g(k) ˚ bruke delvis integrasjon f˚ betegne den k’te integrerte. Ved a ar en da Z Z f g = f g1 − f 1 g1 Z = f g1 − f 1 g2 + f 1 g2 = ··· = f g1 − f 1 g2 + f 2 g3 − · · · + (−1)k+1 f k gk+1 + · · · Dersom det eksister en n slik at f n = 0 eller gn = 0 stopper prossesen etter n-delvise integrasjoner. Vi illustrerer dette med et eksempel. Eksempel 2.7.3. Vi tar utgangspunkt i likning (2.56) og ønsker først a˚ beregne det ubestemte integralet Z (x3 − 1) cos x dx vi velger n˚ a u = x3 , og v = cos x. Grunnen er enkel, etter fire derivasjoner blir u null, mens v er periodisk og blir aldri null. De deriverte og integrerte er gitt i tabell (2.3) Ved a˚ gange sammen uk med vk+1 – som vist med pilene i tabellen – kan integralet skrives som Z (x3 − 1) cos x dx = (x3 − 1) sin x − 3x2 (− cos x) + 6x(− sin x) − 6 cos x = (x3 − 1) sin x + 3x2 cos x − 6x sin x − 6 cos x som ønsket.

66

II

7 Delvis integrasjon Tabell 2.3: Viser de deriverte til u = x3 − 1, og integrerte til v = cos x. u

v

x3 − 1 3x2 6x 6 0

cos x sin x − cos x − sin x cos x

& & & &

Dersom vi videre trenger a˚ bruke delvis tre eller flere ganger kommer vi alltid ˚ skrive opp de deriverte som i tabellen ovenfor. Denne teknikken kan og til a ˚ utlede taylorrekka til ulike funksjoner. Under viser vi dette for ex . brukes til a Eksempel 2.7.4. Vi studererer følgende funksjon Z x I= et dt 0 x

P˚ a den ene siden vet vi at integralet er likt I = [et ]0 = ex − 1, siden et per ˚ definisjon er sin egen derivert. Men vi kan og bruke delvis integrasjon for a bestemme uttrykket la v = 1 og u = et , da f˚ ar vi tabell (2.4) Via sammenlikning Tabell 2.4: Viser de deriverte til u = x3 − 1, og integrerte til v = cos x. u

v t

e et et et et ˚ bruke at og a

& & & &

1 t t2 /2 t3 /6 t4 /24

Rx

tn dx = xn /(n + 1), s˚ a har vi Z x I= et dt x 0 1 3 t 1 4 t 1 2 t x t t e + ··· e − 1 = te − t e + t e − 2 6 24   0 1 1 1 = ex x − x2 + x3 − x4 + · · · 2 6 24 0

Deler vi begge sider p˚ a ex , eller ganger med e−x f˚ ar vi 1 2 x + 2 1 = 1 − x + x2 − 2

1 − e−x = e−x

x−

1 3 x − 6 1 3 x + 6

1 4 x + ··· 24 1 4 x + ··· 24

˚ sette inn u = −x, f˚ som gir oss rekkeutviklingen for e−x , ved a ar en tilsvarende utviklingen for ex .

II

7.3 Tabell og reduksjonsformler

67

For flere kildehenvisninger og eksempler anbefales [? ] p˚ a det varmeste. La ˚ oss ta en titt p˚ a en siste klasse av integraler, hvor tabell-integrasjon viser seg a være nyttig. Eksempel 2.7.5. La oss betrakte følgende integral Z x3 dx K= (1 + x)5 ˚ bruke substitusjonenen u 7→ x + 1, slik at Den normale fremgangsm˚ aten er a Z Z Z 3 x3 (u − 1)3 u − 3u2 + 3u − 1 K= dx = du = du 5 5 (1 + x) u u5 −1 + 4u − 6u2 + 4u3 1 (2x + 1)(2x2 + 2x + 1) = =− 4 u 4 (x + 1)4 Hvor igjen integrasjonskonstanten ble droppet av plassmangel. Bruker vi heller tabell-integrasjon kan K skrives p˚ a en mer givende form (suggestive form). Ved Tabell 2.5: Integrasjon av f (x) = x3 /(1 + x)5 u

v

x3 3x2 6x 6 0

& & & &

− 14 1 12 1 − 24 1 − 24

(1 + x)−5 (1 + x)−4 (1 + x)−3 (1 + x)−2 (1 + x)−1

˚ bruke verdiene fra tabell (2.5) kan K og skrives som a 1 x3 3 x2 6 x 6 1 − − − 4 (1 + x)4 12 (1 + x)3 24 (1 + x)2 24 1 + x "    2  3 # 1 1 x x x =− 1+ + + 4 1+x 1+x 1+x 1+x

K=−

Dersom en har ett godt falkeblikk19 kan en se at " # 1+3 Z 1 x x3 dx = −1 . (1 + x)5 1+3 1+x

(2.57)

(2.59)

˚ følge tankerekken ovenfor kan vi generalisere til følgende eksempel Ved a 19 Legg

1+

merke til at



x 1+x



Pn

+

k=0



=

x 1+x

xn+1 − 1 x−1

2

+



. Dette medfører at vi kan skrive uttrykket i parentesen som

x 1+x

3

=

3  k X x

1+x

 = (1 + x) 1 −



x 1+x

3+1 

k=0

(2.58) ˚ sette inn likning (2.58) inn i (2.57) f˚ Ved a ar vi som ønsket likning (2.59).

68

7 Delvis integrasjon

II

Eksempel 2.7.6. Z I=

xn dx (1 + x)n+2

Vi vil beregne integralet ved akkuratt samme fremmgangsm˚ ate som før. Merk at ˚ beregne dette og liknende integral. senere vil vi se p˚ a andre m˚ ater a 1 n n(n − 1 xn xn−1 xn−2 − − n + 1 (1 + x)n+1 n(n + 1) (1 + x)n (n − 1)n(n + 1) (1 + x)n−1 n−3 n(n − 1)(n − 2) x − − ··· (n − 2)(n − 1)n(n + 1) (1 + x)n−2

I =−

Legg merke til at vi bare har et endelig antall ledd (n + 1, faktisk) siden tilslutt s˚ a vil xn deriveres til null. Integralet kan dermed skrives som "  1  2  n # 1 1 x x x I=− 1+ + + ··· + (n + 1) (1 + x) 1+x 1+x 1+x Som faktisk ble ganske pent. Siden utttrykket inneholder en geometrisk rekke, kan vi forenkle summen til " # n+1 Z xn 1 x I= dx = −1 (1 + x)n+2 n+1 1+x som er ganske stilig.

II

8 Trigonometrske funksjoner

2.8

69

TRIGONOMETRSKE FUNKSJONER

I denne delen vil det bli sett p˚ a en rekke spenstige trigonometriske integral, men først tas noen generelle r˚ ad20 . Anta at integralet kan skrives som Z R(sin x, cos x) dx hvor R er en rasjonell funksjon. La oss ta en titt p˚ a noen spesialtilfeller 1) Gitt at R(sin x, cos x) = R(cos x) sin x dx, da foretrekkes substitusjonen u 7→ cos x. 2) Tilsvarende om R(sin x, cos x) = R(sin x) cos x dx da foretrekkes substitusjonen u 7→ sin x. 3) Om R = R(tan x), anbefales tan x → y. 4) Dersom R = R(sin2 x, cos2 x) s˚ a kan igjen substitusjonen tan x → y benyttes, da er   Z Z 1 t2 dt R(cos2 x, sin2 x) dx = R , 1 + t2 1 + t2 1 + t2 Fungerer intet av det ovenfor, m˚ a en nok krype til korset og benytte seg av den universelle Weierstrass substitusjonen fra avsnitt (2.5.1). La oss se p˚ a unntaket som bekrefter regelen ˚ beregne følgende integral Eksempel 2.8.1. En har tidligere prøvd a Z dx cos3 x ved a˚ bruke weiestrass, dette leder dog til en meget urovekkende delbrøksoppspalting. Et alternativ er a˚ benytte delvis integrasjon med u = sec x ⇒ du = sec x tan x dx, og v 0 = sec2 x dx ⇒ v = tan x. Z Z sec2 x dx = sec x tan x − sec x tan2 dx Z Z = sec x tan x + sec x dx − sec3 x dx Hvor det i siste overgang ble brukt at tan2 x = sec2 x−1. Ved a˚ legge til integralet av sec3 x p˚ a begge sider s˚ a er Z Z 3 2 sec x dx = sec x tan x + sec x dx Det siste integralet har blitt beregnet i eksempel (2.5.4) og dermed s˚ a er Z 1 1 sec3 dx = sec x tan x + log |tan x + csc x| + C 2 2 som var det som skulle vises. 20 Deler

av denne introduksjonen er basert IntegrationOfRationalFunctionOfSineAndCosine

p˚ a

http://planetmath.org/

70

II

8 Trigonometrske funksjoner

Dette var langt enklere enn a˚ integrere den rasjonale funksjonen som oppst˚ ar ˚ bruke weiestrass-substisjonen. La oss se p˚ ved a a to integraler til som kan løses ved ulike metoder Herfra tas et par nyttige egenskaper til trigonometriske funksjoner Proposisjon 2.8.1. Z π Z π π xR(sin x, cos2 x) dx = R(sin x, cos2 x) dx , 2 0 0 hvor R er en rasjonell funksjon. Et elementært bevis er vist i oppgave (5), og det anbefales p˚ a det sterkeste a˚ ˚ vise denne identiteten. Under gis det kort og søtt bevis som ikke er like prøve a intuitivt ˚ vise identiteten for Bevis. Det første vi kan legge merke til er at det holder a f (sin x) siden R(sin x, cos2 x) = R(sin x, 1 − sin2 x) = f (sin x) La oss s˚ a definere følgende funksjon  π f (sin x) . g(x) := x − 2 En kan n˚ a enkelt se at g(π − x) = −g(x), siden R π sin(π − x) = sin x. Funksjonen ˚ putte inn g er alts˚ a symmetrisk omkring π og vi har 0 g(x) dx = 021 . Ved a definisjonen av g f˚ as Z π π x− f (sin x) dx = 0 dx , 2 0 Rπ Beviset fullføres ved a˚ dele opp integralet og legge til π2 0 f (sin x) dx p˚ a begge sider. Et liknende integral er Proposisjon 2.8.2. La p og q være to relle tall, da er Z



Z f (p cos x + q sin x) dx =

Z f (α cos x) dx = 2

0

hvor α =



0

p

π

f (α cos x) dx 0

p2 + q 2 .

Bevis. Venstre siden kan skrives som p p cos x + q sin x = p2 + q 2

p

q

!

p cos x + p sin x = cos(x − a) p2 + q 2 p2 + q 2 Rπ 21 Dette følger direkte fra korollar (2.3.2) med a = π/2 eller ved a ˚ la I = g(x) dx. Via sub0 Rπ R0 stitusjonen x 7→ π − x s˚ a er I = g(x) dx = − −g(u) du = −I . Dette medfører at 0 π 2I = I + (−I) = 0 s˚ a I = 0. Alt dette følger fra at g(π − x) = −g(x).

II

8 Trigonometrske funksjoner

71

Hvor sumpformelen cos(a − b) = cos a cos b + sin a sin b ble benyttet med a = arccos(p/ p2 − q 2 ) = arccos p/α. Integralet kan n˚ a skrives om som følger Z 2π−a Z 2π  f (α cos x) dx f α cos(x − a) dx = −a π

0

Z

Z

0

Z

2π−a

g(x) dx

g(x) dx +

g(x) dx +

=

−a

0

π

hvor forkortelsen g(x) = f (α cos x) ble innført. De to siste integralene kan skrives Z 2π Z 2π−a Z 2π f (α cos x) dx + f (α cos x) dx = f (α cos x) dx π

2π−a

π

Hvor en enten bruker at cos x har en periode p˚ a 2π, eller subsitusjonen x = u−2π p˚ a første integralet. Dette har en og sett før, for eksempel i likning (2.21). Ved a˚ bruke dette har en n˚ a at Z 2π Z π Z 2π  f α cos(x − a) dx = f (α cos x) dx + f (α cos x) dx 0

π

0 2π

Z

Z f (α cos x) dx = 2

= 0

π

f (α cos x) dx 0

p ˚ spare plass. hvor igjen forkortelsen α = p2 + q 2 ble brukt for a ˚ se den siste overgangen p˚ Det er flere m˚ ate a a, den har for eksempel blitt studert i detalj i oppgave (8). Dette fullfører beviset. N˚ a tas en artig trigonometrisk identitet som vil bli mye brukt fremmover. Lemma 2.8.1.  arctan x + arctan

1 x

 =

π 2

˚ vise at f (x) = π/2 er a ˚ se at den deriverte er null. Bevis. En m˚ ate a   d 1 1 1 1 1 f (x) = + · − = − . dx 1 + x2 1 + (1/x)2 x2 1 + x2 1 + x2 Dermed s˚ a m˚ a f være lik en konstant s˚ a lenge x 6= 0. Dette kan en se for ˚ integrere f 0 (x) = 0. For a ˚ bestemme denne konstanten s˚ eksempel ved a a er f (1) = 2 arctan(1). Siden tan(π/4) = 1 s˚ a er arctan 1 = π/4 da arctan y er inversen av tan y. Dermed s˚ a er f (x) = f (1) = π/2, siden f (x) er konstant. ˚ betrakte følgende figur Alternativt kan og identiteten og vises ved a

C α x γ A

1

som er et søtt lite bevis.

B

tan γ = x/1 ⇒ γ = arctan x 1 tan α = 1/x ⇒ α = arctan x π α+γ+ =π 2 π α+γ = 2 1 π arctan x + arctan = x 2

72

Korollar 2.8.1. La a, b ∈ R da er   a+b     arctan   1 − ab  a+b  arctan a + arctan b =  arctan +π   1 − ab   π/2

sett inn refferanse i fotnote.

II

8 Trigonometrske funksjoner

hvis

ab < 1

hvis

ab > 1

ellers

a kort inkluderes det her. Beviset har og vært gitt p˚ a Bevis. Siden beviset er s˚ eksamen p˚ a videreg˚ aende . Tilfellet ab = 1 har allerede blitt drøftet i lemma (2.8.1). Sider ab = 1 impliserer a = 1/b, s˚ a er arctan 1/b + arctan b = π/2. Fra trigonometrien har vi sin(x + y) = sin x sin y + cos x sin y cos(x + y) = cos x cos y − sin a sin b ˚ bruke disse uttrykkene kan tan(x + b) skrives p˚ Ved a a formen (sin x cos y + cos x sin y)/(cos x cos y) sin(x + y) = cos(x + y) (cos x cos y − sin x sin y)/(cos x cos y) tan x + tan y = . 1 − tan x tan y

tan(x + y) =

Setter vi n˚ a inn a = tan x og b = tan y f˚ ar vi tan(arctan a + arctan b) =

a+b , 1 − ab

˚ ta arctan p˚ og beviset fullføres ved a a begge sider av likningen. Anta at ab > 1, ˚ bruke lemma (2.8.1) to ganger har vi da vil alltid a1 · 1b < 1. Ved a 1 1 + arctan a b  a+b   a+b  = π − arctan = arctan +π ab − 1 1 − ab

arctan a + arctan b = π − arctan

˚ vise. Dette fullfører beviset. som var det vi ønsket a Et annet viktig integral er følgende Proposisjon 2.8.3. Z

π

Z

π/2

log(sin x) dx = −π log 2

log(sin x) dx = 2 0

0

˚ vise dette tas først et lite hjelperesultat For a Lemma 2.8.2. Z

π/2

Z

0

π/2

log(sin x) dx

log(sin 2x) dx = 0

II

8 Trigonometrske funksjoner

73

Bevis. Dette lemmaet følger direkte fra likning (2.23) i proposisjon (2.4.2), se oppgave (7) for et bevis. Vi viser likevell resultatet uten bruk av proposisjonen. Via substitusjonen u 7→ 2x s˚ a er π/2

Z

log(sin 2x) dx = 0

1 2

π

Z

log(sin u) du 0

˚ sette Høyresiden av likniningen kan n˚ a skrives om via proposisjon (2.4.5) ved a m = 0, n = 1 og f (x) = log(x) i likning (2.30). Da er ! Z Z π/2 Z π/2 π/2 1 log(sin 2x) dx = 2 log(sin x) dx = log(sin x) dx 2 0 0 0 som ønsket. Alternativt kan en og snitte det integralet p˚ a midten, og bruke substitusjonen u 7→ π − x. Da kan integralet skrives som π

Z

Z

π/2

log(sin u) du = 0

Z

π

f (u) du +

f (u) du π/2

0

Z

π/2

Z

π/2

f (π − u) du = 2

f (u) du +

= 0

Z

0

π/2

f (u) du . 0

Hvor f (u) = log(sin u), og substitusjonen u 7→ π − x ble benyttet i andre over˚ bytte gang. Heldigvis s˚ a er f (π − u) = f (u), siden sin x er periodisk. Ved a tilbake til x som integrasjonsvariabel har en alts˚ a at " Z # Z Z π/2 π/2 π/2 1 log(sin x) dx = log(sin x) dx 2 log(sin 2x) dx = 2 0 0 0 som var det som skulle vises. ˚ vise proposisjon (2.8.3). En er n˚ a klar til a Bevis. Fra proposisjon (2.3.1) eller via substitusjonen u 7→ π/2 − x har en Z

π/2

Z

π/2

log(sin x) dx = 0

 log sin(π/2 − x) dx =

Z

π/2

log(cos x) dx

0

0

˚ legge sammen disse integralene f˚ Ved a ar en at π/2

Z

Z log(cos x) + log(cos x) dx =

0

π/2

log(1/2 sin 2x) dx 0

Dermed kan integralet skrives som π/2

Z

π/2

Z

0

π/2

Z log(sin 2x) dx −

log(sin x) dx =

2

0

log(2) dx 0

˚ benytte lemma (2.8.2) s˚ Siste integralet beregnes relativt enkelt og ved a a er Z

π/2

Z

0

π/2

log(sin x) dx −

log(sin x) dx =

2

0

π log(2) dx 2

74

II

8 Trigonometrske funksjoner

˚ trekke fra det opprinnelige integralet p˚ Beviset fullføres ved a a begge sider. For ˚ vise den siste likheten kan en skrive a Z π Z π/2 Z π/2 log(sin x) dx = 2 log(sin 2u) du = 2 log(sin x) dx = −π log 2 0

0

0

Hvor substitusjonen 2u 7→ x og lemma (2.8.2) ble benyttet igjen. Avslutningsvis tas et integral som vil dukke opp flere ganger senere Eksempel 2.8.2. Gitt at α er en vinkel slik at α ∈ (−π, π) vis at Z 1 dx α I(α) = = . 2 2 sin α 0 x + 2x cos α + 1 ˚ skrive om nevneren Begynner med a x2 + 2x cos α + 1 = x2 + 2x cos α + cos2 α + sin2 α = (x + cos α)2 + sin2 α Integralet kan n˚ a skrives som Z 1 Z 1 dx dx = , 2 2 + 2x cos α + 1 2 x 0 (x + cos α) + sin α 0 Dersom α = 0 s˚ a blir integralet  0 Z 1 1 1 dx = = 2 + 02 (x + cos 0) x + 1 2 0 1 Høyresiden blir tilsvarende lim

α→0

α 1 = 2 sin α 2

 lim

α→0

sin α α

−1 =

1 2

Der det ble brukt at sin x ∼ x n˚ ar x → 0. Anta n˚ a at α 6= 0, vi kan da fritt benytte substitusjonen22 u sin α 7→ x + cos α og du sin α = dx. Dette gir   1 Z 1 Z 1 sin(α) du 1 du 1 x + cos α = = arctan 2 2 2 sin α 0 1 + u2 sin α sin α 0 u sin α + sin α 0 ˚ sette inn grensene blir integralet p˚ Ved a a formen     cos α  1 1 + cos α I(α) = arctan − arctan sin α sin α sin α Dersom differansen av de to siste leddene blir α/2 kan uttrykket skrives som ˚a ˚ n˚ α/2 sin α, som ønsket. Krukset blir a a benytte addisjonsformelen fra korollar (2.8.1)23 ,   x−y arctan x − arctan y = arctan . 1 + xy 22 Hvorfor 23 Hvorfor

kan vi ikke bruke substitusjonen n˚ ar α = 0? x−y brukes arctan 1+xy konsekvent her og ikke arctan

x−y 1+xy

+ π?

II

8 Trigonometrske funksjoner

75

˚ kun se p˚ Ved a a brøken (x − y)/(1 + xy) s˚ a er x − y = 1/ sin α og   x−y 1 + cos α cos α 1 . sin α α 1+ = = = tan 1 + xy sin α sin α sin α 1 + cos α 2 Der det ble benyttet i andre overgang at sin2 α + cos2 α = 1. Siste overgang kan en se fra halv-vinkel formlene for sinus og cosinus sin α = 2 sin(α/2) cos(α/2) og 1 + cos(α) = 2 cos2 (α/2). Integralet forenkles dermed til Z

1

 dx α 1 α = = arctan tan x2 + 2x cos α + 1 sin α 2 2 sin α

0

som ønsket. Oppgaver π

Z

x cos2 x sin x dx

1. Bestem integralet I = 0

2. Vis at følgende likhet holder Z

∞ n

f x +x

−n

0

 arctan x π dx = x 2

Z 0

1

 1 f xn + x−n dx x

for alle funksjoner f . 3. Vis at 1

Z J= 0

holder der α ∈ [0, 2π]. Z ∞ 4. Bestem integralet



1 − x2 π dx = 1 − x2 (sin α)2 4 cos2 α/2

arctan(x) dx n˚ ar a som vanlig er en vinkel i x2 − 2x sin(a) + 1

−∞

a ∈ (−π, π). Z 5. Bestem integralet I = 0

π

x sin x dx 1 + cos2 x

76

II

9 Logaritmiske funksjoner

2.9

LOGARITMISKE FUNKSJONER

I forrige del ble det sett p˚ a integral p˚ a formen Z p(x) log x dx der løsningen var relativt enkel. Benytt delvis integrasjon og sett u = log x og v = P (x), der P 0 (x) = p(x). I denne delen blir studiet av integraler p˚ a formen Z

b

R(x) log dx a

begynt p˚ a. Her er R(x) er en rasjonal funksjon p˚ a formen P (x)/Q(x) med P og Q som polynomer. La oss først ta et lite hjelperesultat Lemma 2.9.1. ∞

Z 0

log x dx = 0 1 + x2

˚ dele integralet. Legg først merke til at Bevis. Fremgangsm˚ aten blir a Z 1



0

Z

log x dx = 1 + x2

log 1/x dx = 1 + (1/u)2 −u2

1

1

Z 0

log 1/u du 1 + u2

(2.60)

˚ dele integralet i to f˚ Via substitusjonen x 7→ 1/u. S˚ a dx = −1/u2 du. Ved a as n˚ a Z 0



log x dx = 1 + x2

Z

1

0

Z = 0

1

log x dx + 1 + x2

Z

log x dx + 1 + x2

Z



1

0

1

log x dx 1 + x2 log 1/x dx = 0 1 + x2

I andre overgang ble likning (2.60) ble benyttet og log 1/x = − log x. En generalisering av dette lemmaet kommer i en senere seksjonen for n˚ a kan ˚ vise følgende resultatet brukes til a Proposisjon 2.9.1. Z 0



log ax π dx = log(ab) x2 + b2 2b

(2.61)

˚ bruke substitusjonen bw = x slik at dw = dx/b. Grensene Bevis. Velger først a blir det samme Z Z ∞ Z ∞ log ax log abw 1 ∞ log abw dx = b dw = dw (2.62) x2 + b2 (wb)2 + b2 b 0 u2 + 1 0 0 Vi kan dele integralet i to siden log abw = log ab + log w, slik at Z ∞ Z Z log ax log ab ∞ dw 1 ∞ log w dx = + dw x 2 + b2 b 1 + w2 b 0 1 + w2 0 0

II

9 Logaritmiske funksjoner

77

Fra lemma (2.9.1) er det siste integralet null, mens det første er likt Z i log ab h log ab ∞ dw π = · arctan(∞) − arctan(0) = log(ab) 2 b 1+w b 2b 0 og en er ferdige. Her brukte en at arctan x → π n˚ ar x → ∞ og arctan 0 = ˚ bruke lemmaet. Ved a ˚ heller bruke 0. Vi kan og vise proposisjonen uten a substitusjonen w 7→ 1/y i likning (2.62) f˚ as Z Z Z 1 0 log ab/y dy 1 ∞ log ab/y 1 ∞ log abw dw = = dy (2.63) b 0 1 + w2 b ∞ 1 + (1/y)2 −y 2 b 0 1 + y2 ˚ ta gjennomsnittet av likning (2.61) og (2.63) f˚ Ved a ar en Z ∞ Z ∞ Z log ax 1 log ay + log ab/y log ab ∞ dy dx = dy = x2 + b2 2b 0 1 + y2 2 1 + y2 0 0 som vist før. Her ble det bare brukt at log A + log B = log AB. La oss videre se p˚ a et integral som dukker opp i [? ]. Eksempel 2.9.1. Z b a

1 log(ab) log x dx = log (x + a)(x + b) 2 b−a



(a + b)2 4ab



Integralet kan dog vises relativt enkelt via substitusjonen x 7→ ab/t. Da er Z b

a

− log ab/t ab dt = ab(b/t + 1)(a/t + 1) t2

Z a

b

log ab − log t dt (t + a)(t + b)

˚ ta gjennomsnittet av integralene f˚ Slik at ved a as Z b Z b log x 1 log x + log ab − log x dx = dx 2 a (x + a)(x + b) a (x + a)(x + b) Merk den fine kanseleringen som skjer med logaritmen i telleren. Det gjennst˚ aende integralet kan beregnes via delbrøksoppspalting   1 1 (b + x) − (x + a) 1 1 1 = = − (x + b)(x + a) b − a (x + b)(x + a) b−a x+a x+b (2.64) ˚ integrere begge sider av (2.64) fra a til b f˚ Ved a ar en at Z a

b

 b   dx 1 x+a 1 a+b 2a = log = log − log (x + b)(x + a) b−a x+b a b−a 2b a+b

Integralet kan dermed skrives som   Z b Z 1 b log ab dx 1 log(ab) (a + b)2 log x dx = = log 2 a (x + a)(x + b) 2 b−a 4ab a (x + a)(x + b) ˚ vise. som var det en ønsket a

78

II

9 Logaritmiske funksjoner

Eksempel 2.9.2. La oss se p˚ a et litt annet integral med en enda rarere substitusjon. Z 0

2

log(1 + x) dx x2 − x + 1

˚ stange hodet mott dette integralet helt til en ser den magiske Det er mulig a substiusjonen som løser alt. Vi begynner med a˚ anta at en substitusjon p˚ a formen x 7→ −1 +

K u+1

vil fungere. Dette vil forenkle logaritmen i nevneren siden   K log (1 + x) = log 1 − 1 + = log K − log(u + 1) u+1 Konstantent K m˚ a bestemmes slik at u(0) = 2 og u(2) = 0 (hvorfor?). Ved a˚ sette inn ser en at magisk nok vil K = 3 fungere. Legg merke til at dx = −3 du/(u+1)2 og at x(u)2 − x(u) + 1 = (u2 − u + 1)

3 (u + 1)

Integralet blir dermed Z 0

2

log(1 + x) dx = − x2 − x + 1

Z 2

0

  Z 2 log 3/(1 + u) log 3 − log(1 + u) du = du u2 − u + 1 u2 − u + 1 0

˚ ta gjennomsnittet av integralene, f˚ Ved a ar en som før at 1 2

Z 0

2

log 3 − log(u + 1) + log(u + 1) du = u2 − u + 1

Z 0

2

√ log 3 du u2 − u + 1

˚ fullføre kvadratet Hvor beregningen av integralet eksempelvis kan gjøres ved a √ Z 2 Z π/3 π 2 3 sec2 y dy 2 π 4 du √ = + = 2 2 3 6 3 −π/6 3 tan y + 3 0 (2u − 1) + 3 √ √ I andre overgangen ble substitusjonen 2u−1 = 3 tan y ⇒ du = 3 /2 sec2 y dx brukt. I tillegg s˚ a er 1 + tan2 x = sec2 x som vanlig. Ved a˚ sette inn og rydde opp f˚ as alts˚ a √ Z 2 Z 2 √ log(1 + x) log 3 du π dx = = √ log 3 2 2 3 0 x −x+1 0 u −u+1 Disse to integralene er begge eksempler p˚ a integral som kan bli løst via den samme substitusjonen. La oss generalisere dette, men først et lite lemma Lemma 2.9.2. La P (x) være et polynom da tilfredstiller P   2 (d u + dc)2 K c − = P (u) P d2 u + cd d Kd2

II

9 Logaritmiske funksjoner

79

hvor K = (c + ad)(db + c) 6= 0, hvis og bare hvis P er p˚ a formen P (x) = Ax2 +

Ac2 + Cd2 − AK x+C, dc

hvor d 6= 0. Dersom c = 0 s˚ a tilfredstilles funksjonallikningen kun dersom P kan skrives p˚ a formen P (x) =

A 2 x + Cx + A , ab

(2.65)

Hvor A og C velges fritt, og a, b 6= 0. ˚ vise dette overlates som en oppgave til leser. Fremgangsm˚ ˚ sette A aten blir a inn et generelt polynom i likningen, og se at høyre side vil ikke være et polynom om ikke graden er to. Dette fører til følgende generalisering Theorem 2.9.1. Gitt at P (x) = Ax2 + Bx + C hvor B=

AK − Ac2 − Cd2 , dc

(2.66)

med K = (ad + c)(db + c). Da holder følgende likhet b

Z

Z

log(dx + c) 1 dx = P (x) 2

a

a

b

log K dx P (x)

for alle A, B. Dersom c = 0 da er Z a

b

log(dx) 1 dx = P (x) 2

Z a

b

log K dx P (x)

med K = dab, for alle polynomer p˚ a formen P (x) = C kan velges fritt.

A ab

x2 + Cx + A. Hvor A og

Dette kan vises direkte ved a˚ benytte seg av substitusjonen u 7→ −c + K/(x + ˚ vise at graden av P maksimalt kan være to overlates til c). Men det i tilleg til a ˚ sette inn et generelt andregradspolynom inn i likning (2.66) kan en leser. Ved a vise følgende Fra dette korollaret faller ?? direkte ut ved a˚ sette A = ab og C = (a + b) inn i likning (2.65)

80

II

10 Ulike tips og knep

2.10 2.10.1

ULIKE TIPS OG KNEP REKURSJONER

OG FUNKSJONSFØLGER

I avsnitt (2.7.3) ble delvis integrasjon gjort enklere ved a˚ skrive om integralet som en funksjonsfølge. I denne delen skal vi se nærmere p˚ a slike følger, og hvordan de kan løeses direkte uten bruk av delvis integrasjon. Eksempel 2.10.1. Z In =

π

cos nx dx = 0 ,

n ∈ Z/{0}

(2.67)

0

Dette eksempelet overlates til leser. Men viser en svært enkel rekursjon In = In−1 . Proposisjon 2.10.1. Z In = 0

π/2

 2 2 2(−1)n   2 − + − · · · + sin nx 3 5 2n + 1 dx = π  sin x  2

dersom

n = 2m

dersom

n = 2m − 1

gitt at m ∈ N. Bevis. Vi begynner med den trigonometriske identiten sin(m + 2)x − sin mx = 2 sin x cos(m + 2)x , ˚ bruke dobbel-vinkel identitene. Ved a ˚ dele likheten p˚ som vises ved a a sin x f˚ as sin mx sin(m + 2)x − = 2 cos(m + 1)x . sin x sin x ˚ dele p˚ Vi m˚ a herfra anta at x 6= 0 slik at vi unng˚ ar a a null. Tilfellet hvor m = 0 ˚ integrere begge sider av likheten f˚ betraktes derfor for seg selv senere. Ved a as Z π/2 Z π/2 Z π/2 sin(m + 2)x sin mx dx − dx = 2 2 cos(m + 1)x dx , (2.68) sin x sin x 0 0 0 ˚ definere som likner en del p˚ a en rekursjonslikning. Ved a Z π/2 sin mx Im = dx sin x 0 har vi alts˚ a vist at Im+2 − Im =

2 π sin (m + 1) m+1 2

(2.69)

hvor vi bare har brukt definisjonen av Im , og beregnet integralet p˚ a høyre side av likning (2.68). Tanken er n˚ a at studerer tilfellet hvor m er odde og like hver for seg. Dersom m er odde kan en skrive m = 2n − 1. Rekursjonen gir da I2n+1 − I2n−1 =

2 π sin (2n) = 0 2n 2

,

II

10.1 Rekursjoner og funksjonsfølger

81

Gitt at n ∈ N s˚ a er alts˚ a I2n+1 = I2n−1 . Rekursjonen sier alts˚ a at dersom m er odde er alle odde potenser like. For eksempel for m = 5 s˚ a f˚ as I5 = I4+1 = ˚ fullføre utregningen m˚ I4−1 = I2+1 = I2−1 = I1 . For a a en alts˚ a bestemme I1 Z

π/2

I1 = 0

sin 1 · x dx = sin x

Z

π/2

dx = 0

π . 2

Alts˚ a har en at n˚ ar m = 2n − 1 hvor n ∈ N s˚ a er Z π/2 Z π/2 sin(2n + 1)x sin(2n − 1)x π dx = dx = sin x sin x 2 0 0 som ønsket. La n˚ a m være like i likning (2.69), da er m p˚ a formen 2n, n ∈ N I2n+2 − I2n =

2 π 2 sin (2n + 1) = (−1)n . 2n + 1 2 2n + 1

˚ vise siste overgang kan en for eksempel bruke dobbel-vinkel identitene For a sin

π π π (2n + 1) = sin πn cos + cos πn sin = cos πn = (−1)n 2 2 2

Rekursjonen sier alts˚ a at hver gang vi minker potensen med 2, s˚ a legger vi til en faktor 2(−1)n /(2n + 1) 2(−1)n + I2n−2 2n + 1 2(−1)n 2(−1)n−2 = + + I2n−4 2n + 1 2(2n − 2) + 1

I2n =

n−1

=

X (−1)n 2(−1)n 2(−1)1 + ··· + + I0 = 2 2n + 1 2+1 2n + 1 k=0

siden I0 = 0. Dette fullfører tilfellet hvor m er like, og ogs˚ a beviset. Lemma 2.10.1. π/2

Z Im = 0

  sin 2mx cos x dx =  sin x

0 m π |m| 2

dersom

m=0

dersom

m ∈ Z/{0}

,

gitt at m ∈ N. Bevis. Det er klart fra definisjonen at I0 = 0 siden sin 0 = 0. Siden sin(−x) = − sin x følger det at I−m = −Im slik at det holder a˚ betrakte Im for n ∈ N. Igjen s˚ a betrakter vi differansen Im+1 − Im for n ≥ 0. Z

π/2

Im+1 − Im = 0

Z

sin(2n + 2)x − sin 2nx cos x dx sin x

π/2

2 cos(2n + 1)x · cos x dx

= 0

Z

π/2

cos(2n + 2)x − cos 2nx dx

= 0

82

10 Ulike tips og knep

II

˚ bekrefte herfra kan vi bruke et resultat som vi lar det være opp til leser a  Z π/2 0 dersom m ∈ Z\{0} cos 2mx = π/2 dersom m = 0 0 Dermed s˚ a er Im+1 = Im , s˚ afremt m 6= 0. Vi har at Z π/2 Z π/2 sin 2x π I1 = cos x dx = 2 cos2 x dx = sin x 2 0 0 R π/2 Der dobbel-vinklel identiten ble brukt sin 2x = 2 cos x sin x og at 0 cos2 x dx = R π/4 cos2 x + sin2 x dx. Fra dette følger det at Im = π/2, n˚ ar m 6= 0 og Im = 0 0 som ønsket. Proposisjon 2.10.2. 2 Z π Z π 1 − cos mx sin mx dx = `2 (m) = dx = |m|π sin x 1 − cos x 0 0

(2.70)

Hvor m ∈ Z. Bevis. Første likhet følger fra at sin2 x + cos2 x = 1 s˚ a (sin mx)2 = 1 − cos mx, 2 ˚ betrakte og (sin x) = 1 − cos x. Vi har at cos(−m) = cos m, s˚ a det holder a m = 0, 1, . . . For m = 0 har vi Z π Z π 1 − cos 0x 1−1 I0 = dx = dx = 0 1 − cos x 1 − cos x 0 0 ˚ drøfte tilfellet m ∈ N. For a ˚ bevise som stemmer. S˚ a herfra gjennst˚ ar det bare a ˚ vise at (In+1 + In−1 )/2 = In . dette ser vi p˚ a en litt annen rekursjon, vi ønsker a Sumformelen for cosinus er cos(A ± B) = cos A cos B ∓ sin A sin B ˚ bruke likningen har vi at Ved a  cos(n + 1)x + cos(n − 1)x = cos nx cos x − sin nx sin x (2.71)  + cos nx cos x + sin nx sin x = 2 cos nx cos x Som vi skal se forenkler dette utregningene v˚ are noe Z π 1 1 − cos(n − 1)x In+1 + In−1 1 − cos(n + 1)x = + dx 2 2 0 1 − cos x 1 − cos x   Z 1 π 2 − cos(n + 1)x + cos(n − 1)x = dx 2 0 1 − cos x ˚ bruke likning (2.71) f˚ Ved a ar vi n˚ a at Z π 1 − cos nx cos x = dx 1 − cos x Z0 π (1 − cos nx) + (1 − cos x) cos nx = dx 1 − cos x Z0 π 1 − cos nx = dx = In 1 − cos x 0

II

10.1 Rekursjoner og funksjonsfølger

˚ vise. Siste overgang følger fra at siden som var det vi ønsket a 1 [sin πn − 0] = 0 for alle n ∈ N. N˚ a har vi vist at n In =

83 Rπ 0

cos nx dx =

In+1 + In−1 ⇒ 2In = In+1 + In−1 ⇒ In − In−1 = In+1 − In 2

Dette beskriver en aritmetrisk progresjon, da differansen mellom to p˚ afølgende ledd er konstant. Vi har alts˚ a In = I0 + (n − 0)d = nd, hvor d er differansen mellom to p˚ afølgende ledd og I0 = 0. Differansen mellom to p˚ aføglende ledd er Z π 1 − cos x d = I1 − I0 = I1 = dx = π 0 1 − cos x og dette fullfører beviset. En triviell triviell generalisering følger fra theorem (2.4.1). Z





`2 (m) = mπ

sin kx sin x

2

Z



dx = mπ

1 − cos kx dx = |knm|π , 1 − cos x

(2.72)

hvor k, n, m ∈ Z. Men dette ser bare mer rotete ut. Fra resultatene ovenfor kan ˚ spørre seg om det være fristende a Z

π



`k (m) = 0

sin mx sin x

k

har en pen lukket form, eller en lukket form i det hele tatt. Vi legger merke til at integralet er null n˚ ar potensen er odde og integranden jevn `1,3,5,··· ,k (2m) = 0 , hvor k ∈ 2N − 1 og m ∈ Z. Via tilsvarende regning som ovenfor kan en vise at  π 1 + 3k 2 4  π `4 (m) = m 1 + 2m2 3  π `5 (k) = 27 + 50k 2 + 115k 4 192  π `6 (m) = m 4 + 5m2 + 11m4 . 20 `3 (k) =

˚ vise den generelle formen Hvor igjen n ∈ 2N − 1 Derimot a Z π `m (n) = 0

sin nx sin x

bm(1−1/n)/2c

m dx = π

X k=0

(−1)

k

  m  m 2 (n + 1) − kn − 1 k m−1

er utenfor denne bokens hensik,t for et bevis se [5] . Her betegner bxc gulvfunksjonen (floor function) som vi vil f˚ a et gjennsyn med i del III. Kort fortalt rundet denne funksjonen ethvert tall ned til det nærmeste heltallet b3.7c = 3, b0.5c = 0 ogs˚ a videre.

http://math.stackexchange.com/ we-simplify-int-0-pileft-frac-sin-nx-sin-xrightmdx

84

II

10 Ulike tips og knep

2.10.2

NYTTIG

FUNKSJONALLIKNING

Lemma 2.10.2. Gitt at R(x) er en funksjon slik at   1 1 R = R(x) x x2 og f (x) en vilk˚ arlig funksjon, da holder følgende Z ∞ Z 1  R(x)f (x) dx = R(x) f (x) + f (1/x) dx 0

0

Bevis. Lemmaet følger direkte fra proposisjon (2.5.2) eller likning (2.38) med g(x) = R(x)f (x) og S = 1. Z ∞ Z 1 R(x)f (x) dx = R(x)f (x) + f (1/x)R(1/x)/x2 dx 0

0 1

Z

 R(x) f (x) + f (1/x) dx

= 0

som ønsket. Vi kan og følge samme fremgangsm˚ ate som i proposisjonen og dele opp integralet Z ∞ Z 1 Z ∞ R(x)f (x) dx = R(x)f (x) dx + R(x)f (x) dx 0

0

Z

1 1

0

Z R(x)f (x) dx −

= 0

Z

1 1

1

Z R(x)f (x) dx +

=

R(y)f (1/y) dy

0

Z

R(1/y) f (1/y) dy y2

0 1

=

 R(x) f (x) + f (1/x) dx

0

og i nest siste overgng y 7→ 1/x benyttet og at R(1/y)/y 2 = R(y). Som vist før. Fra dette følger følgende vidunderlige theorem. Theorem 2.10.1. Gitt at R(x) er en rasjonell funksjon som tilfredstiller funksjonallikningen   1 1 R = R(x) , (2.73) x x2 da holder følgende Z



Z R(x) dx

0

Z

1

R(x)

= 2

(2.74)

0 ∞

R(x) log x dx Z0 ∞

(2.75)

= 0

1 ∞ R(x) dx (2.76) 2 0 Z0 ∞ Z ∞ π R(x) arctan x dx = R(x) dx (2.77) 4 0 0 R∞ Hvor r ∈ R. Likning (2.75) er gyldig viss og bare viss 0 R(x) dx konvergerer. R(x) dx xr + 1

Z

=

II

10.2 Nyttig funksjonallikning

85

˚ vise likning (2.76) og (2.77), hvor de to første likningene blir Bevis. Velger a ˚ benytte overlatt til leser. Merk at alle likningene vises p˚ a samme m˚ ate ved a ˚ bruke lemmaet med f (x) = 1/(xb + 1) f˚ lemma (2.10.2). Ved a as   Z ∞ Z 1 R(x) 1 1 xb R(x) dx dx = + · xb + 1 xb + 1 (1/x)b + 1 xb 0 0   Z 1 1 xb R(x) dx = + xb + 1 1 + xb 0 Z Z 1 1 ∞ R(x) dx . = R(x) dx = 2 0 0 Beviset for siste likning gjøres nesten tilsvarende igjen ved bruk av lemma (2.10.2) n˚ a med f (x) = arctan x.   Z ∞ Z 1 1 R(x) arctan x dx = R(x) arctan x + arctan dx x 0 0 Z Z π ∞ π 1 R(x) dx = R(x) dx . = 2 0 4 0 Der likning (2.74) og arctan x + arctan 1/x = π/2 ble benyttet fra ??. Korollar 2.10.1. Dersom R(x) tilfredstiller R(x) = R(1/x)/x2 s˚ a tilfredstiller ogs˚ a r(x) = (log x)2 R(x) funksjonallikningen. Spesielt s˚ a er Z ∞ (log x)2n−1 R(x) dx = 0 . 0

For alle n ∈ N, gitt at

R∞ 0

R(x) dx konvergerer.

Bevis. Siden (log 1/x)2 = (− log x)2 = (log x)2 følger det direkte at  r

1 x



1 = x2

 2   1 1 1 log R = (log x)2 R(x) = r(x) x x x2

˚ vise siste del av korollaret. Grunntilfellet n = 1, Vi R ∞bruker induksjon for a R(x) log x dx = 0 følger fra likning (2.75). Vi antar s˚ a at det stemmer for 0 Z ∞ (log x)2k−1 R(x) dx = 0 . 0

Alts˚ a at det eksisterer en eller annen funksjon som tilfredstiller R(x) = R(1/x)/x2 ˚ vise at dette medfører at det stemslik at integralet ovenfor er null. Vi ønsker a ˚ skrive om har vi mer for 2(k + 1) − 1 = 2k + 1. Ved a Z ∞ Z ∞   (log x)2k+1 R(x) dx = (log x)2k−1 (log x)2 R(x) dx 0 Z0 ∞ = (log x)2k−1 r(x) dx 0

Fra første del av korollaret s˚ a er r(x) = (log x)2 R(x) en rasjonell funksjon som 2 tilfredstiller r(1/x)/x og resten følger ved induksjon.

86

II

10 Ulike tips og knep

˚ tygge igjennom en betraktelig andel av Theorem (2.10.1) kan benyttes til a vanskelige integral. For eksempel følger lemma (2.9.1) direkte fra (2.75). Eksempel 2.10.2. La oss vende tilbake til en gammel klassiker π/2

Z 0

dθ π = b (1 + (tan θ) 4

(2.78)

˚ bruke substitusjonen x 7→ tan θ f˚ hvor b ∈ R. Ved a as Z 0

π/2

dθ = (1 + (tan θ)b

Z 0



dx 1 = 2 b (1 + x )(1 + x ) 2

Z 0



dx π = 2 1+x 4

I andre overgang ble likning (2.76) ble benyttet og R(x) = 1/(1+x2 ) tilfredstiller selvsagt R(1/x)/x2 = R(x). Et utvalg av funksjoner som tilfredstiller likningen er eksempelvis R(x) ≡

1 1 x 1 1 ≡1+ 2 ≡ ≡ 2 ≡ x x (1 + x2 )2 x +x+1 1 + x2

Spørsm˚ alet blir da, hvordan en kan finne flere funksjoner som tifredstiller likningen? En kan for eksempel legge merke til at R(x) =

h(x) + h(1/x) , x

tilfredstiller likningen hvor h er en rasjonell funksjon. Tilsvarende s˚ a funger  g h(x) + h(1/x) R(x) = x ogs˚ a hvor igjen g og h er rasjonelle funksjoner. En annen m˚ ate a˚ konstruere slike ˚ studere indeksene til polynomene, ved a ˚ skrive ut polynom p˚ a er a x2 x2 = (1 + x2 )2 x4 + 2x2 + 1 Her er koeffisientene til telleren (1), mens i nevneren s˚ a er koeffisientene (1, 2, 1). Alts˚ a er indeksene det samme om de leses fra høyre eller venstre. En definisjon av polynom med denne egenskapen er følgende Definisjon 2.10.1. Et polynom q(x) er symmetrisk24 med indeks k dersom p(x) = xk p(1/x) Eksempelvis er indeksen til q(x) = x3 + x lik 4. Følgende proposisjon25 da vises. 24 Slike

polynom betegnes gjerne som palindom ogs˚ a. Det kan og opplyses om at alle av like grad polynomer som er symmetrisk har komplekse røtter som ligger p˚ a enhetssirkelen. En kan lese mer om dette her http://www.math.uconn.edu/˜kconrad/blurbs/galoistheory/ numbersoncircle.pdf. 25 Takk til flounderer for denne observasjonen. Beviset kan leses her.

II

10.3 Integral par

87

Proposisjon 2.10.3. La p(x) og q(x) være to polynomer, hvor p(x) er symmetrisk med indeks k − 2 og q(x) er symmetrisk med indeks k. La R(x) være definert p(x) q(x)

R(x) =

Da tilfredstiller R funksjonallikningen   1 1 = R(x) R x x2 for alle x. Dette gir oss for eksempel direkte at R(x) =

x4 + 3x3 + 3x2 + x πx7 − x4 − x3 + π

tilfredstiller likningen. Oppgaver 1. Vis at følgende funksjon  R(x) =

x2 x4 + 2ax2 + 1

r 

1 1+ 2 x



tilfredstiller differensiallikningen R(1/x)/x2 = R(x). Her er r > 1 og a ∈ R. 2. Vis at Z



−1

dx =3 1 + x2

Z 3. Bestem integralet 0



Z

1

dx 3 = 2 1+x 2

0

Z 0



dx 1 + x2

xa − xb dx , hvor a, b ∈ R a b (1 + x )(1 + x ) 1 + x2

4. Vis at Z I= 0



(log x)2

1 + x2 dx = 2 1 + x4

Z 0



(x log x)2 dx = 2 1 + x4

Z 0



(log x)2 dx 1 + x4

88

2.10.3

10 Ulike tips og knep

INTEGRAL

II

PA R

I denne delen skal vi se nærmere p˚ a et av favorittintegralene til forfatter. ˚ forst˚ Teknikken som benyttes er relativt enkel a a; anta en har to integral Z Z J = f (x) dx og K = g(x) dx som hver for seg er vanskelig a˚ beregne. M˚ alet blir n˚ a a˚ konstruere et likningsett av to uavhengige lineærkombinasjoner av J og L u1 (x) = a11 J + a12 L u2 (x) = a21 J + a22 K ˚ løse enn J og K hver for seg. p˚ a en slik m˚ ate at likningsettet er enklere a ˚ beregne er Eksempel 2.10.3. Integralene som ønskes a Z Z J = cos log x dx og K = sin log x dx som en har sett før, kan delvis kanselering være gull verdt. Dette gis en sjangse og en beregner J + K Z J + K = cos log x + sin log x dx Det brukes n˚ a delvis integrasjon p˚ a sin log x med u = sin log x og v 0 = 1, s˚ a 0 v = x og u = 1/x · cos log x. Innsatt f˚ as da   Z Z cos log x cos log x dx + x sin log x − x · dx = sin log x + C x Tilsvarende for tilfellet for J − K kan integralet skrives som Z J − K = cos log x − sin log x dx ved samme fremmgangsm˚ ate som før benyttes delvis integrasjon p˚ a cos log x. Hvor u = cos log x, u0 = −(1/x) sin log x og v 0 = 1, v = x.   Z Z − sin log x − sin log x dx + x cos log x − x · dx = cos log x + C x ˚ bruke disse to opplysningene har en alts˚ Ved a a likningssettet x sin log x = J + K x cos log x = J − K ˚ ta gjennomsnittet av likningene f˚ Ved a ar en J=

x sin log x + x cos log x 2

(I) (II)

II

10.3 Integral par

89

˚ ta (I − II)/2 har en ogs˚ og ved a a x sin log x − x cos log x 2

K= Dermed har en at Z

x sin log x + 2 Z x sin log x − sin log x dx = 2 cos log x dx =

x cos log x + C 2 x cos log x + D 2

˚ beregne. som var det en ønsket a ˚ beregne integralet p˚ ˚ Merk at det finnes mange alternative m˚ ater a a. Ved a sette u = log x f˚ as Z Z J = ex sin x dx og K = ex cos x dx . Herfra kan en enten benytte delvis integrasjon p˚ a gammlem˚ aten p˚ a hvert integral. En kan benytte teknikken med integral-par p˚ a J og K. Ellers kan en og skrive om J og K til kompleks form. Merk at alle disse metodene krever noe mer innsats enn metoden ovenfor Dessverre finnes det ikke mange integraler denne metoden er nyttig for, og derfor ender den opp som mer et stillig triks enn en nyttig teknikk. Oppgaver 1. Bestem integralene Z I+ =

sin2 x dx

Z og

I− =

cos2 x dx

2. Bestem integralet Z I=

sin x dx cos x + sin x

˚ finne et passelig J og benytte deg av et velkjent indianertriks. Via Ved a samme fremgangsm˚ ate bestem ogs˚ a integralet π/2

Z I= 0

a cos x − b sin x dx . b sin x + a cos x

3. Bestem integralet Z I= .

3ex + 5 sin x + 10 cos x dx ex + 4 sin x + 3 cos x

90

II

11 Oppgavesammling II

2.11

O P P G AV E S A M M L I N G I I

2.11.1 Z 1. Z 2. Z



INTEGRAL 1 dx x − x2

cos x − sin x dx cos x + sin x

Z 15. Z

dx 1 + sin x 0 Z √ x+1 4. dx x7/2 Z −1 5. dx x3 + x7 Z eπ 6. sin(log x) dx

x cos4 (x) dx

Z

17. 1

ln x x

0

2

18.

Z

19. 0

20.

dx 1 + esin x

π/2 √

0

4a3 dx − a4

x4



0 e

33. 1

0

1 + x4 1 + x6

Z 21.

Z



34.

Z 1 ln x · e + ex dx 22. x

Z

π/2

8.

x

9. 0

Z 10.

x dx 1 + cos x

Z 11. 12. 1

Z 13.

√ dx x+x x

25.



dx p √ x −1 + n x

26.



log x − 1 dx x2 − (log x)2 2x 3x dx − 4x

2

dx Z 5 √ 2 x +x 0 x + x − 2x + 2 35. dx x8 + 1 Z 1 2014 x dx Z ∞ x x2 −1 1 + e 36. dx 2 Z π (x2 − a2 ) + x2 0 sin x sin 2x sin 3x dx 0 Z ∞ 4x5 − 1 √ Z 4 37. dx 7 5 x dx (x + x + 1)2 0 √ √ 7 7−x + 7x 3 r Z ∞ Z ∞ x−1 x dx 38. − dx 2 + 2x + 10 x + 1 x+1 x 1 −∞

Z sinh x + cosh x dx 27. cosh x − sinh x

Z p 14.

24.

dx

p Z

sin x dx 23. cos x + sin x

(x − 1) dx 2x − 1 log(2x − 1)

9x

1

Z

dx 4 + 2x



32.

Z 1 + cos x dx

4

31. Z

4

ln(2x) dx x2 + 9

dx

(x + 2)2 dx (x + 7)5

Z

Z



30.

Z ∞

0

Z 7.

29.

0

Z

x2 (log x − 1) dx x4 − (log x)4

Z

π

16.

π

3.

dx x7 − x

0

π/2

Z √ dx tan x dx a cos2 x + b sin2 x39.

Z r x + x2 + 1 x+1 √ dx 28. dx x2 + 1 x3

Z

π

40. −π

x sin x arctan ex dx 1 + cos2 x

II

11.2 Oppgaver Z

π/2

1 1 41. + dx log tan x 1 − tan x 0 Z log 2 √ 2x e − 2 + e−2x dx 42. ex + e−x 0 Z π/2 sin2(sin x) + cos2(cos x) dx 43. 0

Z 44.

π/2 √

45.

Z

Z 0

Z

Z

Z √

Z

1

8x3 cos4 x · sin2 x dx π 2 − 3πx + 3x2

1



50. ∞

0

2 sin x + x cos x dx

47.

π

49.

51.



sin x ex − sin x − cos x

π

csc x − sin x dx



46.



48.

3/7

0

Z

91

x3

y dy (1 + y 2 ) (y 4 + y 2 + 1)

Z √log 3



x2 + 1 − x2 − 1 √ dx x4 − 1 p p 3 7 1 − x7 − 1 − x3 dx

52. √ log 2

Z

π/4

−π/4

2.11.2

x sin x2 dx sin x2 + sin(log 6 − x2 )

x7 − 3x5 + 7x3 − x + 1 dx cos2 x

53.

0

1 − x2 dx 2x4 − 2x2 + 1

O P P G AV E R

1. Vis at integralene Z

1

Z

I1 = 0

0

1

x2 − y 2 dy dx og (x2 + y 2 )2

Z

1

Z

I2 = 0

0

1

x2 − y 2 dx dy (x2 + y 2 )2

er ulike. a integralet i første oppgave, samt 2. I denne oppgaven skal vi se nærmere p˚ noen liknende integral a) La a, y ∈ R, slik at a > 0, vis at Z



−∞

x2 − y 2 dx = 0 (y 2 + x2 )2

b) Vis videre at vi har Z 0

a

a log a 1 x2 − y 2 log x dx = 2 − arctan 2 2 2 2 (y + x ) y +a y



a y

 (2.79)

˚ og drøft tilfellene a = 0, a = 1 og a → ∞. Det kan være fordelaktig a ˚ bestemme bruke resiltatet fra lemma (2.7.1). Bruk likning (2.79) til a Z a



x2 − y 2 log x dx (y 2 + x2 )2

92

11 Oppgavesammling II

II

˚ vise nytten til c) Det p˚ afølgende integralet blir ofte trukket frem for a kompleks analyse. Men som vi skal se kan det og beregnes mer elementært. Bestem Z ∞ log x I1 = dx (2.80) (1 + x2 )2 0 Hvor en kan f˚ a bruk for en eller flere tidligere deloppgaver. 3. Beregn dobbeltintegralet √ x+ x2 +a2

Z Z K= 1

dt dx , t

hvor a ∈ R. Og vis at løsningen kan skrives som x p + x log a − x2 + a2 + C , K = x · arcsinh a ˚ bruke definisjonen sinh = [exp(−x) + exp(x)] /2. bla ved a 4. Drøft integralet π

Z



0

r2

sin θ dθ , + R2 − 2Rr cos θ

for tilfellene n˚ ar r > R > 0 og R > r > 0. 5. Bestem følgende integral Z

√ 3

x2x

2

+1

  (2x2 +1) + log x2x dx

1

6. En definisjon av naturlige logaritmen er gitt som arealet under 1/t fra t = 0 til t = x. Z x 1 log x := dt . t 1 a) Vis ved a˚ bruke definisjonen ovenfor at logaritmefunksjonen har følgende egenskaper: • log 1 = 0 d • dx log x = x1 • log ar = r log a • log ab = log a + log b b) Som en ekstra utfordring vis at log(1 + xα ) ≤ αx , med likhet hvis og bare hvis α = 1. 7.

x ≥ 0, α ≥ 1

II

11.2 Oppgaver

93

a) La α ∈ R være ett reelt tall. Bestem   Z ∞ 1 + xα dx log I(α) = . 2 log x(1 + x2 ) 0

b) Vis at √

Z



1 + x4+

log



1 + x2+ 1 + x2

0

15

!

5

dx π = log x 4



√ 2+ 6

 q √ 3− 5

Hvor en kan f˚ a bruk for første deloppgave. c)

Konvergerer følgende integral? Z ∞ log (1 + xα ) dx 1 + x2 log x 0

8. Gitt at a og b er to reelle tall slik at a2 + b2 = 52 og Z

b

2ab x − dx 3 x ab

tan θ = a

Vis at cos 2θ kan skrives p˚ a formen −8β 2 + 8β − 1, hvor β = ka, k ∈ R. Finn ogs˚ a cos θ uttrykt ved a. 9. Vis at t m˚ a være periodisk for at Z

t

ex sin x dx =

0

Z

t

ex · cos x +

0

1 dx t

10. Vis at Z 0

π

cos2 x dx = 1 ± cos x sin x

Z 0

π

sin2 x dx 1 ± cos x sin x

og bestem verdien av integralet. 11. Løs integralet 1

1 − x2 √ dx 2 4 0 (1 + x ) 1 + x √ med og uten bruk av substitusjonen 1 + x4 = (1 + x2 ) · cos θ. Z

12. (American Mathematical Monthly # 11457) Vinkelen ϑ(x) er definert utifra følgende figur

94

II

11 Oppgavesammling II

ϑ(x) a x

O

b

Vis at den gjennomsnittlige verdien for ϑ er gitt som 1 b−a

Z

b

ϑ(x) dx = a

π b−a · 4 b+a

der 0 < a < b. Virker utrykket logisk for grensetilfellene a → 0 og b → ∞? ˚ sette Argumenter. Hint: 1) I den delvise integrasjonen er det fordelaktig a u = ϑ(x) og v = x − ab/(a + b). 13. Gitt funksjonen f (x) = arccos x + arcsin x. Vis at f 0 (x) = 0 og bestem Z f (x) dx hvor D er hele definisjonsmengden til f (x). (Alts˚ integralet, a D

alle lovlige x-verdier.) 14. Bestem integralet Z

π/2

I=

 log

0

(1 + sin x)1+cos x 1 + cos x

 dx ,

og vis at svaret er positivt. 15. Vis at Z 0



dx (1 +

αsin x ) (1

+ β cos x )

α, β > 0

er uavhengig av α, β og bestem integralet. Hva er problemet n˚ ar α, β → 0? 16. a)

Vis at Z 1  Z 1  Z 1 4 x +1 dx 2 = Ax + Bx + C dx , 6 2 0 x +1 0 0 Ax + Bx + C

og bestem dermed koeffisientene A,B og C. Hva blir integralet? ˚ bestemme integralene b) Bruk a) til a Z ∞ Z ∞ x4 dx I0 = dx og I4 = 6 1 + x 1 + x6 0 0 ˚ ha klart a). Merk det er ikke nødvendig a

II

11.2 Oppgaver

95

17. Bestem følgende integral π

Z lim

x→∞

0

sin t dt 1 + cos2 (tx)

18. La f og g være to integrerbare funksjoner p˚ a [−8, 8] og la Z 4 Z 4 I= 4f (x) − 8g(x) dx + 8g(x) − 4f (x) dx . −8

8

a

Vis at I = 2 , n˚ ar det oppgis at f er en likefunksjon, g er en oddefunksjon og gjennomsnittsverdien av f og g p˚ a [0, 8] er 1. 19. La f være en strengt voksende og kontinuerlig deriveerbar funksjon p˚ a Z 5 [1, 5] slik at: f (1) = 0 og f (5) = 10. Anta at f (x) dx = 7, bestem 1

integralet

Z

10

f −1 (x) dx

0

der f

−1

betegner inversen til f . √ R π/2 (sin x) 2 +1 dx 0 20. Bestem R π/2 √ (sin x) 2 −1 dx 0 Z 21. Vis at a

b

2x dx p

(x2

− a2 )(b2 − x2 ) Z

1

22. Bruk følgende integral 0

er uavhengig av a og b. Hvor a, b ∈ R.

22 x4 (1 − x)4 ˚ vise at dx til a > π. Vis s˚ a at 2 1+x 7

1 22 1 22 − a. Merk at integralene i a) og b) er h˚ arfint forskjellige. 36. Vis at integralet I kan skrives som Z  I=

arctan x x − arctan x

2 dx =

1 +C tan(β − tan β)

˚ bruke hvor β er en funksjon som avhengiger av x. Det kan være nyttig a tan(a − b) =

tan a − tan b 1 + tan a tan b

(2.81)

som er sumformelen for tangens. 37. Dersom f (0) = 1 , f (2) = 3 og f 0 (2) = 5, bestem Z I=

1

xf 00 (2x) dx

0

Z

π/2

38. Integralet

sin(2x)3 cos(3x)2 dx kan skrives som

π/6

 a b b

√ . Bestem ab + ba + 1

39. La f og g være to funksjoner som har kontinuerlige første og andrederiverte p˚ a [5, 7]. Anta videre at funksjonene er integrerbare p˚ a intervalet. Beregn Z 7 I := f (x)g 00 (x) dx 5

n˚ ar det oppgis følgende: (i) f (x), f 00 (x) > 0 for alle x ∈ [5, 7]. (ii) Funksjonen f /g har kritiske punkt i 5 og 7. (iii) For alle x ∈ [5, 7] s˚ a er g 00 (x) = 1/f 00 (x). 00 (iv) g(x) = 2g (x) − f (x)[g 00 (x)]2 for alle x ∈ [5, 7]. 40. Bestem a og b slik at Z I=

b

√ 2 (sin arccos x dx

a

oppn˚ ar sin maksimale verdi. Hvor stor kan I bli?

II

11.2 Oppgaver

99

41. Vis at dersom f (x) = cos(log x) og g(x) = sin(log x) + cos(log x) s˚ a er 1

Z

1

Z

2

g(x)2 dx

f (x) dx = 0

0

 √ Z 3 2+ 1−y √ dy = 2 (y − 2)3 log y dy 2− 1−y 0 1 Z  Z Z x 2 8t dt 43. Bestem integralet eA 4xe2x dx dx, der A = 2 0 4t + 1 Z

42. Vis at J =

Z 44. Vis at 0

1



(1 − y)



x29 (5x2

+ 49)

17

dx =



492

14! . · 515 · 16!

˚ beregne 45. La oss ta enda en liten digresjon til numerisk integrasjon. For a et integral kan en dele opp intervalet i n deler og konstruere et trapes p˚ a hver del. Dette gir følgende formel n−1

X  h T (n) = f (a) + f (b) + h f (xk ) 2 k=1

a og b er henholdsvis nedre og øvre grense, n er antall interval, h = ˚ beregne (b − a)/n, og xk = a + hk. La oss bruke trapesmetoden til a integralet Z A=

b

f (x) dx a

gitt at integralet er begrenset, og f er en kontinuerlig funksjon. I tillegg opplyses det om at f (x) + f (a + b − x) , er konstant for alle x i intervalet. Vis at 1)

lim T (`)

=

T (1)

T (`)

=

T (1) ∀ ` ∈ N

`→∞

2) Z 3)

b

f (x) dx =

T (1)

a

Dette fungerer ikke bare for trapesmetoden, men enhver metode som bruker lik steglengde for hele intervalet. Eksempelvis midtpunktmetoden, Riemansummer, Simpsons metode og ogs˚ a høyrer ordens metoder. Derimot fungerer det ikke for Romberg integrasjon, eller Gauss-kvadraturer.

46. Anta at en har en beholder som er fyllt med en liten mengde vann. Vi øker s˚ a temperaturen i beholderen. M˚ alet er studere hvordan det molare metningstrykket (heretter kalt damptrykket) varierer som en funksjon av

100

11 Oppgavesammling II

II

˚ gjøre dette lager en en enkel model ved en rekke kjente parameter. For a hjelp av Clausius-Clapeyron’s ligning dp ∆S = dt ∆V Denne beskriver hvor raskt damptrykket forandrer seg, alts˚ a dp/ dt Her er ∆S forandring i entropien27 og gitt som ∆S = nŁf /t hvor Łf er smeltevarme (n˚ ar vann g˚ ar over til damp), n er antall mol og T er temperaturen. Videre s˚ a er ∆V forandringen i volumet og gitt som ∆V = Vg − Ve . Hvor Vg er gassvolumet og Ve er væskevolumet. Tilnærmelsen ∆V h Vg kan ˚ benyttes da det antas en har mye mer gass enn væske. Gassen antas a følge ideel gasslov pV = nRT , hvor n er mol, R er gasskonstanten og T er temperaturen. a)

Vis at Clausius-Clapeyron’s ligning kan omskrives til  d  Lf log p = 2 dT T R

˚ anta at smeltetemperaturen Lf Det a ikke avhengig av tempereturen er gans˚ ke t˚ apelig. I stedet s˚ a er det rimelig a anta at Lf avtar lineært med temperaturen for lave temperaturer. 280 K < T < 440 K. Lf (T ) = α − βT . Bruk vannets trippelpunkt som reffereanse P0 = 611 Pa ved T0 = 273.16 K og ˚ vise at uttrykket for smeltevarmen til a p(T ) kan skrives som p(t) = K · T −µ exp{−Z/T } og kartlegg dermed konstantene Z, K og µ. 47. I denne oppgaven skal en studere nærmere Ω konstanten28 , og noen av dens egenskaper. Omega konstanten er definert slik at den oppfyller ΩeΩ = 1 . a) Vis at Ω konstanten er unikt definert. Vis alts˚ a at likningen xex = 1 har nøyaktig en løsning I resten av oppgaven skal en se nærmere p˚ a funksjonene f (x) = x og g(x) = x2 · log x.

27 Entropi

gir et m˚ al p˚ a hvor mye uorden eller kaos som er i et system funksjonen betegnes og ofte som LambertW(1), og en lengre artikkel finnes p˚ a wikipedia http://en.wikipedia.org/wiki/Omega_constant.

28 Omega

II

11.2 Oppgaver

101

b) Vis at skjæringspunktene mellom f og g er henholdsvis x0 = 0 og x1 = eΩ . c) Vis at arealet avgrenset av f , g, x0 og x1 kan uttrykkes via Ω konstanten som A=

1 1 2Ω 1 1 + e + e3Ω = 6 9 6Ω2 9Ω3

˚ benytte seg av definisjonen til Ω. Hvor det kan bli nyttig a d) Omega konstanten kan og defineres rekursivt via Ωn+1 =

1 + Ωn 1 + eΩn

˚ ansl˚ med startverdi Ω0 . Bruk startverdien Ω0 = 0 og to iterasjoner til a a integralet. En bedre startverdi er Ω0 = log 2, bruk denne og at e ≈ 8/3 til ˚ ansl˚ a a integralet. Benytt n˚ a en iterasjon. 48. Gitt at a = 1 og b2 = 1 − 1/2 log 1/2, der b > 0. Vis at Z

b

xf (x) dx = A log(2) · (1 + log 2) + B

I := a

og kartlegg dermed konstantene A og B. Her er f er implisitt gitt som x2 + f ef = 1 ,

f > −1 .

˚ benytte delvis integrasjon for a ˚ oppn˚ Hint: Det kan lønne seg a a x2 for ˚ bruke substitusjon. deretter a 49. I denne oppgaven skal en s˚ a vidt være innom kvantefysikk. Merk at ingen forkunnskaper utover integrasjon er nødvendig. Oppgaven dreier seg i praksis om Carbon-14 datering, men som sagt fokuserer en her p˚ a det matematiske. N˚ ar et C14 atom henfaller, (brytes ned) skiller det ut blant annet ut en α partikkel, (alfa-str˚ aling). Denne α partikkelen har da et potensial V (r) =

2Ze2 4π0 r

Radien til partikkelen er gitt som r1 , mens str˚ alingen fra partikkelen n˚ ar til til radien r2  r1 , Energien til partikkelen en da være E = V (r2 ) =

2Ze2 4π0 r2

og tilslutt s˚ a er hvor mye str˚ aling som blir avgitt fra partikkelen gitt som   Z q   2 r2 − T ∼ exp 2m V (r) − E dr = } r1

102

II

11 Oppgavesammling II a)

Vis at en kan skrive V (r) = Er2 /r og at log T kan uttrykkes som Z 1 r 2√ 1 ∼ log T = − 2mE r2 − 1 dx } x r2 /r1

˚ undersøke hvordan log T øker n˚ Videre s˚ a ønsker en a ar E avtar. b) Forklar hvordan en kan skrive om integralet til Z 1r Z r1 /r2 r Z 1 r 1 1 1 − 1 dx = − 1 dx − − 1 dx , I= x x x 0 0 r1 /r2 og, beregn integralet eksakt. c) For lettere regning kan en benytte seg av en frekk tilnærming. Det første integralet er lik π/2, og for det andre integralet har en at x < a x−1 − 1 ∼ x−1 . r1 /r2  1 slik at faktoren 1 bidrar tilnærmet ingenting, s˚ Bestem n˚ a et tilnærmet uttrykk for I og log T . d)

Tilslutt leker en oss litt med algebra. Vis at   p Z log T ∼ = −2 K1 √ − K2 Zr1 E

hvor verdiene for K1 og K2 er henholdsvis e2 √ K1 ≡ π 2mc2 4π0 }c

s og

K2 ≡ 4

e2 4π0 }c

r

mc2 }c

III

104

3.1

1 Introduksjon III

III

INTRODUKSJON III

I første del dyppet du forsiktig tærne ned i innsjøen av integrasjonsteknikker. Mens i andre del lærte du, og forh˚ apentligvis mestret en rekke teknikker for ˚ holde deg flytende. Men jeg h˚ ˚ holde pusten for n˚ a aper du er god til a a skal vi dykke dypt, dypt ned i mørket av integrasjonsmetoder. Denne delen vil ha fokus p˚ a to hovedomr˚ ader, spesielle funksjoner og spesielle teknikker. Videre vil du finne flere eksempler p˚ a hvor disse to omr˚ adene virker i skjønn harmoni ˚ løse vanskelige integral. Vi vil studere rekkeutvikling, kompleks anaylyse, for a integraltegnet, omskrivninger til dobbel integraler og mye mer.

III

2.1 Euler–Mascheroni konstanten

3.2

105

V I K T I G E K O N S TA N T E R

˚ I denne seksjonen blir et knippe viktige konstanter tatt opp. Det er viktig a huske p˚ a at det a˚ oppgi svaret som ζ(3) eller Γ(1/4) ikke er noe forskjellig fra et svar som π eller e. Forskjellen kommer i hva en legger i et elementært uttrykk. Herfra blir et par konstanter nevnt, som forfatter mener er elementære.

3.2.1

EULER–MASCHERONI

K O N S TA N T E N

γ = 0.577 215 664 901 532 860 606 512 090 082 402 431 042 159 335 939 92 Definisjon 3.2.1. Euler’s konstant ble først introdusert av Leonard Euler (1707 − 1783) i 1734 som   1 1 1 γ := lim 1 + + + ... + − log(n) (3.1) n→∞ 2 3 n Grunnen til at konstanten ofte omtales som Euler–Mascheroni er at selv om det var Euler som først studerte grensen var det Lorenzo Mascheroni (1750 − 1800) som fra 1790 skrev flere utdypende papir om konstanten, og dermed grunnlaget for videre studier. Nøyaktig n˚ ar notasjonen γ, ble kom i bruk er uvist, men det som er sikkert er at navnet er valgt grunnet konstantens nære relasjon til Gammafunksjonen ∞ h Y 1 z  −z/n i = zeγz e 1+ Γ(z) n n=1

(3.2)

hvor beviset kommer vi tilbake til senere. Formelen gir oss og med en gang relasjonen Γ0 (1) = ψ0 (1) = −γ. Det er fortsatt ukjent om γ er et algebraisk eller trancendentalt. Faktisk vet en ikke engang om γ er irrasjonelt, selv om det er sterke antakelser om det. ˚ være nært tilknyttet gammafunksjonen spiller og funksjonen en Foruten om a viktig rolle innen Analyse (Bessel funksjoner, eksponensial-integral, summer, osv), og dukker og hyppig opp i Tallteori. Det er liten tvil om at selv om konstanten ikke er like kjent som π eller e s˚ a er den en av de aller viktigste matematiske konstantene vi har i dag. Integral-representasjoner av gamma Z ∞ Z ∞ 2 −x γ=− e log x dx = −4 xe−x log x dx 0 0   Z 1 Z 1 1 =− log log dx = Hx dx x 0 0 Z ∞ Z 1 1 1 1 1 = − dx = + dx x−1 x e xe log x 1 − x 0 0   Z ∞ 1 1 −x = − e dx ∀ k > 0 x 1 + xk 0 ˚ vise følgende Som eksempel velger vi her a

106

III

2 Viktige konstanter

Eksempel 3.2.1. Z γ=−

1

 log log

0



1 x



Z

e−x log x dx

dx = − 0

˚ bevise følgende trengs definisjonen av Γ(x) som dessverre kommer Bevis. For a først senere og er gitt som Z



tx−1 e−t dt

Γ(x) = 0

˚ derivere likningen f˚ Merker oss at ved a as d Γ(x) = dx

Z

Γ0 (1) =

Z



tx−1 e−t log t dt

0 ∞

e−t log t dt

(3.3)

0

Dersom vi n˚ a benytter oss av substitusjonen x 7→ log(1/t) i likning (3.3) f˚ as Γ0 (1) =

Z

1

 log log

0

1 x

 (3.4)

dx

˚ vise at Γ0 (1) = −γ kan gjøres ved a ˚ studere se p˚ A a logartimen av likning (3.2), ogs˚ a derivere. Men for øyeblikket overlates dette til leser. Vi har alts˚ a γ = −Γ0 (1) = −

Z

1

 log log

0

1 x



Z dx =



ex log x dx

0

˚ vise. som var det vi ønsket a Vi kommer tilbake til Γ0 (1) n˚ ar vi diskuterer digamma-funksjonen.

3.2.2

C ATA L A N ’ S K O N S TA N T G = 0.915 965 594 177 219 015 054 603 514 932 384 110 774

Catalans konstant er navngitt etter Eug`ene Charles Catalan (1814-1894) og er definert som G :=

1 1 1 1 − 2 + 2 − 2 + ··· 12 3 5 7

Konstanten dukker hyppig opp i problemer innne kombinatorikk, og inneholder mange fine identiter. Det er ikke kjent hvorvidt G er irrasjonell, eller trancendental og noen ganger betegnes og konstanten som Catalan.

III

2.3 Glaisher–Kinkelin konstanten

107

Noen viktige identiter ∞ X

(−1)n (2n + 1)2 n=0 Z 1 Z ∞ arctan t = dt = arctan e−t dt t 0 0 Z π/4 Z π/4 = log cot t dt = log tan t dt

G = β(2) =

0

Z

0 π/4

=

Z

π/4

log(sin 2t) dt = 0

Z

1

= 0

Z = 0

Z = 0

log(cos 2t) dt 0

1

Z

1

dx dy 1 + x2 y 2 0 Z ∞ log t log t dt = 1 + t2 1 + t2 1

π/4

t dt sin t cos t

hvor β er Dirichlet beta funksjonen. Eksempel 3.2.2. I Del II ble følgende resultat bevist Z ∞ a π log ax = log x 2 + b2 2b b 0 ˚ bruke at ved a Z 0



log x =0 x2 + 1

˚ legge merke til at arealet fra 0 til 1 var like stort som fra 1 til Dette s˚ a vi ved a ∞. Noe merkelig var at verdien av disse integralene aldri ble vist! Fra tabellen over ser vi at integralene er lik G, og dette viser vi n˚ a med rekkeutvikling. Bevis.

3.2.3

GLAISHER–KINKELIN

K O N S TA N T E N

108

III

3 Spesialfunksjoner

3.3

SPESIALFUNKSJONER

3.3.1

G U LV

O G TA K - F U N K S J O N E N E

˚ mappe I matematikk og Informattik brukes gulv (floor) og tak (ceiling) til a et reelt tall til det største tidligere heltallet eller minste p˚ afølgende heltall. Definisjon 3.3.1. La m ∈ Z og x ∈ R. Da er bxc = m dxe = m + 1

dersom m ≤ x ≤ m + 1 dersom m ≤ x ≤ m + 1

Definisjonen ovenfor kan ogs˚ a skrives som bxc = max{m ∈ Z | m ≤ x} og bxc = min{n ∈ Z | m ≥ x}. Definisjonen ovenfor vil derimot være noe mer anvenbart I tilleg har vi fraktal delen {x}. Definisjon 3.3.2. La x ∈ R. Da er fraktaldelen av x gitt som {x} = x − bxc for alle x i 0 ≤ {x} ≤ 1. x 1 0.5 π

bxc

dxe

1 0 3

1 1 4

{x} 0 0.5 0.141592 . . .

Eksempel 3.3.1. I forhold til integrasjon er gulv og tak-funksjonene ofte brukt som en enkel ˚ bytte mellom integraler og summer. Som et trivielt eksempel har en m˚ ate a Z n n Z k+1 n X X n(n + 1) dxe dx = k dx = k= 2 0 k k=0

k=0

hvor n ∈ N. Alts˚ a kan vi skrive summen av de n første heltallene som et integral. Proposisjon 3.3.1. La x ∈ R da er Z x bxcbx − 1c btc dt = + bxc{x} (3.5) 2 Z 0x 1 bxcbx − 1c {t} dt = x2 − − bxc{x} (3.6) 2 2 0 Rx Rx Bevis. Her trenger vi bare vise første likning siden 0 {t} dt = 0 t − btc dt = Rt 1 2 a p = bxc, da er 2 x − 0 btc dt. La n˚ Z

x

[t]dt = 0

p−1 Z X k=0

=

p−1 X k=0

k

k+1

Z [t] dt +

x

[t] dt = p

k + p(x − p) =

p−1 Z X k=0

k+1

Z k dt +

k

p(p − 1) + p(x − p) 2

x

p dt p

III

3.1 Gulv og tak-funksjonene

109

Som var det vi ville vise siden bxc ± n = bx ± nc, n ∈ N og x − p = x − bxc = {x}. Proposisjon 3.3.2. La x ∈ R+ da er Z x  1 1 dt = 1 − γ + log x + Hρ − t ρ 0

(3.7)

Hvor ρ = b1/xc + 1 og Hn er summen av de n første leddene i den harmoniske rekken. Korollar 3.3.1. La x ≥ 1 da er Z x 0

1 t

 dt = 1 − γ + log x

(3.8)

og spesielt s˚ a er Z 1 0

1 t

 dt = 1 − γ

(3.9)

for x = 1. ˚ vise dette overlates derfor til leser. For a ˚ vise Bevis. Korollaret følger direkte. A proposisjonen setter vi y 7→ 1/t og ρ = b1/xc.   Z ∞ Z x  Z ρ ∞ Z k+1 X {y} x−k 1 y − (ρ − 1)  + dt = dy = dy dy 2 2 t y x y2 1/x k 0 1/x k=ρ

(3.10) Integralet kan skrives som   Z ρ y+1−ρ 1 = − x + log ρ + log(x) y2 ρ 1/x ˚ skrive ut summen f˚ Ved a ar vi en teleskoperende sum ∞ Z k+1 ∞ X X x−k 1 dy = log(k + 1) − log(k) − = Hρ − log ρ − gamma 2 x 1 + k k k=ρ

k=ρ

˚ sette sammen de to forrige likningene f˚ ved a ar vi alts˚ a     Z x 1 1 1 dt = (ρ − 1) − x + log(x) + Hρ == 1 − γ + log x + Hρ − t ρ ρ 0 som var det som skulle vises. Theorem 3.3.1. La f være en stykkevis kontinuerlig funksjon for x ∈ (0, 1) slik at R1 |f (x)|/x dx < ∞ er endelig. Da har en 0 Z 1   Z 1 1 dx f (x) f = dx x 1−x x 0 0 hvor {x} fortsatt betegner fraktal delen av x.

110

III

3 Spesialfunksjoner

Bevis. Dette beviset tillegnes Olivier Oloa og kan leses i sin opprinnelige form ˚ skrive om integralet til en sum her. Som før er første steg a Z 1   ∞ Z 1/k X 1 dx dx f = f ({1/x}) x 1 − x 1 −x 0 k=1 1/(1+k) Z ∞ k+1 X du = f {u} u(u − 1) k=1 k ∞ Z k+1 X du = f (u − k) u(u − 1) k=1 k Z ∞ X 1 dv = f (v) (v + k)(v + k − 1) k=1 0 Z 1 ∞ X 1 f (v) = dv (v + k)(v + k − 1) 0 k=1 Z 1 dv = f (v) v 0 Hvor bytte av summasjonstegnet og integraltegnet følger fra dominererende konverges theoremet. La v ∈ (0, 1), og la N → ∞, en har da N X k=1

N

X f (v) = f (v) (v + k)(v + k − 1)

k=1



1 1 − v+k−1 v+k

 =

f (v) f (v) − v v+N

˚ se Som g˚ ar mot f (v)/v n˚ ar N → ∞. Mer at 1 = (v + k) − (v + k − 1) for a ˚ teste at summen virkelig konvergerer har vi delbrøksoppspaltingen. For a N f (v) X f (v) |f (v)| |f (v)| |f (v)| 1 − = − ≤ = f (v) (v + k)(v + k − 1) v v+N v v+N v k=1

R1 som følger fra antakagelsen om at 0 |f (x)|/x dx var endelig. Siden absollutt konvergens impliserer konvergens fullfører dette beviset. ˚ f˚ B˚ ade i beviset ovenfor og i theorem 4.22 ble f {x} = f ({x}) brukt, for a a litt penere notasjon. Proposisjon 3.3.3. La a være et reelt tall, da holder Z 1s {1/x} dx 2 arcsin a = 2 {1/x} 1 − x 1 − a a 0 for alle a ∈ (0, 1]. Proposisjon 3.3.4. Z 0

for all n ∈ N\{1}.

1

s n

{1/x} dx π = 1 − {1/x} 1 − x sin(π/n)

III

3.2 Gammafunksjonen

111

Korollar 3.3.2. 1

Z

s n

0

{1/x} dx =π 1 − {1/x} 1 − x

Bevis. Dette integralet følger fra ?? med a = 1 eller proposisjon (3.3.4) med n = 2.

3.3.2

GAMMAFUNKSJONEN

I denne delen skal vi se nærmere p˚ a Γ-funksjonen og noen av dens egenskaper. For et historisk perspektiv anbefalles artikkelen [? ] p˚ a det sterkeste, og for en mer utfyllende introduksjon anbefales [? ] – denne delen er hovedsaklig basert p˚ a disse kildene. Det finnes mange funksjoner innen matematikk som bare er definert for hele tall1 . Eksempelvis s˚ a kan summmen av de n første naturlige tallene skrives som T : N 7→ N

T (n) =

n X

= 1 + 2 + · · · + (n − 1) + n

k=1

ved hjelp av enkel algebra kan summen skrives p˚ a lukket form som n(n + 1) 2

T (n) =

(3.11)

N˚ a er formelen bare gyldig for naturlige tall. Men likevel gir formelen en mulig definisjon av hva det vil si og legge sammen de 5 21 første naturlige tallene. En ˚ legge sammen de n første tallene, vil være a ˚ gange naturlig fortsettelse til det a sammen de n første tallene T : N 7→ N

T (n) =

n Y

k = 1 · 2 · · · (n − 1) · n = n!

k=1

Det ble lenge lett etter et lukket uttrykk for produktet slik som likning (3.11) gir en naturlig utvidelse for addisjon av de naturlige tallene. Motivasjonen er alts˚ a ˚ gi mening til n! n˚ a ar n ikke er et heltall. ˚ konstruere en slik funksjon Leonard Euler viste heldigvis at det var mulig a som følger Definisjon 3.3.3. La s = σ + it, hvor σ, t ∈ R. Vi definerer Γ-funksjonen for σ > 0 som Z ∞ Γ(s) := ts−1 e−t dt 0

Dette integralet betegnes ogs˚ a noen ganger som Eulers første integral, eller ˚ vise at denne funksjonen kondet Euleriske integralet (The Eulerian integral). A vergerer dersom σ > 0 er vist i lemma (C.1.4) i Appendix C. Et sterkere resultat er a˚ vise at Γ-funksjonen er analytisk for σ > 0, dette er og vist i appendiks C (se ˚ vise at denne funksjonen virkelig er lemma 34.15) i For n˚ a begrenser vi oss til a ˚ vise at en utvidelse av fakultetsfunksjonen. Vi ønsker alts˚ aa 1 Slike

funksjoner kalles gjerne for aritmetiske- eller tallteoretiske-funksjoner.

112

III

3 Spesialfunksjoner

Proposisjon 3.3.5. Γ-funksjonen tilfredstiller Γ(x + 1) = xΓ(x)

∀x∈R

(3.12)

og dersom n ∈ N s˚ a er Γ(n + 1) = n! med Γ(1) = 1. ˚ bruke delvis integrasjon s˚ Bevis. Ved a a kan vi skrive Z ∞ Z ∞  0 Γ(x + 1) = tx e−t dt = tx e−t ∞ − −xtx−1 e−t dt = xΓ(x) 0

0

˚ anta Hvor et vokser raskere enn tx ∀, x > 0 slik at tx e−t → 0 n˚ ar t → ∞. Ved a at n ∈ N f˚ as Γ(n + 1) = nΓ(n) = n(n − 1)Γ(n − 1) = n · (n − 1) · · · 2 · 1 · Γ(1) = n! · Γ(1) ˚ vise at Γ(1) = og det er trivielt a

R∞ 0

e−t dt = 1.

˚ finne funksjoner slik at f (1) = 1 og at f (x + 1) = xf (x) er ikke spesielt A vanskelig og det finnes hele funksjonsgrupper som tilfredstiller disse kravene. Eksempelvis har vi ! Γ 1−x d 1 2  log F (x) = Γ(1 − x) dx Γ 2−x 2 eller " f (x) = eg(z) · z

∞  Y n=1

z  −z/m 1+ e m

#−1

Hvor i siste definisjon s˚ a er g alle funksjoner som tilfedstiller g(z + 1) − g(z) = γ + 2kπi k ∈ Z. Som en artig kuriositet f˚ as gammafunksjonen ved a˚ velge den enkleste funksjonen g som g(z) = γz. Enda mer t˚ apelige eksempler inneholder eksempelvis  1/x 0 0 og t ∈ R. Som er vist p˚ a side (223). Selv uten bevis virker lemmaet intuitivt siden vi har e−x = limx→∞ (1 − x/n)n .

˚ ta utgangspunkt I lemmaet ovenfor har vi Bevis. Ved a Z 0

n

 n Z 1 t s−1 s 1− t dt = n (1 − u)us−1 dt n 0 ! 1  s Z n 1 u (1 − u)n s n−1 s + =n (1 − u) u du s s 0 0  Z 1  n = ns (1 − u)n−1 us du s 0 = ··· n(n − 1) · · · 1 s(s + 1) · · · (s − 1 + n) n! ns = s(s + 1) · · · (s + n)

= ns

Z

1

u(s−1)+n du

0

˚ ta grensen n˚ og a ar n → ∞ p˚ a begge sider av likheten fullfører beviset.

˚ bruke likning (3.13) kan følgende theorem utledes Ved a Theorem 3.3.3. Weiserstrass produktet er definert som ∞   z Y 1 z := zeγz 1− exp − Γ(z) n n n=1

(3.14)

for alle z ∈ C forutenom z ∈ −N.( Alts˚ a polene −1, −2, . . . til gammafunksjonen.) Dette er og en analytisk utvidelese av Γ(s) og er kjent som et av Weierstrass produktene. Under følger et kort bevis

116

III

3 Spesialfunksjoner

Bevis. Vi tar utgangspunkt i likning (3.13) og ser p˚ a inversen.   1 z(z + 1) · · · (z + n) = lim n−z n→∞ Γ(z) 1 · 2 · · · (n − 1) · n  n  Y z+k = z lim n−z n→∞ k k=1

= z lim e

−z log n

n→∞

n Y

e

z/k

k=1

n  Y z  −z/k e 1+ k

k=1

= z lim e−z log n ez(1+1/2+1/3+...) n→∞

1+

k=1

= z lim e−z(Hn −log(n) n→∞

= ze−γz

n  Y

n  Y k=1

∞  Y n=1

1+

z  −z/k e k

z  −z/k e k

z  −z/n 1+ e n

Hvor Hn er de n første tallene i den harmoniske serien. det andre Weiestrassproduktet definerer sin πx og er gitt som  ∞  Y z2 1− 2 sin πz = πz n n=1

(3.15)

Som kan bevises formelt via Weierstrass faktoriserings teorem ellers kan en anta at ˚ gange sammen sin πz kan skrives som et produkt av alle sine nullpunkter, ved a røttene p˚ a en smart m˚ ate f˚ as teoremet. For n˚ a tar vi det for gitt at likning (3.15) ˚ vise følgende teorem. stemmer, og ønsker heller a Theorem 3.3.4. (Euler’s refleksjons formel) For alle s ∈ (0, 1), s˚ a holder identeiteten π Γ(s)Γ(1 − s) = (3.16) sin(πs) ˚ vise denne sammenhengen uten Weierstrass’ produktene er noe trasig, men A kan gjøres ved a˚ skrive om høyresiden til et integral som løses via kompleks ana˚ vise at φ(x) = Γ(x)Γ(1 − x) oppfyller en bestemt differensiallikning lyse eller a hvor løsningen gir oss resultatet. Bevis. Ved a˚ ta utgangspunkt i likning (3.13) s˚ a har vi likning (3.14) s˚ a har vi at Γ(1 − z)Γ(z) = −zΓ(−z)Γ(z) og n! n! ns n−s n→∞ s(s + 1) · · · (s + n) −s(−s + 1) · · · (−s + n) 1 π   =− Qn = lim s s n→∞ −s2 sin πs 1− n k=1 1 + n

Γ(s)Γ(−s) = lim

˚ gange Her ble likning (3.14) brukt i siste overgang. Beviset fullføres n˚ a ved a med −s p˚ a begge sider og bruke −sΓ(−s) = Γ(1 − s). Som følger direkte fra proposisjon (3.3.5).

III

3.2 Gammafunksjonen

117

˚ bruke resultatet ovenfor kan vi eksempelvis utlede følgende Ved a Lemma 3.3.3.  Γ

1 2



Z



=

√ 2 e−u du = π

−∞

˚ sette z = 1/2 inn i likning (3.16) f˚ Bevis. Ved a as  Γ

1 2

 2    1 1 π  ⇒ Γ Γ 1− = =π 2 2 sin π2

√ ˚ bruke selve definisjonen av gammaHerfra ser vi at Γ(1/2) = π . Videre ved a funksjonen s˚ a har vi at   Z ∞ Z Z ∞ 2 1 1 ∞ −u2 (1/2−1) −t e du = e−u du Γ = t e dt = 2 2 0 −∞ 0 ˚ kombinere dette resultatet med uttrykket for Γ(1/2) Via substitusjonen t 7→ u2 . A fullfører beviset. Som er et svært viktig resultat innen statistikk og sannsynlighetsregning. Proposisjon 3.3.8. (Dobbel Identiten) For alle s slik at s, s + 1/2 6= 0 s˚ a er   1 = 21−2s π 1/2 Γ(2s) (3.17) Γ(s)Γ s + 2 Vi skal senere se p˚ a et enklere bevis for denne identiten ved bruk av betafunksjonen, men inntil da blir beviset noe tungt ˚ bruke et nyttig uttrykk for Γ(2s), vi kan skrive Bevis. Teknikken blir a n!ns n!ns/2 s(s + 1) · · · (s + n) (s + 12 ) · · · (s +  2s(2s + 1) · · · (2s + 2n) (2n)!(2n)2s   1 (n!)2 n1/2 22n+1 lim 2s n→∞ (2n)!(s + n + 21 )   1 (n!)2 22n+1 lim 1 2s n→∞ (2n)!n1/2 (1 + ns + 2n )   2 2n+1 1 (n!) 2 lim 2s n→∞ (2n)!n1/2

Γ(s)Γ(s + 1/2) = lim n→∞ Γ(2s) × = = =



1 2

+ n)

˚ vise identiteten m˚ For a a vi alts˚ a bestemme den siste grensen. For enkelhetensskyld defineres   (n!)2 22n+1 C = lim n→∞ (2n)!n1/2

118

III

3 Spesialfunksjoner

Uttrykket m˚ a holde for alle s, det m˚ a eksempelvis holde for s = 1/2. Innsetning gir da C=

√ Γ(s)Γ(s + 1/2) 2s Γ(1/2)Γ(1) 2 = 2=2 π Γ(2s) Γ(1)

√ ˚ sette inn uttrykket hvor Γ(1) = 1, og Γ(1/2) = π fra lemma (3.3.3) ble brukt. A ˚ gange med Γ(2s) fullfører beviset. for C, og a Til slutt regner vi ut to siste integral herfra blir resten av resultatene knyttet ˚ vise følgende til gammafunksjonen tatt i oppgavedelen. Vi ønsker a Vi har tidligere sett at de logaritmiske egenskapene til gammafunksjonen er svært viktige. Bla gir har vi et resultat som sier at Theorem 3.3.5. (Stirrlings approximasjon) Gitt følgende grense lim √

n→∞

n! 2πn

n e

n = 1

s˚ a er n! ∼

 n n √ e

2πn

for store n. √ Stirrlings approximasjon eller formel, kan og skrives som, log(n!) ∼ 2πn + n log n − n . Resultatet kan selvsagt generaliseres for gammafunksjonen. Et forenklet bevis for overnevnte finnes i oppgavesammlingen under. Vi velger heller ˚ prøve a ˚ vise følgende a Proposisjon 3.3.9. 1

Z

√ log Γ(x) = log 2π

0

˚ skrive følgende integral Bevis. Beviset begynner med a Z

1

log Γ(x)Γ(1 − x) dx

(3.18)

0

P˚ a den ene siden kan integralet deles opp, slik at Z

1

1

Z log Γ(x)Γ(1 − x) dx =

Z

1

log Γ(1 − x) dx

log Γ(x) dx +

0

0

0 1

Z

Z log Γ(x) dx −

= 0

log Γ(y) dy 1

Z

1

log Γ(x) dx .

=2 0

0

III

3.2 Gammafunksjonen

119

Alternativt kan integralet og skrives om ved a˚ bruke Eulers refleksjonsformel, s˚ a Z 0

1



 π log Γ(x)Γ(1 − x) dx = log dx sin(πx) Z 1 Z 1 = log π − log sin(πx) dx Z

0

0

= log π + log(2) dx ˚ se det siste integralet kan substitusjonen y 7→ πx bli brukt s˚ For a a da f˚ as 1

Z

log sin(πx) dx = 0

1 π

π

Z

log(sin y) dy 0

som er et standard integral som har blitt beregnet før, se proposisjon (2.8.3). Ved sammenlikning har en n˚ a at Z

1

1

Z log Γ(x)Γ(1 − x) dx =

log Γ(x)Γ(1 − x)

0

0

Z

1

log Γ(x) dx = log π + log(2)

2 0

Z

1

√ log Γ(x) dx = log 2π

0

˚ vise. som var det vi ønsket a

Oppgaver Z



1. Vis at Γ(n) =

e−e

x

+nx

dx , hvor n ∈ N.

−∞

2. I denne oppgaven skal vi kartlegge en tilnærmelse til Γ(x), ogs˚ a bedre kjent som Stirrling’s formel. Vi utvikler bare rekken til andre ledd, da videre utvikling krever mer arbeid. Bestem et lukket uttrykk for S=

N X

log i ,

i=1

og videre vis at S tilnærmet er en Riemansum til integralet Z n S≈C· log x dx . 1

˚ bekrefte at Bruk uttrykket for S og integralet til a  N! ≈ C for store N , der C er konstant.

N e

N

120

III

3 Spesialfunksjoner

˚ bli mer 3. I denne oppgaven er ikke integrasjon i fokus. Men heller det a ˚ bruke Γ(x) funksjonen. Uansett er det sterkt anbefalt a ˚ prøve stø i a seg p˚ a følgende oppgaver. Vi tar atter en gang og dypper fingrene ned ˚ lære matematikken og ikke i kvantevannet, merk som vanlig at m˚ alet a fysikken, men litt forklaring trengs likevell. Et kvantifisert magnetisk moment har 2 kvantetilstander som betegnes med verdiene s = ±1. Avhengig av om det magnetiske momentet peker med eller mot p˚ asatt magnetfelt. Tenkt p˚ a en magnet med + og −. Vi betrakter et stort antall N slike magnetiske moment. Hvor N+ har verdien s = +1 og N− = −1. Videre s˚ a er N+ + N− = N . Tilslutt s˚ a er det totale magnetiske momentet gitt som m = (N+ − N− )/N . M˚ alet med blir a˚ bestemme Entropien S (uorden eller kvantevillskapen”) til systemet. Stefan-Boltzans lov at entropien i et slikt magnetisk system er gitt som   N! S = k log N− !N+ ! a) Bruk uttrykket for Entropen og Stirllings formel (se forrige oppgave) ˚ vise at entropien S kan skrives som til a   S = N k A − B(1 + m) log(1 + m) − C(1 − m) log(1 − m) og kartlegg dermed konstantene A, B og C. 4. Bestem følgende integral Z I= 0



e−ax − e−bx xρ+1

der a,b positive konstanter, og ρ < 1. Integralet refferes noen ganger til som et generalisert frullani integral, der det klassiske frullani integralet f˚ as ˚ studere grensetilfellet ρ → 1, som vi skal studere senere. ved a 5. Definer følgende funksjoner Z 1 f (x) := log Γ(x + t) dx 0

Z og

g(x) :=

t

log x dx , 0

vis differansen mellom f og g er høyst en konstant, og kartlegg konstan˚ bestemme integralet ten3 . Bruk dette til a Z 1 log Γ(x + t) dx . 0

˚ derivere høyresiden kan være noe keitete, det anbefales derfor a ˚ bruke A ˚ benytte seg av en luddig substitusjon for deretter a Z h(t) ∂ d d f (x) dx = f (h(t)) h(t) − f (g(t)) g(t) (3.19) ∂t g(t) dx dx 3 Merk

denne metoden bare er derivering under integraltegnet i forkledning. Denne metoden skal granskes nærmere senere.

III

3.3 Betafunksjonen

121

den fundamentale kalkulussetningen i skjønn harmoni med produktregelen. Følgende integral   Z u+1 Γ(x) log √ dx = u log u − u (3.20) 2π u ˚ bestemme integralet relativt enkelt. kan og benyttes til a Likning (3.20) betegnes ofte som Raabes formel [? ]. Bevis Raabes formel uten bruk av derivasjon.

3.3.3

B E TA F U N K S J O N E N

Integralet som vi studerte i forrige seksjon er ogs˚ a kjent som Euler’s andre integral og i denne seksjonen skal vi se nærmere p˚ a Euler’s første integral, kanskje bedre kjent som betafunksjonen og er definert som følger Definisjon 3.3.5. For alle x, y > 0 s˚ a defineres Z 1 B(x, y) = tx−1 (1 − t)y−1 dt

(3.21)

0

Tilsvarende som for gammaintegralet er det viktig a˚ vite at integralet virkelig er konvergent for alle x, y > 0. Eksempelvis om x < 1 s˚ a vil integranden g˚ a mot uendelig n˚ ar vi nærmer oss origo. Tilsvarende s˚ a vil integranden bl˚ ase opp for t = 1 n˚ ar y < 1. beta- og gammafunksjonen er svært nært knyttet til hverandre og vi har følgende resultat. Theorem 3.3.6. La B(x, y) være definert som Z 1 B(x, y) = tx−1 (1 − t)y−1 dt 0

Da har vi for alle x, y ∈ hvor Re(x), Re(y) > 0 relasjonene B(x, y) =

Γ(x)Γ(y) Γ(x + y) Z ∞

= 0

Z =2

tx−1 dt (1 + t)x+y

(3.22) (3.23)

π/2

(sin θ)2x−1 (cos θ)2y−1 dθ

(3.24)

0

Beviset for likning (3.22) overlates til leser se oppgave (3), da igjen gir ˚ komfortabel med beta og Γdet god trening i b˚ ade bevisføring, algebra, og a funksjonens egenskaper. Senere skal vi bevise relasjonen b˚ ade med laplace˚ vise liktransformasjoner og kompleks analyse. Men for n˚ a velger vi heller a ning (3.23) og (3.24). Bevis. Tar utganspunkt i definisjonen Z 1 B(a, b) = ta−1 (1 − t)b−1 dt 0

(3.25)

122

III

3 Spesialfunksjoner

Vi benyttes oss av subtitusjonen x 7→ t/(1 − t) som gir t ∈ [0, ∞), slik at a−1  b−1 Z ∞ Z ∞ x xa−1 1 dx B(a, b) = = dx 1+x 1+x (x − 1)2 (1 + x)a+b 0 0 ˚ vise likning (3.24) bruker vi substitusjonen dette beviser likning (3.23). For a t 7→ (sin θ)2 p˚ a likning (3.25) og f˚ ar følgende uttrykk Z 1 B(x, y) = tx−1 (1 − t)y−1 dt 0 π/2

Z

sin2 θ

=

x−1

1 − sin2 θ

y−1

2 cos θ sin θ dθ

0

Z

π/2

=2

(sin θ)2x−1 (cos θ)2y−1 dθ

0

som fullfører beviset. ˚ vise at Eksempel 3.3.2. Vi ønsker a   Z ∞ α−1 1 u du π/β M (α) := = , β β 1+u 1+u sin (πη) 0

(3.26)

hvor β, α > 1, er reelle konstanter og η = α/β. Bevis. Vi betegner integralet som I og benytter oss av substitusjonen x 7→ 1 + uβ ⇒ u = (x − 1)1/β slik at Z ∞ α−1 Z ∞ u 1 (x − 1)(α−1)/β du = (x − 1)(1/β)−1 dx I= β 1 + u β x 0 1 Settes n˚ a x 7→ 1/v f˚ as   Z 1 1 α α 1 v −α/β (1 − v)−1+α/β dv = B 1 − , β 0 β β β Ved a˚ n˚ a benytte seg av definisjonen til betafunksjonen, og Eulers refleksjonsformel f˚ ar vi Z ∞ α−1 1 Γ (1 − α/β) Γ (α/β) π/β u du = = β 1 + u β Γ (1) sin (πη) 0 ˚ vise. som var det vi ønsket a Resultatet kan og vises ved a˚ ta utgangspunkt i likning (3.23), via substitusjonen t 7→ xβ . Som gir Z ∞ Z ∞ ta−1 taβ−1 B(a, b) = dt = β dt (1 + t)a+b (1 + xβ )a+b 0 0 ˚ dele likningen p˚ og resultatet følger ved a a β og sette b + a = 1, a · β = α s˚ a   Z ∞ α−1 x 1 α α π/β dx = B ,1 − = β 1 + x β β β sin (πα/β) 0 tilsvarende som før4 . 4 Merk

at betafunksjonen er en symmetrisk funksjon, B(x, y) = B(y, x).

III

3.3 Betafunksjonen

123

Korollar 3.3.3. Gitt at n ∈ R s˚ a er Z Z ∞ Z ∞ n−2 1 ∞ 1 + xn−2 dx x π/n dx = = dx = n n 2 0 1 + xn 1 + x 1 + x sin π/n 0 0

(3.27)

Beviset uttelates, men følger direkte fra likning (3.23) og lemma (2.6.1). En svak generalisering av likning (3.26) er følgende Z



π b2 + uα−1 du = a2(−1+1/β) a2 + uβ β

0



b2 a2(1−α)/β + sin(π/β) sin(πα/β)

 (3.28)

hvor η = α/β, men dette overlates til leser. Dette resultatet gir blant annet ∞

Z 0

1 + uα−1 π du = 1 + uβ β



1 1 + sin(π/β) (sin πη)

 (3.29)

˚ sette a = b = 1 i (3.28). Leser kan selv sjekke at likning (3.29) reduseres ved a til (3.27) i tillfellet hvor α − 1 = β − 1. Integraler p˚ a denne formen har blitt studert i forrige seksjon, og vil igjen bli studert i delen med kompleks analyse. Oppgaver 1. Bevis korollar (3.3.3) 1 2

Z 0



1 + xn−2 dx = 1 + xn

Z



0

dx = 1 + xn

Z



0

xn−2 π/n dx = , n 1+x sin π/n

˚ kun bruke likning (3.26). Unng˚ ˚ bruke lemma (2.6.1). ved a a alts˚ aa Z



2. Bestem følgende integral −∞

enx dx n˚ ar n ∈ [0, 1]. 1 + ex

3. Vi skal i denne oppgaven bevise relasjonen mellom beta- og gammafunksjonen B(x, y) :=

Γ(x)Γ(y) Γ(x + y)

og til dette benyttes følgende funksjon φ(x) :=

B(x, y)Γ(x + y) . Γ(y)

Vis at φ(x) oppfyller følgende egenskaper

1)

φ(1) = 1

2)

φ(x + 1) = x · φ(x)

124

III

3 Spesialfunksjoner 3)

log φ(x) er konveks

˚ fullføre beviset. og benytt Bohr-Mullerup teoremet til a

˚ bevise av multiplikasjons theoremet bedre 4. I denne oppgaven ønsker vi a kjent som Legendre’s dobbel formel (Legendre’s duplication formula”).   √ 1 Γ(z)Γ z + = 21−2z z Γ(2z) . 2 Vis først at   1 B(a, a) = 21−2a B a, 2 ˚ bevise Legendre’s dobbel formel. Vis og at identiteten og bruk dette til a kan skrives som   π (2n)! 1 = 2n . (3.30) Γ n+ 2 2 n! dersom n ∈ N. 5. Wallis integralene er nesten like kjent som Wallis produktet og er definert som Z Wn = 0

π/2

(sin x)n dx =

Z

π/2

(sin x)n dx .

0

Vis at integralene er like og bestem verdien til til Wn for odde og like n. Du kan fritt benytte deg av Legendre’s form fra forrige oppgave.

III

3.4 Digamma-funksjonen

3.3.4

125

DIGAMMA-FUNKSJONEN

I denne delen skal vi se nærmere p˚ a digammafunksjonen (psi) Definisjon 3.3.6. Digammafunksjonen er definert som følger ψ(z) =

Γ0 (z) d = log Γ(z) Γ(z) dz

ogs˚ a kjent som den logaritmisk-deriverte av gammafunksjonen. Noen ganger betegnes ogs˚ a funksjonen som ψ0 , som vi skal senere. For n˚ a beholdes den originale notasjonen for enkelhetens skyld. Et viktig resultat er følgende ψ(1 − z) − ψ(z) = π cot(πz)

(3.31)

Bevis. Resultatet ligner veldig p˚ a Eulers refleksjonsformel. S˚ a en naturlig start ˚ derivere formelen, vi har da er a d d π Γ(x)Γ(1 − x) = dx dx sin(πx) 1 cos(πz) Γ0 (x)Γ(1 − x) − Γ(x)Γ0 (1 − x) = −π 2 sin(πz) sin(πz) 1 ψ(z)Γ(z)Γ(1 − z) − Γ(z)ψ(1 − z)Γ(1 − z) = −π 2 cot(πz) sin(πz)     π Γ(z)Γ(1 − z) ψ(1 − z) − ψ(z) = π cot(πz) sin(πz) ψ(1 − z) − ψ(z) = π cot(πz) ˚ vise. Her ble det brukt at Γ0 (x) = ψ(x)Γ(x), Eulers som var det vi ønsket a refleksjonsformel, og noen velkjente trigonometriske identiter. Enda en viktig identitet er følgende ψ(1 + z) − ψ(z) =

1 z

(3.32)

Bevis. Vi starter med følgende likning Γ(1 + z) = z. Γ(z)

(3.33)

Dette følger selvsagt direkte fra at per definisjon s˚ a er Γ(z + 1) = zΓ(z). Derivasjon av begge sider gir  Γ(1 + z) ψ(1 + z) − ψ(z) = 1 Γ(z) ˚ bruke likning (3.33) ogs˚ og beviset fullføres ved a a dele begge sider p˚ a z. Her ble det ikke brukt noe mer enn kvotientregelen og definisjonen av digammafunksjonen. Vi tar et raskt lite eksempel for a˚ vise hvor kraftig disse enkle relasjonene kan være

126

III

3 Spesialfunksjoner

Eksempel 3.3.3. Vis at Z 0



log x (1 +

2 x2 )

dx = −

π 4

˚ definere følgende integral Bevis. Vi begynner med a Z ∞ xa I(a) := 2 dx (1 + x2 ) 0 ˚ bestemme er I 0 (0). Hvorfor vi Der vi legger merke til at integralet vi ønsker a ˚ bytte om integral og derivasjonstegnet kommer senere, men for n˚ har lov til a a holder det a˚ si at dette er lovlig da funksjonen er begrenset. Utregningen av I(a) g˚ ar som følger Z ∞ xa I(a) = 2 dx (1 + x2 ) 0   Z 1 ∞ a+1 tβ 2 = dx t=x ,β= 2 0 (1 + t)2 2   1 a+1 a+1 = B ,2 − 2 2 2     a+1 a+1 1 Γ 2− = Γ 2 2 2 Dersom vi n˚ a deriverer I(a) med hensyn p˚ a a f˚ as         1 a+1 a+1 a+1 a+1 I 0 (a) = Γ Γ 2− ψ −ψ 2− 4 2 2 2 2 settes n˚ a a = 0 f˚ as         1 1 3 1 3 1 π π 0 I (0) = Γ Γ ψ −ψ = (−2) = − 4 2 2 2 2 4 2 4 og vi er ferdige. Under blir noen av detaljene i beviset gjennomg˚ att  Γ

1 2

    2 1 1 1 1 √ 2 π Γ 1+ = Γ = π = , 2 2 2 2 2

hvor Γ(x + 1) = xΓ(x) ble benyttet. Tilslutt s˚ a er          1 3 1 1 1 ψ −ψ =− ψ 1+ −ψ = − −1 = −2 , 2 2 2 2 2 hvor likning (3.32) benyttet. La oss vise en annen representasjon av ψ(x), nemlig ∞

ψ(z) = −γ −

X 1 1 1 + − z n n + z n=1

(3.34)

III

3.4 Digamma-funksjonen

127

Bevis. Snur vi oppmerksomheten v˚ ar mot likning (3.14) og tar logaritmen f˚ ar vi  log  log

1 Γ(z)



1 Γ(z)



γz

= log ze

∞  Y n=1

!  z z 1− exp − n n

= log exp{γz} + log z +

∞ nzo  z X − log exp log 1 + n n n=1 n=1 ∞ X

Brukes n˚ a at log 1/A = − log A og begge sider ganges med −1 kan vi skrive uttrykket p˚ a en enkel deriverbar form ! ∞ h  X z i z −γ · z − log z + − log 1 + n n n=1   ∞ X Γ0 (z) 1 1 1 = ψ(z) = −γ − + − Γ(z) z n 1+z n=1 d d (log Γ(z)) = dx dz

som var det som skulle vises. Overgangen fra sum til logaritme kan virke noe underlig, vi ser p˚ a produktet an /bn og tar logaritmen ∞ Y an K = log b n=1 n

!

 = log

a1 · a2 . . . b1 · b2 · . . .



= (log a1 + log a2 + . . .) − (log b1 + log b2 + . . .) ! ! ∞ ∞ X X = log an − log bn n=1

n=1

hvor tilsvarende utregning kan bli brukt p˚ a produktet i likning (3.2). Fra denne likningen kan vi enkelt finne et par konkrete verdier for digammafunksjonen. ψ(1) = Γ0 (1) = −γ   1 ψ = −γ − 2 log 2 2   1 π 3 ψ = −γ − √ − log 3 3 2 2 3   1 π ψ = −γ − − 3 log 2 4 2 Disse kan utledes ved a˚ studere likning (3.34) nøyere. Første verdi faller direkte ut da ∞ X 1 1 ψ(1) = −γ − 1 + − = −γ n n + 1 n=1

128

3 Spesialfunksjoner

III

Siden rekken er teleskoperende s˚ a har en at k X 1 1 − n n+1 n=1         1 1 1 1 1 1 1 = 1− + − + ··· + − + − 2 2 3 k−1 k k k+1 1 =1− . k+1 Slik at n˚ ar k → ∞ s˚ a vil S → 1. Bruksomr˚ adene til digammafunksjonen og ulike integralrepresentasjoner vil komme i form av sm˚ a drypp fremmover n˚ ar ˚ skrive opp vi utforsker ulike integrasjonsteknikker. For n˚ a begrenser vi oss til a noen grunnleggende identiter.

Sk :=

Noen viktige identiter ∞ X 1 1 − k z+k k=1 Z 1 1 − tz ψ(z + 1) = −γ + dt 1−t 0 Z ∞ −t e e−zt ψ(z) = − dt t 1 − e−t 0 ∞ X 1 ψ 0 (z) = (n + z)2 n=0

ψ(z + 1) = −γ +

ψ 0 (1) =

π2 6

Oppgaver 1. Vis følgende ∂ log B(x, y) = ψ(x) − ψ(x + y) ∂x 2. Vis følgende  ψ

1 2

 = −2 log 2 − γ

˚ beregne integralet og bruk resultatet til a Z 1 √ t −1 √ I= dt . t (1 − t) 0 3. Bestemm integralet Z I(m) =

1

ψ(x) sin 2λπx dx 0

hvor λ ∈ Z. Hva skjer n˚ ar λ → ∞? Strider dette mot Rieman-Lesgue lemmaet, hvorfor/hvorfor ikke?

III

3.6 Riemann zeta funksjonen

3.3.5

129

P O LY G A M M A - F U N K S J O N E N

Denne seksjonen blir noe kort, da polygamma-funksjonen ikke er like nyttig som digamma funksjonen. Funksjonen er defiert som følger dm dm+1 ψ(x) = log Γ(z) . dz m dz m+1 alts˚ a er ψ0 (z) = ψ(z) fra forrige seksjon. De fleste identitetene som holdt for digammafunksjonen holder og for polygammafunksjonen, eksempelvis ψm (z) =

ψm (z + 1) − ψm (z) = (−1)m

m! z m−1

som kan vises via induksjon da ψ(z + 1) − ψ(z) = 1/x, som vi har vist før.For trigammafunksjonen ψ1 f˚ ar vi eksempelvis 1 z2 Videre s˚ a gjelder fortsatt refleksjonsformelen slik at ψ1 (z + 1) − ψ1 (z) = −

(−1)m ψm (1 − z) − ψm (z) = π

dm cot(πz) dz m

hvor vi f˚ ar igjen eksempelvis har at for ψ1 s˚ a er  2 π ψ1 (1 − z) + ψ1 (z) = sin(πz) Dersom vi ser bort i fra ψ(x) s˚ a kan vi for alle m > 0 skrive polygammafunksjonen som følgende sum ψm (z) = (−1)m+1 m! ·

∞ X k=0

1 = (−1)m+1 m! · ζ(m + 1, z) (z + k)m+1

Der ζ(s, z) er Hurwitz zeta funksjonen som vi snart skal se p˚ a.

3.3.6

RIEMANN

Z E TA F U N K S J O N E N

Riemann zeta funksjonen er en av de viktigste og mest fascienerende funksjonene innen moderne matematikk. M˚ aten funksjonen oppst˚ ar p˚ a er veldig naturlig da den dreier seg om potenserekker av naturlige tall ∞ X 1 , n2 n=1

∞ X 1 , n3 n=1

∞ X 1 , usw. n4 n=1

Definisjon 3.3.7. Riemann Zeta funksjonen er definert som ζ(s) =

∞ X 1 ns n=1

for Re(s) > 1.

Her er s et komplekst tall med real del større enn 1. Men vi skal bare se p˚ a tilfellet der s er reell. Som vanlig var det Euler som først betraktet funksjo˚ nen, men det var Bernhard Riemann (1826 − 1866) som inns˚ a viktigheten av a bektrakte s som kompleks. Det var Euler som kartla verdien av ζ(2n) hvor n er et naturlig tall. Men før vi ser p˚ a det trengs en kort introduksjon av Bernoulli tallene.

130

III

3 Spesialfunksjoner

˚ bestemme Bernoulli tallene Jacob Bernoulli (1654 − 1705) var opptatt med a en formel for Sk (n) = 1k + 2k + · · · + (n − 1)k ˚ finne et hvor k ∈ N og n ∈ N\{1}. Ved lave verdier er det relativt enkelt a uttrykk for summen n(n − 1) 2

S2 (n) =

som kjent fra før. Utrolig nok fant bernoulli ut at  k  1 X k+1 Sk (n) = Bj nk+1−j . 1 + k j=0 j der Bj st˚ ar for det j’te Bernoulli tallet. Tallene kan bli beskrevet rekursivt som  k  X k+1 Bj = 0 , k ≥ 1 j j=0 med initalverdi B0 = 1. P˚ a magisk vis oppst˚ ar og Bernoulli tallene i taylorutviklingen av visse funsjoner. Eksempelvis ∞ X z 1 z 1 z6 1 z4 zn B = 1 − · + · + · = n ez − 1 1! 2 2! 6 4! −30 n! n=0

i tabbel 1 er de første bernoulli-tallene oppført. n

0

1

2

3

4

5

6

7

8

9

10

11

12

Bn

1

− 12

1 6

0

1 − 30

0

1 42

0

1 − 30

0

5 66

0

691 2730

Proposisjon 3.3.10. La ζ(z) betegne Riemann zeta funksjonen da er Z



ζ(z)Γ(z) = 0

tz−1 dt , et − 1

hvor Re(z) > 1. Bevis. Vi har fra den geometriske rekken ∞ X

rk =

k=0

1 1−x

|x| < 1

˚ sette r = e−z f˚ Ved a as ∞ X k=0

e−k·z =

1 ez = z −z 1−e e −1

III

3.8 Polylogaritmen

131

˚ bruke dette resultatet kan integralet n˚ Ved a a beregnes Z ∞ ∞ X 1 tz−1 e−t dt ζ(z)Γ(z) = kz 0 k=1 z−1 ∞ Z ∞ X dt t = e−t k k 0 k=1

˚ bruke substitusjonen u = t/n f˚ Ved a ar vi n˚ a = =

∞ Z X



(u)

k=1 0 ∞ Z ∞ X k=0

Z

z−1 −ku

e

uz−1 e−u e−ku du

0 ∞ X



u

=

z−1 −u

e

0

Z

! e

−ku

du

k=0 ∞

uz−1 e−u

=



0

Z

du



= 0

1 1 − e−u

 du

uz−1 du eu − 1

˚ vise. som var det vi ønsket a

3.3.7

HURWITZ

3.3.8

P O LY L O G A R I T M E N

Z E TA F U N C T I O N

Som vi har sett tidligere s˚ a er Rieman-zet funksjonen svært viktig innen matematikk. I denne delen studerer vi en lignende funksjon. Polylogaritmen er ogs˚ a kjent som Jonqui`ere’s function og betegnes som Lis (z), hvor s er graden. Li2 (z) betegnes som dilogaritmen, Li3 (z) som trilogaritmen usw. Per definisjon s˚ a er Definisjon 3.3.8. Polylogaritmen er definert som følger Li( z) =

∞ X zk z2 z3 = z + + ks 2s 3s

k=1

Fra definisjonen ser vi at Lin (1) =

X k

=

1 = ζ(n) kn

P∞ hvor notasjonen k er kortformen av k=1 . Dette viser sammenhengen mellom polylogaritmen og zetafunksjonen. For dilogaritmen n = 2 har en P

Li2 (1) = ζ(2) =

π2 6

132

3 Spesialfunksjoner

III

som vi kommer mer tilbake til senere. Legg merke til at for n = 1 s˚ a har en Li1 (z) =

X zk = − log(1 − z) k k

som vi gjennkjente som maclaurinrekken til log(1 − z) mer generelt s˚ a har vi følgende theorem. Theorem 3.3.7. for n ∈ N s˚ a kan polylogaritmen defineres rekursivt som Z z Lin (t) Lin+1 (z) = dt . t 0 Bevis. Vi tar utgangspunkt i høyresiden og bruker definisjon (3.3.8) s˚ a ! Z z Z z Lin (t) 1 X tk dt dt = t kn 0 0 t k X 1 Z z = tk−1 dt kn 0 k X 1  tk z = kn k 0 k

=

X k

zk k n+1

= Lin+1 (z)

Derivasjon av theorem (3.3.7) gir n˚ a z

∂ Lin+1 (z) = Lin (z) ∂z

som er en nyttig identitet.

3.3.9

DILOGARITMEN

Av alle polylogaritmene er nok kanskje Li2 (z) den mest nyttige, noen ganger er funksjonen og kjent som Spence’s funksjon etter den skotske matematikkeren William Guthrie Spence (1846–1926), som gjorde en systematisk studie av funksjonen p˚ a begynnelsen av 1900-tallet. Vi tar utgangspunkt i følgende definisjon for dilogarithmen Definisjon 3.3.9. Dilogaritmen er definert som følger Z z X zk log(1 − t) Li2 (z) = =− dt 2 k t 0 k

for alle z < 1.

III

3.9 Dilogaritmen

133

Den første identiteten vi skal vise er tillegn Abel, og refferes noen ganger som Pentagon Identiteten. Theorem 3.3.8. (Abels identitet) Gitt at x, y 6=∈ [1, ∞) da er log(1 − x) log(1 − y) = Li2 (u) + Li2 (v) − Li2 (uv) − Li2 (x) − Li2 (y)

(3.35)

hvor u = x/(1 − y) og v = x/(1 − y). ˚ først derivere høyre og venstre side av identiteten og Beviset g˚ ar ut p˚ aa vise at disse er like. Integrasjon gir da at høyre og venstre side høyst kan være ˚ vise at konstanten er null. forskjellige ved en konstant. Beviset fullføres ved a ˚ fullføre omskrivningen av likning (3.35) f˚ Ved a as     u(1 − v) v(1 − u) Li2 (u) + Li2 (v) = Li2 (uv) + Li2 + Li2 1 − uv 1 − uv     1−v 1−u log + log 1 − uv 1 − uv

(3.36)

som ofte reffereres til som pentagon identiteten. Ved hjelp av Abels theorem og vanlig derivasjon kan en vise at dilogaritmen tilfredstiller en rekke funksjonallikninger Proposisjon 3.3.11. text π2 Li2 (z) + Li2 (1 − z) = − log z log(1 − z) (Refleksjonsformelen) 6   1 π2 1 − log2 (−z) (Inversjonsformelen) Li2 (z) + Li2 =− z 6 2   1 z Li2 (−z) + Li2 = − log2 (z + 1) (Landen’s Identitet) 1+z 2 1 Li2 (z) + Li2 (−z) = Li2 (z 2 ) (Kvadrat identiteten) 2

(3.37) (3.38) (3.39) (3.40)

˚ vise første og andre identitet. Setter vi x = z og y = 1 − z i Bevis. Velger her a likning (3.35) f˚ as log(1 − z) log(z) = Li2 (1) − Li2 (z) − Li2 (1 − z) Li2 (z) − Li2 (1 − z) =

π2 − log z log(1 − z) 6

der det ble benyttet at Li2 (1) = ζ(2) = π 2 /6, dette viser Eulers refleksjonsformel.

˚ vise Inversjonsformelen. Ved a ˚ derivere Li2 (−1/z) f˚ Bevis. Ønsker s˚ aa as   d 1 1 Li2 − = 2 dz z z

log 1 + z1 − −1/z

! =

log(1 + z) − log z z

134

3 Spesialfunksjoner

III

Integrasjon med hensyn p˚ a z gir n˚ a at   Z z 1 log(1 + t) log z Li2 − = − dt z t z 0 Z −z log(1 − t) 1 = dt − log2 (z) + C t 2 0 1 = Li2 (−z) − log2 (z) + C 2 ˚ bestemme konstanten settes z = 1 s˚ for a a C = 2 Li2 (−1). Men vi har at Li2 (−1) = −

∞ X π2 (−1)n = − n2 12 n=1

setter vi n˚ a inn dette og lar z = −z f˚ ar vi   1 π2 1 Li2 (z) + Li2 =− − log2 (−z) , z 6 2 som ønsket. Ved samme metode kan Landens Identitet bestemmes, ta utgangspunkt i ˚ vise dette og kvadrat identiten overlates til Li2 [z/(z − 1)] og derivere, men a leser. Selv om digammafunksjonen tilfredstiller en rekke funksjonallikninger er det dessverre bare en h˚ andfull verdier som kan regnes ut eksplisitt. Disse er √ 1± 5 0, ±1 , ±1 ± 2 √ hvor vi gjennkjenner sistnevnte som det gylne snittet. Merk at ϕ = (1 + 5 )/2 ≈ 1.6180 . . . > 1 slik at Li2 (ϕ) divergerer. Derimot s˚ a konvergerer den konjugerte Ψ = 1 − ϕ som vi skal se i det p˚ afølgende eksempelet. Eksempel 3.3.4. Vis at √  √  5 −1 π2 5 −1 2 Li2 = − log 2 10 2 √    √ 2 π 5 −1 3− 5 2 Li2 = − log 2 15 2 √   √  2 1− 5 π 1 5 −1 Li2 =− + log2 2 15 2 2 Bevis. La oss for enkelhetensskyld definere √ 1− 5 α= , 2 da kan vi enten se eller regne ut at α er en løsning av x2√ + x = 1. Bedre kjent som den konjugerte av det gylne snittet, heldigvis s˚ a er 5 > 1 s˚ a α < 1, og integralene konvergerer. Videre s˚ a er √  √    1− 5 3− 5 = −α og =1−α 2 2

III

3.11 Elliptiske Integral

135

˚ sette x = y = 1 − α inn i Abels Identitet (3.35) f˚ Ved a as n˚ a 2 Li2 (α) − 3 Li2 (1 − α) = log2 α

(3.41)

der det ble benyttet at α2 = 1 − α. Via refleksjonsformelen fra likning (3.37) med z = α f˚ as Li2 (α) + Li(1 − α) =

π2 π2 log α log(1 − α) = − 2 log2 α 6 6

(3.42)

˚ bruke kvadrat identiteten fra likning (3.40) f˚ Og sist men ikke minst ved a as Li2 (α) + Li2 (−α) −

1 Li2 (1 − α) 2

(3.43)

˚ betegne hvor det ble brukt igjen at α2 = 1 − α. Ved a A = Li2 (α) ,

B = Li2 (1 − α) ,

C = Li2 (−α)

gir likningene (3.41) til (3.43) følgende likningssett 2A − 3B = log2 α A + A +

π2 − 2 log2 α 6 1 B C = 2

B =

løser en dette settet for A, B, C f˚ as  √ π2 5 −1 A = Li2 = − log2 α 2 10 √   3− 5 π2 B = Li2 = − log2 α 2 15 √   π2 1 1− 5 C = Li2 =− + log2 α 2 15 2 ˚ vise. som var det vi ønsket a 1. Vis Abels identitet via derivasjon log(1 − x) log(1 − y) = Li2 (u) + Li2 (v) − Li2 (uv) − Li2 (x) − Li2 (y) hvor u = x/(1 − y) og v = x/(1 − y). 2. Vis Kvadrat identiten og Landen’s identitet

3.3.10

DILOGARITMEN

136

III

3 Spesialfunksjoner

3.3.11

ELLIPTISKE INTEGRAL

P˚ a begynnelsen av 1900-tallet var studiet av kurver og buelengden av disse et av de sentrale omr˚ adene som ble studert. Sinus og cosinus kunne brukes til ˚ beregne buelengden av sirkler, og buelengden av hyperbolske kurvene kunne a beregnes tilsvarende med de hyperbolske trigonometriske funksjonene (sinh og ˚ beregne buelengden av ellitptiske cosh). elliptiske funksjonene ble innført for a kurver, og fullførte dermed treenigheten. Studiet av disse funksjonene var lenge svært sentralt og ble studert allerede av Gauss, som vi skal se senere. Innen moderne matematikk defineres et elliptisk integral som alle funksjoner f som kan skrives p˚ a formen Z x  p  f (x) := R t, P (t) dt c

hvor R er en rasjonell funksjon, og P er et polynom av enten grad 3 eller 4, og c er konstant. Her skal vi bare studere komplette elliptiske integral og disse skrives som f (π/2) med c =. Det er som regel tre integral som forbinnes med elliptiske funksjoner, disse er Definisjon 3.3.10. Det første komplette elliptiske integralet er definert som π/2

Z K(k) := 0

Z

dt p

1−

k2

2

sin (t)

1

= 0

dt p

(1 −

t2 )(1

− k 2 t2 )

Definisjon 3.3.11. Det andre komplette elliptiske integralet er definert som Z π/2 p Z 1√ 1 − k 2 t2 2 2 √ E(k) := 1 − k sin t dt = dt 1 − t2 0 0 Definisjon 3.3.12. Det tredje komplette elliptiske integralet er definert som π/2

Z 1 dt dx p p K(k) := = 2 2 2 2 2 2 2 2 (1 + n sin t) 1 − k sin (t) 0 0 (1 − n x ) (1 − x )(1 − k x ) √ Hvor k kalles modulusen og k 0 = 1 − k 2 kalles den komplimentære modulusen. Slik at k 2 + (k 0 )2 = 1. Generelt sett s˚ a kan ikke de elliptiske integralene utregnes eksplisitt i form av mer kjente funksjoner, men la oss vise et lite knippe Z

Eksempel 3.3.5. √ K

 1 −1 = √ Γ2 4 2π



1 4



˚ sette inn og bruke substitusjonen y 7→ x4 f˚ Bevis. Ved a as √

 −1 =

Z

1

Z dx 1 Γ(1/4)Γ(1/2) = y −3/4 (1 − y)−1/2 dy = 4 4 4Γ(3/4) 1 − x 0 √ ˚ bruke eulers-refleksjonsformel med herfra s˚ a er Γ(1/4)Γ(3/4) = π 2 , √ ved a a = 1/4. Tilsvarende s˚ a er Γ(1/2) = π og dette fullfører beviset. K



III

3.11 Elliptiske Integral

137

Vi kan vise et par tilsvarende former for de elliptiske integralene, men først trengs følgende lemma. Lemma 3.3.4. La f være en funksjon med odde periode a. Da er Z



0

f (x) π dx = x a

a/2

Z

f (x) cot

 πx  a

0

dx

(3.44)

˚ dele integralet inn i perioder p˚ Bevis. Vi begynner beviset med a a a alts˚ a Z ∞ Z a Z 2a Z 3a = + + +··· 0

a

0

2a

˚ gjøre dette kan integralet skrives som ved a Z 0





X f (x) dx = x

Z

k=0

a(k+1)

ak



X f (x) dx = x

k=0

a

Z 0

f (u + ak) du . u + ak

Der substitusjonen x 7→ u + ak ble brukt i siste overgang. Siden f har en periode p˚ a a s˚ a er f (x + ak) = f (x) for alle k ∈ Z. Vi deler atter en gang opp integralet slik at  Z a Z a ∞ hZ a ∞  X X f (x) dx i 1 1 f (x) dx + = f (x) − dx x + ak x + ka (k + 1)a − x a/2 x + ak 0 0 k=0

k=0

(3.45) Der substitusjonen x 7→ u − a/2 ble brukt i siste overgang. Ved litt smart algebramagi s˚ a har vi at summen kan skrives som ∞ X k=0





X X 1 1 1 1 − = + x + ka (k + 1)a − x x + ka x − ka =

k=0 ∞ X

k=1

1 x + ka

k=−∞ ∞ X

1 k + x/a k=−∞  πx  π = cot a a

=

1 a

(3.46)

Der det ble benyttet at π · cot(πx) = lim

N →∞

N X n=−N



X 1 1 2x = + 2 − n2 x+n x x n=1

Ved n˚ a a˚ sette likning (3.46) inn i likning (3.45) og sette konstantleddet utenfor f˚ as Z ∞ Z ∞ Z  πx   πx  f (x) π π a/2 dx = f (x) cot dx = f (x) cot dx x a a a 0 a 0 0 som var det som skulle vises.

138

III

3 Spesialfunksjoner

Korollar 3.3.4. La f være en like-funksjon med periode a. Da er Z



0

 πx  f (x) π dx = sin x a a

a/2

Z

f (x) dx 0

og spesielt s˚ a er Z 0



f (x) sin x dx = x

π/2

Z

f (x) dx 0

˚ benytte oss av lemma (3.3.4) p˚ Bevis. Ved a a funksjonen f (x) = sin(πx/a) som er odde med periode 2a. Videre s˚ a gir halv-vinkel formlene v˚ are tan

x sin x = 2 1 + cosx

˚ sette inn f˚ ved a ar vi n˚ a Z Z ∞  πx   πx   πx  π a/2 f (x) sin dx = f (x) sin cot dx x a a 0 a a 0 Z h  πx i π a/2 = f (x) 1 + cos dx a 0 a Z π a/2 = f (x) a 0 N˚ a siden cos(πx/a) er en odde funksjon omkring a/4 s˚ a er integralet null. SOMETHING DARK SIDE Som en liten digresjon avslutningsvis tar vi og ser p˚ a to integral som likner p˚ a Elliptiske integral, men kan bestemmes eksplisitt. Eksempel 3.3.6. For alle b ∈ (0, 1) s˚ a er Z

π/2

0

Z 0

π/2

cos t dt arcsin k p = 2 k 1 − k 2 sin t   sin t dt 1 1+b p = log 2b 1−b 1 − k 2 sin2 t

Bevis. Vi kaller integralene henholdsvis for F1 (k) og F2 (k). Via substitusjonen x 7→ k sin t s˚ a er Z π/2 Z 1 b cos t dt dx arcsin k p √ F1 (b) = = = 2 2 2 b k 1 − x 0 0 1 − k sin t der substitusjonen x 7→ sin y fullfører beviset. La oss løse F2 (b) noe liknende, ved √ a˚ la k∗ = 1 − k 2 , da er k 2 +k∗2 . Videre s˚ a er 1−k 2 sin2 t = 1−(1−k∗2 ) sin2 t = 2 2 2 2 2 k ∗ +k cos t. Siden 1 − sin x = cos x fra pytagoras. N˚ a er Z F2 (b) = 0

π/2



sin t dt 1 = k b ∗2 +k 2 cos2 t

Z 0

π/2



dx 1 = arcsinh k k ∗2 +x2



k k∗



III

3.11 Elliptiske Integral

139

Der substitusjonen a x 7→ k ∗ y. Per definisjon s˚ a er √x = k sin   t ble brukt, ogs˚ arcsinh = log 1 + 1 + u2 , via litt algebra s˚ a er n˚ a 1 + k 2 /k∗2 = 1/b∗2 slik at F2 (b) =

1 1 1+k 1 1+k (1 − k 2 )1/2 = log = log log b k∗ b b (1 − k 2 )1/2 (1 − k 2 )1/2

˚ bruke at a log b = log ba fullfører n˚ det a a beviset.

140

3.4

4 Transformasjoner

III

TRANSFORMASJONER

En integral transformasjon kan bli sett p˚ a som en type operator som virker p˚ a en funksjon f . Transformasjonen eller avbildningen skrives gjerne som Z F (x) =

b

K(x, t)f (t) dt a

hvor K(x, t) betegner kernelen. En av de viktigste egenskapene med integral transformasjonen er at det er en lineær operator5 . Definisjon 3.4.1. En lineær transformasjon mellom to vektorrom V og W er en avbildning T : V 7→ W slik at T (αv1 + βv2 ) = αT (v1 ) + βT (v2 ) , holder for alle vektorer v1 og v2 i V og for enhver skalar α ∈ R. Setter vi den formelle notasjonen til side er motivasjonen for slike tranforns˚ forst˚ fjormasjoner lett a a. Det eksisterer mange klasser av problemer som er vanskelig, eller umulig a˚ løse algebraisk - i deres opprinnelige form. En integraltranformasjoner avbilder en likning fra dets opprinnelige omr˚ adet til et annet ˚ behandle. Deromr˚ adet. Likningen i det nye omr˚ adet kan være langt enklere a etter kan en mappe funksjonen tilbake til det opprinnelige omr˚ adet ved a˚ bruke den inverse av integral-tranformasjonen. Dette gjelder integraler og. For en lengre liste over ulike integral-transformasjoner anbefales [? ]. Direkte relatert til integral transformasjoner er konvolusjoner. Dersom F og G er integral transformasjonene til f og g, s˚ a er konvolusjonen f ∗ g lik funksjonen som har transformasjonen F G. Konvolusjonen uttrykkes ofte som et integral med hensyn p˚ a f og g, men ikke nødvendigvis kernelen K(x, t).

3.4.1

LAPLACE

TRANSFORMASJONEN

Laplace transformasjonen er en av de mest brukte integral-transformasjonene innen matematikk og har flere bruksom˚ ader innen fysikk og ingeniørvitenskap. ˚ beregne differensiallikninger, funksjonallikninger, og visse Den blir brukt til a integral. For et historisk perspektiv anbefales artikklene [? ? ] fra Michael A. B. Deakin varmt. Definisjon 3.4.2. La F (t) være en funksjon av t. Laplace Transformasjonen av F er da definert som L Z ∞  L f (t) = F (s) = e−st f (t) dt 0

˚ vise at vi tar laplace transformasjonen Noen ganger skrives ogs˚ a Lf (s) for a av funksjonen f . Ellers brukes og den tosidide laplace-transormasjonen Z ∞  L 2 F (t) = e−st F (t) dt −∞ 5 Kjært

barn har mange navn. Alternative navn er en lineær transformasjon, lineær mapping, lineær funksjon osv.

III

4.1 Laplace transformasjonen

141

Den inverse L -transformasjonen betegnes som L −1 (f (t) eller Lf−1 (s), og kan skrives som et kontur-integral Z γ+iT 1 lim est F (s) ds , f (t) = L−1 {F }(t) = L−1 {F (s)}(t) = s 2πi T →∞ γ−iT ˚ bruke tabell (3.1) til a ˚ bestemme inversene. men det er ofte langt enklere a Proposisjon 3.4.1. Transformasjonen L er en lineær transformasjon Dette følger fra at selve integralet danner en lineær-transformasjon K(f ) = Bevis. R f , men dette glemmer vi for n˚ a. Direkte innsetning gir Z ∞  L αf (t) + βg(t) = (αf (t) + βg(t))e−st f (t) dt 0 Z ∞ Z ∞ = αf (t)e−st dt + βg(t)e−st dt 0 0 Z ∞ Z ∞ −st =α f (t)e dt + β g(t)e−st dt 0 0   = αL f (t) +βL f (t) som ønsket. Vi har allerede sett p˚ a laplace-transformasjonen av F (t, x) = ts−1 , da dette nesten er Γ-funksjonen. En rekke slike integral er vist i tabell (3.1). merk at Tabell 3.1: Et utvalg L -transformasjoner av elementære funksjoner. f (t)

Lf (s)

Betingelser

1 ta eat cos ωt sin ωt cosh ωt sinh ωt σ(t − a) = e−as

1/s Γ(a + 1)/sa+1

Re(a) > −1

1 s−a s s2 +ω 2 ω s2 +ω 2 s s2 −ω 2 ω s2 −ω 2



ω∈R s > |Im(ω)| s > |Re(ω)| s > |Im(ω)|

1/s om ecs /s om

H[c](t)

c≤0 c>0

transformasjonen av de trigonometriske integralene er vist i lemma (3.8.2) og ˚ utlede a ˚ utelates proposisjon (2.7.1). Uttrykkene i tabellen er ikke vanskelig a derfor. I tabellen ble to nye funksjonen innført Heavyside funksjonen Hc og Dirac delta funksjonen δ, disse er definert som følger Definisjon 3.4.3. Dirac delta funksjonen δ er definert som  +∞ dersom x = 0 σ(x) = 0 ellers og tilfredstiller Z



δ(x) dx = 1 −∞

142

III

4 Transformasjoner

Definisjon 3.4.4. Heavyside funksjonen er definert som  0 dersom n < 0 H[n] = 1 dersom n ≥ 0 ˚ studere egenskapene Laplace Transformasjonen har, Mer spennende er det a enn spesfifikke verdier. tabell (3.2) viser et utvalg egenskaper laplace transformasjonen har. Vi skal se nærmere p˚ a et par av disse egenskapene. Tabell 3.2: Noen av de mest kjente egenskapene til L -transformasjonen. tdomenet

sdomenet

af (t) + bf (t) tf (t) tn f (t) f 0 (t) f 00 (t) f (n) f (t)/t Rt f (τ ) dτ 0 f (a)t eat f (t) F (t − a)H(t − a)

aF (s) + bG(s) −F 0 (s) (−1)n F (n) (s) sF (s) − f (0) s2 F (s) − sf − f 0 (0) P(0) n n sR F (s) − k=1 sk−1 f (n−k) (0) ∞ F (σ) dσ s F (s)/s  s 1 |a| F a F (s − a) e−as f (s)

Proposisjon 3.4.2. G(s) = L



f (t) t



Z



F (σ) dσ

= s

 Hvor L f (t) = F (σ). a Bevis. Vi begynner med a˚ sette inn definisjonen av Laplace-transformasjonen p˚ høyre side  Z ∞ Z ∞ Z ∞ Z ∞Z ∞ F (σ) dσ = f (t)e−σt dt dσ = f (t)e−σt dσdt s

s

0

0

s

I siste overgang byttet vi om integrasjonsrekkefølgen. Dette følger fra fubinis sats siden vi krever at |f (t)e−σt | skal være begrenset. Siden f (t) ikke er avhengig av σ kan vi sette ledddet utenfor integrasjonen   Z ∞ Z ∞ Z ∞ Z ∞ e−st f (t) F (σ) dσ = f (t) e−σt dσdt = f (t) dt = L t t s 0 s 0 som var det som skulle vises. Korollar 3.4.1. Z 0



f (t) = t

Z



F (σ) dσ 0

˚ sette s = 0 i proposisjon (3.4.2). Korolaret faller ut direkte ved a

III

4.1 Laplace transformasjonen

143

Eksempel 3.4.1. Fra korolaret følger Dirichlet integralet nesten direkte. Z ∞ sin ω =π ω −∞ ˚ Siden integranden er symmetrisk trenger vi bare se p˚ a omr˚ adet ω > 0. Ved a bruke korollar (3.4.1) med f (t) = sin t har vi Z ∞ Z ∞ Z ∞ f (ω) 1 ds dω = L (sin t) ds = 2 ω s + 12 0 0 0 Hvor laplace transformasjonen av sin x ble tatt fra tabell (3.1) mens utledningen finnes i lemma (3.8.2) og proposisjon (2.7.1). Alts˚ a har vi  ∞ Z ∞ Z sin ω 1 1 1 ∞ ds = = arctan s =π ω 2 0 s2 + 12 2 −∞ 0 Eksempel 3.4.2. Bestem den inverse L -transformasjonen av funksjonene F (s) =

1 s(1 + s)(1 + s2 )

via standard delbrøksoppspalting kan F skrives som F (s) =

1 1 1 1 1+s + − 2 1+s s 2 1 + s2

R∞ Vi kan n˚ a sammenlikne med tabellen og ser at L (1) = 1/s, siden 0 e−sx dx = 1/s. Dette betyr at L −1 (1/s) = 1. Videre kan det siste uttrykket deles opp i to deler       1 s 1+s −1 −1 −1 =L +L = cos t + sin t L 1 + s2 1 + s2 1 + s2 Som følger fra at b˚ ade L og L -transformasjonen lineære transformasjoner. For det siste leddet kan vi bruke faseforskyvnings egenskapene L (eat f (t)) = F (s − a). Vi har at F (s) = 1/s, og F (s + 1) = 1/(1 + s) slik at     1 1 −1 −s −1 L =e L = e−s 1+s s oppsumert har en alts˚ a  e−t cos t + sin t L −1 F (s) = 1 − − 2 2 som var det som skulle bestemmes. ˚ forenkle eller løse differenHovedsaklig blir L -transformasjoner brukt til a ˚ evaluere visse integral. Under er et siallikninger. Men de kan og brukes for a ˚ beregne L -transformasjonen av integralet, enn eksempel hvor det er enklere a integralet i seg selv. Eksempel 3.4.3. Z 0



cos xt π dt = e− |x| 1 + t2 2

144

III

4 Transformasjoner

˚ ta den Bevis. Vi kaller integralet for I og beregner i stedet først L (I), for s˚ aa inversee L -transformasjonen. Dette gir Z ∞ Z ∞Z ∞ cos(xt) −sx −sx e L f (s) = f (x)e dx = dt dx 1 + t2 0 0 0 ˚ bytte om grensene. At integralet fortsatt tar samme verdi Det neste steget blir a følger fra fubinis sats siden absoluttverdien av integralet konvergerer6 Z ∞ Z ∞ Z ∞ 1 1 s −sx = cos(xt)e dx dt = dt 2 2 s2 + t2 1 + t 1 + t 0 0 0 Bruker vi delbrøksoppspalting p˚ a den gjennst˚ aende integranden f˚ as     Z ∞ 1 1 s π 1 π 1 s − dt = −1 = . = 2 2 2 2 2 1−s 0 s +t 1+t 1−s 2 s 2 1+s Vi har n˚ a alts˚ a beregnet L -transformasjonen av integralet v˚ art Z ∞  cos xt π 1 L dt = 2 1 + t 2 1+s 0 ˚ ta den inverse L -transformasjonen p˚ ved a a begge sider av likningen f˚ as   Z ∞ cos xt π 1 π dt = L −1 = e− |x| 2 1 + t 2 1 + s 2 0 som var det som skulle beregnes. Grunnen til at vi skriver |x| er fordi vi ønsker at svaret skal holde for alle x. Siden cos x er en likefunksjon s˚ a er cos(−xt) = cos(xt), s˚ a vi f˚ ar samme uttrykk for negative x. Konvolusjon Konvolusjon er en m˚ ate a˚ definere en relasjon mellom to funksjoner p˚ a. Akkuratt som en definerer addisjon og subtraksjon danner en operasjon p˚ a de reelle tallene. For integraler og spesielt laplace-transformasjoner er følgende definisjon viktig Definisjon 3.4.5. La f (t) og g(t) være to funksjoner av t. Vi definerer operasjonen (f ∗ g)(t) som følger Z ∞ f ∗g = f (t − τ )g(τ ) dτ −∞

som og er en funksjon avhengig av t. Proposisjon 3.4.3. Dersom f og g er en-sidede funksjoner, s˚ a er Z t (f ∗ g)(t) = f (t − τ )g(τ ) dτ 0 6

Siden | cos(xt)| < 1 har vi at

Z



Z

|I| ≤ 0

som komvergerer.

0



e−sx dt dx = 1 + t2

Z 0



h

π −sx e 2

i∞ 0

h

dt dx = −

π −sx e 2s

i∞ 0

=

π 2s

III

4.1 Laplace transformasjonen

145

Alts˚ a at f (t) = a(t)H(t) og g(t) = b(t)H(t), der H igjen betegner heavyside funksjonen og a og b er funksjoner av t. Det interessante er hva som skjer n˚ ar vi tar L -transformasjonen av f ∗ g. Proposisjon 3.4.4. La f, g : [0, ∞) 7→ R være to en-sidede funksjoner, og la F (s) og G(s) betegne deres respektive L -transformasjoner. Da er L {(f ∗ g)(t)} = F (s)G(s) ⇔ L −1 {F (s)G(s)} = (f ∗ g)(t) Proposisjonen kan og skrives som L (f ∗ g) = L (f ) · L (g) som noen ganger er en hendigere form. Her ser vi alts˚ a operasjonen ∗ gjør L -transformasjonen til en komplett-multiplikativ funksjon7 . ˚ skrive om F (s)G(s) som dobbeltintegralet Bevis. Vi begynner med a Z ∞Z ∞ F (s)G(s) = e−s(x+y) f (x)g(y) dx dy . 0

0

˚ bruke substitusjonen t 7→ x + y ogs˚ Ved a a y = τ f˚ ar vi at integralet kan skrives Z ∞Z ∞ F (s)G(s) = e−st f (t − τ )g(τ ) dt dτ . τ

0

Herfra snur vi integrasjonsrekkefølgen, og at dette er lov følger igjen fra fubinis sats. Z t  Z ∞Z t Z ∞ −st −st F (s)G(s) = e f (t − τ )g(τ ) dτ dt = e f (t − τ )g(τ ) dτ dt . 0

0

0

0

Rt

˚ innfører notasjonen h(t) = 0 f (t − τ )g(τ ) dτ har en n˚ Ved a a vist Z ∞ F (s)G(s) = e−st h(t) dt = L {(f ∗ g)(t)} , 0

˚ vise den andre likheten tar en invers L -transformasjon av som ønkset. For a likningen over. Dette fullfører beviset. Tanken er n˚ a at i stedet for a˚ beregne integralet f ∗ g kan en i stedet beregne den inverse L -transformasjonen av produktet av F og G Z t f (t − τ )g(τ ) dτ = L −1 {F (s)G(s)} 0

la oss vise et tilfellet hvor dette er nyttig. Eksempel 3.4.4. Vi ønsker a˚ vise bevise theorem (3.3.6) ved hjelp av konvolusjonstheoremet. Alts˚ a at Z 1 Γ(x)Γ(y) B(x, y) = sx−1 (1 − s)y−1 ds = Γ(x + y) 0 holder for alle komplekse x,y med realdel større enn null. 7 Alts˚ a

at f (n ∗ m) = f (n)f (m) holder. For tallteoretiske funksjoner har vi en tilsvarende konvolusjonen nemlig Dirichlet konvolusjonen.

146

III

4 Transformasjoner

˚ beregne f ∗ g har vi Bevis. Vi velger f (t) = tx og g(t) = ty . Ved a Z t sx (t − s)y ds (tx ∗ ty ) = 0

som nesten er definisjonen av beta-integralet. P˚ a den ene siden s˚ a er L (tx ∗ty ) = x y L (t )L (t ), slik at L (tx ∗ ty ) = L (tx )L (ty ) =

x! · y! sx+y+2

Fra konvolusjons-theoremet har vi da at   Z t x! · y! tx+y+1 x y −1 s (t − s) ds = L = x! · y! (3.47) sx+y+2 (x + y + 1)! 0   ˚ sette inn hvor det ble brukt at L −1 Γ(a + 1)/sa+1 = ta igjen fra tabell. Ved a x = x − 1, y = y − 1 og t = 1 kan likning (3.47) skrives som Z 1 Γ(x)Γ(y) (x − 1)!(y − 1)! sx−1 (1 − s)y−1 ds = = (x + y − 1)! Γ(x + y) 0 hvor n! = Γ(n + 1). Dette fullfører beviset. 1. La f være en funksjon med periode T . Vis at Z T 1 Lf (s) = f (t)e−st dt 1 − e−T s 0 2. La f være en funksjon slik at L -transformasjonen av f eksisterer. Vis at Z t  1 L f (w) dw = F (s) s 0

3.4.2

FOURIER-TRANSFORMASJON

Parseval og Plancherel Theorem 3.4.1. (Parseval’s identitet) La f og g være kvadratisk integrerbare funksjoner f, g ∈ L2 slik at indreproduktet er definert. Da holder Z ∞ Z ∞ hf, gi = f (x)g(x) dx = fˆ(x)ˆ g (x) dx = hfˆ, gˆi −∞

−∞

Korollar 3.4.2. (Plancherels Theorem) Spesielt dersom f = g i theorem (3.4.1), s˚ a er Z ∞ Z ∞ ˆ 2 2 2 kf k2 = |f (x)| dx = f (x) dx = kfˆk22 −∞

−∞

III

4.4 Landen’s transformasjon

3.4.3

MELLIN

147

TRANSFORMASJONEN

Mellin transformasjonen er en multiplikativ transformasjon og er relatert til den tosidige laplace transformasjonen. Integralet er dypt knyttet til Dirichlet rekker, og sentral innen analytisk tallteori. Definisjon 3.4.6. Mellin transformasjonen av f er definert som Z ∞ M(f (x)) = ϕ(s) = xs−1 f (x) dx 0

˚ skrive Mf (s) eller ϕ(s) dersom det er innSom før kommer vi enten til a lysende hvilken funksjon vi tar transformasjonen av. Som et grunleggende og viktig eksempel innen tallteori s˚ a er M-transformasjonen av f (x) = e−x lik Γ-funksjonenen. Mellin-transformasjonen er faktisk en Fourier-transformasjon, som kan vises ved en substitusjon. Merk at bruksomr˚ adene til Fouerier og Mellintransformasjonene er noe forskjellig. Proposisjon 3.4.5. Gitt at f er en kontinuerlig funksjon p˚ a (0, ∞), at f (t) ≤ Ct−α for α ∈ R n˚ ar t → 0, og at f (t) g˚ ar mot null raskere enn enhver potens av t n˚ ar t → ∞. S˚ a holder følgende 1) Mellin-transformasjonen av f konvergerer absolutt for Re(s) > α og definerer en holomorf funksjon i det halvplanet. 2) La s = σ + iT , hvor σ > α og la gσ (t) = e−2πσt f e−2πt



3) Den inverse mellin-transformasjonen er definert som et komplekst linjeintegral og konvergerer for alle σ > α for alle x > 0 Z σiT 1 f (x) = x−s Mf (s) ds 2πi σ−iT n˚ ar T → ∞.

3.4.4

LANDEN’S

TRANSFORMASJON

Som introduksjon forklares først hva det aritmetiske-geometriske snittet er. Definisjon 3.4.7. (Aritmetiske-geometriske snittet) For to positive reelle tall x og y definer som følger 1 (a + b) √2 b1 = ab

a1 =

Der a1 er den aritmetriske middelverdien av a, b og b1 er den geometriske middelverdien av a, b. Deniner s˚ a følgen 1 (an + bn ) 2 p = an · bn

an+1 = gn+1

Følgene konvergerer mot samme verdi og kalles det aritmetiske-geometriske snittet av a og b. Der snittet som M (a, b) eller Agm(a, b).

148

III

4 Transformasjoner

Eksempelvis s˚ a er Agm(1, 2) ≈ 1.45679 og Agm(1.5) ≈ 2.604008. La oss først vise et lemma som relaterer det ellitpiske integralet, og Agm(x, y). Lemma 3.4.1. Integralet Z



I(a, b) := 0

dθ p 2 2 a cos θ + b2 sin2 θ

er uforandret om a og b byttes ut sitt aritmetriske og geometriske snitt   √ 1 I(a, b) = I (a + b), ab . 2 Bevis. Benytter substitusjonen tan θ = x/b s˚ a dθ = cos2 (θ)/b s˚ a Z ∞ dθ dx p p I(a, b) = = 2 2 2 2 2 (x + a2 )(x2 + b2 ) 0 0 a cos θ + b sin θ √ Benyttes n˚ a substitusjonen x = t + t2 + ab f˚ as Z ∞ dt r = 2  −∞ 2 t2 + a+b (t2 + ab) 2 Z ∞ dt r =  √ 2   0 t2 + ( ab )2 t2 + a+b 2 Z

π/2

Kvadratroten ble skrevet om som følger s p



(x2 + a2 )(x2 + b2 ) = 2x t2 +

a+b 2

2

√ og dx = (x/ t2 + ab ) dt. Tidlig p˚ a 1800-tallet var Gauss interest i lengden av leminskater alts˚ a kurver som ligner p˚ a ∞. Etter en numerisk beregning av en spesiell leminskate observerte han at Z 1 π 1 dx √ √ og 2 0 Agm(1, 2 ) 1 − x4 stemte til 11 desimaler. Med forbausende klarsyn deduserte Gauss følgende theorem Theorem 3.4.2. Z I=

π/2

dθ p

a2

cos2

b2

2

=

π 2 Agm(a, b)

θ + sin θ   √ a har vi alts˚ a Bevis. Siden I(a, b) = I 12 (a + b), ab s˚ 0

 I(a, b) = I(a1 , b1 ) = I(a2 , b2 ) = · · · = I M (a, b), M (a, b)

(3.48)

III

4.5 Cauchy-Schl¨ omilch transformasjonen

149

Setter vi dette inn i likningen f˚ as Z ∞ Z ∞ dx 1 dy π p = = I= 2 2 2 2 2 M (a, b) 0 1 + y 2M (a, b) (x + M (a, b) )(x + M (a, b) ) 0 der substitusjonen x 7→ M (a, b)y ble benyttet. At dette integralet er nært knyttet til de elliptiske integralene er allerde hintet til i starten, via substitusjonen b2 = a2 (1 − k 2 ) f˚ as I=

1 a

Z 0

π/2

dθ 1 π  1 p = F , k = K(k) 2 2 a 2 a 1 − k sin θ

(3.49)

Settes likning (3.48) inn i likning (3.49) og løser med hensyn p˚ a K f˚ as K(k) =

a π √ . 2 Agm(a, a 1 − k 2 )

fordelen med dette er at iterasjonen av Agm(x, y) konvergerer kvadratisk, alts˚ a ˚ tilnærme dobbles antall rette siffer per iterasjon. Dette er alts˚ a særs nyttig for a lengden av ellptiske kurver.

3.4.5

¨ MILCH C A U C H Y- S C H L O

TRANSFORMASJONEN

Utledningene og informasjonen her er stort sett hentet fra [? ] og mer utdyppende informasjon finnes der. Teknikken vi skal se p˚ a her ble popularisert av Oscar Xavier Schl¨ omilch (1823-1901), men kan finnes i notater til Cauchy tidligere, derav navnet. Eksempel 3.4.5. Først tar vi en titt p˚ a et klassisk integral fra P.Laplace. r Z ∞  1 π −2√ab 2 −2 exp −ax − bx dx = e 2 a 0 ˚ beregne integralet fullføres kvadratet slik at Bevis. For a Z ∞ Z ∞ √ √ √ 2 2 −2 −2 ab exp(−ax − bx ) = e e−( a x− b /x) dx 0

0

Definerer s˚ a siste integral som K s˚ a Z ∞ √ √ 2 K := e−( a x− b /x) dx 0

p Benyttes n˚ a substitusjonen t = x1 b/a f˚ as r Z ∞ √ √ 2 dt b e−( a x− b /x) 2 . K= a 0 t Tar en gjennomsnittet av disse to integralene f˚ as Z ∞   √ √ √ 2 √ 1 K= √ e−( a x− b /x) a + b /x2 dx . 2 a 0

(3.50)

150

III

4 Transformasjoner

√ √ Substitusjonen y 7→ a x − b /x gir s˚ a r Z ∞ 1 π 1 −y 2 e dy = K= √ 2 2 2 a −∞ og dette fullfører beviset da r Z ∞ √  1 π −2√ab exp −ax2 − bx−2 dx = Ke−2 ab = e . 2 a 0

Teknikken om blir brukt her ble utvidet av O.Schl¨ omilch til følgende theorem. Theorem 3.4.3. (Cauchy-Schl¨ omilch) Gitt a, b > 0 og f en kontinuerlig funksjon da er Z Z ∞   1 ∞ 2 f (y 2 ) dy , (3.51) f (ax − b/x) dx = a 0 0 gitt at integralene i likning (3.51) konvergerer. Bevis. Benytt substitusjonen t 7→ b/(ax) s˚ a Z ∞  Z ∞    dt b 2 2 I := f (ax − b/x) dx = f (at − b/t) a 0 t2 0 ˚ ta gjennomsnittet av disse to integralene f˚ Ved a as   Z ∞ Z  1 b 1 ∞ 2 = a + 2 f (at − b/t) dt = f (y 2 ) dy a 0 t a 0 Der substitusjonen y 7→ at − b/t ble brukt i siste overgang. Dette fullfører beviset. Denne relativt enkle transformasjonen kan løse langt vanskeligere integral enn hva dagens symbolske kalkulatorer klarer. Her sammenlikner vi med Maple 14. Eksempel 3.4.6. Z



 1 exp −(x − b/x)2n = Γ n −∞



1 2n



n

Bevis. Tar utgangspunkt i likning (3.51) med f (x) = e−x slik at Z ∞ Z ∞  2n 2n exp −(x − b/x) dx = e−y dy −∞ −∞ Z ∞ 1 e−t t1/(2n)−1 dt = n 0   1 1 = Γ n 2n 2

der substitusjonen t 7→ y 2n ble brukt i andre overgang. At e−x er symmetrisk ˚ fullføre omkring origo, og definisjonen av gammafunksjonen ble benyttet til a beviset.

III

4.5 Cauchy-Schl¨ omilch transformasjonen

151

I det neste theoremet ser vi p˚ a en naturlig utivdelse av Cauchy-Schl¨ omilch transformasjonen Theorem 3.4.4. La s være en kontinuerlig synkende funksjon fra R+ til R+ . Anta at s er sin egen invers, alts˚ a at s−1 (x) = s(x) ∀ x ∈ R+ da er Z Z ∞  1 ∞ f (y 2 ) dy (3.52) f [ax − s(ax)]2 dx = a 0 0 der a > 0 og gitt at integralene konvergerer. Bevis. La først t 7→ ax og bytt tilbake til x som integrasjonsvariabel da f˚ as Z ∞ Z ∞   1 f [ax − s(ax)]2 dx = I= f [x − s(x)]2 dx a 0 0 Videre la t 7→ s(x), dette gir Z Z   1 ∞ 1 ∞ I= f [x − s(x)]2 dx = − f [s(t) − t]2 s0 (t) dt a 0 a 0 Gjennomsnittet av disse to integralene gir Z ∞ Z   1 1 ∞ 2 0 f [x − s(x)] 1 − s (t) dx = f (y 2 ) dy 2a 0 a 0 der substitusjonen y 7→ x − s(x) ble benyttet i siste overgang. Dette fullfører beviset. Det finnes flere funksjoner som er sin egen invers, den enkleste f (x) = 1/x har vi sett grundigere p˚ a før. I følgende eksempel vises noen andre slike funksjoner Eksempel 3.4.7. En nyttig funksjon som er sin egen invers er s(x) = x −

1 log (eαx − 1) α

˚ benytte oss av likning (3.52) f˚ ved a as  Z ∞  Z ∞ 1 2 αx log (e − 1) dx = f f (y 2 ) dy α2 0 0 Interessant! Ved n˚ a og sette f (x) = e−x f˚ as   √ Z ∞ Z ∞ π 1 2 αx −y 2 . exp log (e − 1) dx = e dy = 2 α 2 0 0 ˚ forenkle høyresiden, dette gir merk og at integralet kan løses ved a   Z ∞ Z ∞ Z ∞ 2 1 2 αx (x−1/α)2 exp log (e − 1) dx = e dx = e−y dy 2 α 0 0 0 via y 7→ x − 1/α, uansett er dette en artig generalisering.

152

III

4 Transformasjoner

3.4.6

DIVERSE

TRANSFORMASJONER

I denne delen skal vi blant annet beregne Z





I(a, b; r) := 0

x2 x4 + 2ax + 1

r

x2 + 1 dx xb + 1 x2

˚ løse oppgaven innledningsvis trengs følgende og noen spesialtilfeller. For a omskrivning Lemma 3.4.2. Z ∞  f 0

x2 4 x + 2ax2 + 1



Z



 f

dx = 0

1 x2 + 2(a + 1)

 dx

for alle funksjoner f med a > 0. ˚ dele p˚ Bevis. Ved a a x2 og faktorisere gir Z



J=

 f

0

x2 4 x + 2ax2 + 1



Z



 f

dx = 0

1 (x − x−1 )2 + 2(a + 1)

 dx

˚ La oss n˚ a benytte Cauchy-Schl¨ omilc transformasjonen fra theorem (3.4.3) til a beregne integralet. Her settes f (t) = g 1/(t2 + 2a + 2) . Da har vi Z



 g

0



1 (ax − b/x)2 + 2(a + 1)

dx =

1 a

Z



 g

0

1 2 y + 2(a + 1)

 dy

˚ sette a = b = 1. og beviset fullføres ved a ˚ gyve løs p˚ Vi er endelig klar til a a problemet innledningsvis, n˚ a med barsk og bram. Theorem 3.4.5. (V.Moll’s MasterTheorem) La r Z ∞ x2 + 1 dx x2 · · I1 = x4 + 2ax2 + 1 xb + 1 x2 0 r Z ∞ x2 dx I2 = · 2 4 + 2ax2 + 1 x x 0 r Z ∞ 2 x I3 = dx 4 + 2ax2 + 1 x 0   Z ∞ r  1 x2 1 I4 = · 1 + 2 dx 2 0 x4 + 2ax2 + 1 x Da er I1 = I2 = I3 = I4 og verdien er I(a, b; c) = der λ = r − 1/2.

  1 2−(1+λ) B λ, (1 + a)λ 2

(3.53)

III

4.6 Diverse transformasjoner

153

Bevis. Heldigvis s˚ a tilfredstiller  R(x) =

x2 x4 + 2ax2 + 1

r ·

x2 + 1 x2

funksjonallikningen R(y) = R(1/y)/y 2 , og fra theorem (2.10.1) s˚ a er R(x)/(xb + ˚ vise I4 kan en enten as I2 , b = −2 gir I3 . For a 1) uavhengig av b. Settes b = 2 f˚ sette b = 0 i I1 eller ta gjennomsnittet av I2 og I3 . Dette viser at integralene er like. For a˚ beregne integralet tar en utgangspunkt i lemma (3.4.2) med f (u) = ur dette gir Z 0





x2 x4 + 2ax2 + 1

r

r 1 dx x2 + 2(a + 1) 0 Z 1 −r+1/2 1 2(a + 1) ur−3/2 (1 − u)−1/2 2 0   1 1 2−1−r+1/2 , B r − 2 2 (1 + a)r−1/2   −(1+λ) 2 1 B λ, (1 + a)λ 2

Z dx = = = =





Der substitusjonen u = x22(a+1) +2(a+1) ble benyttet i andre overgang. Avslutningvis ble definisjonen av betafunksjonen benyttet og λ = r − 1/2. Herfra følger par eksempler p˚ a spesialtilfeller av dette integralet Eksempel 3.4.8. Ved a = 7 og r = 5/4 i I2 f˚ as Z 0





Γ2 (3/4) x √ = (x4 + 14x2 + 1)5/4 4 2π

˚ derivere med hensyn p˚ Et par flere eksempler f˚ as ved a a variablene i lik˚ sette a = 1/2 og derivere med hensyn p˚ ning (3.51). Eksempelvis ved a ar f˚ as r r Z ∞ x2 dx π Γ(λ) = · 4 − x2 + 1 2 x x 4 Γ(r) 0 r r   Z ∞ 2 2  x dx π Γ(λ) x log = · ψ(λ) − ψ(r) 4 − x2 + 1 4 − x2 + 1 2 x x x 4 Γ(r) 0 (3.54) Hvor λ = r − 1/2 og ψ(x) = Γ0 (x)/Γ(x). Eksempel 3.4.9. I dette eksempelet setter vi inn tre verdier for r i likning (3.54). r=1      Z ∞ √ x x2 1 1 log = 2π Γ ψ( ) − ψ(1) x4 − x2 + 1 2 2 (x4 − x2 + 1)3/2 0 = −π log 2

154

4 Transformasjoner

III

√ Hvor det ble benyttet at ψ(1) = −γ, ψ(1/2) = −γ − 2 log 2 og Γ(1/2) = π . Via tilsvarende regning f˚ as    r  3/4     Z ∞ x x2 π Γ 41 1 1  ψ 1− log =− −ψ 3 4 − x2 + 1 4 − x2 + 1 x x 4 4 4 Γ 4 0 r   1 π 2 1 Γ =− 2 2 4 for r = 3/4. Her ble det brukt at ψ(1−z)−ψ(z) = π cot(πz) ⇒ ψ(3/4)−ψ(1/4) = π og Γ(1 − z)Γ(z) = π/ sin(πz) ⇒ 1/Γ(3/4) = Γ(1/4) sin(π/4)/π. Dette klarte ˚ beregne. ikke Maple 14 a

III

5 Diverse applikasjoner

3.5

155

DIVERSE APPLIKASJONER

Theorem 3.5.1. Anta f er en kontinuerlig, deriverbar8 og strengt økende funksjon Rb p˚ a [a, b]. Let m = (a + b)/2 være midpunktet, da har g(x) = a |f (t) − x| dt et unikt minimum over x ∈ R. Minumumet er Z b Z m f (t) dt − f (t) dt m

a

and skjer ved x = f (m). Bevis. Dersom x < f (a) eller x > f (b) s˚ a er Z b Z b |f (t) − x| dt > |f (t) − f (c)| dt, a

a

hvor c er a dersom x < f (a) og b dersom x > f (b). Et minimum m˚ a alts˚ a skjer i for f (a) ≤ x ≤ f (b). Fra middelverdi setningen (B.1.1) s˚ a eksisterer det en x = f (ξ) hvor ξ ∈ [a, b]. S˚ a Z b Z ξ Z b |f (t) − f (ξ)| dt = f (ξ) − f (t) dt + f (t) − f (ξ) dξ a

a

ξ

˚ snu grensene i siste Legg merke til at f (ξ) er konstant og kan beregnes. Ved a integral f˚ ar en da Z b Z ξ  = 2ξ − (a + b) f (ξ) + f (t) dt − f (t) dt (3.55) ξ

a

Siden vi har antatt at f er deriverbar s˚ a kan vi trygt derivere likning (3.55). Z b  d |f (t) − f (ξ)| dt = 2ξ − (a + b) f 0 (ξ) = 0 dξ a N˚ a siden f er strengt økende p˚ a intervalet medfører dette at f 0 (ξ) > 0 for alle ξ ∈ [a, b]. S˚ a den deriverte er null n˚ ar ξ = (a + b)/2. Den dobbelderiverte er negativ ved x = ξ slik at ξ virkelig er et minimum. La oss n˚ a vise at ξ virkelig er et unikt minimum. Siden ingenting forandres ved a˚ legge til en konstant til f , eller skalere intervalet skalerer vi slik at a = −a og f (0) = 0. Da er Z b Z b Z ζ  |f (t) − f (ζ)| dt − |f (t) − f (ξ)| dt = 2 ζf (ζ) − ξf (ξ) − 2 f (t) dt. −b

−b

ξ

Dersom vi velger ξ = 0 s˚ a blir differansen Z ζ Z 2ζf (ζ) − 2 f (t) dt = 2 0

ζ

f (ζ) − f (t) dt > 0,

0

og tilsvarende om vi velger ζ = 0, s˚ a blir den Z 0 Z 0 −2ξf (ξ) − 2 f (t) dt = 2 f (ξ) − f (t) dt < 0, ξ

ξ

dette viser at x = f (m) virkelig gir et unikt globalt minimum. 8 Kravet

om deriverbarhet er ikke nødvendig, men det forenkler beviset noe.

156

5 Diverse applikasjoner

3.5.1

G U LV

O G TA K - F U N K S J O N E R

3.5.2

ITERERTE

INTEGRAL

III

III

6 Derivasjon under integraltegnet

3.6

157

D E R I VA S J O N U N D E R I N T E G R A LT E G N E T

One thing I never did learn was contour integration. I had learned to do integrals by various methods shown in a book that my high school physics teacher Mr. Bader had given me. That book also showed how to differentiate parameters under the integral sign—it’s a certain operation. It turns out that’s not taught very much in the universities; they don’t emphasize it. But I caught on how to use that method, and I used that one damn tool again and again. So because I was self-taught using that book, I had peculiar methods of doing integrals. The result was, when guys at MIT or Princeton had trouble doing a certain integral, it was because they couldn’t do it with the standard methods they had learned in school. If it was contour integration, they would have found it; if it was a simple series expansion, they would have found it. Then I come along and try differentiating under the integral sign, and often it worked. So I got a great reputation for doing integrals, only because my box of tools was different from everybody else’s, and they had tried all their tools on it before giving the problem to me. (Feynman [? , s47-48]) Metoden som Mr. Feynman i denne passasjen refferer til g˚ ar ofte under navnet derivasjon under integraltegnet, derivasjon med hensyn til en parameter, eller ˚ navngi metoden, til og med Feynman integrasjon. Uansett hvordan en ønsker a ˚ beregne s˚ a ligger appelen og skjønnheten i at metoden kan bli benyttet til a tilsynelatende umulige integral, uten bruk av mer enn elementær kalkulus9 . Vi skal studere hvilke situasjoner følgende likhet (som vi inderlig ønsker skal være sann) holder Z Z d ∂ f (x, y) dy = f (x, y) dy . (3.56) dx Y ∂x Y ˚ bruke definisjonen av den deriverte ser vi at utsagnet er ekvivalent med Ved a følgende Z Z lim f (x, y) dy = lim f (x, y) dy (3.57) x→n

Y

Y x→a

Følgende teorem viser n˚ ar utsagnet stemmer. Theorem 3.6.1. (Elementær form). La f : [a, b] × Y → R være en funksjon, der [a, b] er et lukket interval og Y er et kompakt underrom av Rn . Da er Z Z d ∂ f (x, y) dy = f (x, y) dy . dx Y ∂x Y R Dersom b˚ ade f (x, y) og ∂f (x, y)/∂x er kontinuerlige for alle x, y og Y f (x, y) dy en kontinuerlig, deriverbar funksjion med hensyn p˚ a x ∈ [a, b]. 9 Om

˚ bevise theorem (3.6.2) en ser bort fra den lille desjen med m˚ alteori som er nødvendig for a

158

6 Derivasjon under integraltegnet

III

Et bevis av overnevnte ville brukt mye vanskelig notasjon og teoremer fra funksjonalanalysen. S˚ a det droppes for leserens helse og velbehag. I korte trekk vil et bevis ta i bruk at Y er kompakt, som impliserer uniform konvergens. ˚ bytte om grenseverdien (Selve definisjonen av kompakthet). Herfra kan det a og integralet rettferdigjøres. Dessverre s˚ a er denne definisjonen ofte noe snever, da vi gjerne vil se p˚ a omr˚ ader som ikke er kompakte. Eksempelvis halv˚ apne interval [x, ∞), eller ˚pne interval (−∞, ∞). Heldigvis finnes det en mer generell verisjon av theoa rem (3.6.1). Som bruker enda mer abstrakt notasjon, mer spesifikt m˚ alteori. alteori utgave) La X være et a Theorem 3.6.2. (M˚ ˚pent underrom av R og la Ω være et m˚ alrom. Anta f : X × Ω → R tilfredstiller følgende betingelser 1) f (x, ω) er en Lebesgue-integrerbar funksjon av ω for enhver x ∈ X. 2) For nesten alle ω ∈ Ω, s˚ a eksisterer den deriverte ∂f (x, ω)/∂x for enhver x ∈ X. 3) Def finnes en integrerbar funksjon Θ : Ω → R slik at |∂f (x, ω)/∂x| ≤ Θ(ω) for enhver x ∈ X. Da for enhver x ∈ X s˚ a er Z Z ∂ d f (x, y).dω = f (x, y) dω . dx Ω Ω ∂x ˚ deriverre under inteDette teoremet gis oss og grunnlaget for ikke bare a graltegnet n˚ ar omr˚ adet er Y er endelig, men og for Y = (0, ∞) gitt at kravene ovenfor holder. For leserens trygghet holder heldigvis de to første kravene for nesten alle integraler. Om funksjonen er kontinuerlig eller glatt, eksisterer den deriverte for alle verdier. Om funksjonen konvergerer absolutt er den Lebesgue integrerbar, eller om funksjonen er Riemann integrerbar p˚ a et endelig intervall. Det som stort sett m˚ a sjekkes er at b˚ ade integranden, og dens deriverte er bundet p˚ a intervalet. For enkelhetens skyld gis og følgende teorem som konkretiserer dette Theorem 3.6.3. Begge sider av likningen Z Z d ∂ f (x, y) dy = f (x, y) dy . dx Y ∂x Y eksisterer og er like i et punkt t = t0 gitt at følgende to krav holder 1) f (x, t) og ∂f (x, t)/∂t begge kontinuerlige n˚ ar x ∈ Y og t befiner seg i et lite omhegn innenfor t0 2) Det eksisterer funksjoner slik at f (x, t) ≤ A(x) og ∂f (x, t)/∂t ≤ B(x) Rb Rb som er uavhengig av t slik at a A(x) dx og a B(x) dx konvergerer. Teoremet noe svakere (Det gjelder ikke for Lebesgue-integral), men holder for de funksjonene som blir studert her. Men n˚ a har det vært fryktelig mye snakk, ˚ vise hvor nyttig denne teknikken kan være vi sl˚ ar til med barsk og bram for a

III

6 Derivasjon under integraltegnet

159

˚ bestemme følgende integral Eksempel 3.6.1. Vi ønsker a Z 1 2 x −1 dx log x 0 Integralet har ingen kjent antiderivert, og vi m˚ a derfor ty til andre metoder. Den vante leser vil kanskje legge merke til enten at integralet kan skrives som et dobbeltintegral, eller at det er et frullani integral i forkledning (u 7→ log x). Men ˚ derivere med hensyn dette sparer vi til en senere annledning. Vi trenger noe a p˚ a, og innfører paramteren a > 0 som følger Z 1 a x −1 dx I(a) = log x 0 Her vil selvsagt I(2) gi oss integralet vi er ute etter. Derivasjon med hensyn p˚ aa gir Z 1 a Z 1 Z 1 d x −1 ∂ xa − 1 1 dx = dx = xa dx = I 0 (a) = da 0 log x ∂a log x 1 + a 0 0 Integrasjon gir oss at I(a) = log(1 + a) + C ˚ betrakte a = 0. Da er I(0) = 0 og log(1) = 0 Konstanten kan bestemmes ved a slik at C = 0. Vi f˚ ar da alts˚ a at Z 1 2 x −1 dx = I(2) = log(1 + 2) = log 3 log x 0 ˚ finne. som var det vi ønsket a Theorem 3.6.4. (Dirichlet integral) er definert som følger Z ∞ sin ω π dω = ω 2 0 Dette integralet vil dukke opp gjentatte ganger videre og er svært viktig innen signalanalyse. Ofte blir forkotelsen sinc(x) = sin(x)/x benyttet. En alternativ ˚ først normalisere funksjonen slik at sinc(x) = sin(πx)/πx, som definisjon er a har en rekke kjente definisjoner.  ∞  Y sin(πx) x2 1 = 1− 2 = πx n Γ(1 + x)Γ(1 − x) n=1 Har en noe kjennskap til laplacetransformasjoner eller singnalanalyse vil følgende ˚ innføre variabelen a utregning av integralet gi mer mening. Vi prøver først a som følger. Z ∞ sin ax I(a) = dx x 0 Men ved derivasjon f˚ as I 0 (a) = cos(ax) som ikke konvergerer! La oss heller se p˚ a følgende metode

160

6 Derivasjon under integraltegnet

III

Bevis. Vi definerer følgende funksjon Z ∞ I(b) := sinc(x) · e−bx dx 0

Dette er faktisk laplce-transformasjonen av sinc(x) som vi skal se senere. Herfra ˚ la b = 0. Derivasjon gir ses at det originale integralet oppst˚ ar ved a Z ∞ Z ∞ ∂ −bx 1 I 0 (b) = e−bx sin x dx = − +C e sinc(x) dx = − ∂b 1 + b2 0 0 Hvor det siste integralet har blitt løst flere ganger før. Enten skriv om integralet til kompleks form, eller benytt delvis integrasjon to ganger. Integrasjon gir n˚ a I(b) = C − arctan(b) Igjen gjennst˚ ar det a˚ beregne konstanten C. Dersom vi lar b → ∞ s˚ a vil I(b) → 0 og arctan b → π/2. Som medfører at I(b) =

π − arctan(b) 2

˚ la b = 0 f˚ Ved a ar vi v˚ art originale integral, slik at Z ∞ sin x = 2 · I(0) = π x −∞ ˚ finne. som var det vi ønsket a Et annet forholdsvis klassisk eksempel er følgende integral √ x dx = π log 2 tan x

π/2

Z 0

˚ betrakte følgende integral som kan løses ved a Z

π/2

I(b) = 0

tan(b arctan x) dx tan x

Der I(1) gir oss v˚ art opprinnelige integral. Mellomregningene overlates til leser ˚ skrive ut taylorrekka til tan x. og i neste seksjon vil integralet beregnes ved a Vi snur derimot oppmerksomheten v˚ ar mot et moteksempel. I p˚ afølgende eksempel viser det seg at høyre og venstre side i likning (3.56) ikke nødvendigvis er like. Eksempel 3.6.2. For alle x, t ∈ R, s˚ a definerer vi følgende funksjon ( xt3 hvis x = 0 eller t 6= 0, (x2 +t2 )2 f (x, y) := 0 hvis x = 0 og t = 0. La videre Z F (t) :=

1

f (x, t) dx . 0

III

6 Derivasjon under integraltegnet

161

Integralet beregnes for t = 0 og t 6= 0 som følger Z 1 Z 1 F (0) = f (x, 0) dx = 0 dx = 0 0

0 1

Z F (t) = 0

3

xt dx (x2 +

2 t2 )

Z

1+t2

= t2

t3 du 1 t = 2u2 2 1 + t2

Formelen er og gyldig for t = 0. Ved derivasjon f˚ as d 1 1 − t2 F (t) = dt 2 (1 + t2 )2

∀t ∈ R

P˚ a den andre siden s˚ a er f (0, t) = 0 ∀t, slik at ∂f (0, t)/∂x = 0. N˚ ar x 6= 0 s˚ a blir den deriverte 2   x2 + t2 (3xt2 ) − xt3 · 4t x2 + t2 xt2 3x2 − t2 ∂ f (x, t) = = 4 3 ∂t (x2 + t2 ) (x2 + t2 ) Kombineres resultatet f˚ ar vi alts˚ a at ( xt2 (3x2 −t2 ) ∂ (x2 +t2 )3 f (x, t) = ∂t 0

hvis hvis

x 6= 0 x=0

(3.58)

Som motbevis til likning (3.56) benyttes heller den tilsvarende definisjonen vist i likning (3.57) hvor vi ser p˚ a a = 0. Da er Z 1 Z 1 xt3 1 0 = F (0) = og lim lim f (x, t) = 0 t→0 0 (x2 + t2 )2 2 0 t→0 Alts˚ a er grensene ulike! Dette gjelder uansett om x = 0 eller ei. Problemet er er at ∂f (x, t)/∂t ikke er en kontinuerlig funksjon, spesifikt bl˚ aser den opp i origo. Som et aller siste eksempel s˚ a studerer vi enda en gang det Gaussiske integralet. Det finnes enklere metoder a˚ beregne integralet p˚ a enten via dobbeltintegraler se spesifikt eksempel (3.8.2) og (3.8.3). Eller ved gammafunksjonen som vist i ??. Eksempel 3.6.3. Vi skal vise det liknende resultatet √ r Z ∞ 2π π −x2 /2 e dx = = 2 2 0 ˚ f˚ og f˚ ar a a det klassiske √ gaussiske integralet kan resultatet ovenfor benyttes via substitusjonen u = 2 x. Vi definerer følgende integral for t > 0 Z t 2 −x2 /2 A(t) := e dx . 0

˚ la t → ∞ ogs˚ Hvor vi f˚ ar v˚ art ønsekede integral ved a a ta kvadratroten. Derivasjon med hensyn p˚ a t gir oss Z t  2 2 2 A0 (t) = 2 e−x /2 dx e−t /2 = 2e−t /2 A(t) 0

162

III

6 Derivasjon under integraltegnet

Benyttes substitusjonen x = ty n˚ a f˚ as Z 1 Z 2 2 2 A0 (t) = 2e−t /2 te−t y /2 dy = 0

1

2te−(1+y

2

)t2 /2

dy

0

˚ bestemme den antideriverte med hensyn p˚ Funksjonen er n˚ a enkel a a t, da f˚ as Z Z 1 2 2 2 2 1 −(1+y )t /2 ∂ 2e−(1+y )t /2 d e dy = −2 dy A0 (t) = − 2 ∂t 1 + y dt 1 + y2 0 0 For n˚ a definerer vi integralet som en ny funksjon s˚ a Z 1 −(1+y2 )t2 /2 e B(t) := dy 1 + y2 0 Slik at vi har likningen A0 (t) = −2B 0 (t) ∀ t > 0. Integrerer vi begge sider ser vi at det finnes en konstant C slik at A(t) = −2B(t) + C

∀t > 0.

Lar vi n˚ a t → 0+ f˚ ar vi at A(t) → 0 mens B(t) → Alts˚ a s˚ a er

R1 0

(3.59)

 dy/(1 + y 2 ) = π/4.

0 = −2(π/4) + C ⇒ C = π/2 . Lar vi n˚ a t → ∞ i likning (3.59) f˚ as Z

t

−x2 /2

e

lim

t→∞

2 dx

= lim

0

Z



−x2 /2

e 0

π − 2

t→∞

Z 0

1

2

e−(1+y )t 1 + y2

2

!

/2

dy

(3.60)

2 Z 1 2 2 π e−(1+y )t /2 dx = − lim dy 2 1 + y2 0 t→∞

˚ benytte oss av likning (3.57). Herfra ser vi at høyresiden g˚ Ved a ar mot π/2 da e−ax → 0 n˚ ar x → ∞ ∀a > 0. Slik at r Z ∞ π −x2 /2 e dx = , 2 0 da integralet m˚ a være positivt. Det finnes mange flere integral som kan løses ved hjelp av derivasjon under integraltegnet men disse blir plassert som oppgaver for leser, b˚ ade her og p˚ a slutten av dokumentet. 1. Bruk integralet ∞

Z

e−tx dx

F (t) := 0

˚ vise at til a Z n! = 0

hvor n ∈ N.



xn e−x dx

III

6 Derivasjon under integraltegnet 2. Bestem og integralet Z



F (t) := 0

xa − xb dx log x

via derivasjon under integraltegnet. Vis og at derivasjonen er lovlig. 3. Vis at ∞

Z J(a, b) :=

a cos2

0

˚ beregne og bruk dette til a Z ∞ I :=

1 π dx = √ 2 x + b sin x 2 ab

1 a cos2

0

x + b sin2 x

2 dx

. Her er a og b reelle tall. 4. Bestem følgende integral π

Z

 log 1 − 2α cos(x) + α2 ,

ϕ(α) := 0

hvor |α| = 6 1 og α ∈ R. 5. Vi definerer følgende integral Z T (u) = 0

1

√ arctan(u 2 + x2 ) √ dx (1 + x2 ) 2 + x2

˚ vise at Bruk dette integralet til a √ Z 1 arctan 2 + x2 5π 2 √ dx = . 2 2 96 0 (1 + x ) 2 + x Dette integralet kalles for Ahmeds Integral [? ], og senere skal vi og se p˚ a generaliseringer av integralet.

163

164

III

7 Uendelige rekker

3.7

UENDELIGE REKKER

Ofte er kan integraler forenkles betraktelig om en kan omskrive integralet ˚ gjøre dette p˚ ˚ skrive ut taylorrekka til hele til en sum. En vanlig m˚ ate a a er a eller deler av integranden. For a˚ konkretisere la oss se p˚ a følgende integral, som kanskje mange kjenner igjen fra tidligere 1

Z 0

log(1 − z) dz . z

Dette har blitt beregnet før som − Li2 (1). La oss n˚ a heller skrive ut taylorrekka til logaritmen slik at vi f˚ ar Z

1

log(1 − z) dz = z

0

1

Z 0

∞ X zn − n n=1

1 z

!

Z dz = −

∞ 1X

0 n=0

zn dz n+1

˚ kunne n˚ Som vi ser hadde det vært veldig behagelig a a bytte rekkefølge p˚ a summasjonen og integrasjonen. Følgende teorem gir oss en grunn n˚ ar vi kan gjøre dette Theorem 3.7.1. (Monoton konvergens teoremet) Anta at funksjonene f1 , f2 , f3 , . . . er positive p˚ a et interval (a, b) (Alts˚ a at fk ≥ 0 for enhver x). Da er Z

∞ bX

fn (x) dx =

a n=1

∞ Z X n=1

b

fn (x) dx

a

som har gyldighet for negative funksjoner. ˚ bruke teoremet f˚ Ved a ar vi n˚ a relativt direkte at Z 0

1

∞ Z 1 ∞ X X log(1 − z) zn 1 π2 dz = − dz = − =− 2 z n+1 n 6 n=0 0 n=1

˚ sammenligne med verdien vi fikk tidligere for Li2 (1) kan vi anta at bytte Ved a ˚ være sikker ser vi at xn /n + 1 er positiv for alle verdier, og vi var lovlig. For a kan puste lettet ut. Eksempel 3.7.1. Vi ser n˚ a p˚ a et liknende integral hvor a > 0 Z I :=

1

 log 1 + e−ax dx

0

hvor a > 0. Vi ser herfra p˚ a taylorrekka til log(1 + x) som konvergerer for |x| < 1.

III

7 Uendelige rekker

165

Heldigvis s˚ a er e−ax < 1 p˚ a x ∈ [0, 1] n˚ ar a > 0. Direkte f˚ ar vi da Z ∞  I= log 1 + e−ax dx 0 ∞ ∞X

(−1)n+1 −n·ax e dx n 0 n=1  ∞ ∞ X e−n·ax (−1)n+1 = − dx n na 0 n=0 Z

=

∞ 1 X (−1)n−1 (0 − 1) dx a n=1 n2   1 1 1 = 1 − 2 + 2 + ... a 2 3 2 π = 12a

=

Eksempel 3.7.2. Z

1

log(1 − z) log z dz = 2 −

I := 0

π2 6

Bevis. Skriver ut taylorrekka til log(1 − z) slik at integralet kan skrives som ! Z 1 ∞ X zn I= dz log z n 0 n=1 Z 1 ∞ X 1 = z n · log z dz n 0 n=1 ( ) 1 Z 1 ∞ X z n+1 log z 1 zn dz − = n z+1 0 n+1 0 n=1 ∞ X 1 1 = · n (n + 1)2 n=1

=

∞ X 1 1 − n n+1 n=1



!

= 1 + 1 − ζ(2) = 2 −



X 1 1 − + 2 (0 + 1) (n + 1)2 n=0

!

π2 6

Her ble det brukt i andre overgang at taylorrekka til log(1 − z) er analytisk p˚ a (0, 1) og i siste mellomregning ble det benyttet at første sum er en teleskoperende rekke slik at         1 1 1 1 1 1 1 S = 1− + − + ... + − + − 2 2 3 k−1 k k k+1 1 =1− k+1 slik at n˚ ar k → ∞ s˚ a vil S → 1. I siste sum brukte en at S(1) = −a0 + S(1).

166

7 Uendelige rekker

III

I gjennom denne delen har vi gjentatte ganger benyttet oss av Basel pro˚ bestemme følgende blemet, mer eller mindre bevisst. Problemet g˚ ar ut p˚ aa sum   ∞ X 1 1 1 1 = lim 1 + + + . . . + n→∞ n2 4 9 n2 n=1 ˚ bestemme summen p˚ men det blir først i neste seksjon at vi viser en m˚ ate a a. ˚ avslutte med et lemma som vil hjelpe oss senere. Her nøyer vi med a Lemma 3.7.1. ∞ ∞ X 4 X 1 1 = 2 n 3 n=0 (2n + 1)2 n=1

˚ kalle summen for S og deler den i odde og like ledd Bevis. Vi begynner a S :=

∞ ∞ ∞ X X X 1 1 1 = + 2 2 n (2n) (2n + 1)2 n=1 n=1 n=0

Herfra ser vi nærmere p˚ a første sum, s˚ a ∞ X

∞ 1 1 X 1 S = = (2n)2 4 n=1 n2 4

n=1

Bruker vi dette i forrige likning f˚ ar vi n˚ a at ∞ ∞ ∞ X X 1 1 4 X 1 1 ⇒ S= = S= S+ 2 2 4 (2n + 1) n 3 (2n + 1)2 n=0 n=1 n=0

˚ vise. som var det vi ønsket a Oppgaver Z 6. Bestem integralet 0



cos log x dx .

III

8 Dobbel Integraler

3.8

167

DOBBEL INTEGRALER

I denne delen skal vi fokusere hovedsaklig p˚ a dobbeltintegral og se hvordan ˚ løse vanskelige integral. Metoden kan delvis bli sammenliknet de kan hjelpe a ˚ derivere under integraltegnet. med a Det kanskje viktigste theoremet n˚ ar det kommer til evalueringen av slike dobbeltintegraler er følgende. Theorem 3.8.1. (Fubini’s Teorem.) Anta at X og Y er to lukkede m˚ albare rom. Anta videre at f (x, y) er X × Y m˚ albart. Da er   ZZ Z Z Z Z f (x, y) d(x, y) = f (x, y) dx dy = f (x, y) dy dx X×Y

Y

X

X

Y

hvis og bare hvis følgende gjelder Z Z f (x, y) d(x, y) < ∞ 1) X×Y

Her kan gjerne X og Y eksempelvis være relle interval X = [0, 1]. Men theoremet holder og for mye mer abstrakte rom. Dette theoremet er langt mer abstrakt enn hva vi trenger, og et bevis kan finnes i [? , s. 163]. Essensen er at vi ikke kan bytte om integralgrensene n˚ ar det passer oss. Et enkelt eksempel p˚ a dette vist under Eksempel 3.8.1. La oss først se p˚ a følgende funksjon f (x, y) =

x2 − y 2 (x2 + y 2 )2

og integralet over enhetskuben S = [0, 1] × [0, 1]. Ved a˚ regne ut absoluttverdien av funksjonen f˚ as  Z 1Z 1 Z 1 Z y Z 1 y 2 − x2 x2 − y 2 |f (x, y)| dx dy = dx + dx dy 2 2 2 2 2 2 0 0 0 0 (x + y ) y (x + y )  Z 1 1 1 1 = + − 2 dy 2y 2y y +1 0 Z 1 Z 1 1 1 = dy − dy. 2 0 y 0 1+y Slik at absoluttverdien av integralet g˚ ar mot uendelig. For a˚ g˚ a mellomregningene litt mer i sømmene kan en se p˚ a likning (2.53) Her f˚ ar en at Z x2 − y 2 y dx = (x2 + y 2 )2 x2 + y 2 Z x2 − y 2 x dx = − 2 (x2 + y 2 )2 x + y2 Dermed s˚ a blir Z 1 Z 0

Z 0

1

0 1

Z 0

1

 1 Z 1 y x2 − y 2 π dy dx = dx = 2 + y2 (x2 + y 2 )2 x 4 0 0    Z 0 1 x2 − y 2 x π dx dy = dy = − 2 2 2 2 2 (x + y ) x +y 1 4 0

168

III

8 Dobbel Integraler

At vi f˚ ar to ulike verdier kommer av at absoluttverdien ikke er endelig. Akkuratt i dette eksempelet s˚ a er absoluttverdien av integralet ikke endelig fordi funksjonen ˚ forandre p˚ har en singularitet i origo. Ved a a grensene forandrerer en ogs˚ a hvordan en nærmer seg dette punktet med sine riemansummer. Oppsumert kan en alts˚ a si at hvis ZZ |f (x, y)| d(x, y) = ∞ S

(der S = [a, b] × [c, d]) s˚ a kan godt b

Z

d

Z

Z

a

d

b

Z

og

f (x, y) dy dx

f (x, y) dx dy

c

c

a

ha ulike verdier. Som en ap`ertiff p˚ a det som kommer senere, tar vi atter en kikk p˚ a det klassiske Gaussiske integralet ∞

Z

1 √ π

2

e−x dx = 1 ,

−∞

Som er essensiel innen blant annet sannsynlighetsregning. Fra før har vi allerede vist integralet ved hjelp av gamma og betafunksjonen. Den klassiske utledningen er derimot ved dobbeltingralet og vist i det p˚ afølgende eksempelet Eksempel 3.8.2. Vi definerer følgende integral, og ser p˚ a kvadratet. Z



I :=

e

−x2

Z

2



dx ⇒ I =

−∞

e

 Z

−x2



e

dx

−∞

−x2

 dx

−∞

˚ si n˚ Variabelnavnet har ingenting a ar vi integrer, s˚ a vi lar heller y være integrasjonsvariabelen i siste integral og f˚ ar 2

Z



I =

−x2

e



 Z dx

−∞

e

−y 2

 dy

−∞

Z



Z



= −∞

2

2

e−x e−y dx dy

−∞

Merk her ble integrasjonsrekkefølgen byttet, og i følge Fubinis sats er dette ikke lov om ikke absoluttverdien av integralet konvergerer. Videre regning gir Z

2



e−(x

I =

2

+y 2 )

dx dy .

−∞

Herfra bytter vi til polarkoordinater, hvor x2 + y 2 = r2 og r dr dθ = dx dy Z



Z



−r 2

re

= 0

0

 ∞ 1 −r2 dr dθ = 2π − e =π 2 0

√ Slik at vi f˚ ar I = π som ønsket. At vi kan bytte om grensene følger fra at ˚ se dette s˚ funksjonen er absolutt konvergent. For a a kan vi benytte at e−µ >

III

8 Dobbel Integraler

169

0 ∀ µ ∈ R s˚ a Z



I= −∞ −1

Z −x2 dx = e

Z

e

=

−x

2

Z dx +

−∞ 1 −x2

e

2

−x · e−x dx +



2

e−x dx Z

2

e−x dx

1

Z

1

e0 dx +

−1

−∞



dx +

−1

−∞ −1

Z



Z



2

x · e−x dx

1

= 2 + e−1 Og siden 2 + e−1 < ∞ s˚ a konvergerer integralet absolutt. Her ble det bla brukt at funksjonen er symmetrisk og synkende. I videre utledninger vil det stort sett ikke bli vist at integralet konvergerer absolutt. Da det antas fremmover at integralene konvergerer absolutt om ikke annet blir sagt. Men det er noe a˚ ha i tankende, og uten tvil en god øvelse a˚ vise at integralet konvergerer absolutt. Eksempel 3.8.3. En alternativ m˚ ate a˚ beregne integralet p˚ a er a˚ studere følgende integral ZZ  J= x exp −x2 (y 2 + 1) d(x, y) S

hvor S = [0, ∞) × [0, ∞). P˚ a den ene siden s˚ a er Z ∞Z ∞  J= x exp −x2 (y 2 + 1) dx dy 0

0

˚ bruke substitusjonen u = x2 (y 2 + 1) f˚ ved a as Z ∞  Z ∞  1 dy −u = e du 2 y2 + 1 0 0 ∞  −u 0 1 = arctan y 0 · e ∞ 2    1π = −0 · 1−0 2 2 π = 4 ˚ ha motsatte grenser via t = xy f˚ Beregner vi derimot integralet ved a as Z ∞Z ∞  J= x exp −x2 (y 2 + 1) dy dx Z0 ∞ Z0 ∞  = exp −t2 − x2 dt dx 0 0 Z  Z ∞  ∞  2  2 = exp −t dt · exp −x dx 0

 =

I 2

√ Slik at I = π som før.

0

2

170

III

8 Dobbel Integraler

˚ anta at et integral er et dobbelintegral, som har blitt integrert feil Det a veier mye brukt. Alts˚ a at en har integrert først x ogs˚ a med hensyn p˚ a y i ste˚ skrive om integralet som et dobbeltintegral defor motsatt. Teknikken blir da a igjen, snu grensene og beregne det forh˚ apentligvis enklere integralet. Dette blir forh˚ apentligvis klarere i det følgende eksempelet I forbindelse med gamma- og betafunksjonene ble følgende integral betraktet Z ∞ exp(−ax) − exp(−bx) dx I= xn 0 Her vil vi studere spesialtilfellet hvor n = 1, da har vi Z ∞ exp(−ax) − exp(−bx) I= dx x 0 Z ∞Z b e−y·x dy dx = a ∞

0

Z

b

Z

e−y·x dx dy

= a

Z

0 b

= a

Z



e0 e−xy − lim x→∞ y y

 dx

b

dy y a a = log b =

Tilsvarende metode kan bli benyttet p˚ a en rekke integral. Faktisk har vi følgende teorem Theorem 3.8.2. (Frullani’s Theorem). La f ∈ C 1 (kontinuerlig deriverbar), og la f (∞), f (0) ∈ R. Da er   Z  f (ax) − f (bx) b dx = f (0) − ` log , ∀ a, b ∈ (0, ∞) , x a der limx→∞ f (x) = ` og limx→0 = f (0). Bevis. Theoremet gjelder under svakere betingelser, men gitt de ovenfor kan et artig dobbeltintegral benyttes. Vi definerer ZZ Y = −f 0 (xy) dx dy D

 der D = (x, y) ∈ R2 : x ≥ 0 , a ≤ y ≤ b . Vi beregener integralet begge veier. P˚ a den ene siden s˚ a er  0 Z b Z ∞ Z b f (xy) 0 Y = −f (xy) dx dy = dy y a 0 a ∞   Z b  f (0) − ` b = dy = f (0) − ` log y a a

III

8 Dobbel Integraler

171

˚ bytte om integrasjons rekkefølgen f˚ Ved a as derimot ! a Z ∞ Z b Z ∞ Z ∞ f (xy) f (ax) − f (bx) 0 −f (xy) dy dx = Y = dx dx = x x a 0 0 0 b og vi er ferdige. For at bytte av integrasjonsrekkefølgen skal være lovlig m˚ a vi ha at |f 0 (xy)| < 0 hvor y ∈ [a, b] og x ∈ (0, ∞). Flere steder blir teoremet gitt med at ` = 0, men det er ikke vanskelig a˚ finne eksempler p˚ a hvor f (0) = 0 og ` 6= 0. Eksempel 3.8.4.     Z ∞  q arctan(px) − arctan(qx) π π p log = 0− = log x 2 p 2 q 0 Hvor f (0) = arctan 0 = 0 og ` = arctan(∞) = π/2. N˚ a har vi tatt for oss Fubini og Frullani integral, s˚ a da er det passende at vi tar for oss Fresnel integralene avslutningsvis. Theorem 3.8.3. (Fresnel’s Integral) er definert som følger r Z ∞ Z ∞ π 2 2 sin x dx = cos x dx = 8 −∞ −∞ Dette kan bevises ved hjelp av eksempelvis kompleks analyse, men her brukes ˚ vise dette trengs et par lemma først: selvsagt dobbeltintegralet. Men for a Lemma 3.8.1. Z ∞ 2 1 1 √ = √ e−tx dx π −∞ t √ √ as direkte Bevis. Bruk substitusjonen u = t x ⇒ dx = du/ t , da f˚ Z ∞ Z ∞ 2 2 1 1 1 √ e−tx dx = √ e−u du = √ π −∞ πt −∞ t √ Der det ble benyttet at verdien av det gaussiske integralet over R er π som ble vist i eksempel (3.8.2) og eksempel (3.8.3). Lemma 3.8.2. Z ∞ 2 e−tx sin t dt = 0

1 1 + x4

Z og 0



2

e−tx cos t dt =

x2 1 + x4

Bevis. Det har blitt vist i Del II (proposisjon (2.7.1)) at Z  e−αt  I = e−αt sin βt dt = − 2 α sin βt + β cos βt α + β2 Z  e−αt  J = e−αt cos βt dt = β sin βt − α cos βt 2 2 α +β

172

III

8 Dobbel Integraler

Resutatet kan vises ved a˚ skrive om p˚ a kompleks form eller via delvis integrasjon. Med innsatte grenser f˚ as10 Z ∞ β I= e−αt sin βt dt = 2 (3.61) α + β2 Z0 ∞ α e−αt cos βt dx = 2 J= (3.62) α + β2 0 ˚ sette β = 1 og α = x2 . og beviset fullføres ved a Lemma 3.8.3. Z 0



1 π/n = 1 + xn sin(π/n)

Bevis. En mer generell form er vist før i likning (3.26), men vi gjennomg˚ ar ˚ bruke substitusjonen x 7→ 1 + uβ ⇒ u = likevell beviset i korte trekk. Ved a (x − 1)1/β kan integralet skrives som Z ∞ Z ∞ 1 1 1 I= (x − 1)(1/n)−1 dx du = n 1+u β 1 x 0 hvor β er et reellt positivttall og substitusjonen Settes n˚ a x = 1/v f˚ as   Z 1 1 1 1 1 I= v −1/n (1 − v)−1+1/n dv = B − + 1, n 0 n n n Ved a˚ benytte seg av definisjonen til betafunksjonen, og Eulers refleksjonsformel f˚ as I=

1 Γ (1 − 1/n) Γ (1/n) π/n = n Γ (1) sin (π/n)

og dette fullfører beviset. Alternativt er integralet lik Mellin-transformasjonen av f (x) = xn + 1 med s = 1. ˚ beregne Fresnel integralene, definerer som Bevis. Vi er n˚ a endelig klar til a følgende Z ∞ Z ∞ F1 := sin x2 dx og F2 := cos x2 dx −∞

−∞

2

Via substitusjonen t = x omskrives integralene til Z ∞ Z ∞ sin t cos t √ dt , √ dt F1 = F2 = t t 0 0 Benyttes n˚ a lemma (3.8.1) f˚ as dobbeltintegralene Z ∞Z ∞ Z ∞Z ∞ 2 2 2 2 F1 = √ e−tx sin t dx dt , F2 = √ e−tx cos t dx dt π 0 π 0 0 0 10 Disse

likningene beskriver alts˚ a laplace-transformasjonen av sin βt og cos βt.

III

8 Dobbel Integraler

173

Snus grensene og lemma (3.8.2) benyttes s˚ a f˚ as Z ∞Z ∞ Z ∞ 2 2 2 dx e−tx sin t dt dx = √ F1 = √ π 0 π 0 1 + x4 0 Z ∞Z ∞ Z ∞ 2 2 2 x2 F2 = √ e−tx sin t dt dx = √ dx π 0 π 0 1 + x4 0 Det første integralet kan beregnes, men det vises først at Z ∞ Z ∞ 2 Z ∞ 2 Z ∞ dx x dx x dx dx = ⇒ − =0 4 4 4 1 + x 1 + x 1 + x 1 + x4 0 0 0 0 ˚ se dette kan vi enten bruke lemma (2.6.1) med n = 4 eller legge merke til For a at   Z ∞ 2 Z 0 Z 1 2 x −1 (1/u)2 − 1 du u −1 dx = − = − du 4 4 1 + x4 u2 1 1 1 + (1/u) 0 1+u ˚ dele integralet i to s˚ Ved n˚ aa a har en Z 0

1

x2 − 1 dx + 1 + x4

Z



1

x2 − 1 dx = 1 + x4

Z 0

1

x2 − 1 dx − 1 + x4

Z

1

0

u2 − 1 du 1 + u4

S˚ a I = 0 som ønsket. Dette medfører at F1 = F2 og 2 F1 = F2 = √ π

Z 0



dx 2 π/4 = √ · = 4 1+x π sin(π/4)

r

π 8

Hvor lemma (3.8.3) ble benyttet. Dette fullfører beviset. Som lovet i forrige del tar vi avslutningsvis og ser p˚ a følgende problem. Eksempel 3.8.5. (Basel problemet) Problemet er gammelt og ber oss beregne ˚ summen av inversen av alle kvadrattallene. Det var først Euler som klarte a bestemme summen ∞ X 1 π2 = , n2 6 n=1

gjennom forholdsvis lange utregninger som kan være noe vanskelig a˚ henge med p˚ a. Vi fokuserer heller p˚ a et bevis som kom senere av J.D Harper. Bevis. Vi tar utgangspunkt i følgende integral ZZ x J := d(x, y) 2 2 2 D (1 + x ) (1 + x y )  hvor D (x, y) ∈ R2 : x ≥ 0, 0 ≤ y ≤ 1 . P˚ a den ene siden har vi at integralet kan skrives som Z 1  Z ∞Z 1 Z ∞ 1 dy x dy dx = dx J= 2 2 2 2 2 x(1 + x2 ) 0 0 (1/x) + y 0 0 (1 + x ) (1 + x y )

174

III

8 Dobbel Integraler

det innerste integralet kan løses via substitusjonen y = x−1 tan u som gir 1  ∞ Z ∞ Z ∞ arctan(xy) arctan x arctan2 x π2 J= dx = = = 1 + x2 1 + x2 2 8 0 0 0 0 Bruker vi n˚ a fubinis sats og regner ut integralet ved motsatte grenser f˚ as Z ∞Z 1 x J= dx dy 2 ) (1 + x2 y 2 ) (1 + x 0 0 Det innerste integralet kan løses via delbrøkoppspalting som gir gir   2x x 1 2y 2 x − = (1 + x2 ) (1 + x2 y 2 ) 2 (y 2 − 1) 1 + x2 y 2 1 + x2     1 d d 2 2 2 = log 1 + x y − log 1 + x 2 (y 2 − 1) dx dx Der y ble betraktet som en konstant. Setter vi inn og forenkler, skrives integralet om til   ∞ Z 1 Z ∞ 1 + x2 y 2 log y 1 log dy = dy J= 2 2 2 2 (y − 1) 1+x 0 y −1 0 0  Hvor det ble benyttet at n˚ ar x → ∞ s˚ a vil log (x−2 +y 2 )/(x−1 +1) → log(y 2 /1) = 2 log y og x → 0 s˚ a vil log(1/1) = 0. Vi har alts˚ a n˚ a vist at Z 1 2 log y π dy = 2 8 0 y −1 Som ikke var akkuratt det vi skulle ha... En kan n˚ a legge merke til at y 2 < 1 for 2 y ∈ (0, 1) slik at vi kan rekkeutvikle 1/(1 − y ) som en geometrisk serie. Da f˚ as Z 1 Z 1 ∞ ∞ Z 1 ∞ X X X 1 log y 2n 2n dy = − log y y = − y log y dy = − 2 1−y (2n + 1)2 0 0 n=0 n=0 0 n=0 benyttes n˚ a lemma (3.7.1) f˚ ar vi at ∞ X n=0

1 π2 = (2n + 1)2 8

∞ 4 X 1 4 π2 = · 3 n=0 (2n + 1)2 3 8 ∞ X 1 π2 = 2 n 6 n=1

˚ vise. som var det vi ønsket a Oppgaver 7. Beregn følgende integral Z I= 0

hvor p, q ∈ R.



1 log x



p + qe−ax p + qe−bx

 dx

III

8 Dobbel Integraler

175

8. Beregn følgende integral Z I= 0



sin(px) · sin(qx) dx 2

hvor p, q ∈ R. 9. Vis at Z 0



e2x − ex dx = x(ex + 1)(e2x + 1)

og bestem integralet.

Z 0



e−x − e−2x dx x

176

3.9

9 Kompleks Integrasjon

KOMPLEKS INTEGRASJON

III

III

9.1 Typer integraler

3.9.1

TYPER

177

INTEGRALER

˚ regne ut de ulike typene La oss først ta to sm˚ a lemma som vil hjelpe oss a integral Lemma 3.9.1. (M-L ulikheten) La f være en kompleks, kontinuerlig funksjon p˚ a konturen Γ og dersom absoluttverdien |f (z)| er mindre enn en konstant M for alle z p˚ a Γ da er Z F (z) dz ≤ M · `(Γ) . Γ

Hvor `(Γ) betegner buelengden til Γ og M = sup |f (z)|11 . z∈Γ

Bevis. Fra den kontinuerlige trekantulikheten har en at Z Z f (x) dx ≤ |f (x)| dx fra dette følger det at integralet kan skrives om som Z Z b  f (z) dz = f γ(t) γ(t) ˙ dt C a Z b Z b Z  γ(t) ≤ ˙ dt ≤ M ˙ dt = M dz = M L f γ(t) γ(t) a

a

C

hvor kurven ble parametrisert via γ(t), og M er den maksimale teoretiske verdien f (γ) kan ha. I andre overgang ble det brukt direkte at |a · b| ≤ |a| |b|

i CD

−R

R

x

CR x −R

CR

R

CD ∗ i

(a) Illustrasjonen av konturen i det øvre halvplanet

(b) Illustrasjonen av konturen i det nedre halvplanet

Figur 3.2 11 Dersom

et maksimum eksisterer s˚ a vil supremum og maksimum være like. Derimot s˚ a er det noen ˚ prate om en minste teoretisk ganger maksimum ikke eksisterer, men det likevell er ønskelig√a øvre grense. Et klassisk eksempel er mengden S = {0 < r < 2 , r = Q}, her finnes ikke noe maksimum siden en alltid kan finne et litt større ar mot √ √ rasjonellt tall 1.4, 1.41, 1.414, . . . Følgen g˚ 2 , men aldri helt kommer dit s˚ a sup S = 2 .

178

III

9 Kompleks Integrasjon

Rasjonale funksjoner Theorem 3.9.1. La P (x) og Q(x) være polynomer slik at deg(Q) ≥ deg(P ) + 2 da er   Z ∞ m X P (x) P (x) dx = 2πi , zk Res Q(x) −∞ Q(x) k=1

hvor z1 , . . . , zm er singularitene til P (x)/Q(x) i det øvre halvplanet y > 0. Bevis. Her gis en kort skisse av et bevis for theoremet. Beviset deles inn i tre deler. (1) vi viser at integralet konverger. (2) Vi lager en kontur i det komplekse ˚ avslutte beviset brukes planet, og viser at ’bue’-integralet er null. (3) For a residue theoremet. (1) For det første har ikke Q(x) noen nullpunkter p˚ a den reelle aksen, slik at P (x)/Q(x) er definert p˚ a hele R. Vi ser først p˚ a om integralet konvergerer absolutt, for dette impliserer konvergens. For store verdier av x, s˚ a domineres polynomene av deres første ledd, s˚ a Z Z ∞ P (x) C|x|n−2 dx Q(x) ≤ D|x|n −∞ og siden integralet p˚ a høyre side konvergerer, konvergerer ogs˚ a integralet v˚ art. ˚ være helt formelle burde vi ha delt opp integralet i to, og vist at integralet For a over (0, ∞) og (−∞, 0) konvergerer hver for seg. (2) Vi integrerer funksjonen langs konturen vist i figur neger, for s˚ a a˚ la R → ∞. Da er integralet Z Z Z P (x) P (x) P (x) = + R Q(x) C1 Q(x) C Q(x) ˚ vise at integralet i det komplekse planet, som følger kurven Γ er M˚ alet er n˚ aa ˚ bruke M L-ulikheten s˚ null. Ved a a er Z P (x) P (x) C|z|n−2 C ≤ πR sup ≤ πR ≤ πR n D|z| DR2 z∈Γ Q(x) C1 Q(x) som g˚ ar mot null n˚ ar R → ∞. (3)

Fra Cauchys integral formel har en n˚ a I  Z ∞ Z P (x) P (x) P (x) dx = lim dz − dz R→∞ −∞ Q(x) C Q(x) C1 Q(x)   I m X P (x) P (x) = lim dz − 0 = 2πi Res , zk R→∞ C Q(x) Q(x) k=1

som var det som skulle vises.

III

9.1 Typer integraler

179

Integralet over P (x)/Q(x) konvergerer ogs˚ a dersom deg P + 1 ≤ deg Q, men ulempen er da at integralet over halvsirkelen ikke g˚ ar mot null. Da m˚ a en beregne dette integralet eksplisitt, og det kan være vanskeligere enn det opprinnelige integralet. ˚ bruke Dersom b˚ ade graden til P og Q er partall, s˚ a er P/Q like. Ved a symmetrien omkring x-aksen f˚ ar en da   Z ∞ m X P (x) P (x) dx = πi , zk Res Q(x) Q(x) 0 k=1

slik at vi ikke bare kan beregne integraler over R, men ogs˚ a R+ og R− . Theoremet sier at vi m˚ a studere polene til P/Q i det øvre halvplanet, men en kunne like gjerne studert polene i det nedre halvplanet. Da m˚ a en integrere ’motsatt’ vei, og fra omløpstallet f˚ ar en et negativt bidrag fra hver pol, slik at   Z ∞ n X P (x) P (x) dx = −2πi , zk Res Q(x) −∞ Q(x) k=1

hvor zn er polene i det nedre halvplanet. En kan alts˚ a fritt velge det halvplanet som har færrest eller penest singulariter. Merk og at det ikke er nødvendig at P og Q er polynomer, men at det holder at P er analytisk i det halvplanet en studerer. Eksempel 3.9.1. I dette eksempelet skal vi beregne følgende integral Z ∞ 2 x +2 dx x4 + 1 0 ved hjelp av kompleks analyse. Integralet er et spesialtilfellet av ??, men brukes her for a˚ vise frem kompleks integrasjon. Siden teller er av grad 4, mens nevner er av grad 2, medfører at integralet konvergerer. Videre s˚ a er integralet symetrisk omkring x aksen, eller like s˚ a Z ∞ 2 Z x +2 1 ∞ x2 + 2 dx = dx (3.63) x4 + 1 2 −∞ x4 + 1 0 ˚ benytte seg av theorem (3.9.1) er oppfyllt. Integralet er Slik at vilk˚ arene for a dermed likt   Z ∞ 2 m X x +2 P (x) dx = 2πi , z Res (3.64) k 4 Q(x) −∞ x + 1 k=1

Singuaritenene til funksjonen er hvor z 4 = −1, n˚ a kan en skrive −1 = ei(π+2πk) s˚ a røttene blir z0 z2

= eiπ/4 = ei5π/4

z1 z2

= ei3π/4 = ei7π/4

Av disse ligger bare z0 og z1 i det øvre halvplanet og i tillegg er de enkle singula˚ bruke ?? fra ?? kan residyene skrives som riterer. Ved a  2  z +2 z2 + 2 1 1 Res 4 , zk = lim = + z→zk z +1 4z 3 4zk 2zk3

180

9 Kompleks Integrasjon

III

˚ dele likning (3.64) p˚ Ved a a 2, og s˚ a benytte seg av likning (3.63) har en at      Z ∞ 2 x +2 P (x) P (x) dx = πi Res , z0 + Res , z1 x4 + 1 Q(x) Q(x) 0     1 1 1 1 + = πi + + 4z0 2z03 4z1 2z13 Merk at n˚ a kan en utnytte symmetrien til enhetsrøttene en har at z13 = z0 og at 3 z0 = z1 . Dette gir at       Z ∞ 2 x +2 1 1 1 1 3π 1 1 dx = πi + + + = i + x4 + 1 4z0 2z1 4z1 2z0 4 z1 z0 0 ˚ skrive om røttene til deres real og kompleks del ved hjelp av av eulers Ved a formel eiω = cos ω + i sin ω f˚ as     Z ∞ 2 3π 1 i 1 i 3π x +2 √ √ √ √ dx = i − + − − = √ 4+1 x 4 2 2 2 2 2 2 0 ˚ g˚ som var det som skulle vises. For a a detaljene litt i sømmene s˚ a har en at z13 = ei9π/4 = e−2πi ei9π/4 = eπ/4 = z0 og tilsvarende s˚ a er z03 = ei3π/4 = z1 som en og kan se siden røttene er symmet˚ regne ut røttene eksplisitt f˚ risk om den imaginære aksen. Ved a ar en fra eulers formel π 1 i 1 π − sin = √ −√ = e−iπ/4 = cos z0 4 4 2 2 i 1 3π 3π 1 −i3π/4 =e = cos − sin = −√ − √ . z1 4 4 2 2 ˚ bestemme integralet noe enklere kunne en brukt symmmetrien av z0 og For a z1 .   Z ∞ 2  x +2 3π 1 1 3π 3π π dx = i + =− i 2iImz1 = sin 4 x + 1 4 z z 4 2 4 1 0 0 ˚ bestemme en trigonometrisk verdi. som gjorde at vi bare trengte a ˚ bestemme integralet ble det kun brukt enkel algebra, trigonometriske For a ˚ beregne verdier og at i2 = −1. P˚ a mange m˚ ater er dette en enklere m˚ ate a integraler over brøker, enn det som ble vist i seksjon to, ulempen er selvsagt at ˚ bruke disse nye verktøyene en trenger en dypere matematisk innsikt for a Eksempel 3.9.2. I denne delen skal vi studere et kjent integral Z ∞ dx In = 1 + xn 0 der n ∈ N2 er et partall. Igjen tar vi utgangspunkt i den kjente konturen som er vist i figur (3.3). Grunnen til at vi kan bruke denne konturen er at integralet er symmetrisk omkring origo Z ∞ Z dx 1 ∞ dx In = = . n 1+x 2 −∞ 1 + xn 0

III

9.1 Typer integraler

181

i CD x −R

R

CR

Figur 3.3: Sløyfen C = CR ∩ CD , hvor polene til 1/(1 + x10 ) i det øvre halvplan er markert. Som følger fra at n er like. Integralet langs CR blir som før null da graden av nevner er 2 eller høyre, mens graden til teller alltid er konstant 1. Theorem (3.9.1) gir da at   Z m X 1 1 ∞ dx In = dx = πi Res , zk (3.65) 2 0 1 + xn 1 + xn k=1

hvor vi betrakter residyene til funksjonen i det øvre halvplanet. Først m˚ a vi se hvor xn + 1 = 0, og dette blir bare enhetsrøttene zk = eiπ(2k+1)/n ,

0≤k≤

n −1 2

˚ regne ut en residye kan vi igjen bruke ?? fra ?? slik at For a Res(zk ) = lim

z→zk

1 1 = − eiπ(2k+1)/n (1 + z n )0 n

hvor minustegnet kommer av at vi betrakter residyene i det øvre halvplanet. N˚ a har vi at   n/2−1 m X X 1 eiπ(2k+1)/n Res , z − = k 1 + xn n k=1 k=0 " # n/2 n/2−1 ei2π/n −1 eiπ/n X  i2π/n k eiπ/n =− e =− n n ei2π/n − 1 k=0 ˚ n˚ hvor den siste overgangen følger fra den geometriske rekken. Ved a a forenkle algebraen kan en skrive summen av residyene som m X k=1

Res (zk ) =

eiπ/n 2 i2π/n n e −1

=

1 2i 1 1/n · iπ/n = . −iπ/n in e i sin(π/n) −e

˚ sette inn dette i hvor bare (z + z −1 )/2i = (eiz + e−iz )/2i sin z ble brukt. Ved a likning (3.65) f˚ ar en n˚ a   Z ∞ m X dx 1 π/n = πi Res , z . (3.66) k = n n 1 + x 1 + x sin(π/n) −∞ k=1

som var det som skulle vises.

182

III

9 Kompleks Integrasjon

Men stemmer likning (3.66) dersom n ikke er odde? Svaret er overraskende nok ja, men da m˚ a vi bruke en alternativ kontur som vi skal se p˚ a senere. Dette er fordi dersom n ikke er like, s˚ a er ikke funksjonen symmetrisk. Den neste type integral vi skal se p˚ a er p˚ a formen p(x)eiax /q(x), men før det et viktig lemma Lemma 3.9.2. (Jordan’s lemma) La f være en kontinuerlig funksjon i det komplekse planet, definert p˚ a følgende kontur CR = {z : z = Reiθ , θ ∈ [0, α]} med radius R > 0, 0 < α ≤ π og sentrum i origo. Dersom funksjonen f er p˚ a formen f (z) = eiaz g(z) , z ∈ CR og a > 0, s˚ a er integralet over CR begrenset av Z  π sup g Reiθ f (z) dz ≤ a θ∈[0,π] CR med likhet kun n˚ ar g(z) er identisk lik null. Merk at dersom a = 0 følger det fra ML-ulikheten at Z   2πR f (z) dz ≤ sup g Reiθ ≤ πR sup g Reiθ α θ∈[0,π] θ∈[0,π] CR Et helt likt argument holder for en kontur i det nedre halvplanet dersom a < 0. Dette er p˚ a mange m˚ ater en utvidet utgave av jordans originale lemma siden da er α = π. Sett inn ref til linjeintegral

˚ bruke substitusjonen z = Reiθ = R(cos θ + i sin θ), da Bevis. Vi begynner med a blir grensene 0 til α. Z Z α  f (z) dz = g Reiθ eiaR(cos θ+i sin θ) iReiθ dθ CR 0 Z α  =R g Reiθ eaR(i cos θ−sin θ) ieiθ dθ . 0

Vi kan n˚ a ta av integralet og bruke den kontinuerlige trekant R absoluttverdien R ulikheten f ≤ |f | Z Z α Z π  ≤R g Reiθ e−aR sin θ dθ ≤ RMR f (z) dz e−aR sin θ dθ . (3.67) CR

0

0

iz

Hvor det ble brukt blant annet at |e | = | cos z + i sin z| = | cos z| = 1. Siden integralet er positivt s˚ a betrakter vi heller omr˚ adet fra 0 til π. Vi innførte og  MR = sup g Reiθ θ∈[0,π]

˚ forenkle notasjonen litt. Da kan ulikheten skrives som for a Z Z π Z π/2 −aR sin θ ≤ RMR f (z) dz e dθ ≤ 2RM e−aR sin θ dθ R CR

0

0

III

9.1 Typer integraler

183

hvor vi i siste overgang brukte symmetrien av sin θ = sin(π − θ). Dette kan vises ˚ dele intervalet i to eller bruke likning (2.30) fra proposisjon (2.4.5) med ved a m = 0, n = 1 og f (x) = exp(−aRx). Funksjonen sin θ er konkav p˚ a intervalet θ ∈ [0, π/2]. Alts˚ a at funksjonen ligger over den rette linja mellom endepunktene 2 θ ≤ sin θ , π

0≤θ≤

π 2

(3.68)

˚ vise, men kan sees intiutivt fra figur 8. Ved a ˚ bruke dette overlates til leser a

1 y sin θ 2θ/π θ 0

π/2

Figur 3.4: Figuren viser f (θ) = 2θ/π, og g(θ) = sin 2θ dette f˚ ar vi endelig at Z Z f (z) dz ≤ 2RMR CR

π/2

e−2aRθ/π dθ =

0

 π π 1 − e−aR MR ≤ MR a a

som var det som skulle vises. Fra dette følger det direkte at Korollar 3.9.1. Dersom de eneste singularitetene til f (z) er poler, da er Z lim g(z)eiaz dz = 0 R→∞

CR

forutsatt at a > 0, |f (z)| → 0 n˚ ar R → ∞. For at linjeintegralet over p(z)/q(z) skulle g˚ a mot null, kreves det at p(z)/q(z) ∼ 1/R2 n˚ ar z → ∞. Mens her ser vi at s˚ a lenge p(z)/q(z) g˚ ar mot null, uansett hvor sakte s˚ a vil linjeintegralet over p(z)eiz /q(z) g˚ a mot null. Dette har med ˚ tvinge funksjonen til a ˚ konverge raskere. hvordan eiz bidrar til a Proposisjon 3.9.1. La P (x) og Q(x) være funksjoner slik at P (x)/Q(x) → 0 n˚ ar x → ±∞. Da er   Z ∞ m P (x) iax a X P (x) e dx = 2πi Res , zk (3.69) |a| Q(x) −∞ Q(x) k=1

gitt at Q(x) ikke har noen singulariter p˚ a den reelle aksen. Her er a ∈ R/{0} er en reell konstant ulik null og z1 , . . . , zm er singularitene til P (x)/Q(x) i det øvre halvplanet dersom a > 0 og det nedre halvplanet dersom a < 0.

184

9 Kompleks Integrasjon

III

˚ Dersom P og Q er polynomer m˚ a en alts˚ a ha deg(Q) ≥ deg(P ) + 1 for a sikre konvergens. Dette er et snillere krav enn integraler p˚ a formen P/Q, hvor vi trengte deg(Q) ≥ deg(P ) + 2 for konvergens. Først s˚ a kommer leddet a/|a| fra at i det nedre halvplanet integrerer vi ’motsatt’ vei, og f˚ ar et negativt bidrag fra hver singularitet. Dette leddet defineres gjerne som sign a = a/|a|, og gir oss fortegnet til a. En skisse for beviset gis, og fremmgangsm˚ aten er tilsvarende som i (3.9.1). Vi kan anta at integralet langs den reelle aksen konvergerer siden her er |eiaz | = 1, og vi f˚ ar Z Z Z N N P (x) N P (x) P (x) ix ix dx dx ≤ e ≤ e dx −M Q(x) −M Q(x) −M Q(x) som er samme tilfellet som i (3.9.1), og integralet konvergerer dermed. Merk at eix = |cos x + i sin y| = |cos x| = 1. Vi studerer s˚ a absoluttverdien av integralet langs sirkelbuen Z Z   P (z) iaz P (z) iaz dz ≤ πR sup |P (z)| eiaz e dz ≤ e |Q(z)| z∈C1 CR Q(z) CR Q(z) Legg merke til at |eiaz | = |ea(ix−y) | = e−ay . Dermed s˚ a er Z 1 P (z) iaz e dz ≤ πR 2 · e−ay R C1 Q(z) hvor vi som før benyttet at P/Q ∼ 1/R2 n˚ ar R → ∞. For at vi skal f˚ a samme form som i likning (3.69) m˚ a integralet langs sirkelbuen g˚ a mot null n˚ ar R → ∞12 . Velger vi en halvsirkel i det øvre planet s˚ a vil y → ∞ n˚ ar R → ∞. For her er y den største y-verdien til halvsirkelen. Dersom a < 0, s˚ a vil e−ay g˚ a mot uendelig. Vi m˚ a alts˚ a velge det nedre halvplanet n˚ ar a < 0, slik at y → −∞ n˚ ar R → ∞. Eksempel 3.9.3. Et av de mest klassiske integralene p˚ a denne formen er Z ∞ cos x 1 + x2 −∞ Dette integralet kan skrives som Z ∞  Z ∞ cos x eiz dx = Re dz , 2 2 −∞ 1 + x −∞ 1 + z ˚ bruke eulers formel eiω = cos ω + i sin ω. M˚ ˚ beregne ved a alet vil n˚ a være a integralet p˚ a høyresiden og s˚ a ta realdelen av hva det n˚ a enn er vi f˚ ar som svar. Siden a > 0, s˚ a ser vi p˚ a singularitene til funksjonen i det øvre halvplanet. z1 = i ,

z0 = −i ,

her er det bare z1 som ligger i øvre halvplan, s˚ a residyen blir  iz  e 1 −1 eiz Res , i = lim = e . 2 z→i 2z 1+z 2i 12 Det

er ikke alltid dette er tilfellet og slike integral vil bli diskutert senere.

III

9.1 Typer integraler

185

˚ bruke residue theoremet s˚ Ved a a har vi at "Z # Z Z R eix eiz eiz lim dz = lim + . 2 2 R→∞ Γ 1 + z 2 R→∞ −R 1 + x CD 1 + z I det øvre halvplanet g˚ ar integralet over CD mot null, mens høyresiden er gitt ˚ løse likningen f˚ som 2πi Res(i). Ved a ar en alts˚ a ∞

Z

−∞

eix dx = 2πi Res(i) + lim R→∞ 1 + x2

Z CD

eiz π dx = . 1 + x2 e

˚ ta henholdsvis realdelen og imaineær delen av likningen f˚ Ved a ar en Z ∞ Z ∞ cos x π sin x dx = dx = 0 , og 2 2 e −∞ 1 + x −∞ 1 + x hvor det siste integralet kunne en ha forventet da funksjonen er symmetrisk omkring origo. ˚ Kunne vi heller ha skrevet om 2 cos z = e−iz + eiz ? Faktisk ikke! For a begrense veksten til eiaz m˚ atte vi taktisk velge halvplan, derimot om vi bruker ˚ velge ’rett’ halvplan siden e−iz bl˚ 2 cos z = e−iz + eiz er det umulig a aser opp i iz nedre og e i øvre. Hele diskusjonen med valg av plan er noe meningsløs. Anta at du skal integrere en funksjon cos(−x)P/Q, da kan en helt rett velge e−iz P/Q, men ˚ bruke at cos(−x) = cos x kunne en ogs˚ ved a a like gjerne ha brukt e−iz P/Q. Tilsvarende for sin x s˚ a er sin(−z)xP/Q = − sin zxP/Q siden sin x er odde. Det a˚ velge rett kontur, og det a˚ kunne vite n˚ ar en kan velge er derimot svært viktig. Og for et knippe funksjoner s˚ a kan singularitene i det nedre halvplanet ˚ regne ut enn i det øvre. være enklere a Trigonometriske integral I denne delen skal vi igjen studere integraler p˚ a formen Z



R(cos x, sin x) dz 0

˚ beregne disse p˚ og vise atter en m˚ ate a a. Proposisjon 3.9.2. La R(x, y) være en rasjonell funksjon, da er Z



0



I R(cos x, sin x) dx =

R

z + z −1 z − z −1 , 2 2i



dz iz

|z|=1

˚ bestemme n˚ ˚ bruke Det er noen ganger vanskelig a ar det er fordelaktig a Weierstrass-substitusjon eller proposisjonen ovenfor. En løs tommelfinger regel er at dersom integralet krever delbrøksoppspalting etter Weiersttrass-substitusjonen, ˚ foretrekke. Dersom røttene til nevner har høy multiplier proposisjon (3.9.2) a ˚ foretrekke. sitet kan likevell Weiestrass være a

186

III

9 Kompleks Integrasjon

˚ bruke substitusjonen ovenfor s˚ Ved a a har en Z 2π dx 2 dz = 2 + cos x i 1 + 4z + z2 0 som er mer komplisert a˚ integrere enn fremgangsm˚ aten i eksempel (2.5.5). Som vanlig er dette siste metode som bør testes ut for a˚ løse slike integral, da en som regel kan finne langt mer elegante løsninger. Eksempel 3.9.4. La oss bestemme integralet Z 2π sin2 x dx 2 + cos x 0 ˚ bruke proposisjon (3.9.2) kan integralet skrives som Ved a  2 . " 2 # Z 2π I  z − z −1 sin2 x dz z − z −1 2+ dx = 2 + cos x 2i 2i iz 0 C I 2 2 i (z − 1) = dz 2 2 C z (1 + 4z + z 2 ) Det neste steget er a˚ bestemme hvor residuene til f (z) befinner seg. Vi ignorerer alts˚ a faktoren i/2 frem til slutten. Konturen C er alts˚ a med klokken omkring enhetssirkelen som vist i figur (3.5). Singularitene til funksjonen er

i

vokser

synker

synker

vokser

x

Figur 3.5 √ √ z0 = 0 , z1 = −2 + 3 , og z2 = −2 − 3 Her er det bare z1 og z3 , som ligger i enhetsdisken, og begge er enkle poler. Residyen til z0 blir Res(0, f ) = lim

z→0

(z 2 − 1)(2 + 3z + 6z 2 + z 3 ) d 2 z f (z) = lim = −4 z→0 dz (1 + 4z + z 2 )2

For litt pene notasjonen skriver vi ω = z2 , den første singulariteten kan regnes ut som √ (ω 2 − 1)2 h(z) Res(f, ω) = lim 0 = = −2 3 2 z→ω g (z) 2ω(1 + 6ω + 2ω )

III

9.1 Typer integraler

187

˚ g˚ For a a mellomregningene litt mer i sømmene har en at 1 + 6ω + 2ω 2 = (1 + 2 a er 4ω + ω ) + (2ω + ω 2 ) = ω(2 + ω), siden ω er en rot av f (z) = z 2 + 4z + 1, s˚ følgelig ω 2 + 4ω + 1 = 0. En kan da skrive om brøken som følger. (ω 2 − 1)2 1 (ω 2 − 1)2 1 (ω − 1/ω)2 = = ω(1 + 6ω + 2ω 2 ) 2 ω(2 + ω) 2 2+ω √ √ Beregningene fullføres n˚ a ved a˚ bruke at ω − 1/ω = 2Imω = 2 3 og 2 + ω = 3 . Totalt s˚ a er alts˚ a integralet Z 2π I sin2 x i (z 2 − 1)2 dx = dz 2 + cos x 2 C z 2 (1 + 4z + z 2 ) 0 h √ i √ i  = 2πi Res(0) + Res(−2 + 3 ) = 2π(2 − 3 ) = −2πω 2 ˚ beregne residuen til ω, kunne en først ha som fullfører beregningene. For a skrevet om funksjonen Res(ω, f ) = lim

z→ω

√ 1 (ω − 1/ω)2 (z 2 − 1)2 /z 2 = = −2 3 2 0 [1 + 4z + z ] 2 2+ω

˚penbar. som før. Klart raskere, men kanskje ikke like a Fresnell Integralene Z ∞

sin(x2 ) dx =

−∞

Z



cos(x2 ) dx =

−∞

r

π 2

Denne seksjonen er hovedsaklig basert p˚ a [? ], men fremmgangsm˚ aten er mye ˚ skrive om funksjonen p˚ eldre. Det første vi gjør er a a kompleks form, slik at Z ∞  Z ∞  Z ∞ Z ∞ 2 ix2 2 ix2 sin(x ) dx = Im e dx og cos(x ) dx = Re e dx −∞

−∞

−∞

−∞

Legg merke til at vi ikke uten videre kan bruke substitusjonen −z 2 = ix2 , siden dette ødelegger grensene helt13 . Integralet har ingen singulariter i det komplekse planet, slik at vi ikke kan ˚ beregne integralet. Et annet problem er at bruke den vante halvsirkelen til a 2 f (z) = eiz vokser mot uendelig n˚ ar Im(f ) > 0 og Re(f ) < 0, se figur 8. Dette medfører at dersom vi bruker den vante halvsirkelen vil linjeintegralet g˚ a mot 2 uendelig. Vi m˚ a alts˚ a integrere eix over en kvadrant hvor den synker for at linjeintegralet skal konvergere. Valget blir derfor mellom I og III kvadrant. For enkelhetensskyld velges I, men akkuratt samme bevis holder og for III kvadrant. Forskjellene er at i III kvadrant m˚ a en holde tungen litt benere i munnen med ˚ integrere over hele første kvadrant tanke p˚ a fortegn. Dersom vi velger a C = {x : 0 ≤ x ≤ R} ∩ {z : z = Reiθ , 0 ≤ θ ≤ 13 Se

π } ∩ {y : 0 ≤ y ≤ R} 2

(3.70)

for eksempel http://math.stackexchange.com/questions/163946/ ˚ la are-complex-substitutions-legal-in-integration for en diskusjon av problemet. Ved a √ ˚ arbeide med to substitusjoner ix2 7→ −z 2 f˚ ar en z = ± −i x, slik at en ender opp med a a en √p˚ gang. √ Spesielt ligger problemet i at dette blir et vei integral i det komplekse planet fra ± −i ∞ ˚ tolke enn v˚ til ∓ −i ∞ som er vanskeligere a art opprinnelige integral!

188

9 Kompleks Integrasjon

C3

III

C2

C1 2

Figur 3.6: Viser hvor funksjonen eix vokser og minker. vil vi uheldigvis bare komme frem til at integralene er like14 Z ∞ Z ∞ cos(x2 ) dx sin(x2 ) dx = −∞

−∞

som er nyttig og interessant i seg selv, men ikke hjelper oss beregne verdien av ˚ prøve a ˚ heller bruke integralene. Vi m˚ a alts˚ a velge en mindre kontur. Ved a halvparten av første kvadrant C = {x : 0 ≤ x ≤ R} | {z } C1

π } ∩ { z : z = reiπ/4 , 0 ≤ r ≤ R } ∩ { z : z = Reiθ , 0 ≤ θ ≤ | {z } 4} | {z C3

(3.71)

C2

˚ beregne integralet. Konturen C er vist n˚ s˚ a vil en klare a a i figur (3.7). Fremm-

C3

C2

C1 Figur 3.7: Viser konturen C = C1 ∩ C2 ∩ C3 fra likning (3.71), og polene til eiz p˚ a omr˚ adet. gangsm˚ aten blir som før at vi bruker at linjeintegralet over en lukket sløyfe i det komplekse planet er lik singularitetene I m X 2 ix2 e dx = 2π Res(eix , zk ) = 0 C

k=0

da funksjonen aldri er null i det komplekse planet. En har da at I I I 2 2 2 eix dx + eix dx − eix dx = 0 C1

C2

C3

da C = C1 ∩ C2 ∩ C3 , og C3 g˚ a mot klokken. Vi beregner n˚ a ett og ett integral. 14 Dette

er gitt som øvelse til leser, se oppgave 4.

III

9.1 Typer integraler

189

C1) Vi kan parametrisere integralet ved a˚ bruke γ(t) = y · 0 + t · x, fra t = 0 til t=R I Z R 2 iz 2 e dz = eit dt (3.72) C1

0

som vi ser blir integralet v˚ art n˚ ar R → ∞. C2) Dette integralet følger en kvartsirkel og det vil være behagelig om dette ˚ bruke ML-ulikheten direkte integralet g˚ ar mot null n˚ ar R → ∞. La oss prøve a Z   2 2 Rπ Rπ eiz dz ≤ sup eiR exp(i2θ) ≤ sup e−R sin 2θ 4 θ∈[0,π/4] 4 θ∈[0,π/4] γ2 hvor det ble brukt at z 2 = (Reiθ )2 = R2 ei2+θ , og Eulers formel. For θ ∈ [0, π/4] 2 s˚ a er sin 2θ en strengt voksende funksjon. Alts˚ a synker e−R sin 2θ for θ ∈ [0, 4] og den største verdien utrykket kan f˚ a er alts˚ a n˚ ar θ = 0. Dette gir Z 2 Rπ Rπ −R2 sin 2·0 e ≤ eiz dz ≤ 4 4 γ2 Dersom vi n˚ a lar R g˚ a mot uendelig har vi alts˚ a vist Z 2 Rπ eiz dz ≤ lim lim ≤∞ R→∞ 4 R→∞ γ 2 som sier oss fint lite. Dersom vi i stedet hadde studert intervalet15 (0, π/4) s˚ a 2 ˚penbart g˚ ville integralet asymptotisk g˚ att som Re−R , som a ar mot null. Det ˚ vise dette. Vi trenger alts˚ er alts˚ a bare s˚ avidt ML-ulikehen ikke klarer a a en skarpere ulikhet enn hva ML-ulikheten kan gi oss. La oss heller bruke samme teknikk som i beviset til Jordans lemma. Via trekant-ulikheten har vi Z Z Z π/4 2 iz2 iz 2 ≤ e dz |dz| ≤ R e−R sin 2θ dθ (3.73) e C2

C2

0

˚ bruke z = Reiθ , s˚ hvor atter en gang har parametrisert integralet ved a a z2 = 2 i2θ 2 R e = R (cos 2θ + i sin 2θ). Ganger en m˚ a med i, og tar absoluttverdien f˚ as uttrykket ovenfor. Tanken er n˚ a igjen a˚ bruke at sin 2θ er konkav. Ved a˚ sette inn θ = 2x i likning (3.68) f˚ ar en 2 π 2x ≤ sin 2x , 0 ≤ 2x ≤ π 2



4 π x ≤ sin 2x , 0 ≤ x ≤ π 4

bruker vi dette i likning (3.73) f˚ ar en n˚ a Z Z π/4 2 2 eiz dz ≤ R e−R (4x/π) dx ≤ R · C2

0

 π  π −R2 1 − e ≤ 2 4R 4R

som heldigvis g˚ ar mot null n˚ ar R → ∞. Dette er et eksempel p˚ a hvor MLulikheten ikke fungerer, men integralet likevell konvergerer mot null. 15 Men

˚pne interval! Dette medfører at sløyfen v˚ vi kan ikke studere a ar ikke hadde vært sammenhengene og Cauchy’s integralformel ville ikke vært gyldig.

190

III

9 Kompleks Integrasjon

C3) For det siste integralet bruker vi følgende parametrisering 1+i z(t) = ei(π/4)t = √ t , 2

0 ≤ t ≤ t0

p √ og dz = (1 + i) dt/ 2 . Her er t0 valgt p˚ a en slik m˚ ate at 02 + z(t0 )2 = R, alts˚ a at avstanden til origo er R n˚ ar t = t0 . Integralet kan da skrives som (  2 ) Z t0 Z Z  2 1 + i 1 + i t0 1 + i iz exp i √ t dt = √ exp i2 t2 dt e dz = √ 2 2 2 0 0 Cr 2

˚ sette inn 25 26 hvor det siste uttrykket selvsagt gjeennskjennes som e−t . Ved a og 27 inn i 47 har vi Z R I Z 1 + i t0 −t2 ix2 ix2 √ e dx + e dx − e dt = 0 2 0 C2 0 Vi kan n˚ a la R → ∞. Da vil integralet langs C2 g˚ a mot null, slik at r r √ Z ∞ Z π π π 1+i 1 + i ∞ −t2 ix2 e dx = √ = +i e dt = √ · 2 8 8 2 2 0 0 2

Siden eix = cos x2 + i sin x2 har vi n˚ a r r Z ∞ Z ∞ π π 2 2 +i cos x dx + i sin x dx = 8 8 0 0 som var det som skulle vises. For a˚ være ekstra pedantisk kan en n˚ a ta realdelen av likningen og imagineærdelen og bruke at begge funksjonene er symmetrisk omkring origo. Men dette f˚ ar leser klare p˚ a egenh˚ and. ˚ beregne Merk at vi endte opp med a˚ bruke det gaussiske-integralet likevell. A dette integralet med kompleks integrasjon er noe vanskelig men for et slikt bevis kan en se seksjon 8 eller oppgave 4. Det gaussiske integralet I dette avsnittet skal vi studere følgende integral I 2 eπi(z−1/2) dz −2πiz ∂DR 1 − e ˚ bruke integralet ovenfor til a ˚ vise at i fra Gamelin. M˚ alet er a Z ∞ √ 2 e−s ds = π

(3.74)

−∞

som vi har sett flere ganger før. Konturen som vi integrerer omkring er parallelogrammet DR med sider ± 12 ± (1 + i)R som er vist i figur (3.8). En skisse av beviset er som følger, integralet over hele konturen kan bestemmes fra Cauchy’s residue theorem, den eneste polen er z = 0 slik at I i−1 f (z) dz = 2πi · Res [f (z), z = 0] = eiπ/4 = √ (3.75) 2 ∂DR

III

9.1 Typer integraler

y

191

γ1

1 γ4 −3

−2

−1

x 1

2

3

γ2 −1

γ3

Figur 3.8: Illustrasjon av konturen en integrerer rundt, de stiplede linjene er henholdsvis γ1 og γ3 . En kan og betrakte integralet som en sum av fire linjeintegral I Z Z Z Z = + + + ∂DR

γ1

γ2

γ3

γ4

Det kan videre vises at n˚ ar R → ∞ s˚ a vil integralene langs γ2 og γ4 g˚ a mot null. ˚ bruke dette kombinert med likning (3.75) f˚ Ved a ar en Z Z i−1 √ = f (z) dz + f (z) dz 2 γ1 γ3 Integralet langs γ1 kan parametrises ved z = 1/2(1 + i)R med −R < t < R Z

Z

R

f (z) dz = −R

γ1

2

eπi(z−1/2) dz = (1 + i) 1 − e−2πiz

Z

R

−R

2

e−2πt dt 1 + e−2πi(1+i)t

og tilsvarende kan γ3 parametrises ved z = −1/2(1 + i)R for −R < t < R slik at Z

Z

−R

f (z) dz = γ3

R

2

eπi(z−1/2) dz = (1 + i) 1 − e−2πiz

Z

R

−R

˚ legge sammen disse integralene og la R → ∞ f˚ Ved a ar fra at Z



(i + 1) −∞

 2 e−2πt 1 + e−2πi(1+i)t 1+i dt = √ −2πi(1+i)t 1+e 2

som medfører at Z



2 1 e−2πt dt = √ 2 −∞

som er ekvivalent med likning (3.74).

2

e−2πi(1+i)t e−2πt dt 1 + e−2πi(1+i)t

192

III

9 Kompleks Integrasjon

Poisson Kernel En svært viktig funksjon innen kompleks analyse er poisson kernel, ogs˚ a noen ganger kjent som poisson kjernen og er en integral transformasjon p˚ a lik linje med laplace eller fourier-transformasjonene. Lemma 3.9.3. Poisson kernel funksjonen er definert Pr (θ) =

∞ X n=−∞

r

|n| inθ

e

1 − r2 = = Re 1 − 2r cos θ + r2



1 + reiθ 1 − reiθ

 , r ∈ [0, 1)

Andre likhet følger fra substitusjonen ϕ → ϑ − ϕ og at poisson kernel funksjonen Pr (θ) har en periode p˚ a 2π. Definisjon 3.9.1. Poisson kernel transformasjonen er definert som Z 2π Z 2π dϕ dϕ ˜ iθ ) = h(re h(eiϕ )Pr (ϑ − ϕ) h(ei(ϑ−ϕ )Pr (ϕ) = 2π 2π 0 0 Dette er en transformasjon som tar inn en funksjon og dersom h er integrerbar p˚ a enhetssirkelen s˚ a er u harmonisk p˚ a omr˚ adet D = {z : |z| < 1}. Vi skal her hovedsaklig studere ulike m˚ ater a˚ bestemme poisson kernel transformasjonen av h = 1 p˚ a, alts˚ a en konstant funksjon. Proposisjon 3.9.3. La R > r > 0 da er Z 2π R2 − r 2 1 dϑ = 1 PR,r = 2 2π 0 R − 2Rr cos(θ − ϑ) + r2 Bevis. 1) Legg merke til at integralet ikke er akkuratt det samme som poisson transformasjonen. Det første vi gjør er a˚ enten a˚ bruke substitusjonen ϑ → ϕ − ϑ ˚ se at eller a ∂ ∂ =− ∂ϕ ∂ϑ slik at integralet er uavhengig av ϕ. Videre la oss dele p˚ a R og innføre konstanten ρ = r/R Z 2π 1 R2 − r 2 PR,r = dϑ 2 2π 0 R − 2Rr cos ϑ + r2 ˚ forenkle integranden via kompleks analyse. Da er fremmgangsm˚ aten n˚ a blir a 1 − (r/R)2 1 − ρ2 = 12 − 2(r/R) cos ϑ + (r/R)2 (1 − ρeiϑ )(1 − ρe−iϑ ) 1 ρe−iθ = + 1 − ρeiϑ 1 − ρe−iϑ ∞ X = ρ |n| einϑ n=−∞

=1+2

∞ X n=1

ρn cos(nϑ)

III

9.1 Typer integraler

193

tanken er n˚ a at n˚ ar vi integrerer over 0 ≤ ϑ ≤ 2π s˚ a vi alle integralene g˚ a mot null. Siden vi har fra at eksempel (2.10.1) at Z π cos nx dx = 0 , n ∈ Z/{0} 0

dermed s˚ a blir integralet 2π

R2 − r 2 dϑ − 2Rr cos ϑ + r2 0 Z 2π ∞ X 1 = 1+2 ρn cos(nϑ) dϑ 2π 0 n=1 Z 2π ∞ X =1+2 cos(nϑ) dϑ = 1 ρn

PR,r =

Z

1 2π

R2

n=1

0

som var det som skulle vises. 2) La oss og løse integralet p˚ a en litt annen m˚ ate, vi kan gjøre følgende omskrivning PR,r =

1 2π



Z 0

R2 − r 2 R2 − r 2 dϑ = 2 2 2 R − 2Rr cos ϑ + r R + r2

Z 0



dx 1 2π 1 − a cos x

Hvor x = ϑ og a = 2rR/(r2 + R2 ). Siste integralet har vi løst i Del II, men la oss n˚ a løse det via kompleks integrasjon. Da er I 1 dx 1 dz/zi 1 1 = = 2π 1 − a cos x 2π 1 − a(z + z −1 )/2 πi z=1 a + 2z + az 2 ˚ beregne integralet. Nullpunktene er Vi kan n˚ a bruke √ residue-theoremet til a z1,2 = (−1 ± 1 − a2 )/a, og det er bare z1 som ligger i enhetssirkelen, s˚ a ! √ Z 2π 1 dx 1 −1 + 1 − a2 = 2πi Res 2π 0 1 − a cos x πi a = 2 lim

z→z1

1 1 1 = = √ 2 + 2az 1 + az1 1 − a2

˚ sette inn f˚ Ved a ar en n˚ a PR,r =

R2 − r 2 1 R2 + r2 2π

Z 0



dx R2 − r 2 1 √ = 2 =1 2 1 − a cos x R +r 1 − a2

som før. Det siste steget er bare en rekke ufine algebraforenklinger hvor en kun bruker at a = 2rR/(R2 + r2 ), men disse stegene kan leser kan f˚ a kose seg med alene. Logaritmer Nøkkel konturer

194

III

10 Oppgavesammling III

3.10

O P P G AV E S A M M L I N G I I I

3.10.1 Z

1/2

1. 0

Z

INTEGRAL



2.

2

1 − e−1/x dx

0

3.10.2

Z



log(1/x) dx 5. (1 + x2 )2 0   Z ∞ 1 dx 4. LambertW x2 0

log(x) log(1 − x) 3. dx x(1 − x)

Z 0

1

log(1 − x) dx 1+x

O P P G AV E R

1. Beregn Dirichlet integralet Z



0

sin ω π dω = ω 2

ved hjelp av dobbeltintegralet ZZ I= e−st sin t d(s, t) D

hvor D = [0, ∞) × [0, ∞). 2. Bestem følgende sum S=

∞ X n=1

1 (v − u)2n+1

Z

v

(x − u)n (v − x)n dx

u

hvor v > u. 3. (Tom Apostol - 10.23) Definer Z ∞ F (y) := 0

sin xy dx x (x2 + 1)

∀ y > 0.

a)

Vis at F (y) tilfredstiller følgende differensiallikning π F 00 (y) − F (y) = , 2 ˚ vise at og bruk dette til a  1 F (y) = π 1 − e−y . 2 b)

˚ bestemme følgende tre integral Bruk F (y) til a Z ∞ Z ∞ Z ∞ sin xy cos xy x sin xy , , og . 2 2 2 2 x (x + a ) x +a x2 + a2 0 0 0

4. Vi skal i denne oppgaven studere følgende integral Z 1 xn In = dx 0 x+1 hvor n ∈ N.

III

10.2 Oppgaver

195

a) Vis at lim In = 0 n→∞

b) Bestem In+1 + In c)

Bestem følgende sum

P∞

k=0

(−1)k k+1

5. Beregn integralet π

Z

 arctan

0

γ sin x 1 − γ cos x



dx sin x

der γ, betegner den velkjente Euler–Mascheroni konstanten. Vis at integralet kan skrives som √ π log(2 + 3 ) 2 √ ˚ benytte seg av tilnærmingen γ ≈ 1/ 3 . ved a 6. Denne oppgaven handler om gamma og beta-funksjonene. a) Vis at 1

Z I=

−1

(1 + x)2m−1 (1 − x)2n−1 dx = 2m+n−2 B(m, n) (1 + x2 )m+n

hvor m, n > 0. ˚ vise at b) Bruk forrige oppgave til a Z

π/4

J= −π/4

 cos x + sin x cos α cos x − sin x

dx =

π 2 sin π cos2

α 2

.

n˚ ar α ikke er en multipel av π (α 6= kπ, k ∈ N). 7. I del II viste vi at for alle n ∈ N π/2

Z

sin(2n + 1)x π = , sin x 2

0

˚ vise at se proposisjon (2.10.1). Bruk dette til a (2n+1)π/2

Z In = 0

sin x dx x

g˚ ar mot π/2 n˚ ar n → ∞. Z 8. Bestem følgende integral 0



xa − xb dx hvor a, b ∈ R (1 + xa ) (1 + xb )

(3.76)

196

10 Oppgavesammling III

9. I denne oppgaven skal vi studere følgende to integral Z ∞ Z ∞ −tx sin(x − t) e dx , I1 (t) = dx og I2 (t) = x 1 + x2 t 0

III

(3.77)

Vis at begge integralene tilfreldstiller differensiallikningen y 00 + y =

1 , t

t > 0.

˚ vise at I La I(t) = I1 (t) − I2 (t), og bruk differensiallikningen ovenfor til a m˚ a være p˚ a formen I(t) = A sin(t + B) Bestem konstantene A og B og vis endelig at Z ∞ Z ∞ sin x dx π dx = = . 2 x 1+x 2 0 0

10. Beregn følgende integral Z

π/2

−π/2

log(1 + b sin x) dx sin x

hvor |b| ≤ 1. 11. Tidligere har en studert følgende integral Z ∞ 1 In (a, b) := dx 2 x + b sin2 x)n (a cos 0 For n = 1 og n = 2. Anta n > 1 og vis at In tilfredstiller   ∂ ∂ + In−1 = ∇In−1 = (1 − n)In ∂α ∂β hvor ∇ er summen av de partiellderiverte. Bestem og et lukket uttrykk for I√ n ved hjelp av partiellderiverte. Det kan fritt benyttes at I1 (a, b) = π/(2 ab ). 12. (Putnam 2008−B2) Merk, oppgaven er ikke helt den samme. La F0 (x) = log x. For n ≥ 0, defineres Z x Fn+1 = Fn (t) dt 0

Bestem følgende sum  lim log n − n! · Fn (1)

n→∞

III

10.2 Oppgaver

197

13. La A være definert som følger Z ∞ −ϕx e − e(1−ψ)x A= dx x 0 Hvor ψ og ϕ er henholdsvis minste og største løsning av x2 + x = 1 Bestem uttrykk for sinh(A) og

cosh(A) ,

√ og skriv de s˚ a enkelt som mulig. Hint: ψ + ϕ = ϕ · ψ, ϕ − ψ = 5 . 14. (Putnam 2005−A5) Det integralet som kanskje har høstet mest oppmerksomhet gjennom Putnam’s historie er følgende er følgende Z 1 log(1 + x) I= dx 1 + x2 0 ˚ bestemme integralet i denne oppgaven ser vi nærmere p˚ a ulike m˚ ater a p˚ a. a) Løs integralet ved hjelp av substitusjonen x = tan θ. b) Innfør en parameter α i teller og bestem integralet. ˚ bec) Benytt den noe uvanlige substitusjonen (1 + x)(1 + y) = 2, til a stemme integralet. 15. I denne oppgaven skal vi se hvordan Euler kan ha kommet frem til antakelsen om at betafunksjonen var relatert til gammafunksjonen. Se p˚ a følgende integral Z 1 1 π √ I= dx = 2 2 1 − x 0 ˚ vise hvorfor det kan være rimelig a ˚ anta at betafunkBruk integralet til a sjonen kan skrives som Z 1 Γ(x)Γ(y) B(x, y) = tx−1 (1 − t)y−1 dt = . Γ(x + y) 0

˚ komme frem til a ˚ komme frem an16. I forrige oppgave viste vi en m˚ ate a takelsen om sammenhengen mellom betafunksjonen og gammafunksjonen. Denne sammenhengen ble vist i theorem (3.3.6) ved hjelp av Bohrmullerup theoremet. Siden den gang har vi (forh˚ apentligvis) lært mange ˚ bevise sammenhengen og vi studeflere teknikker. Disse kan benyttes til a rer nærmere to av disse.

198

III

10 Oppgavesammling III

˚ bevise theoa) Skriv Γ(x)Γ(y) som et dobbeltintegral og bruk det til a rem (3.3.6). b) Bruk konvolusjonstheoremet for laplace-transformasjoner med f (t) = ˚ bevise theorem (3.3.6). tx og g(t) = ty til a 17. Følgende likhet har tidligere blitt vist Z 1 Z √ log Γ(x + t) dx = log 2π + 0

1

x log tx dx

∀t>0

0

og kan benyttes fritt i denne oppgaven. Bestem følgende integral ZZ B(x, y) d(x, y) S

hvor S = [0, 1] × [0, 1] er enhetsfirkanten (”unit square”). 18. I denne oppgaven skal en svak generalisering av Ahmed’s integral √ Z 1 arctan x2 + 2 dx 5π 2 √ = 2 2 x +1 96 x +2 0 ˚ vise at studeres. Bruk at 1/q + 1/p = (p + q)/pq til a ZZ ZZ dx dy dx dy + 2 2 2 2 2 2 2 2 2 2 S (a + x )(2a + x + y ) S (a + y )(2a + x + y ) Z 1 Z 1 dx dy = 2 + y2 2 + y2 a a 0 0 Der a er en reell konstant, og S er enhetskuben S = [0, 1] × [0, 1]. Integralene p˚ a høyre side er like. Vis da at √ Z 1Z 1 Z 1 π 2 2 1 dy a2 2 − arctan 2a + x √ dx = dx 2 2 2 2 2 2 2 2a2 + x2 0 0 a + x 2a + x + y 0 a +x ˚ endelig vise at og bruk dette til a √  2 Z 1 2a2 arctan 2a2 + x2 1 1 √ √ dx = π arctan − arctan . 2 2 a 2a2 + x2 2a2 + 1 0 a +x Hvor a som før er en reell konstant. 19. Sinus funksjonen er definert som følger Z z Z Si(z) := sinc(x) dx = 0

0

z

sin x dx , x

og vi ser at limz→∞ Si(z) = π/2 gir oss Dirichlet integralet som har blitt studert før. I denne oppgaven skal vi studere en rekke relaterte integral. a)

Vis at Z

∞ 2

Z

0



sinc(x) dx .

sinc (x) dx = 0

III

10.2 Oppgaver

199

a) Uttrykk integralene ∞

Z 0

sin4 x dx x2

Z



og 0

sin4 x dx x4

ved hjelp av Dirichlet integralet. 20. I denne og den neste oppgaven studerer vi nærmere Wallis integralet Z

(

π/2

Wn =

n

sin (x) dx = 0

2p π 22p+1 p  2p 2p+1 2 p+1 n



n˚ ar n = 2p , n˚ ar n = 2p + 1

(3.78)

som vi har sett tidligere.

R π/2 a) Vis først at Wn = 0 cosn dx, og deretter at høyre og venstre side av likheten tilfredstiller differensiallikningen nWn = (n − 1)Wn−2 for alle n ≥ 2.

b) Vis følgen er synkende og at for store n s˚ a er Wn+1 ∼ Wn . Bruk dette ˚ vise at for store n s˚ til a a er r π Wn ∼ , 2n ˚ betrakte følgen yn = (n + 1)Wn Wn+1 . hvor det kan være lurt a

c) Avslutningsvis viser vi noe av nytteverdien til Wallis integralet og beregningene vi har utført. Fra tidligere har vi vist tilnærmet at √  n n n! ∼ C n , e

der C ∈ R+

(3.79)

ogs˚ a kjent som stirrlings formel. Tidligere har vi antatt at C = 1, men vi kan finne en enda bedre tilnærmelse for denne konstanten. Bruk tilnærmelsen for W2n fra deloppgave b) og det eksakte uttrykket fra likning (3.78) ˚ bestemme C. Du kan fritt benytte deg av at likning (3.79) holder for til a store n. 21. I denne oppgaven skal vi se nærmere p˚ a det gaussiske integralet. La f være definert som 2 √ f (x, t) = xt · e−x / π

200

10 Oppgavesammling III

III

a) Vis at f (x, 0) er en sannsynlighetsfordeling. Alts˚ a at integralet over R er lik 1. For en kontinuerlig fordelig er forventingsverdien til en sannsynlighetstetthet er gitt som Z ∞ E(x) = hxi = xf (x) dx −∞

og beskriver det mest sannsynlige utfallet til funksjonen. Variansen er hvor nært sentrert funksjonen er omkring funksjonen gitt som Z ∞ 2 Var(x) = x − E(x) f (x) dx −∞

b)

Bestem forventingsverdien og variansen til f (x, 0).

Videre I denne oppgaven blir det sett nærmere p˚ a den moment-genererende funksjonen til f . Den er gitt som Z ∞ Mf (p) = exp f (x)dx −∞

c)

Vis at den momentgenererende funksjonen til f (x, 0) kan skrives som Mf (p) = ep

2



/4

π

Og vis videre at lim Mf (p)

t→0

d)

0

= E(x) og

00 lim Mf (p) = Var(x)

t→0

Vis at integralet Z



I(t) :=

αf (x, t) dx

(3.80)

−∞

tilfredstiller funksjonallikningen I(t) =

t−1 I(t − 1) 2

dersom t er et partall. Hva skjer dersom t er odde? Bestem integralet I(t) og finn α slik at I(t) blir en sannsynlighetsfordeling. Eksisterer det ˚ være en sannsynlighetsfordeling? t-verdier slik at det er umulig for I a

IV

˚ gi leser en pustepause og for a ˚ la Denne siden er med hensikt blank, for a forfatter sl˚ ass mot dinosaurer.

I

203

Tillegg A

A.1

KONVERGENS

N˚ ar vi studerer integraler er spørsm˚ alet om integralet har en verdi, minst like viktig som selve svaret. Hva er konvergens og hva menes med at integralet har en verdi? Definisjon A.1.1. Et uegentlig integral er et integral som skrives p˚ a formen Z b Z b lim f (x) dx , lim f (x) dx , b→∞

a→−∞

a

a

eller p˚ a formen Z

Z f (x) dx ,

lim

b→∞

b

a

a→−∞

b

f (x) dx ,

lim

a

Hvor grenseverdiene f (x) enten er udefinert, eller g˚ ar mot ±∞. En sum av uegentlige integral, er ogs˚ a uegentlig. Definisjon A.1.2. Et integral kalles konvergent, dersom det kan skrives som en endelig verdi. Alle integral som ikke er konvergent, kalles divergente. Eksempel A.1.1. For motivasjon studeres følgende integral Z 2 dx x −1 0 ˚ ignorere diskusjonen omkring integralet konverger eller ikke1 og vi velger a regner blindt ut integralet. Z 2 h i2 dx = log |x − 1| = log |−1| + log |1| = 0 0 0 x−1 Dette virker noe rart om en studerer grafen til funksjonen vist i figur 8. Da funksjonen klart bl˚ aser opp nære x = 1. S˚ a funksjonen ovenfor konvergerer, men vi vil gjerne skille mellom ulike typer konvergens. Vi har for eksempel Rieman-integrerbare funksjoner, LebesgueIntegrerbare funksjoner osv. Funksjonen ovenfor konvergerer om en betrakter prinsipial verdien av integralet, men divergerer ellers. Om vi sier at en funksjon er integrerbar, vil betydningen alltid være at funksjonen er Rieman-integrerbar. 1 Siden

vi ikke engang har diskutert hva konvergens er engang!

204

1 Konvergens

I

Definisjon A.1.3. Et uegentlig integral konvergerer hvis og bare hvis integralet kan skrives som en sum av uegentlige integral, som alle konvergerer. Ved a˚ bruke denne definisjonen av konvergens p˚ a integralet i eksempel (C.1.1) s˚ a m˚ a b˚ ade Z 1 Z 2 dx dx og x − 1 x −1 0 1 ˚ bryte opp integralet i interval (a, b) hvor kun konvergere. Teknikken er alts˚ aa en av grenseverdiene limx→a f (x) eller limx→b f (x) ikke eksisterer. I tilleg til at f (x) er øvre begrenset p˚ a [a, b]. Hvorfor vi velger a˚ definere konvergens p˚ a denne m˚ aten gir mer mening n˚ ar vi studerer begrepet uendelig nærmere. Uendelig er ikke noe tall, men noe vi reiser mot og aldri helt n˚ ar. For a˚ slippe hodeverk bør gjerne hastigheten p˚ a hvor ˚ si, siden vi uansett aldri kommer helt frem. raskt vi reiser ikke ha noe a Eksempel A.1.2. La oss betrakte om følgende integral Z ∞ 2x dx 2 −∞ 1 + x konvergerer eller divergerer. Fra diskusjonen ovenfor er m˚ ate a˚ studere hvordan integralet oppfører seg n˚ ar vi g˚ ar mot uendelig i ulikt tempo. Z a h ia 2x dx lim = lim log 1 + x2 = 0. 2 a→∞ −a 1 + x a→∞ −a Derimot om vi lar funksjonen vokse ’dobbelt’ s˚ a raskt mot uendelig f˚ as Z 2a h i2a 2x dx = lim log 1 + x2 lim = 2 log 2 . 2 a→∞ a→∞ −a 1 + x −a vi kan dermed konkludere med at integralet er udefinert da det er avhengig av ’hastigheten’ Dette problemet med hastigheter løses dersom en deler opp integralet i uegentlige integral. Slik at hvert integral bare har en singularitet. P-integral Z ∞ dx Proposisjon A.1.1. Integralet divergerer for alle relle eller komplekse p xp 0 mens Z 1 Z ∞ dx dx , p < 1 og , p>1 p x xp 0 1 konvergerer. Bevis. Dersom p < 1 s˚ a er  1 Z 1  dx 1 1 1 p−1 = x = lim 1 − c1−p = p 1−p 1 − p x→0 1−p 0 x 0

I

1 Konvergens

og likeledes s˚ a er Z ∞ 1

dx = lim c→∞ xp

Z

c

1

205

 dx 1 = lim 1 − c1−p = ∞ p c→∞ x 1−p

Deretter studeres tilfellet n˚ ar p > 1 s˚ a   1  Z 1 dx 1 1 1 p−1 = lim x lim 1 − p−1 = ∞ = p c→0 1 − p 1 − p c→0 c 0 x c og likeledes s˚ a er Z c Z ∞  dx 1 1 dx = lim = lim 1 − c1−p = p p c→∞ c→∞ x x 1 − p 1 − p 1 1 Siden (0, 1) konvergerer for p < 1 og (1, ∞) konvergerer for p > 1, s˚ a kan ikke (0, ∞) = (0, 1) ∪ (1, ∞) konvergere n˚ ar p < 1 eller p > 1. Da gjennst˚ ar det bare ˚ teste tilfellet hvor p = 1. a Z ∞ Z 1 Z ∞ dx dx dx = + = lim (− log x) + lim log x = ∞ x→∞ x→0 x x x 0 0 1 som begge divergerer. Dette fullfører beviset. Dette resultatet er svært nyttig n˚ ar vi bruker det i kombinasjon med sammenliknings testen. Proposisjon A.1.2. (Sammenliknings Testen) La f (x) og g(x) være to funksjoner definert p˚ a (a, b) slik at 0 ≤ f (x) ≤ g(x) er sant for alle x ∈ [a, b]. Dersom Z

b

g(x) dx a

konvergerer s˚ a konvergerer integralet over f (x). Dersom integralet over g(x) divergerer s˚ a divergerer integralet over f (x). Eksempel A.1.3. La oss se om følgende integral konvergerer Z ∞ (sin x)2 dx x2 0 dersom x ≥ 1 s˚ a har vi at (sin x)2 1 ≤ 2 x2 x dermed har vi fra p-testen at integralet ovenfor konvergerer siden p = 2 > 1. Dersom 0 ≤ x < 1, kan vi stedet bruke følgende sammenlikning (sin x)2 1 ≤ √ x2 x

206

I

1 Konvergens

˚ gange med x2 , p˚ ved a a begge sider s˚ a har en at (sin x)2 ≤ x3/2 . Dette stemmer 3/2 siden sin x ≤ x ≤ x , for x ≥ 0 som en for eksempel kan se fra taylorrekken til sin x. Dermed konvergerer integralet fra sammenlikningstesten og vi har Z ∞ Z 1 Z ∞ 1 (sin x)2 1 √ + 0≤ dx ≤ =3 2 x x2 x 0 0 0 som er en fin tilnærming. ˚ se p˚ En kunne fint ha f˚ att enda strammere grenser ved a a taylorrekken til (sin x)2 /x2 , men det er helt uviktig. Legg merke til at metoden ovenfor ikke kan ˚ vise at (sin x)/x konvergerer, selv om integralene har samme verdi. brukes for a Proposisjon A.1.3. (Grensesammenliknings Testen) La f og g enten være to strengt positive eller strengt negative funksjoner p˚ a [a, b] og la funksjonene være udefinert i punktet a. Dersom g(x) ∼ f (x) n˚ ar x ∼ a s˚ a konvergerer Z b Z b f (x) dx hvis og bare hvis g(x) dx konvergerer. a

a

Hvor g(x) ∼ f (x) n˚ ar x ∼ c betyr at limx→c f (x)/g(x) = 1. Begge testene ovenfor krever at funksjonene f og g ikke skifter fortegn p˚ a intervalet. S˚ a det er logisk a˚ lure p˚ a hvordan en skal behandle integraler som ikke er strengt positive eller negative. En m˚ ate a˚ tvinge en funksjon til a˚ være positiv p˚ a er a˚ ta absoluttverdien. Det er klart at b˚ ade f (x) og |f (x)| har samme singulariter s˚ a et naturlig Rb spørsm˚ al er hvilken konklusjon vi kan trekke om a f (x) , dersom vi vet noe om Rb integralet a |f (x)| dx konvergerer, eller divergerer. Et delvis svar p˚ a spørsm˚ alet er gitt i følgende proposisjon Proposisjon A.1.4. La f (x) være en funksjon dersom integralet Z b |f (x)| dx a

konvergerer, s˚ a konvergerer ogs˚ a integralet Z b f (x) dx . a

Rb Merk at det kontrapositive ikke stemmer! Dersom integralet a |f (x)| dx Rb divergerer betyr dette ikke at a f (x) dx divergerer. Et eksempel p˚ a dette er vist under Dirichlet integralet Lemma A.1.1. La f være definert som følger2   sin x dersom f (x) = x  0 ellers 2 Det

x 6= 0 , R∞

er i utgangspunktet ingen grunn til a˚ definere f slik, da integralet (sin x)/x dx er Riemann0 ˚ definere f slik at funksjonen blir kontinuerlig, deriverbar. integrerbart. Derimot er fordelen ved a

I

1 Konvergens Z

207



da konvergerer integralet

f (x) dx, men ikke absolutt. −∞

Bevis. Først kan en legge merke til at integralet er symmetrisk omkring origo ˚ betrakte (0, ∞). f (−x) = f (x), slik at det holder a ˚ vise at funksjonen ikke konvergerer absoFørste del av bevisetRvil g˚ a ut p˚ aa ∞ lutt, og deretter vise at 0 f (x) dx, konvergerer. Vi betrakter først integralet av funksjonen over et endelig interval Z N f (x) dx , 0

La n˚ a N ∈ N, da er Z 2πN N −1 Z 2π(n+1) X sin x sin x dx = x dx x 0 n=0 2πn Z 2π(n+1) N −1 X 1 ≥ |sin x| dx 2π(n + 1) 2πn n=0 Z 2π N −1 X 1 = |sin x| dx 2π(n + 1) 0 n=0 =

N −1 1 2 X π n=0 n + 1

som klart divergerer n˚ ar N → ∞. Dette viser at integralet ikke konvergerer ˚ bruke testene ovenfor i kombinasjon med p-integralene er det som absolutt. A ˚ vise de aller fleste integralene konvergerer. vil bli brukt for a ˚ Men dette integralet krever litt mer triksing. Trikset i denne oppgaven blir a atter en gang vende tilbake til delvis integrasjon. Vi deler igjen integralet ved x=1 Z N Z N h cos x iN Z N cos x sin x cos N cos x dx = dx = cos(1) − − − 2 x x x N x2 1 1 1 1 ˚ vise at siste integral konvergerer kan en som konvergerer n˚ ar N → ∞. For a bruke p-testen med x2 siden cos x/x2 ≤ 1/x2 n˚ ar x ≥ 1. Tilfellet N˚ ar x ∼ 0 s˚ a er sin x ∼ x s˚ a (sin x)/x ∼ 1. S˚ a Z 1 Z 1 sin x dx ≤ 1 dx x 0 0 dette følger fra grensesammenlikningstesten og medfører konvergerens. Dette medfører ogs˚ a at integralet konvergerer. ˚ gjøre. Først s˚ At integralet ovenfor konvergerer har med flere ting a a bl˚ aser ikke integralet opp nære origo da (sin x)/x ≤ 1 for |x| < 1. Faktisk s˚ a er (sin x)/x ≤ 1 ∀ x ∈ R, siden 1/x ’drar’ funksjonen mot null. Funksjonen sin x bidrar til at funksjonen oscillerer over og under x-aksen, og det er kanseleringen av disse omr˚ adene som bidrar til at funksjonen konvergerer. N˚ ar en ser p˚ a absoluttverdien av funksjonen f˚ ar en derimot ikke disse kanseleringene. I alle eksemplene tidligere har vi sett p˚ a funksjoner som g˚ ar mot 0 n˚ ar

208

I

1 Konvergens

x → ∞. At dette alltid er tilfellet for konvergente integral er feil. Det eksisterer alts˚ a funksjoner ar mot en verdi ulik R ∞ hvor limx→∞ f (x) ikke eksisterer – eller g˚ null – men k f (x) dx konvergerer. Fresnell integralene Lemma A.1.2. x

Z

Z

 sin t2 dt , C(x) =

Integralene S(x) = 0

x

 cos t2 dt konvergerer ∀x ∈ R.

0

Lemma A.1.3. Z



 sin x2 dx og

Integralene −∞

Z



 cos x2 konvergerer.

−∞

R ˚ forst˚ A a hvorfor R sin x2 dx konvergerer og ikke R sin x dx er ikke s˚ a veldig vanskelig. For sin(x2 ) har funksjonen et en topp over x-aksen, ogs˚ a en bunn under x-aksen. Arealet av disse toppene og bunnene blir mindre og mindre n˚ ar x → ∞. Dette er ikke tilfellet for sin x som oscillerere med samme hastighet hele tiden. Fra den alternerende rekketesten konvergerer alts˚ a integralet over sin(x2 ), men ikke sin x. ˚ bevise lemma (C.1.3) siden dette ogs˚ Vi kommer kun til a a beviser lem˚ vise at S(x) og C(x) konvergerer siden ma (C.1.2). Selv om det er rett frem a en integrerer en begrenset funksjon over et endelig interval. R

˚ betrakte intea holder det a Bevis. Funksjonene er symmetrisk omkring origo, s˚ gralene over (0, ∞). Her vil vi bare vise at integralet over sin(x2 ) konvergerer, men et nøyaktig likt bevis kan føres for cos(x2 ). Alternativt kan en vise først at integralene er like store, slik at det holder a˚ vise at ett av integralene konvergerer. ∞

Z

 sin x2 dx =

1

Z

0

 sin x2 dx +

Z

0



 sin x2 dx

1

Funksjonen er integrerbar p˚ a intervalet [0, 1], den bl˚ aser ikke opp og har ingen singulariterer. Vi m˚ a alts˚ a vise at integralet Z lim

N →∞

N

 sin x2 dx ,

1

√ √ ˚ bruk substitusjonen t = x2 , s˚ konvergerer. Ved a a x = t og 2 t dx = dt f˚ ar en Z

N

sin x2

2

Z dx =

1

1

N2

sin t cos N 2 cos 1 1 √ dt = − √ + − 2 4 2 t 2 N

Z 1

N2

cos t dt t3/2

˚ sammenlikne med p-integralene ser vi at siste integralet kjenner vi igjen. Ved a integraler konvergerer, siden 3/2 > 1 og cos t/t3/2 ≤ 1/t3/2 . Γ-funksjonen Z Lemma A.1.4. 0



tx−1 e−t dt konvergerer for alle x ∈ (0, ∞)

I

1 Konvergens

Bevis. Først s˚ a deler vi integralet i Z Z ∞ x−1 −t t e dt =

1 x−1 −t

t

e



Z

tx−1 e−t dt

dt + 1

0

0

209

og det neste steget er a˚ vise at de uegentlige integralene p˚ a høyre side konverge˚ finne en funksjon f (t) slik at 0 ≤ tx−1 e−t ≤ f (t), rer. Fremmgangsm˚ aten blir a og vise at integralet over f (t) er konvergerer. For det første integralet s˚ a er e−t ≤ 1 for t ≥ 0, slik at   x 0  Z 1 Z 1 cx t 1 x−1 −t x−1 − . 0≤ t e dt ≤ lim t dt = lim = lim c→0 c c→0 x c c→0 x x 0 Dersom x > 0, s˚ a vil ax → 0 n˚ ar a → 0+ , s˚ a integralet konvergerer mot 1/x. R 1 x−1 −t Dette viser at integralet over 0 t e dt konvergerer. For det neste integralet vil vi bruke at limt→∞ tr e−t/2 = 0 for alle r ∈ R. Alts˚ a for x > 0 eksisterer det en kx ∈ R slik at 0 ≤ tx e−t/2 ≤ 1 for t ≥ kx . Vi holder x konstant og deler integralet ved kx Z ∞ Z kx Z ∞ x−1 −t x−1 −t t e dt = t e dt + tx−1 e−t dt . 1

kx

1

Det første integralet p˚ a høyre side er over et endelig interval og har ingen singulariter p˚ a intervalet, alts˚ a konvergerer integralet. For det neste har vi at t ≥ kx slik at tx−1 e−t = (tx−1 e−t/2 )e−t/2 ≤ e−t/2 , s˚ a Z ∞ Z ∞   tx−1 e−t dt ≤ e−t/2 dt = lim 2e−kx /2 − e−c/2 = 2e−kx /2 . kx

c→∞

kx

Merk at uansett hva kx er s˚ aR konvergerer integralet. Dette viser at det andre ∞ integralet er konvergent, s˚ a 1 tx−1 e−t dt konvergerer. Dette fullfører beviset.

Betaintegralet Z Lemma A.1.5.

1

tx−1 (1 − t)y−1 dt konvergerer for alle x, y ∈ (0, ∞)

0

Bevis. Slik som beviset for konvergens av gammaintegralet deles integralet opp Z 1 Z 1/2 Z 1 x−1 y−1 x−1 y−1 t (1 − t) dt = t (1 − t) dt + tx−1 (1 − t)y−1 dt . 0

0

1/2 x−1

y−1

x−1

For det første integralet merk at t (1 − t) ≤t for 0 ≤ t ≤ 1/2, s˚ a   Z 1/2 Z 1/2 (1/x)x − cx 0≤ tx−1 (1 − t)y−1 dt ≤ tx−1 dt = lim c→0 x 0 0 N˚ a, dersom x > 0 s˚ a vil ax → 0 n˚ ar a → 0+ , s˚ a det første integralet konvergerer dersom x > 0. Tilsvarende for det neste integralet s˚ a er tx−1 (1 − t)y−1 ≤ y−1 (1 − t) for 1/2 ≤ t ≤ 1 s˚ a   Z 1/2 Z 1/2 (1/2)y − (1 − c)y 0≤ (1 − t)y−1 dt ≤ (1 − t)y−1 dt = lim− x c→1 0 0

210

1 Konvergens

I

N˚ a, dersom y > 0 s˚ a vil (1 − a)y → 0 n˚ ar a → 1− , s˚ a det første integralet R 1 x−1 konvergerer dersom y > 0. Dermed s˚ a konvergerer 0 t (1 − t)y−1 dt for alle x, y > 0 som var det som skulle vises.

A.1.1

KONVERGENSRADIUS

A.1.2

P R I N S I PA L

VERDI

Cauchys prinsippielle verdi (Cauchy principal value) er en m˚ ate a˚ gi en verdi til integralet som i utganspunktet divergerer.

II

211

Tillegg B

B.1

A N A LY S E N S F U N D A M E N TA L T H E O R E M .

Før vi kan vise analysens fundamentaltheorem., trengs det først to proposisjoner. Proposisjon B.1.1. (mellomverdi setningen) La I = [a, b] være et a ˚pent endelig interval interval, hvor a, b ∈ R. La f være en reell funksjon som er kontinuerlig p˚ a (a, b). Dersom k ∈ R er tall slik at f (a) < k < f (b) eller

f (a) > k > f (b)

da eksisterer det en konstant c ∈ (a, b) slik at f (c) = k. At dette stemmer er ikke vanskelig a˚ forst˚ a. Theoremet sier at dersom vi har en kontinuerlig funksjon som har verdiene f (a) og f (b) i hvert endepunkt av intervaet, s˚ a tar funksjonen alle verdier mellom f (a) og f (b) i ett eller annet punkt i intervalet. For et bevis se for eksempel [? , thm 4.23]. Proposisjon B.1.2. (mellomverdi setningen for integraler) La f være en reell kontinuerlig funksjon p˚ a det lukkede intervalet [a, b], a, b ∈ R. Da eksisterer det et reellt tall k ∈ [a, b] slik at Z

b

f (x) dx = f (k)(b − a)

(B.1)

a

Bevis. Siden funksjonen er kontinuerlig p˚ a et lukket interval, s˚ a er den ogs˚ a Rieman-integrerbar p˚ a [a, b]. Det følger ogs˚ a fra at f er kontinuerlig p˚ a et lukket interval at funksjonen har et minimum og et maksimum f (m) = max f (x) x∈[a,b]

f (M ) = max f (x) x∈[a,b]

Vi kan n˚ a definere følgende ulikhet Z

b

Z f (m) dx ≤

a

b

Z f (x) dx ≤

a

b

f (M ) dx a

212

II

1 analysens fundamentaltheorem.

Siden f (m) og f (M ) er konstaner kan en skrive Z b 1 f (x) dx ≤ f (M ) f (m) ≤ b−a a Fra mellomverdi setningen proposisjon (B.1.1) har vi n˚ a at det eksisterer en k ∈ (a, b) slik at Z b 1 f (x) dx = f (k) b−a a ˚ gange likningen med b − a fullfører beviset. A Theorem B.1.1. (Analysens fundamentaltheorem. - Del I) La f være en kontinuerlig funksjon p˚ a [a, b]. Funksjonen g definert som Z x f (t) dt g(x) = a

er da kontinuerlig p˚ a [a, b], deriverbar p˚ a (a, b) og g 0 (x) = f (x) for alle x ∈ (a, b). Her betegner integralet som vanlig arealet under funksjonen f (x) fra a til x. Bevis. Siden f er kontinuerlig og øvre begrenset s˚ a er ogs˚ a F kontinuerlig og øvre begrenset. La oss ta den deriverte av F , da har vi fra definisjon g(x + h) − g(x) d g(x) = lim h→0 dx h Vi kan skrive om teller som følger Z x+h Z g(x + h) − g(x) = f (t) dt − a

x+h

(B.2)

Z

x+h

f (t) dt =

a

f (t) dt x

Siden vi har at Z

x

Z

x+h

f (t) dt + a

Z

x+h

f (t) dt = x

f (t) dt x

Dette er intuitivt fra at vi legger sammen to omr˚ ader ved siden av hverandre. Formelt kan dette ogs˚ a vises via rieman-summer. Likning (B.2) kan da skrives som Z d 1 x+h g(x) = lim f (t) dt h→0 h x dx ˚ skrive om Tanken er n˚ a at vi bruker mellomverdi setningen for integraler til a integralet. Direkte bruk av likning (B.1) fra proposisjon (B.1.2) gir Z 1 x+h 1 d g(x) = lim f (t) dt = lim f (k)[x − (x + h)] = lim f (k) h→0 h→0 h→0 dx h x h ˚ fullføre beviset m˚ For a a vi bestemme k, for husk at k er avhengig av hva x er. Fra definjsjonen s˚ a er x ≤ k ≤ x + h, slik at n˚ ar h → 0 s˚ a vil k → x. Oppsumert s˚ a er alts˚ a Z d 1 x+h g(x) = lim f (t) dt = f (x) (B.3) h→0 h x dx ˚ vise. som var det vi ønsket a

II

1 analysens fundamentaltheorem.

213

Fra dette theoremet kan vi vise følgende korollar a [a, b] og la F være en Korollar B.1.1. La f være en kontinuerlig funkjson p˚ kontinuerlig funksjon p˚ a [a, b] slik at F 0 (x) = f (x) for alle x ∈ (a, b) da er b

Z

f (t) dt = F (b) − F (a) a

Mekr at korollaret krever kontinuetet p˚ a hele intervalet og ikke bare stykkevis ˚ vise det kontinuitet. Vi viser først et kort bevis for theoremet, for deretter a sterkere resulatet ved hjelp av middelverdi setningen. Bevis. Anta at F er en antiderivert av f , hvor f er kontinuerlig p˚ a [a, b]. Vi definerer Z x G(x) = f (t) dt (B.4) a

fra theorem (B.1.1), s˚ a vet vi at G ogs˚ a er en antiderivert av f . Dermed s˚ a m˚ a F og G høyst avike med en eller annen konstant c. G(x) = F (x) + c for alle ˚ la x = a s˚ x ∈ [a, b]. Ved a a har vi Z a F (a) + c = G(a) = f (t) dt = 0 a

som medfører at c = −F (a). Alts˚ a s˚ a er G(x) = F (x) − F (a) eller med andre ord Z b f (t) dt = F (b) − F (a) a

som ønsket. Proposisjon B.1.3. (Middelverdi setningen) La f : [a, b] → R være en kontinuerlig funksjon p˚ a det lukkede intervalet [a, b], og deriverbar p˚ a det a ˚pne intervalet (a, b) hvor a < b. Da eksisterer det en c ∈ (a, b) slik at f 0 (c) =

f (b) − f (a) b−a

Igjen uttelates beviset som bygger p˚ a Rolles sats. Men det kan finnes i de fleste innføringsbøker til Kalkulus. Deriblant [? , thm. 5.10]. Theorem B.1.2. (Analysens fundamentaltheorem. - Del II) La f og F ta reelle verdier p˚ a intervalet [a, b], hvor a, b ∈ R. Slik at den deriverte av F er f . Alts˚ a at F 0 (x) = f (x) for alle x ∈ [a, b]. Dersom f er Rieman-integrerbar p˚ a [a, b] s˚ a er Z

b

f (x) dx = F (b) − F (a) a

Bevis. Dette er et bevis som baserer seg p˚ a Riemann summer. Siden f er stykkevis kontinuerlig p˚ a et endelig interval s˚ a er f Rieman-integrerbar. La F være den

214

II

1 analysens fundamentaltheorem.

antideriverte til f p˚ a [a, b]. Vi tar utgangspunkt i F (b) − F (a), og la x1 , . . . , xn være tall slik at a = x0 < x1 < x2 < · · · < xn−1 < xn = b Vi antar videre at f er kontinuerlig p˚ a (xi−1 , xn−1 ) hvor i g˚ ar fra 1 til n. Siden vi har definert x0 = a og xn = b har vi F (b) − F (a) = F (xn ) − F (x0 ) vi kan skrive om høyresiden som en teleskoperende rekke slik at F (b) − F (a) =

n X

F (xi ) − F (xi−1 )

i=1

˚ skrive ut høyresiden og sammenlikne. Fra middelhvor det overlates til leser a verdi setningen (B.1.3) har vi n˚ a at F (xi ) − F (xi−1 ) = F 0 (ci )(xi − xi−1 ) hvor xi−1 < ci < xi . Dette er lov siden f er kontinuerlig p˚ a (xi−1 , xn−1 ). Ved innsetning i har vi n˚ a F (b) − F (a) =

n X

F 0 (ci )(xi − xi−1 ) =

i=1

n X

f (ci )∆xi

i=1

siden F (x) = f (x) for alle x ∈ (a, b). Dette kan bli sett p˚ a som et Riemann˚ la sum hvor ∆xi beskriver lengden av partisjonene og f (ci ) er høyden. Ved a k∆xi k → 0 f˚ as F (b) − F (a) =

lim

k∆xi k→0

n X i=1

Z f (ci )∆xi =

b

f (x) dx a

som var det som skulle vises. Den siste summen er hvordan vi opprinnelig definerte arealet av en funksjon. Alts˚ a som summen av rektangler hvor bredden g˚ ar mot null.

III

215

Tillegg C

C.1

KONVERGENS

N˚ ar vi studerer integraler er spørsm˚ alet om integralet har en verdi, minst like viktig som selve svaret. Hva er konvergens og hva menes med at integralet har en verdi? Definisjon C.1.1. Et uegentlig integral er et integral som skrives p˚ a formen Z b Z b lim f (x) dx , lim f (x) dx , b→∞

a→−∞

a

a

eller p˚ a formen Z

Z f (x) dx ,

lim

b→∞

b

a

a→−∞

b

f (x) dx ,

lim

a

Hvor grenseverdiene f (x) enten er udefinert, eller g˚ ar mot ±∞. En sum av uegentlige integral, er ogs˚ a uegentlig. Definisjon C.1.2. Et integral kalles konvergent, dersom det kan skrives som en endelig verdi. Alle integral som ikke er konvergent, kalles divergente. Eksempel C.1.1. For motivasjon studeres følgende integral Z 2 dx x −1 0 ˚ ignorere diskusjonen omkring integralet konverger eller ikke1 og vi velger a regner blindt ut integralet. Z 2 h i2 dx = log |x − 1| = log |−1| + log |1| = 0 0 0 x−1 Dette virker noe rart om en studerer grafen til funksjonen vist i figur 8. Da funksjonen klart bl˚ aser opp nære x = 1. S˚ a funksjonen ovenfor konvergerer, men vi vil gjerne skille mellom ulike typer konvergens. Vi har for eksempel Rieman-integrerbare funksjoner, LebesgueIntegrerbare funksjoner osv. Funksjonen ovenfor konvergerer om en betrakter prinsipial verdien av integralet, men divergerer ellers. Om vi sier at en funksjon er integrerbar, vil betydningen alltid være at funksjonen er Rieman-integrerbar. 1 Siden

vi ikke engang har diskutert hva konvergens er engang!

216

1 Konvergens

III

Definisjon C.1.3. Et uegentlig integral konvergerer hvis og bare hvis integralet kan skrives som en sum av uegentlige integral, som alle konvergerer. Ved a˚ bruke denne definisjonen av konvergens p˚ a integralet i eksempel (C.1.1) s˚ a m˚ a b˚ ade Z 1 Z 2 dx dx og x − 1 x −1 0 1 ˚ bryte opp integralet i interval (a, b) hvor kun konvergere. Teknikken er alts˚ aa en av grenseverdiene limx→a f (x) eller limx→b f (x) ikke eksisterer. I tilleg til at f (x) er øvre begrenset p˚ a [a, b]. Hvorfor vi velger a˚ definere konvergens p˚ a denne m˚ aten gir mer mening n˚ ar vi studerer begrepet uendelig nærmere. Uendelig er ikke noe tall, men noe vi reiser mot og aldri helt n˚ ar. For a˚ slippe hodeverk bør gjerne hastigheten p˚ a hvor ˚ si, siden vi uansett aldri kommer helt frem. raskt vi reiser ikke ha noe a Eksempel C.1.2. La oss betrakte om følgende integral Z ∞ 2x dx 2 −∞ 1 + x konvergerer eller divergerer. Fra diskusjonen ovenfor er m˚ ate a˚ studere hvordan integralet oppfører seg n˚ ar vi g˚ ar mot uendelig i ulikt tempo. Z a h ia 2x dx lim = lim log 1 + x2 = 0. 2 a→∞ −a 1 + x a→∞ −a Derimot om vi lar funksjonen vokse ’dobbelt’ s˚ a raskt mot uendelig f˚ as Z 2a h i2a 2x dx = lim log 1 + x2 lim = 2 log 2 . 2 a→∞ a→∞ −a 1 + x −a vi kan dermed konkludere med at integralet er udefinert da det er avhengig av ’hastigheten’ Dette problemet med hastigheter løses dersom en deler opp integralet i uegentlige integral. Slik at hvert integral bare har en singularitet. P-integral Z ∞ dx Proposisjon C.1.1. Integralet divergerer for alle relle eller komplekse p xp 0 mens Z 1 Z ∞ dx dx , p < 1 og , p>1 p x xp 0 1 konvergerer. Bevis. Dersom p < 1 s˚ a er  1 Z 1  dx 1 1 1 p−1 = x = lim 1 − c1−p = p 1−p 1 − p x→0 1−p 0 x 0

III

1 Konvergens

og likeledes s˚ a er Z ∞ 1

dx = lim c→∞ xp

Z

c

1

217

 dx 1 = lim 1 − c1−p = ∞ p c→∞ x 1−p

Deretter studeres tilfellet n˚ ar p > 1 s˚ a   1  Z 1 dx 1 1 1 p−1 = lim x lim 1 − p−1 = ∞ = p c→0 1 − p 1 − p c→0 c 0 x c og likeledes s˚ a er Z c Z ∞  dx 1 1 dx = lim = lim 1 − c1−p = p p c→∞ c→∞ x x 1 − p 1 − p 1 1 Siden (0, 1) konvergerer for p < 1 og (1, ∞) konvergerer for p > 1, s˚ a kan ikke (0, ∞) = (0, 1) ∪ (1, ∞) konvergere n˚ ar p < 1 eller p > 1. Da gjennst˚ ar det bare ˚ teste tilfellet hvor p = 1. a Z ∞ Z 1 Z ∞ dx dx dx = + = lim (− log x) + lim log x = ∞ x→∞ x→0 x x x 0 0 1 som begge divergerer. Dette fullfører beviset. Dette resultatet er svært nyttig n˚ ar vi bruker det i kombinasjon med sammenliknings testen. Proposisjon C.1.2. (Sammenliknings Testen) La f (x) og g(x) være to funksjoner definert p˚ a (a, b) slik at 0 ≤ f (x) ≤ g(x) er sant for alle x ∈ [a, b]. Dersom Z

b

g(x) dx a

konvergerer s˚ a konvergerer integralet over f (x). Dersom integralet over g(x) divergerer s˚ a divergerer integralet over f (x). Eksempel C.1.3. La oss se om følgende integral konvergerer Z ∞ (sin x)2 dx x2 0 dersom x ≥ 1 s˚ a har vi at (sin x)2 1 ≤ 2 x2 x dermed har vi fra p-testen at integralet ovenfor konvergerer siden p = 2 > 1. Dersom 0 ≤ x < 1, kan vi stedet bruke følgende sammenlikning (sin x)2 1 ≤ √ x2 x

218

III

1 Konvergens

˚ gange med x2 , p˚ ved a a begge sider s˚ a har en at (sin x)2 ≤ x3/2 . Dette stemmer 3/2 siden sin x ≤ x ≤ x , for x ≥ 0 som en for eksempel kan se fra taylorrekken til sin x. Dermed konvergerer integralet fra sammenlikningstesten og vi har Z ∞ Z 1 Z ∞ 1 (sin x)2 1 √ + 0≤ dx ≤ =3 2 x x2 x 0 0 0 som er en fin tilnærming. ˚ se p˚ En kunne fint ha f˚ att enda strammere grenser ved a a taylorrekken til (sin x)2 /x2 , men det er helt uviktig. Legg merke til at metoden ovenfor ikke kan ˚ vise at (sin x)/x konvergerer, selv om integralene har samme verdi. brukes for a Proposisjon C.1.3. (Grensesammenliknings Testen) La f og g enten være to strengt positive eller strengt negative funksjoner p˚ a [a, b] og la funksjonene være udefinert i punktet a. Dersom g(x) ∼ f (x) n˚ ar x ∼ a s˚ a konvergerer Z b Z b f (x) dx hvis og bare hvis g(x) dx konvergerer. a

a

Hvor g(x) ∼ f (x) n˚ ar x ∼ c betyr at limx→c f (x)/g(x) = 1. Begge testene ovenfor krever at funksjonene f og g ikke skifter fortegn p˚ a intervalet. S˚ a det er logisk a˚ lure p˚ a hvordan en skal behandle integraler som ikke er strengt positive eller negative. En m˚ ate a˚ tvinge en funksjon til a˚ være positiv p˚ a er a˚ ta absoluttverdien. Det er klart at b˚ ade f (x) og |f (x)| har samme singulariter s˚ a et naturlig Rb spørsm˚ al er hvilken konklusjon vi kan trekke om a f (x) , dersom vi vet noe om Rb integralet a |f (x)| dx konvergerer, eller divergerer. Et delvis svar p˚ a spørsm˚ alet er gitt i følgende proposisjon Proposisjon C.1.4. La f (x) være en funksjon dersom integralet Z b |f (x)| dx a

konvergerer, s˚ a konvergerer ogs˚ a integralet Z b f (x) dx . a

Rb Merk at det kontrapositive ikke stemmer! Dersom integralet a |f (x)| dx Rb divergerer betyr dette ikke at a f (x) dx divergerer. Et eksempel p˚ a dette er vist under Dirichlet integralet Lemma C.1.1. La f være definert som følger2   sin x dersom f (x) = x  0 ellers 2 Det

x 6= 0 , R∞

er i utgangspunktet ingen grunn til a˚ definere f slik, da integralet (sin x)/x dx er Riemann0 ˚ definere f slik at funksjonen blir kontinuerlig, deriverbar. integrerbart. Derimot er fordelen ved a

III

1 Konvergens Z

219



da konvergerer integralet

f (x) dx, men ikke absolutt. −∞

Bevis. Først kan en legge merke til at integralet er symmetrisk omkring origo ˚ betrakte (0, ∞). f (−x) = f (x), slik at det holder a ˚ vise at funksjonen ikke konvergerer absoFørste del av bevisetRvil g˚ a ut p˚ aa ∞ lutt, og deretter vise at 0 f (x) dx, konvergerer. Vi betrakter først integralet av funksjonen over et endelig interval Z N f (x) dx , 0

La n˚ a N ∈ N, da er Z 2πN N −1 Z 2π(n+1) X sin x sin x dx = x dx x 0 n=0 2πn Z 2π(n+1) N −1 X 1 ≥ |sin x| dx 2π(n + 1) 2πn n=0 Z 2π N −1 X 1 = |sin x| dx 2π(n + 1) 0 n=0 =

N −1 1 2 X π n=0 n + 1

som klart divergerer n˚ ar N → ∞. Dette viser at integralet ikke konvergerer ˚ bruke testene ovenfor i kombinasjon med p-integralene er det som absolutt. A ˚ vise de aller fleste integralene konvergerer. vil bli brukt for a ˚ Men dette integralet krever litt mer triksing. Trikset i denne oppgaven blir a atter en gang vende tilbake til delvis integrasjon. Vi deler igjen integralet ved x=1 Z N Z N h cos x iN Z N cos x sin x cos N cos x dx = dx = cos(1) − − − 2 x x x N x2 1 1 1 1 ˚ vise at siste integral konvergerer kan en som konvergerer n˚ ar N → ∞. For a bruke p-testen med x2 siden cos x/x2 ≤ 1/x2 n˚ ar x ≥ 1. Tilfellet N˚ ar x ∼ 0 s˚ a er sin x ∼ x s˚ a (sin x)/x ∼ 1. S˚ a Z 1 Z 1 sin x dx ≤ 1 dx x 0 0 dette følger fra grensesammenlikningstesten og medfører konvergerens. Dette medfører ogs˚ a at integralet konvergerer. ˚ gjøre. Først s˚ At integralet ovenfor konvergerer har med flere ting a a bl˚ aser ikke integralet opp nære origo da (sin x)/x ≤ 1 for |x| < 1. Faktisk s˚ a er (sin x)/x ≤ 1 ∀ x ∈ R, siden 1/x ’drar’ funksjonen mot null. Funksjonen sin x bidrar til at funksjonen oscillerer over og under x-aksen, og det er kanseleringen av disse omr˚ adene som bidrar til at funksjonen konvergerer. N˚ ar en ser p˚ a absoluttverdien av funksjonen f˚ ar en derimot ikke disse kanseleringene. I alle eksemplene tidligere har vi sett p˚ a funksjoner som g˚ ar mot 0 n˚ ar

220

III

1 Konvergens

x → ∞. At dette alltid er tilfellet for konvergente integral er feil. Det eksisterer alts˚ a funksjoner ar mot en verdi ulik R ∞ hvor limx→∞ f (x) ikke eksisterer – eller g˚ null – men k f (x) dx konvergerer. Fresnell integralene Lemma C.1.2. x

Z

Z

 sin t2 dt , C(x) =

Integralene S(x) =

x

 cos t2 dt konvergerer ∀x ∈ R.

0

0

Lemma C.1.3. Z



 sin x2 dx og

Integralene −∞

Z



 cos x2 konvergerer.

−∞

R ˚ forst˚ A a hvorfor R sin x2 dx konvergerer og ikke R sin x dx er ikke s˚ a veldig vanskelig. For sin(x2 ) har funksjonen et en topp over x-aksen, ogs˚ a en bunn under x-aksen. Arealet av disse toppene og bunnene blir mindre og mindre n˚ ar x → ∞. Dette er ikke tilfellet for sin x som oscillerere med samme hastighet hele tiden. Fra den alternerende rekketesten konvergerer alts˚ a integralet over sin(x2 ), men ikke sin x. ˚ bevise lemma (C.1.3) siden dette ogs˚ Vi kommer kun til a a beviser lem˚ vise at S(x) og C(x) konvergerer siden ma (C.1.2). Selv om det er rett frem a en integrerer en begrenset funksjon over et endelig interval. R

˚ betrakte inteBevis. Funksjonene er symmetrisk omkring origo, s˚ a holder det a gralene over (0, ∞). Her vil vi bare vise at integralet over sin(x2 ) konvergerer, men et nøyaktig likt bevis kan føres for cos(x2 ). Alternativt kan en vise først at integralene er like store, slik at det holder a˚ vise at ett av integralene konvergerer. ∞

Z

 sin x2 dx =

1

Z

0

 sin x2 dx +

Z

0



 sin x2 dx

1

Funksjonen er integrerbar p˚ a intervalet [0, 1], den bl˚ aser ikke opp og har ingen singulariterer. Vi m˚ a alts˚ a vise at integralet Z lim

N →∞

N

 sin x2 dx ,

1

√ √ ˚ bruk substitusjonen t = x2 , s˚ konvergerer. Ved a a x = t og 2 t dx = dt f˚ ar en Z

N

sin x2

2

Z dx =

1

1

N2

sin t cos N 2 cos 1 1 √ dt = − √ + − 2 4 2 t 2 N

Z 1

N2

cos t dt t3/2

˚ sammenlikne med p-integralene ser vi at siste integralet kjenner vi igjen. Ved a integraler konvergerer, siden 3/2 > 1 og cos t/t3/2 ≤ 1/t3/2 . Γ-funksjonen Z Lemma C.1.4. 0



tx−1 e−t dt konvergerer for alle x ∈ (0, ∞)

III

1 Konvergens

Bevis. Først s˚ a deler vi integralet i Z Z ∞ x−1 −t t e dt =

1 x−1 −t

t

e



Z

tx−1 e−t dt

dt + 1

0

0

221

og det neste steget er a˚ vise at de uegentlige integralene p˚ a høyre side konverge˚ finne en funksjon f (t) slik at 0 ≤ tx−1 e−t ≤ f (t), rer. Fremmgangsm˚ aten blir a og vise at integralet over f (t) er konvergerer. For det første integralet s˚ a er e−t ≤ 1 for t ≥ 0, slik at   x 0  Z 1 Z 1 cx t 1 x−1 −t x−1 − . 0≤ t e dt ≤ lim t dt = lim = lim c→0 c c→0 x c c→0 x x 0 Dersom x > 0, s˚ a vil ax → 0 n˚ ar a → 0+ , s˚ a integralet konvergerer mot 1/x. R 1 x−1 −t Dette viser at integralet over 0 t e dt konvergerer. For det neste integralet vil vi bruke at limt→∞ tr e−t/2 = 0 for alle r ∈ R. Alts˚ a for x > 0 eksisterer det en kx ∈ R slik at 0 ≤ tx e−t/2 ≤ 1 for t ≥ kx . Vi holder x konstant og deler integralet ved kx Z ∞ Z kx Z ∞ x−1 −t x−1 −t t e dt = t e dt + tx−1 e−t dt . 1

kx

1

Det første integralet p˚ a høyre side er over et endelig interval og har ingen singulariter p˚ a intervalet, alts˚ a konvergerer integralet. For det neste har vi at t ≥ kx slik at tx−1 e−t = (tx−1 e−t/2 )e−t/2 ≤ e−t/2 , s˚ a Z ∞ Z ∞   tx−1 e−t dt ≤ e−t/2 dt = lim 2e−kx /2 − e−c/2 = 2e−kx /2 . kx

c→∞

kx

Merk at uansett hva kx er s˚ aR konvergerer integralet. Dette viser at det andre ∞ integralet er konvergent, s˚ a 1 tx−1 e−t dt konvergerer. Dette fullfører beviset.

Betaintegralet Z Lemma C.1.5.

1

tx−1 (1 − t)y−1 dt konvergerer for alle x, y ∈ (0, ∞)

0

Bevis. Slik som beviset for konvergens av gammaintegralet deles integralet opp Z 1 Z 1/2 Z 1 x−1 y−1 x−1 y−1 t (1 − t) dt = t (1 − t) dt + tx−1 (1 − t)y−1 dt . 0

0

1/2 x−1

y−1

x−1

For det første integralet merk at t (1 − t) ≤t for 0 ≤ t ≤ 1/2, s˚ a   Z 1/2 Z 1/2 (1/x)x − cx 0≤ tx−1 (1 − t)y−1 dt ≤ tx−1 dt = lim c→0 x 0 0 N˚ a, dersom x > 0 s˚ a vil ax → 0 n˚ ar a → 0+ , s˚ a det første integralet konvergerer dersom x > 0. Tilsvarende for det neste integralet s˚ a er tx−1 (1 − t)y−1 ≤ y−1 (1 − t) for 1/2 ≤ t ≤ 1 s˚ a   Z 1/2 Z 1/2 (1/2)y − (1 − c)y 0≤ (1 − t)y−1 dt ≤ (1 − t)y−1 dt = lim− x c→1 0 0

222

1 Konvergens

III

N˚ a, dersom y > 0 s˚ a vil (1 − a)y → 0 n˚ ar a → 1− , s˚ a det første integralet R 1 x−1 konvergerer dersom y > 0. Dermed s˚ a konvergerer 0 t (1 − t)y−1 dt for alle x, y > 0 som var det som skulle vises.

C.1.1

KONVERGENSRADIUS

C.1.2

P R I N S I PA L

VERDI

Cauchys prinsippielle verdi (Cauchy principal value) er en m˚ ate a˚ gi en verdi til integralet som i utganspunktet divergerer.

III

C.2

2 Funksjonalanalyse

223

F U N K S J O N A L A N A LY S E

Lemma C.2.1. Anta fn er en funksjonsfølge definert p˚ a en a ˚pen delmengde D av C. Dersom fn konvergerer uniformt p˚ a ethvert kompakt (lukket og begrenset) undermengde av D til grensefunksjonen f , da er f analytisk p˚ a D. Videre s˚ a er sekvensen av deriverte fn0 konvergerer ogs˚ a uniformt mot f 0 p˚ a enhver komptakt undermengde av D. Lemma C.2.2. (Derivasjon under integraltegnet) La D være en a ˚pen mengde og la γ være en kontur av endelig lengde L(γ). Anta ϕ : {γ} × D → C er en kontinuerlig funksjon, og definer g : D → C ved Z g(z) = ϕ(w, z) dw γ

Da er g kontinuerlig. Videre dersom ∂ϕ/∂z eksisterer og er kontinuerlig p˚ a {γ}×D da er g analytisk med derivert Z dϕ 0 ϕ(w, z) dw g (z) = γ dz Korollar C.2.1. La D være en a ˚pen mengde og ϕ : [a, ∞] × D → C være en funksjon med kontinuerlig partiellderiverte ∂ϕ/∂z. Dersom integralet Z g(z) = ϕ(w, z) dw γ

konvergerer uniformt p˚ a en kompakt delmengde av D, da definerer det en analytisk funksjon p˚ a omr˚ adet og har deriverte Z dϕ g 0 (z) = ϕ(w, z) dw γ dz Lemma C.2.3. (Produkt representasjonen av Γ(s)) n Z ∞ t ts−1 dt Γ(s) = 1− n 0 dersom s = it + σ, hvor σ > 0 og t ∈ R. Bevis. Vi innfører funksjonen Z fn (s) = 0

n



t 1− n

n

ts−1 dt

˚ vise at F (s) − fn (s) g˚ og ønsker a ar mot null, n˚ ar n → ∞. Vi kan skrive Z n Z ∞  n  s−1 −t Γ(s) − fn (s) = e 1−t n t dt + e−t ts−1 dt , (C.1) 0

n

og ser at siste leddet g˚ ar mot null n˚ ar n vokser. Det virker rimelig at første ledd og g˚ ar mot null siden første ledd blir mindre og mindre. Tanken er n˚ a at vi ˚ vise at ønsker a  n t 0 ≤ e−t − 1 − ≤ n−1 t2 e−t . (C.2) n

224

III

2 Funksjonalanalyse

Dersom denne ulikheten stemmer s˚ a kan likning (C.1) skrives som Z n 1 Γ(s) − fn (s) ≤ 1 e−t tσ+1 dt < Γ(σ + 2) n 0 n som følger fra ulikheten ovenfor. Dette uttrykket g˚ ar mot null n˚ ar n → ∞ siden Γ(σ + 2) er endelig. Alts˚ a konvergerer fn (s) uniformt mot Γ(s) som ønsket. ˚ vise likning (C.2) tar vi utgangspunkt i at for 0 ≤ y ≤ 1 s˚ For a a har vi 1 + y ≤ ey ≤ (1 − y)−1 . For store n kan vi sette y = t/n, slik at 

t 1− n

n ≤e

−t

 ≤

t 1− n

−n

hvor vi ogs˚ a opphøyde ulikheten i n, s˚ a (e−t/n )n = e−t . Vi skriver om venstresiden av ulikheten ytterligere    n n  t t = e−t 1 − et 1 + 0 ≤ e−t − 1 + n n   n  n  t t ≤ e−t 1 − et 1 + 1− n n   n  2 t = e−t 1 − et 1 − 2 n Det neste steget blir a˚ bruke at dersom 0 ≤ a ≤ 1 s˚ a er (1 − a)n ≥ 1 − na, gitt at 2 2 na < 1. Lar vi a = t /n s˚ a har vi for store n  n t2 t2 1− 1− 2 ≤ n n Dermed s˚ a har vi 0≤e

−t



t − 1− n

n

≤ n−1 t2 e−t

og dette fullfører beviset. Proposisjon C.2.1. Γ-funksjonen definert som Z ∞ Γ(s) = e−t ts−1 dt , 0

er analytisk for alle σ > 0. R∞ Bevis. Først legger vi merke til at for a > 0 s˚ a er funksjonen a e−t ts−1 dt ana˚ vise dette m˚ lytisk. For a a vi bare vise at funksjonen konvergerer unfomormt p˚ a enhver kompakt undermengde av D. Deretter kan vi bruke korollar (C.2.1) siden alle de andre vilk˚ arene er tilfredstilt. Som forventet s˚ a dominerer eksponensialen integralet for store n Z n Z ∞ Z ∞ Z ∞ −t s−1 −t s−1 −t s−1 = ≤ e t dt − e t dt e t dt e−t tσ−1 dt a

a

n

n

III

2 Funksjonalanalyse

225

˚ derivere og se Funksjonen e−t/2 tσ−1 er øvre begrenset av, som vi kan se ved a p˚ a ekstrema. Merk at ekstrema ikke befinner seg i noen av endepunktene. Dette medfører at vi kan skrive  Z ∞ −t/2 σ−1 |fn − f | ≤ max e t e−t/2 dt ≤ 2C(σ)e−n/2 n

Hvor vi ikke bryr oss om hva C er, bare at den avhengier av σ og ikke n. Slik at n˚ ar n → ∞ s˚ a vil |fn − f | = 0, som er selve definisjonen p˚ a uniform konvergens. ˚ vise at funksjonen er analytisk for Dette holder bare for n ≥ 1, og vi ønsker a σ ∈ (0, ∞). Vi definerer følgende funksjon Z ∞ fn (s) = e−t ts−1 dt 1/n

Ved argumentet ovenfor s˚ a er hver fn analytisk. Anta at σ ≥ c ≥ 0. For 0 < t ≤ 1 s˚ a har vi e−t < 1 og tσ−1 ≤ tc−1 . Alts˚ a for n > m, Z Z   1/m 1/m 1 1 1 −t s−1 c−1 e t dt ≤ t dt = − 1/n c ma na 1/n Vi m˚ a n˚ a vise at uttrykket ovenfor g˚ ar mot null. Gitt ε > a kan vi alltid velge 0 s˚ 0 ≤ δ ≤ 1 slik at (m−a − n−a ) /c ≤  n˚ ar m−1 − n−1 ≤ δ. Alts˚ a tilfredstiller fn Cauchys betingelse for uniform konvergens p˚ a en kompakt undermengde av det positive halvplanet σ > 0. Fra lemma (C.2.1) følger det at Γ-funksjonen er analytisk for σ > 0.

226

C.3

III

3 Bohr-Mullerup Theoremet

BOHR-MULLERUP THEOREMET

Beviset her baserer seg p˚ a Rudin’s bevis [? , thm. 8.19] og hele kapitel 4 fra [? ] se spesielt theorem 4.4. Før vi beviser theoremet trengs et lite lemma Lemma C.3.1. La f : (a, b) → R være en konveks funksjon, da holder f (t) − f (s) f (u) − f (s) f (u) − f (t) ≤ ≤ t−s u−s u−t for alle u, t, s ∈ (a, b) slik at a < s < t < u < b. Bevis. La a < s < t < u < b da er f (λs + (1 − λ)) ≤ λf (s) + (1 − λ)f (u) siden f er konveks. Vi lar λ =

u−t u−s

(C.3)

slik at t = 0, λ = 0 og t = s, λ = 1. Da er 

λs + (1 − λ)u = λ(s − u) + u =

t−u u−s

 (u − s) + u = t

Fra dette følger det at likning (C.3) kan skrives som f (t) ≤ f (u) + ˚ bruke at Ved a

u−t u−s

=1+

s−t u−s

 u−t f (s) − f (u) u−s

(C.4)

i likningen ovenfor f˚ ar vi

f (t) ≤ f (s) +

 s−t f (s) − f (u) u−s

(C.5)

˚ dele p˚ Ved a a s − t kan likning (C.4) skrives p˚ a formen f (u) − f (s) f (u) − f (t) ≤ u−s u−t

(C.6)

˚ dele ulikheten p˚ Tilsvarende s˚ a kan vi skrive om likning (C.5) ved a a s − t, f (t) − f (s) f (u) − f (s) ≤ t−s u−s

(C.7)

alts˚ a s˚ a er f (u) − f (s) f (u) − f (t) f (t) − f (s) ≤ ≤ t−s u−s u−t ˚ vise. som var det vi ønsket a Theorem C.3.1. (Bohr–Mollerup) Gitt en funksjon f : (0, ∞) → (0, ∞) som tilfredstiller 1) f (1) = 1 2) f (x + 1) = xf (x) 3) log f er konveks.

III

3 Bohr-Mullerup Theoremet

227

Da følger det at f (x) = Γ(x) ∀ x ∈ (0, ∞). ˚ Bevis. Vi vet allerede at Γ(s) tilfredstiller punkt (1) til (3), og det gjennst˚ ar a vise at f (x) er unikt definert utifra disse restriksjonene. I tillegg er det fra (2) ˚ se p˚ nok a a tilfellet hvor x ∈ (0, 1). Vi innfører funksjonen ϕ = log f . Punkt (1) sier at ϕ(1) = 0, punkt (2) gir at ϕ(x + 1) = ϕ(x) + log x

(C.8)

mens punkt (3) betyr at ϕ er konveks. La x ∈ (0, 1), og la n ∈ N. Vi bruker oss n˚ a av første del av lemma (C.3.1), og setter s = n, t = n + 1 og u = n + 1 + x i likning (C.6) ϕ(n + 1) − ϕ(n) ≤

ϕ(n + 1 + x) − ϕ(n + 1) x

˚ sette inn s = n + 1, t = n + 1 + x og u = n + 2 i likning (C.6) Tilsvarende ved a gir ϕ(n + 1 + x) − ϕ(n + 1) ≤ ϕ(n + 2) − ϕ(n + 1) x Likning (C.8) sier n˚ a at ϕ(n + 1) − ϕ(n) = log n, og i tillegg ved a˚ bruke (2) har vi ϕ(n + 2) − ϕ(n + 1) = log(n + 1). Vi kan kombinere ulikehetene til log n ≤

ϕ(n + 1 + x) − ϕ(n + 1) ≤ log(n + 1) x

(C.9)

˚ bruke ??, n + 1 ganger p˚ Ved a a ϕ(n + 1 + x) s˚ a f˚ ar vi ϕ(x + n + 1) = ϕ(x + n) + log(x + n) = ϕ(x + n − 1) log(x + n) + log(x + n − 1) .. .   = ϕ(x) + log (x + 1)(x + n − 1) · · · (x + 1)x Ved a˚ sette x = 0 i utledningen ovenfor v˚ ar vi ogs˚ a ϕ(n + 1) = log n! siden n ∈ N. Vi kan n˚ a skrive om likning (C.9)   ϕ(x) + log (x + 1)(x + n − 1) · · · (x + 1)x − log n! ≤ log(n + 1) log n ≤ x ˚ gange ulikheten med x, som er gyldig siden x ∈ (0, 1) og Det neste steget blir a ˚ trekke fra log(nx ) fra hvert ledd a   0 ≤ ϕ(x) + log (x + 1)(x + n − 1) · · · (x + 1)x − log n! − log nx ≤ log(n + 1)x − log nx ˚ forenkle likningen ovenfor f˚ Ved a ar en     n!nx 1 0 ≤ ϕ(x) − log ≤ x log 1 + x(x + 1) · · · (x + n) n

228

3 Bohr-Mullerup Theoremet

˚ n˚ Dette uttrykket holder for alle n og ved a a la n → ∞ f˚ ar vi   n!nx ϕ(x) = lim log n→∞ x(x + 1) · · · (x + n)

III

(C.10)

siden log(1 + 1/n) → log 1 = 0. Ved a˚ ta exp p˚ a begge sider av likning (C.10) f˚ as   n!nx f (x) = lim n→∞ x(x + 1) · · · (x + n) siden eϕ(x) = f (x). Dette viser at funksjonen f har en unik representasjon, det eksisterer alts˚ a bare en funksjon som tilfredstiller punkt (1) til (3). Siden vi vet at Γ-funksjonen ogs˚ a tilfredstiller punkene, m˚ a f (x) = Γ(x). Dette fullfører beviset.

V

230

V

1 Kortsvar

5.1

K O R T S VA R 1.3.3

1.2 1. 2π 2. π/2 1. π/3

3. 

log 2 0

a 6= 0 ellers



log 2 0

a 6= 0 ellers

I= 2. π/3 4. 1.3.2 hello I=

1. x + 3 ln |x| + C 5. 6. log 5 √

27. 2 7. log 5

√ √ 2 x − 4 ln x + 2 + C

49.

8. π 2 9. 6 10. 6

70. π

11. π/2 13. log 5

72.

x3 − log |x + 1| 3

√ 14. log 2 > 1/ 2

V

8.1 Oppgaver

2.3

1. like f (−x) = f (x) 2. 0 3. 1/2 4. 0 6. π 2.4

8. 9. π/2 10. −1 + 2 log 2 11. π/2 2.5

12.

π 2cd  13. log

2 π



14. Del opp og la u = ex 15.

π 2a

16. Del opp og la u = ex

231

232

V

1 Kortsvar

2.5.1

2.8

1. 1. π/3 2.6.2 2. 2π 1. √ 3 3

π cos a 2

4.

2. arctan √

x +2

x2

5. π 2 /4 3π 3. √ 2 2

2.10.2

2.7.1  1. log

1. ex sin x + C

2 π

√ 2. π /2





2 π





2 π



2. log 3. c = 3, k = 2 

2 π





2 π



4. log

5. log

2.7.2

1. (x − 1)4 ex + C

3. 0

4. log

2.10.3

1. 4I± = sin 2x + 2x

2. (x − 1)4 ex + C 3. (x − 1)4 ex + C 4. (x − 1)4 ex + C

2. 4I± = sin 2x + 2x

3. 4I± = sin 2x + 2x

V

14.1 Oppgaver

2.11.1

233

18. 2

1. arcsin(2x − 1) + C 2. ln (cos x + sin x) + C

19. π 20. 2 21. arctan x +

3. 2

1 arctan(x3 ) 2

 3/2     22. 1Roger 1 1 1+ 3 1+ −5 +C x x 1 23. 2013 1 1 5. + arctan x2 24. 0 2x 2

2 4. 15

6.

1 π e 2

x−a − 1 arctan( x ) + C 7. ln x+a a a 8. ln x · ex + C

25.

1 2

26. 2 π 27. √ 2 ab r

9. π/4 10. π/4

29.

1 x − log x 1 x log + arctan 4 x + log x 2 log x

30.

1 π ln(6) 6

q √ 11. 4 1 + x 12. nπ 13.

1 2x e +C 2

q p 14. 2 x + x2 + 1 + C 1 1 15. log 1 − 6 + C 6 x 16.

3π 2 16

17. 2 18.

1 (x + 2)3 (x + 22) +C 300 (x + 7)4

√ √  1 + 2 log x + 1 + x x

28. −2 1 +

31. 1/2 32.

2 2 log2 (2)

1 33. log 2



e−1 e+1

34.

log 5 2 log(2/3)

36.

π 2

37. −

x x5 + x + 1



234

1 Kortsvar

38. 2 log 2 − 1

π 39. √ 2 ab

40.

 π 3 2

V

V

14.2 Oppgaver

235

2.11.1

2.11.2

41. π/4

1. I1 = π/4 I2 = −π/4 2. I1 = π/4 I2 = −π/4

42. log(5/4)C

  p 3. arcsinh(x) = log x + x2 + 1 43. π 4. 1/r n˚ ar r > R > 0, 1/R n˚ ar R > r > 0

44. 2

5. I1 = π/4 I2 = −π/4

√  45. 2x sin x + C

6. 46. log

√ x2 − 1 + x √ x2 + 1 + x

! +C

7. I(α) = απ/4 8. β = a2 /5 2ab cos θ = 25

47. 0

9. t = 48. 1

√ 10. π/ 3

49. π 2 /4

π 11. √ 4 2

50. 42 /32

12. t =

1 π + πn n ∈ N 2

13. π

π 51. √ 3 6

52. 1/2 log

1 π + πn n ∈ N 2

14. −1 + 2 log 2 p 3/2

15. −1 + 2 log 2 16. A = B = 1, C = 0, π/3

53. 2

√ 17. 2 18. 2

236

V

1 Kortsvar 

19. 47

41. √

20. 2 − 2 42. 21. I = π/2

4 3

2

26 15 − log 3 3 2

43. xe2∗x

2

22. I = positiv = 22/7 − π 44. hint: u = x2 , π 23. 4φ 

4 3

24.



29.

4 3

u=t

45. π 46. π

2

√ 47. a = 3, b = 4 s˚ a ab + ba = 12

25. π 2 26.

5 49

48. B = 1/4, A = −1/4. √ a ab + ba = 12 49. a = 3, b = 4 s˚

2

π2 22n+1

  2n n

3.3.2 3. arcsin(2x − 1) + C

30. π/4 4. 31. π/4

aρ − bρ Γ(1 − ρ) ρ

√ 5. C = log 2π .

32. Hint: La integralene f˚ a samme grenser. 3.3.3 33.

1.

√ 34. a = 3, b = 4 s˚ a ab + ba = 12 35. I =

x(x − 2) , a = 0, b = 16/9 2

π/n sin π/n 2. π/ sin πn

3.3.4

36. x = tan tan β

3. I(m) =

37. 2

3.3.9

√ 38. a = 3, b = 4 s˚ a ab + ba = 12

1.

39. 2 40. a = 0, b = 1 max(I) = 1/2

2.

π 2

sign m, I(0) = 0.

V

1 Kortsvar

3.6

3.10.2

1. arcsin(2x − 1) + C

1. π/2

2. arcsin(2x − 1) + C 1 2

3.



1 1 + a b



π √ 2 ab

4. arcsin(2x − 1) + C 5. arcsin(2x − 1) + C 3.7

237

2π 2. √ − 1 3 3 3. π/2 4. π/2 5. π arctanh γ 6. π arctanh γ 8. Hint xa − xb = (xa + 1) − (xb + 1)

3.8  7. log

2 π







8. log

9. log

10. π arcsin b π(−1)n n!

2 π



11.

2 π



12. π/2

14. π/2

1. ζ(3)

15. π/2

3.

π 4

4. ζ(3) 5.

1 π2 log2 (2) − 2 12

∂ ∂ + ∂α ∂β

√ 13. −2 ± 5

3.10.1

√ 2. π



16. π/2 √ 3 17. log 2π + 4 18. π/2 √ 3 19. log 2π + 4 20. π/2

n

1 √ ab

238

5.30

V

30 Langsvar

L A N G S VA R

1.2

˚ benytte proposisjon (2.8.1) 1. Det logiske her blir a

˚ benytte proposisjon (2.8.1) siden uttrykket er p˚ 2. Det logiske her blir a a formen xR(sin x, cos2 x). Dermed s˚ a er Z π Z π π (cos x)2 sin x dx . I= x(cos x)2 sin x dx = 2 0 0 ˚ bruke substitusjonen u 7→ cos x med du = − sin x. Videre Det neste steget blir a s˚ a blir grensene u = cos 0 = 1, u = cos π = −1. π = 2

Z

−1 2

Z

−u dx = π 1

1 2

x dx = π 0



x3 3

1 , 0

hvor vi snudde grensene og brukte at x2 er symmetrisk omkring origo. Dermed Z π π x cos2 x sin x dx = , 3 0 som var det som skulle vises. 1.3.2 hello 1. Vi legger merke til at vi kan dele opp integralet i to, som vist under Z 2 Z 2 x + 3x x 3x dx = + 2 dx 2 2 x x x Z 2 Z x 3x = dx + dx x2 x2 The first integral kan forkortes til 1, mens i det andre kan vi sette konstantleddet utenfor, og faktorisere ut en x i teller og nevner. Z Z 1 = 1 dx + 3 dx x = x + 3 ln |x| + C Hvor vi tilslutt brukte at (ln x)0 = 1/x

70. Dette integralet kan se noe vanskelig ut, men løses relativt greit ved substitusjon. Vi kan for eksempel velge u 7→ ex eller t 7→ ex/2 . Dersom vi bruker

V

3.3 Oppgaver

239

y

y = x + 2π y = sin x x = sin y

D x

Figur 5.1: Omr˚ adet D avgrenset av de tre funksjonene. første substitusjon f˚ ar vi at du == ex dx = u dx. Videre s˚ a blir ex/2 = (ex )1/2 = √ u. Z ∞ Z ∞ √ Z ∞ ex/2 u du du √ dx = = x 1 + e 1 + u u u (1 + u) −∞ 0 0 ˚ integrere. La oss heller prøve den andre som ikke virker spesielt mye enklere a substitusjonen. Da er dt = 12 ex/2 dx = 2t dx, og ex = (ex/2 )2 = t2 Z ∞ Z ∞ Z ∞ 2 dt ex/2 t du dx = = 2 =π x 1 + t2 t 1+t −∞ 1 + e 0 0 Mye enklere. Siden t = ex s˚ a vil t → ∞ n˚ ar x → ∞ og t → 0 n˚ ar x → −∞ siden t = lim ex = lim e−x = lim x→−∞

x→∞

x→∞

1 =0 ex

I aller siste overgang ble integralet av arctan funksjonen brukt, se. 1.3.3 2. ˚ bruke delvis integrasjon to ganger. Vi ser 5. Integralet kan beregnes ved a først p˚ a det ubestemte integralet og f˚ ar Z Z 1 2x 1 2 2x 2 (x − 1)e dx = (x − 1) · e − 2x · e2x dx 2 2   Z 1 1 = (x2 − 1)e2x − x · e2x − e2x dx 2 2   1 2 1 1 = x − x− e2x + C 2 2 4

240

V

30 Langsvar

˚ gjøre ting I første delvise integrasjon ble u = x2 − 1 brukt og v = e2x /2. For a litt enklere skrives integralet om til Z   22 (x2 − 1)e2x dx = 2x2 − 2x − 1 e2x + C = 2 · x(x − 1) − 1 e2x + C Setter vi inn grensene v˚ are f˚ ar vi n˚ a Z ϕ h   iϕ 22 x2 − 1 e2x dx = 2 · x(x − 1) − 1 e2x ψ

ψ

  = 2 · ϕ(ϕ − 1) − 1 e2ϕ − 2 · ψ(ψ − 1) − 1 e2ψ   = 2 · 1 − 1 e2ϕ − 2 · 1 − 1 e2ψ = e2ϕ − e2ψ I den andre overgangen ble det brukt at ψ og ϕ løser likningen x(x − 1) = 1. ˚ beregne ϕ og ϕ direkte for a ˚ beregne integralet. Merk det var ikke nødvendig a ˚ slippe den delvise integrasjonen kan vi gjøre følgende antakelse For a Z   x2 − 1 · e2x dx = Ax2 + Bx + C · e2x Dette virker logisk for n˚ ar vi deriverer er polynom ganget med en eksponensialfunksjon, har polynomet i svaret like høy grad. [xex ]0 = [1ex + xex] = [1 + x]ex for eksempel. Da derivasjon og integrasjon er motsatte opperasjoner bør dette holde her og. Derivasjon gir   (x2 − 1) · e2x = 2A · x2 + 2(A + B) · x + (2C + B) · e2x ˚ sammenlikne koeffisientene s˚ Ved a a er 2A = 1, 2(A + B) = 0 og 2C + B = −1. Slik at A = 1/2, B = −A = −1/2 og C = (B − 1)/2 = −1/4 og   Z  1 2 1 1 2 2 2x 2 x − x− e2x = 2 · x(x − 1) − 1 e2x 2 (x − 1) · e dx = 2 2 2 4 som før. 6. 5 7. Vi har via substitusjonen u 7→ 1/t, s˚ a er du = − dt/t2 ⇒ dt = − du/u2 . Z f (1/x) = 1

1/x

log t dt = 1+t

Z

x

− 1

log 1/u du = 1 + (1/u) u2

Z 1

x

1 log u du u 1 + u2

Legger vi sammen funksjonenene f˚ ar vi totalt sett Z x Z x Z x log t 1 log t t log t + log t f (x) + f (1/x) = dt + dt = dx 1 + t t 1 + t t(1 + t) 1 1 1 Z x Z x (1 + t) log t log t (log x)2 = dt = = t(1 + t) t 2 1 1 Dermed s˚ a blir f (e) + f (1/e) = (log e)2 /2 = 1/2. Som ønsket.

V

30 Langsvar

241

8. Vi beregner først de to første integralene 2π

Z

π

Z dφ

0

0

 π  sin θ dθ = 2π − cos θ 0 = 2π − cos π + cos 0 = 4π

˚ beregne det siste Siden cos π = −1 og cos 0 = 1. Det enkleste en kan gjøre for a integralet er delvis integrasjon. Vi har Z Z 2r 2 du 2r 1 1 dr = u =− =− 2 2 1 2r u 1 + r2 (1 + r ) via substitusjonen u = 1 + r2 , s˚ a er du = 2r dr slik at dr = du/2r. Vi bruker n˚ a delvis integrasjon med r 2 1 u0 = 2

v0 =

og

u =

2r 2

(1 + r2 ) 1 v =− 1 + r2

og

kan integralet skrives som Z



I = 4π

2r (1 +

0

2 r2 )

·

r dr = 4π 2



−r 2 (1 + r2 )

∞

Z − 4π

− 0

0



1 dr 2 1 + r2

Den første delen av integralet g˚ ar mot null slik at ∞

Z = 4π

r2

Z 2

(1 + r2 )

0

dr = 2π 0



 π  ∞ dr − 0 = 2π arctan r = 2π 0 1 + r2 2

der det siste integralet er den deriverte av arctan r. Oppsumert s˚ a er alts˚ a Z



Z

0

π

Z



sin θ dθ

dφ 0

0

˚ beregne. som var det vi ønsket a

9. 7

10. 8

13.

14.

r2 (1 +

2 r2 )

Z dr = 2π 0



dr = π2 1 + r2

242

V

30 Langsvar

2.3

˚ vise dette integralet p˚ 2. Det er flere ulike m˚ ater a a   Z 2π (1 + sin x)1+cos x log dx = 0 . 1 + cos x 0 Merk at integralet bl˚ aser opp n˚ ar x = π og x = 3π/2 slik at det er naturlig a˚ dele inn integralet som følger. Z



π/2

Z f (x) dx =

0

|0

Z π Z 3π/2 Z 2π f (x) dx + f (x) dx + f (x) dx + f (x) dx π 3π/2 {z } | π/2 {z | {z } } | {z } I

III

II

IV

En kort skisse av løsningene er gitt under Løsning a) Alle integralene kan mappes til intervalet (0, π/2), deretter fra symmetri er integralet null. Løsning b)

Integalene kan beregnes direkte I = −1 + 2 log 2 II + IV = −2 log 2 III = 1

hvor en ser med en gang at summen er 0. Løsning c) En viser at Z 2π Z (1 + cos x) log(1 + sin x) dx = 0



log(1 + cos x) dx

0

Alle disse metodene fungerer utmerker. Her blir a) og c) studert nærmere. De individuelle integralene vil bli studert senere i heftet. ˚ mappe hvert integral p˚ a) Begynner a a (0, π/2). Z π II = (1 + cos x) log(1 + sin x) − log(1 + cos x) dx π/2 π/2

Z

(1 − sin y) log(1 + cos y) − log(1 − sin y) dy

= 0

Der substitusjonen x 7→ y + π/2 ble brukt. Z

3π/2

(1 + cos x) log(1 + sin x) − log(1 + cos x) dx

III = π Z π/2

(1 − cos y) log(1 − sin y) − log(1 − cos y) dy

= 0

V

30 Langsvar

243

Der substitusjonen x 7→ y + π ble brukt. Siste interval blir Z 2π (1 + cos x) log(1 + sin x) − log(1 + cos x) dx IV = 3π/2 π/2

Z

(1 + cos y) log(1 − sin y) − log(1 + cos y) dy

= 0

˚ legge sammen II + III + IV f˚ Med bruk av x 7→ 2π − x. Ved a ar en Z π/2 II + III + IV = log(1 − sin y) − log(1 − cos y) − sin y log(1 + cos y) 0

˚ bruke De to første leddene kanselerer hverande, som kan bli sett siden ved a z 7→ π/2 p˚ a en av de, slik at Z π/2 Z π/2 II + III + IV = − log(1 + cos y) sin y dy = − log(1 + sin y) cos y dy 0

0

Igjen via enten z 7→ π/2 − y eller siden integralvet over f (x) er likt integralet ˚ legge dette sammen med over f (π − x) (proposisjon (2.3.1)). Ved a Z π/2 II = (1 + cos x) log(1 + sin x) − log(1 + cos x) dx 0

f˚ ar en det koselige resultatet Z I + II + III + IV =

π/2

log(1 + sin x) − log(1 + cos x) dx = 0

0

˚ se at integralet er null Siden v˚ art originale integral var delt opp i I til IV. For a kan en igjen benytte seg av enten z 7→ π/2 − y eller proposisjon (2.3.1). Z π/2 Z 0 Z π/2 log(1 + sin x) = − log(1 + sin(π/2 − z)) dz = log(1 + cos z) dz . π/2

0

0

som ønsket. c)

Integralet kan skrives som Z 2π (1 + cos x) log(1 + sin x) − log(1 + cos x) dx 0

M˚ alet blir n˚ a a˚ vise at (1 + cos x) log(1 + sin x) = log(1 + cos x). Kall integralene for henholdsvis A og B. Ved a˚ bruke substitusjonen x 7→ π − y kan første del av integralet skrives som Z 2π A= (1 + cos x) log(1 + sin x) dx 0 Z π = (1 − cos y) log(1 + sin y) dy −π 2π

Z

(1 − cos x) log(1 + sin x) dy

= 0

244

V

30 Langsvar

˚ ta Der en benyttet substitusjonen x 7→ y + 2π ble brukt i siste overgang. Ved a gjennomsnittet av første og siste integral f˚ ar en 1 2

A=



Z

Z



2 log(1 + sin x) dx =

log(1 + cos x) dx = B

0

0

Dermed s˚ a er Z 2π Z (1 + cos x) log(1 + sin − log(1 + cos x) dx = 0



A − B dx = 0

0

siden A = B 1. ˚ dele integralet som 3. En mulig fremgangsm˚ ate er a ∞

Z

dx = 1 + x2

−1

1

Z

dx + 1 + x2

−1

Z



dx 1 + x2

−1

(5.1)

Første integralet kan beregnes via (2.3.2) s˚ a Z

1

Z

dx = 1 + x2

−1

1

−1

1

Z

1 1 + dx = 2 1 + x2 1 + (−x)2

0

dx 1 + x2

(5.2)

˚ bruke substitusjonen y 7→ 1/x slik at Siste integralet kan vises ved a ∞

Z 1

dx =− 1 + x2

Z

0

dx dy = 2 1 + (1/y) y 2

1

1

Z 0

dy 1 + y2

(5.3)

˚ sette inn likning (5.2) og (5.3) i (5.1) f˚ Ved a ar en som ønsket at ∞

Z

−1

dx =2 1 + x2

1

Z 0

dx + 1 + x2

Z

1

0

dx 1 + y2

(5.4)

˚ vise siste likhet kan en ta utgangspunkt i For a Z 0



dx = 1 + x2

Z 0

1

dx + 1 + x2

Z 1



dx =2 1 + x2

Z

1

0

dx 1 + x2

˚ gange likheten Hvor igjen likning (5.2) ble benyttet i siste overgang. Ved a ovenfor med 3/2 s˚ a er 3 2

Z 0



dx =3 1 + x2

Z 0

1

dx = 1 + x2

Z



−1

dx 1 + x2

som var det som skulle vises. I siste overgang ble (5.4) brukt. 5. La oss benytte oss av den gamle og kjente identiteten Z a Z a f (x) dx = f (a − x) dx 0 1 Takk

til Rob Johnson for denne løsningen

0

V

30 Langsvar

245

slik at integralet kan skrives om som følger Z π (π − x)R(sin x, cos2 x) dx

(5.5)

0

Der det ble brukt at sin(π − x) = sin x og at cos2 (π − x) = (− cos x)2 = cos2 x. N˚ a er Z π Z π Z π xR(sin x, cos2 x) dx = πR(sin x, cos2 x) dx − xR(sin x, cos2 x) dx 0

0

0

˚ dele likning (5.5) i to. Ved a ˚ legge til det Der det ikke ble gjort annet enn a opprinnelige integralet p˚ a begge sider f˚ ar en Z π Z π 2 2 xR(sin x, cos x) dx = π R(sin x, cos2 x) dx 0

0

˚ dele likningen p˚ Beviset fullføres n˚ a ved a a 2. Det samme gjelder ikkefor ˚ R(cos x, sin2 x) siden ved a bruke π − x f˚ a r en R cos(x − π), sin2 (π − x) =  2 R − cos x, sin x . Minustegnet ødelegger dessverre for bruken av denne teknikken. 2.4 7. a)

˚ vise dette er rett frem A Z T Z nT Z T 1 f (nx) dx = f (u) du = f (x) dx n 0 0 0

Hvor substitusjonen u = nx ⇒ dx = du/n. Grensene blir u = n · 0 = 0, og u = nT . I siste overgang ble det benyttet at (2.18) er gyldig siden f (x) er periodisk. b) Denne og neste deloppgave følger samme fremmgangsm˚ ate. Vi bruker først substitusjonen y 7→ 2x, slik at Z π/2 Z Z π/2 1 π f (sin 2x) dx = f (sin y) dy = f (sin x) dx 2 0 0 0 Hvor proposisjon (2.4.5) ble brukt i siste overgang med m = 0 og n = 1. Dette ˚ dele integralet i to, og benytte u 7→ π − y p˚ kan og vises ved a a den siste delen Z π Z 0 Z π/2  f (sin y) dy = −f sin(π − u) du = f (sin u) du π/2

π/2

0

˚ være Siden sin(π − u) = sin π cos u − cos π sin u = 0 − (−1) sin u = sin u. For a pinlig nøyaktig. Dette medfører Z π Z π/2 Z π Z π/2 f (sin y) dy = f (sin y) dy + f (sin y) dy = 2 f (sin y) dy 0

som vist før.

0

π/2

0

246

V

30 Langsvar

c) Denne oppgaven vises p˚ a samme m˚ ate som likning (2.18). Merk at vi ikke kan benytte oss av a), siden sin x har periode 2π og ikke π/2. Derimot s˚ a er π/2

Z 0

Z

1 f (sin nx) = n

nπ/2

f (sin x) 0

˚ bruke u 7→ xn. Siden n ∈ Z/{0} er et heltall kan integralet deles inn i ved a perioder, slik at Z 0

π/2

   n Z n Z 1 X π/2 kπ 1 X kπ/2 f (sin x) = f sin f (sin nx) = −y n n 2 (k−1)π/2 0 k=1

k=1

Hvor substitusjonen x 7→ kπ/2 − y ble benyttet i siste overgang. variabelen ble sløyfet grunnet plassmangel, men er dx, dx og dy. argumentet f˚ ar en  cos y ,          kπ kπ kπ sin y , sin − y = sin cos y − cos sin y = − cos y ,  2 2 2   − sin y ,

IntegrasjonsSkriver en ut k k k k

= 4m + 1 = 4m + 2 = 4m + 3 = 4m

Hvor m ∈ Z. Tanken er at argumentet alltid reduseres til enten ± sin x eller ± cos y. Dermed kan summen n˚ a skrives som π/2

Z 0

n Z X

π/2

   kπ f (sin nx) = f sin − y du 2 k=1 0 Z π/2 Z π/2 = f (cos y) dy + f (sin y) dy Z

0 π/2

0 π/2

Z

f (− sin y) dy + · · ·

f (− cos y) dy +

+ 0

0

Siden f (x) = f (−x) s˚ a er f (− sin x) = f (sin x) og f (− cos x) = f (cos x). Vi kan se at integralene over sin x og cos x er like Z

π/2

Z

π/2

f (sin y) dy = 0

f (cos y) dy

(5.6)

0

Ra Ra ˚ bruke y 7→ π/2 − w, 0 f (x) dx = 0 f (a − x) eller lemma (2.4.1). ved enten a ˚ sette inn dette kan integralet skrives som Ved a Z

nπ/2

Z

π/2

f (sin x) = 0

Z

π/2

f (sin y) dy + 0 Z π/2

f (sin y) dy 0 Z π/2

f (sin y) dy + · · ·

f (sin y) dy +

+ 0

Z

0 π/2

f (sin y) dy

=n 0

˚ dele begge siden p˚ som ønsket. Beviset fullføres ved a a n. Litt mer formelt s˚ a er alle tall heltall 0, 1, 2 eller 3 tall unna 4 gangen. Slik at alle heltall kan skrives

V

30 Langsvar

247

p˚ a formen 4m, 4m + 1, 4m + 2 eller 4m + 3 hvor n ∈ Z. Dette gir at vi kan dele opp summen som følger n X

X

=

1≤4m≤n

k=1

X

+

1≤4m+1≤n kπ 2

 ˚ summere over f sin Ved a

   n X kπ −y = f sin 2

k=1

−y



X

X

+

1≤4m+2≤n

(5.7)

1≤4m+3≤n

f˚ ar en da X

f (− sin y) +

1≤4m≤n

f (cos y)

1≤4m+1≤n

X

+

X

+

X

f (sin y) +

1≤4m+2≤n

(5.8) f (− cos y)

1≤4m+3≤n

Integrerer vi likning (5.8) fra 0 til π/2 og bruker (5.6) f˚ ar vi n Z X

I=

k=1

π/2

0

0

0

Z

X

f (sin y) dy +

1≤4m+2≤n

f (cos y) dy 0

1≤4m+1≤n

π/2

Z

X

π/2

Z

X

f (sin y) dy +

1≤4m≤n

+

π/2

Z

X

=

   kπ f sin − y dy 2

π/2

f (cos y) dy

1≤4m+3≤n

0



 X

=

+

1≤4m≤n

X

+

1≤4m+1≤n

X 1≤4m+2≤n

+

Z

X 1≤4m+3≤n

π/2

f (sin y) dy

 0

Summene i parentesen er bare summen fra k = 1 til n se likning (5.7), slik at integralet reduseres til Z

π/2

   n Z 1 X π/2 kπ f sin − y dy n 2 k=1 0 Z Z π/2 n 1 X π/2 = sin y dy = sin y dy n 0 0

f (sin nx) dx = 0

k=1

R π/2 som var det som skulle vises. Beviset for 0 f (cos x) dx føres samme m˚ ate. Her vil en dele opp argumentet som    cos y ,        kπ kπ kπ sin y , cos − y = cos cos y + sin sin y = − cos y ,  2 2 2   − sin y ,

akkuratt p˚ a k k k k

= 4m = 4m + 1 = 4m + 2 = 4m + 3

og argumentere p˚ a samme m˚ ate som før. Dette fullfører beviset.

˚ jukse en smule og ikke eksplisitt regne ut I(1) her. Derimot 8. Vi velger a

248

30 Langsvar

henvises det til oppgave (1) fra seksjonen om substitusjoner. r Z π/2 dθ 2 2−1 2 π π arctan = √ = √ = √ 2 2 + sin θ 2+1 3 6 3 3 2 −1 0 Z π dθ 2π 2π = √ = √ 3 22 − 1 0 2 + sin θ

V

(5.9) (5.10)

Fra likning (2.42) og (2.43) med a = 2. Ved a˚ bruke substitusjonen nθ = y s˚ a er Z π/2 Z nπ/2 dy dθ 1 = . 2 + sin nθ n 2 + sin y 0 0 Herfra benyttes samme tankegang som i oppgave (7). Vi vet at n alltid er p˚ a formen 4k, 4k + 1, 4k + 2 eller 4k + 3 og drøfter hvert tilfellet. La n = 4k + r, hvor 0 ≤ r ≤ 3 Z π/2 Z 2πk+rπ/2 dθ 1 dy = 2 + sin(4k + r)θ 4k + r 0 2 + sin y 0 ! Z 2π Z rπ/2 1 dy dy = k + 4k + r 2 + sin y 2 + sin y 0 0 Hvor likning (2.19) fra theorem (2.4.1) ble brukt i andre overgang. Bruker vi n˚ a likning (5.10) f˚ as Z π/2 Z rπ/2 1 k 2π dy dθ = ·√ + . 2 + sin(4k + r)θ 4k + r 4k + r 0 2 + sin y 3 0 Siden vi har k like integral. La n˚ a først n = 4k + 0,r = 0 da forsvinner siste integral slik at Z π/2 dθ k 2π π = · √ +0= √ 2 + sin 4kθ 4k + 0 3 2 3 0 √ Som aldri er likt π/3 3 . Dersom n = 4k + 1, s˚ a er " # Z π/2 Z π/2 1 2kπ dy 3k + 1 π dθ √ + √ = = 2 + sin(4k + 1)θ 4k + 1 2 + sin y 2k +1 3 3 3 0 0 √ Igjen se (5.9). Som bare er likt π/3 3 hvis k = 0. Om n = 4k + 2, s˚ a er Z π/2 Z π dθ k 2π 1 dy = ·√ + 2 + sin(4k + 2)θ 4k + 2 4k + 2 2 + sin y 3 0 0 k 2π 2 π 3k + 1 π √ + √ = √ , = 4k + 2 3 4k + 2 3 3 2k + 1 3 3 √ R π/2 Som bare er likt π/3 3 dersom k = 0. Siste overgang følger fra med 0 f (sin x) dx = Rπ 1 2 0 f (sin x) dx. For eksempel (2.30) fra proposisjon (2.4.5) med n = 1, m = 0. Tilslutt om n = 4k + 3 s˚ a er Z π/2 dθ k 2π 1 4π 6k + 4 π √ + √ = √ , = 2 + sin(4k + 3)θ 4k + 3 4k + 3 4k + 3 3 3 3 3 3 0

V

30 Langsvar

249

Dette viser at integralene bare er like for n = 1 og n = 2 π/2

Z

dθ = 2 + sin θ

0

π/2

Z 0

dθ π = √ 2 + sin 2θ 3 3

Som fullfører første del av oppgaven. Regningen ovenfor viser og at Z

π/2

0

dθ π = √ 2 + sin 4kθ 2 3

Alts˚ a kun dersom n = 4k, k ∈ N.

9. Merk at Z b+nT

Z

b+(n−m)T

Z

f (x) dx =

b+(n−m)T

f (u + mT ) du =

a+mT

a

f (x) dx a

Der substitusjonen x = u + mT ble benyttet. Integralet kan n˚ a skrives som b+(n−m)T

Z

b

Z f (x) dx =

Z

b+(n−m)T

f (x) dx +

a

a b

Z

f (x) dx b

Z

(n−m)T

f (x) dx +

= a

f (x) dx 0

b

Z

Z

T

f (x) dx + (n − m)

= a

f (x) dx 0

og dette fullfører beviset. I andre overgang ble likning (2.17) brukt, og i siste (2.18).

˚ bruke likning (2.19) s˚ 10. Ved a a er Z

3π/2+19(2π)

I=

Z

3π/2

f (x) + (19 − 11)

f (x) dx = π+11(2π)

Z

π



f (x) dx 0

Det det siste integralet er null Z



0

dx dx = 0 , (1 + 2sin x ) (1 + 2cos x )

Dette har blitt vist i en tidligere oppgave, se oppgave (2). N˚ a gjennst˚ ar bare det første integralet som kan beregnes relativt smertefritt. Siden integralet over f (x) og f (a + b − x) er like store s˚ a er I= =

1 2 1 2

Z

3π/2

f (x) + f (5π/2 − x) dx π Z 3π/2

log(1 + sin x) cos x + log(1 + cos x) sin x dx π

250

V

30 Langsvar

˚ bruke u 7→ sin x og det andre leddet kan Det første integralet kan løses ved a ˚ bruke u 7→ cos x s˚ løses ved a a  Z −1 Z −1 Z 0 1 log(1 + u) = 1 log(1 + u) du = I= log(1 + u) du − 2 0 −1 0 oppsumert har en alts˚ a ’utrolig’ nok at Z

71π/2

(1 + sin x)1+cos x 1 + cos x

 log

23π

 dx = 1

som var det som skulle beregnes.

11. a er h(−x) − h(x), og dersom de er like s˚ a er a) Dersom funksjonene er odde s˚ h(−x) = h(x) direkte regning gir −x

Z

x

Z f (t) dt = −

G(−x) = 0

f (−u) du = −G(x) 0

siden f er en likefunksjon. Dermed s˚ a er G odde. Tilsvarende s˚ a er Z

−x

x

Z g(t) dt = −

F (−x) =

g(−u) du = F (x)

0

0

en likefunksjon, der det ble brukt at g er odde. ˚ bestemme G(2nL) og F (2nL) benyttes selvsagt (2.18). S˚ b) For a a Z

2L

Z f (t) dt og

G(2nL) = n

2L

F (2nL) = n

0

g(t) dt , 0

Videre benyttes akkuratt samme fremmgangsm˚ ate p˚ a begge funksjonene For enkelhetensskyld studeres G(2L) først. Z

L

G(2L) =

Z f (t) dt +

Z

f (t) dt L Z −L

0 L

f (t) dt −

=

2L

0

f (t + 2L) dt 0

= G(L) − G(−L) = 2G(L) Hvor det ble brukt at G er odde, f er periodisk og at Z

2L

Z

0

L

Z f (t + 2L) du = −

f (t) dt = −L

−L

f (t + 2L) du 0

V

30 Langsvar

251

ble brukt i første overgang med substitusjonen t 7→ x − 2L. P˚ a samme m˚ ate kan F (2L) beregnes L

Z F (2L) =

2L

Z g(t) dt +

g(t) dt L −L

0 L

Z

Z g(t) dt −

=

g(t + 2L) dt

0

0

= F (L) − F (−L) =0 siden F er like. Oppsumert s˚ a er alts˚ a G(2nL) = 2nG(L) F (2nL) = 0 Merk at her kunne en ogs˚ a benyttet lemma (2.3.1) direkte med likning (2.13). Dette gir L

Z

Z

L

f (x) + f (2L − x) dx =

G(2L) =

f (x) + f (x) dx = 2G(L)

0 L

Z

Z

0 L

g(x) + g(2L − x) dx =

F (2L) = 0

g(x) − g(x) dx = 0 0

c) For at en funksjon skal være periodisk kreves det at F (x + 2L) = F (x), ved ˚ sette inn verdier s˚ a a er Z x+2L Z x Z 2L F (x + 2L) = g(t) dt = g(t) dt + g(t) dt = F (x) + F (2L). 0

0

0

Som selvsagt er periodisk siden F (2L) = 0, og andre overgang kan vises ved a˚ benytte (2.19). For at G(x) skal være periodisk følger en samme fremmgangsm˚ ate x+2L

Z G(x + 2L) =

Z

x

f (t) dt = 0

Z

2L

f (t) dt + 0

f (t) dt = G(x) + 2G(L). 0

Igjen se (2.19). Alts˚ a trengs kravet om at G(L) = 0 for at G skal være periodisk.

2.5

˚ benytte seg av substitusjonen x → t−1 . 12. Det enkleste blir a Z

b

J= a

Z

dt p = t (t − a)(t − x)

1/b

=− 1/a

dx p

(1 − xa)(xb − 1)

Z

1/b

x −1 p dx −1 −1 (x − a)(b − x ) x2

1/a

Z

1/a

= 1/b

dx p

ab(x −

b−1 )(a−1

− x)

252

V

30 Langsvar

Her brukte vi at b−1 < a−1 , og snudde grensene. Herfra innfører vi α = 1/b og β = 1/a slik at Z

1 J= √ ab

β

α

π = √ ab

dx p

(x − α−1 )(β −1 − x)

hvor vi benyttet oss av eksempel (2.5.1) i siste overgang.

13. For a˚ forenkle regningen noe innføres g(x) = f (x + a2 /x). En ønsker da ˚ vise at a Z a Z a  dt 2 dt g t = . g (t) t t 1 1 ˚ benytte seg av den logiske substitusjonen y 7→ t2 p˚ Ved a a venstre side f˚ as 1 2

Z

a2

g(y) 1

dy . y

(5.11)

Hvor dy = 2t dt ⇒ dt/t = dy/2y. Der siste likhet f˚ as ved a˚ dele p˚ a t2 = y. Dette integralet kan n˚ a deles opp p˚ a en smart m˚ ate 1 2

a

Z 1

a2

Z

dy 1 g(y) + y 2

dy , y

g(y) a

(5.12)

og det siste integralet p˚ a høyre side kan n˚ a skrives om via y 7→ a2 /t. Z

a2

a

dy g(y) = y

Z

a

g(t) 1

dt . t

(5.13)

a − dy/y = dx/t. Likning (5.14) skrives da om til Hvor dy = −a2 /t2 dx s˚ 1 2

Z

a

g(y) 1

dy 1 + y 2

Z

a

dt t

g(t) 1

(5.14)

Fra dette følger det direkte at Z

a

g t2

1

som fullfører beviset.

 dt 1 = t 2

Z

a2

g(y) 1

dy = y

Z

a

g(y) 1

dy y

V

30 Langsvar

253

14. Vi deler opp integralet som følger  Z ∞  1 f x− dx I= x −∞ Z 0 Z ∞   −1 f x − x−1 dx f x−x dx + = −∞ 0   Z ∞   y Z ∞   y e − e−y e − e−y y = f 2 e dy + f 2 e−y dy 2 2 −∞ −∞ Z ∞ Z ∞ y = f (2 sinh y) e dy + f (2 sinh y) e−y dy −∞ −∞  y  Z ∞ e + e−y f (2 sinh y) 2 = dy 2 −∞ Z ∞ = f (2 sinh y) 2 cosh y dy −∞ Z ∞ = f (x) dy −∞

Den første substitusjonen ble x = ey benyttet, i det andre integralet ble x = e−y brukt. I aller siste overgang ble x = 2 sinh y slik at dx = 2 cosh y dy, og det fullfører beviset.

15. Løsning 1

Legg merke til at funksjonen er symmetrisk, slik at Z ∞ Z dx 1 ∞ dx =   1 2 1 2 2 2 2 0 −∞ a + x − a + x− x x

Ved n˚ a og bruke resultatet Z



 f

−∞

1 x− x



Z



f (x) dx

dx = −∞

som ble vist i en tidligere oppgave har en alts˚ a at Z Z ∞ 1 ∞ dx dx =   2 1 2 −∞ a2 + x − 1 2 0 a2 + x − x x Z 1 ∞ dx = 2 −∞ a2 + x2   x ∞ 1 = arctan a a 0 π = 2a

254

V

30 Langsvar

Løsning 2 ∞

Z I := 0

1 2

Z

1 = 2

Z

=



0

0

|



dx 2

a2

+ (x − 1/x)   1 + 1/x2 + 1 − 1/x2

dx 2 a2 + (x − 1/x) Z 1 + 1/x2 1 − 1/x2 1 ∞ dx dx + 2 2 0 a2 + (x − 1/x)2 a2 + (x − 1/x) | {z } {z } A

B

For det første integralet benyttes substitusjonen u = x−1/x s˚ a du = (1+1/x2 ) dx. Da er Z ∞ Z ∞ h1  u i∞ du 1 + 1/x2 π dx = A= =2 = arctan 2 2 2 2 + (x − 1/x) a + u u a a 0 a −∞ 0 Videre s˚ a er B = 0 da integranden er odde. Alternativt la x = −u i B s˚ a − du = dx, Z ∞ Z ∞ 1 − 1/(−u)2 1 − 1/(−u)2 (− du) = − B= 2 2 du = −B . a2 + (−u − 1/u) a2 + (−1)2 (u − 1/u) 0 0 Siden B = −B s˚ a er 2B = 0 og resultatet følger. Oppsumert har en n˚ a Z ∞ A+B π dx = I= = 2 + (x − 1/x)2 2 2a a 0 som er relativt pent svar. 16. 2.5.1 1. Ved a˚ bruke den kjente a˚ kjære substitusjonen t 7→ tan(x/2) og theorem (2.5.3) f˚ ar en Z Z Z dθ 1 2 dt 2a dt = = (5.15) 2t 2 a + b sin θ (at + b)2 + a2 − b2 a + b 1+t2 1 + t Siden tan(π/4) = 0, tan(0/2) = 1, f˚ ar en √ Z 1 Z π/2 Z dθ 2a dt a2 − b2 dy = = 2 2 2 2 a + b sin θ (a − b2 )(1 + y 2 ) 0 0 (at + b) + a − b √ Via y a2 − b2 7→ at + b. Det siste integralet kjenner vi igjen som arctan y, s˚ a   1 Z π/2 dθ 2 at + b = √ arctan √ (5.16) a + sin θ a2 − b2 a 2 − b2 0 0      2 a+b b = √ arctan √ − arctan √ a2 − b2 a2 − 1 a2 − 1

V

9.2 Oppgaver

255

x−y ˚ bruke arctan x − arctan y = arctan 1+xy ˚ forenkle Det neste steget er a . For a √ regningen a x − y/(1 + xy) med x = (a + 1)/ a2 − b2 og √ noe ser vi først p˚ 2 2 y = b/ a − b r  . a+1 a−b a a a−b a−b √ 1+ 2 · = = √ = √ 2 2 2 2 2 a −1 a a+b a −b a −1 a −b

˚ bruke Siden (a + b)/(a2 − b2 ) = 1/(a − 1) forenkles nevner til a/(a − b). Ved a x−y forrige likning og arctan x − arctan y = arctan 1+xy p˚ a likning (5.16) f˚ ar vi n˚ a Z 0

π/2

dθ = a + sin θ

Z 0

1

2a dt 2 = √ arctan 2 (at + b)2 + a2 − b2 a − b2

r

a+b a−b

x−y som var det som skulle vises. Overgangen arctan x − arctan y = arctan 1+xy q a+b er gyldig, siden xy = a−b > 1 n˚ ar |a| > |b|. Tilfellet hvor a = b m˚ a drøftes

for seg selv, og har blitt gjort tidligere. Bruker vi igjen likning (5.15) s˚ a kan det neste integralet skrives som √ Z ∞ Z ∞ Z 2π dθ 2a dt a2 − b2 dt = = 2 2 2 2 2 2 a + b sin θ 0 −∞ (at + b) + a − b −∞ (a − b )(1 + y ) √ Igjen via y a2 − 1 7→ at + 1. N˚ a forandres ikke grensene og vi f˚ ar Z 2π Z ∞ 2 2π dθ dy = √ = √ 2 2 2 2 a + b sin θ 1 + y a −b a − b2 0 −∞ ˚ vise. som var noe enklere a 2.6.2 1. Det logiske her blir a˚ benytte lemma (2.6.1) med n = 3 fra dette følger det at integralene er like, og at Z ∞ Z ∞ Z dx x 1 ∞ 1+x = dx = dx 1 + x3 1 + x3 2 0 1 + x3 0 0 En kan faktorisere teller som x3 + 1 = (x + 1)(x2 − x − 1), som medfører at Z Z Z ∞ 1 ∞ 1+x 1 ∞ dx 2 dx dx = = 2 0 1 + x3 2 0 x2 − x − 1 (2x − 1)2 + 3 0 √ ˚ n˚ Ved a a bruke substitusjonen 2x − 1 = 3 tan y kan integralet forenkles til Z Z π/2 1 ∞ 1+x 1 sec2 y dy 1 π π √ √ dx = = + 2 0 1 + x3 2 6 3 −π/6 1 + (tan y)2 3 Siden sec2 y = 1 + tan2 y som har blitt vist før. Oppsumert har en alts˚ a Z ∞ Z ∞ dx x 2π = dx = √ 3 3 1 + x 1 + x 3 3 0 0

256

V

30 Langsvar

˚ beregne som det som skulle vises. Denne m˚ aten er noe raskere enn a R ∞ var dx direkte. 1+x3 0

˚ dele integralet i to slik at 3. a) En fremmgangsm˚ ate er a Z



2 + x2 dx = 1 + x4

0



Z

1 + x2 dx + 1 + x4

0

Z



0



Z

dx 1+2 = 1 + x4 2

0

1 + x2 1 + x4

siden 1 + 1/2 = (1 + 2/2). Siste overgang følger direkte fra lemma (2.6.1) siden ∞

Z

1 2

0

1 + x4−2 dx = 1 + x4



Z 0

dx 1 + x4

Verdien av siste integralet følger fra likning (2.51) i eksempel (2.6.7). Vi f˚ ar da ∞

Z 0

2 + x2 3 dx = 4 1+x 2

Z



1 + x2 3π dx = √ 4 1+x 2 2

0

som var det som skulle bestemmes. En kunne og ha delt inn integralet i ∞

Z

2 + x2 dx = 1 + x4

0

Z



0

x2 dx + 1 + x4



Z 0

2 dx =3 1 + x4



Z 0

dx 1 + x4

For deretter a˚ ha brukt likning (2.52), uten at det ville ha gjort utregningen mer spennende eller kortere. b) Proposisjonen sier at Z



0

a + bxn−2 a+b dx = n 1+x 2



Z 0

1 + xn−2 dx , 1 + xn

(5.17)

˚ sette inn a = 0, b = 1 og b = 0, a = 1 f˚ ved a as henholdsvis Z 0



xn−2 dx = 1 + xn

Z 0



1 + xn−2 dx 1 + xn

Z



og 0

1 dx = 1 + xn

Z 0



1 + xn−2 dx. 1 + xn

˚ bevise det motDette viser at lemma (2.6.1) følger fra proposisjonen. For a satte tilfellet, alts˚ a at proposisjonen følger fra lemmaet tar vi utgangspunkt i venstresiden av likning (5.17). ∞

Z 0

a + bxn−2 dx = a 1 + xn

Z 0



xn−2 dx + b 1 + xn



Z 0

dx dx 1 + xn

˚ bruke likning (2.48) fra lemmaet f˚ ved a ar vi at Z 0



a + bxn−2 a dx = 1 + xn 2

som var det som skulle vises.

Z 0



1 + xn−2 b dx + 1 + xn 2

Z 0



1 + xn−2 dx 1 + xn

V

10.1 Oppgaver

257

2.7.1 1. Dette er et svært enkelt eksempel hvor delvis integrasjon kommer til nytte. Z I = ex sin x + ex cos x dx   Z Z x x = e sin x − e cos x dx + ex cos x dx = ex sin x + C Hvor den den delvise integrasjonen ble utført p˚ a ex sin x. Med v 0 = ex , v 0 = ex og u = sin x, u = cos x. Helt tilsvarende kunne en og brukt delvis integrasjon p˚ a ex cos x, men n˚ a m˚ atte en ha valgt u = ex , u0 = ex og v 0 = cos x, v = sin x. Z I = ex sin x + ex cos x dx   Z Z x x x = e sin x dx + e sin x − e sin x dx = ex sin x + C ˚ hamre inn poenget kunne en og ha brukt produktregelen baklengs Bare for a Z Z Z ex sin x + ex cos x dx = (ex )0 sin x + ex (sin x)0 dx = (ex sin x)0 dx Som gir akkuratt det samme som før. 2. Dette problemet er hakket mer utfordrende enn forrige problem. Rett frem 2 delvis integrasjon vil ikke fungere her. Vi kan ikke velge v 0 = e−x siden en da −x2 0 ˚ finne v. En mulighet er da a ˚ velge u = e ikke klarer a og v = x2 . Dette gir da  3 ∞ Z Z 2 4 −x2 x −x2 2 −x2 x e dx = − e x e dx 3 R R 3 −∞ Hvor forkortelsen (−∞, ∞) = R ble benyttet. For det første konvergerer ikke uttrykket i klammene, og for det andre er ikke det siste integralet noe enklere. 2 ˚ velge v 0 = xe−x , Det m˚ a alts˚ a finnes en annet valg for u og v. Trikset blir n˚ aa 2 v = −e−x /2, og u = x, u0 = 1. Dette gir det mye enklere resultatet √ Z  Z 0 h x i 1 π −x2 −x2 −x2 I= x −e /2 dx = − e + e dx = 2 2 R 2 R R ˚ vise grenseverdien Hvor det oppgitte integralet ble brukt i siste overgang. For a 2 kan en se at e−x g˚ ar mye raskere mot null enn x g˚ ar mot uendelig. Alternativt l’hˆ opital lim

x→±∞

x −x2 1 e = 2 2

lim

x→±∞

x 1 2 = x 4 e

lim

x→±∞

1 =0 xex2

˚ bruke mot en s˚ som blir litt meste laget a a søt og liten grenseverdi.

258

30 Langsvar

V

3. Anta at vi kan skrive c summen av to reelle tall a + b. Da kan en skrive integralet som Z Z Z √ √ √ (x + c) ex x dx = (x + a) ex x dx + b ex x dx (5.18) √ √ N˚ a kan delvis integrasjon p˚ a siste leddet med u = ex x , u0 = ex (1 + √ √ en bruke x)/2 x og v 0 = 1, v = x. Dermed kan en se at u0 v = xex (1 + x)/2 og Z Z √ √ √ b b ex x dx = bx xex − xex (1 + x) dx 2 ˚ sette dette inn i likning (5.18) f˚ Ved a ar en at Z Z √ √  √ b (x + c) ex x dx = bx xex + (x + a) − (1 + x) xex dx 2 √ For at integralet skal skulle skrives p˚ a formen k xex m˚ a det siste integralet være null. Problemet er at en vil ende i en evig runddans med delvis integrasjon som aldri vil forenkle integralet. Eneste mulighet for at siste integralet blir null, er dersom a = 1 og b = 2 − Siden a, b ikke kan avhengige av x. En har alts˚ a at Z √ √ (x + 3) ex x dx = 2 ex x dx med valgene c = 3 og k = 2. 5. Dette er et av de morsomste integralene jeg vet om, og deler av integralet ˚ først dele integralet i to er vist p˚ a forsiden av dokumentet. Det enkleste er a biter Z Z 1 1 1 dx + − dx I = log(log x) + log x log x (log x)2 For det første integralet kan en bruke delvis integrasjon p˚ a log(log x) med v 0 = 1, v = x og u = log(log x), u = 1/(x log x) s˚ a   Z Z Z dx x dx dx log(log x) + = x log(log x) − = x log(log x) + log x x log x log x Hvor konstanten ble unnlatt med vilje. Helt tilsvarende kan en bruke delvis integrasjon p˚ a det andre integralet n˚ a med u = 1/ log x, u0 = −1/[x log2 x] og 0 v = 1, v = x.  Z  Z Z 1 dx x x dx dx x − = + − = 2 2 2 log x (log x) log x log x x log x log x hvor igjen konstanten ble utelatt med vilje. Ved a˚ kombinere disse to resultatene s˚ a kan integralet skrives som Z 2 1 x log(log x) + − dx = x log(log x) + +C 2 log x (log x) log x Noe som er spesielt siden ingen av integralene individuelt har en elementær antiderivert.

V

30 Langsvar

259

2.7.2

1. Begynner som før med antakelsen om at svaret er et polynom av samme grad ganget med eksponensialfunksjonen. Alts˚ a at Z (x4 − 6x2 + 8x − 3)e−x dx = (ax4 + bx3 + cx2 + dx + e)ex + C ˚ derivere kan høyre side skrives som Ved a  4  ax + (4a + b)x3 + (3b + c)x2 + (2c + d)x + e + d ex ˚ sammelikne koeffisienter p˚ Ved a a samme m˚ ate som før f˚ as liknignssystemet. 1=a 0 = 4a + b −6 = 3b + c 8 = 2c + d −3 = e + d Nest øverste øverste likning gir b = −4a = −4, neste gir c = 12 − 6 = 6. S˚ a f˚ ar en fra neste at d = 8 − 2c = −4, og tilslutt s˚ a er e = 4 − 3 = 1. Oppsumert s˚ a har en alts˚ a at Z (x4 − 6x2 + 8x − 3)e−x dx = (x4 − 4x3 + 6x2 − 4x + 1)ex + C = (x − 1)4 ex + C Der siste likhet kan ses fra binomialformelen, og eventuelt Pascals trekant. Merk at selv større polynom løses relativt raskt. Delvis integrasjon hadde her blitt langt mer kronglete.

2. Begynner som før med antakelsen om at svaret er et polynom av

3. Begynner som før med antakelsen om at svaret er et polynom av

4. Begynner som før med antakelsen om at svaret er et polynom av

260

V

30 Langsvar

2.8 a formen 1. Det logiske her blir a˚ benytte proposisjon (2.8.1) siden uttrykket er p˚ xR(sin x, cos2 x). Dermed s˚ a er Z π Z π π I= x(cos x)2 sin x dx = (cos x)2 sin x dx . 2 0 0 ˚ bruke substitusjonen u 7→ cos x med du = − sin x. Videre Det neste steget blir a s˚ a blir grensene u = cos 0 = 1, u = cos π = −1. =

π 2

Z

−1

−u2 dx = π

1

Z 0

1

x2 dx = π



x3 3

1 , 0

hvor vi snudde grensene og brukte at x2 er symmetrisk omkring origo. Dermed Z π π x cos2 x sin x dx = , 3 0 som var det som skulle vises. 2. Legg først merke til at via y 7→ 1/x s˚ a kan en skrive Z ∞ Z 1  arctan x  arctan 1/y f xn + x−n dx = dy f y −n + y n x y 1 0 ˚ dele opp Hvor det ble ble brukt at dx = − dy/y 2 ⇒ dx/x = − dy/y. Ved a integralet ved x = 1 og benytte seg av forrige resultat kan integralet skrives som Z 1  arctan x + arctan 1/x f xn + x−n dx (5.19) x 0 ˚ benytte seg av ??. Merk at en kunne ha f˚ Beviset fullføres n˚ a ved a att lik˚ ha brukt proposisjon (2.5.2), med S = 1. ning (5.19) direkte ved a ˚ bestemme ater a dette integralet p˚ a. Vi begynner med 3. Det er mange m˚ √ substitusjonen x = sin θ, slik at 1 − x2 = | cos θ| og dx = cos θ dθ. √ Z 1 1 − x2 J= dx 2 2 0 1 − x (sin α) Z π/2 Z π/2 dt | cos θ| cos θ dθ = dθ = 2 2 2 (θ) − tan2 (θ) sin2 (α) 1 − sin (θ) sin (α) sec 0 0 I første overgang ble grensene θ = arcsin(0) = 0 og θ = arcsin(1) = π/2. Videre s˚ a er cos θ positiv p˚ a intervalet 0 ≤ θ ≤ π/2 s˚ a | cos θ| = cos θ. Siste overgang delte vi teller og nevner p˚ a cos2 θ og brukte at 1/ cos(θ) = ˚ sec(θ) og sin θ/ cos θ = tan θ. Videre kan vi bruke at sin2 (θ) = 1 − cos2 (θ), til a skrive om nevner. Legg merke til at 1 + tan2 x =

cos2 x sin2 x sin2 x + cos2 x 1 + = = = sec2 x 2 2 2 cos x cos x cos x cos2 x

V

30 Langsvar

261

Dette medfører at 1 = sec2 x − tan2 x, dermed s˚ a har vi Z π/2 Z π/2 dθ dθ  J= = 1 + tan2 (θ) cos2 (α) sec2 (θ) − tan2 (θ) 1 − cos2 (α) 0 0 Herfra benyttes substitusjonen u 7→ tan x og du = 1 + tan2 (x) dx = 1 + u2 dx.  Z ∞ Z ∞ du/ 1 + u2 1 1 cos2 (θ)  = − = du 1 − cos2 (α) 0 1 + u2 1 + u2 cos2 (θ) 1 + u2 cos2 (θ) 0 I siste overgang ble vanlig delbrøksoppspalting benyttet. Begge disse integralene er elementære og beregne. Merk at 1 − cos2 (α) = − sin2 (α) slik at   π   ∞ 1 1 π arctan u − cos a arctan cos(a) · u = = − cos(α) 2 2 2 sin (α) sin (α) 2 0 Dette medfører at vi kan skrive integralet v˚ art som √ Z 1 π 1 π 1 − x2 dx = = 2 (sin α)2 2 (α/2) 1 − x 2 1 + cos(α) 4 cos 0 som var det som skulle vises. I den første overgangen ble det kun brukt at (1−cos α)(1+cos α) = 1−cos2 α = sin2 α =⇒ (1−cos α)/ sin2 α = 1/(1+cos α). Tilsvarende i siste overgang ble dobbelidentiten cos(2x) = cos2 (x) − sin2 (x) = 2 cos2 (x) − 1 brukt. Som medfører at 1 + cos(2x) = 2 cos2 (x), dette er akkuratt ˚ se delbrøksoppspaltingen kan en for det samme som vi har med x = α/2. For a eksempel gjøre det som følger 1 1 1 − a2 + (a2 u2 − a2 u2 ) = (1 + u2 )(1 − a2 u2 ) 1 − a2 (1 + u2 )(1 + a2 u2 ) 1 (1 + u2 a2 ) − a2 (1 + u2 ) = 1 − a2 (1 + u2 )(1 + a2 u2 )   1 a2 1 = − 1 − a2 1 + u2 1 + a2 u2 hvor forkortelsen a = cos2 α ble innført. 4. Dette integralet benytter en rekke av resultatene vi allerede har sett p˚ a. ˚ dele opp integralet f˚ Ved a as Z Z 0 Z ∞ arctan x dx arctan x dx arctan x dx = + 2 2 2 x − 2x sin a + 1 R x − 2x sin a + 1 −∞ x − 2x sin a + 1 0 ˚ benytte substitusjonen x 7→ −y kan det integralet skrives som ved a Z ∞ Z ∞ arctan x dx arctan x dx − 2 − 2x sin a + 1 2 + 2x sin a + 1 x x 0 0 ˚ dele dem opp i (0, 1) ∩ (1, ∞) Begge disse integralene kan forenkles ved a Z 1 Z ∞ Z 1 arctan x dx arctan x dx arctan x + arctan(1/x) dx + = 2 ± 2x sin a + 1 2 ± 2x sin a + 1 x x x2 ± 2x sin a + 1 0 1 0

262

30 Langsvar

V

hvor substitusjonen ble brukt x 7→ 1/y i den siste overgangen. Herfra brukes (??) med arctan x + arctan 1/x = π/2 slik at Z Z arctan xdx 1 π 1 1 = − 2 dx 2 − 2x sin a + 1 2 − 2x sin a + 1 x 2 x x + 2x sin a + 1 R 0 Resten er n˚ a er elementær algebra. Merk at x2 ±2x sin a+1 = (x±sin a)2 +cos2 a dermed s˚ a er Z 1 Z 1 dx dx J= = 2 ± 2x sin a + 1 2 + cos2 a x (x ± sin a) 0 0 ˚ sette u cos a 7→ x ± sin a og holde fortegne bent i munnen forenkles Ved a integralet til      1 sin a sin a ± 1 1 π a −1 −1 J =± tan − tan = ± cos a cos a cos a cos a 4 2 Via enten sumformelen for arctangens eller diverse trigonometriske identiteter. Settes alt sammen f˚ ar en at   Z π 1 π a a arctan xdx 1 π π a = + − − = 2 − 2x sin a + 1 x 2 cos a 4 2 cos a 4 2 2 cos a R som var det som skulle beregnes. 2.10.2  1. log

2 π



 2. log

2 π



3. Legg merke til at xa − xb = (1 + xa ) − (1 + xb ). En kan dermed skrive (1 + xa ) − (1 + xb ) 1 1 xa − xb = = − . (1 + xa )(1 + xb ) (1 + xa )(1 + xb ) 1 + xb 1 + xa Integralet kan n˚ a skrives p˚ a følgende form Z ∞ Z ∞ dx 1 dx xa − xb = − . a b 2 b 2 a (1 + x )(1 + x ) 1 + x (1 + x )(1 + x ) (1 + x )(1 + x2 ) 0 0 R∞ π Fra eksempel (2.10.2) og spesielt likning (2.78) har vi 0 (1+xadx )(1+x2 ) = 4 for alle a. Alts˚ a er Z ∞ xa − xb dx π π = − = 0, a b 2 (1 + x )(1 + x ) 1 + x 4 4 0 som var det som skulle vises.   2 4. log π

V

14.1 Oppgaver

263

2.10.3

2.11.1

1. Begynn med substitusjonen u = 2x − 1 da er du = 2 dx og   1 1 1 − u2 2 x − x = (u + 1) (u + 1) − 1 = 2 2 4 Innsatt f˚ ar en da direkte at Z Z Z r 1 dx 4 √ √ dx = I= = du = arcsin u = arcsin(2x − 1) + C 2 2 1−u x−x 1 − u2 ˚ fullføre kvadratet i teller. Da Om denne løsningen virker noe rar, hjelper det a ser en at    2 1 1 1 1 − x2 − x + + = − x− 4 4 4 2 Og dette hinter kraftig til at substitusjonen x − 1/2 = u/2 ⇒ u = 2x − 1 vil virke. Det siste integralet p er standard, men alternativt kan substitusjonen u = sin y benyttes. Siden 1 − sin2 y = cos y og du = cos dy.

2. Legg merke til at (cos x + sin x)0 = cos x − sin x slik at     Z Z d (cos x + sin x)0 (cos x + sin x)0 (cos x + sin x)0 I= dx = log dx = log +C cos x + sin x dx cos x + sin x cos x + sin x hvor det ble benyttet at du

Z z 0}| { Z f (x) du dx = = log f (x) dx f (x) u |{z} u

holder for alle funksjoner.

4. En kan skrive integralet som følger Z √ J=

x+1 = x7/2

Z √

x + 1 dx = x1/2 x3

Z

1 x

r 1+

1 dx x x2

264

V

30 Langsvar

Ved hjelp av substitusjonen t2 = 1 + (1/x) s˚ a er 2t dt = − dx/x2 og 1/x = t2 − 1 innsatt f˚ as da Z  √ J =− t2 − 1 ( t2 )2t dt 2 2√ 2 2 √ t t − t4 t2 + C 3 5  2 2√ 2 = t t 3t2 − 5 + C 15  3/2     2 1 1 = 1+ 3 1+ −5 +C 15 x x

=

5. Ved faktorisering f˚ as Z I=

−1 =− 3 x + x7

Z

1 + x4 − x4 dx = x3 (1 + x4 )

Z

x dx − 1 + x4

Z

dx x3

Sistnevnte integral er rett frem, og førstnevnte kan løses via u = x2 s˚ a du = 2x dx Z  2 1 1 1 du + = + arctan x2 + C I= 2 2 2 2 1+u 2x 2x 2

6. Dette integralet har blitt løst i eksempel (2.10.3) s˚ a Z



π

sin(log x) dx = 0

ie 1h 1 x sin log x − x cos log x = eπ 2 2 0

Alternativt kan integralet skrives ved substitusjonen u = log x til Z π I= eu sin u du −∞

Hvor en har lagt merke til at eu = x og at du = dx/x ⇒ dx = eu du. Dette er et standard integral somkan beregnes p˚ a samme m˚ ate som før ved a˚ innføre hjelpe˚ skrive det om p˚ integralet eu cos u, via delvis integrasjon eller ved a a kompleks form. Legg merke til at Z  Z  Z u iu u u Im e e du = Im e cos u + i · e sin u du = eu sin u du Der Eulers identitet eiωx = cos ωx + i sin ωx

V

14.1 Oppgaver

265

˚ benytte seg av at ble benyttet. P˚ a den andre siden s˚ a er Dette oppn˚ as ved a ˚ gange med den konjugerte. i2 = −1 og a Z    1 i − 1 iu u Im eu eiu du = Im · e e i+1 i−1   1 (1 − i) (cos u + i sin u)eu = Im 2  h i i  1 1h = Im sin u + cos u eu + i · sin u − cos u eu 2 2 i 1h = sin u − cos u eu 2 ˚ sammenlikne de reelle delene i stedet ville en f˚ Ved a att integralet av eu cos u. En f˚ ar n˚ a at integralet kan skrives som Z π   h Z π i π 1 1 sin u − cos u eu = eπ eu sin u du = Im eu eiu du = 2 2 −∞ −∞ −∞ som vist før. ater a˚ angripe dette integralet p˚ a. Det letteste er nok a˚ bruke 9. Det er flere m˚ korollar (2.3.3) med f (x) = sin x, a = 0 og b = π/2. Da f˚ ar en direkte Z π/2 Z π/2 sin x sin x π/2 π dx = dx = = , cos x + sin x sin(π/2 − x) + sin x 2 4 0 0 ˚ bruke theorem (2.3.1) siden Som ønsket. Integralet kan og beregnes ved a g(x) + g(π/2 − x) er konstant. Helt elementært kan en og ta utgangspunkt i Z a Z a f (x) dx = f (a − x) dx . 0

0

Slik at integralet kan skrives som Z π/2 Z π/2 Z π/2 sin x dx sin (π/2 − x) dx cos x dx = = . cos x + sin x cos (π/2 − x) + sin (π/2 − x) sin x + cos x 0 0 0 Dobbler en integralet v˚ art f˚ ar en at Z π/2 Z π/2 sin x sin x 2I = dx + dx cos x + sin x cos x + sin x 0 0 Z π/2 Z π/2 sin x cos x = dx + dx cos x + sin x sin x + cos x 0 0 Z π/2 Z π/2 sin x + cos x π = dx = dx = cos x + sin x 2 0 0 Slik at en f˚ ar da direkte at Z π/2 Z π/2 sin x π cos x dx = = dx cos x + sin x 4 cos x + sin x 0 0 Som vist før. 14. ˚ angripe denne sauen i f˚ Det er flere m˚ ater a areklær.

266

30 Langsvar

V

Løsning 1 Vi prøver seg p˚ a den virkelige v˚ agale substitusjonen p √ 0 √ q 2+1 p x + x 1 x + x2 + 1 du √ = u = x + x2 + 1 , = p √ dx 2 x2 + 1 2 x + x2 + 1 Alts˚ a ser en at √ q Z p p x + x2 + 1 √ dx = 2 x + x2 + 1 + C x2 + 1 Alternativt kan en benytte seg av substitusjonen da har en videre at p √ 1 x + x2 + 1 1 u √ √ du = dx = dx 2 2 2 2 x +1 x +1 Alts˚ a bli integralet v˚ ar følgende p √ √ Z p Z x + x2 + 1 1 x + x2 + 1 √ √ dx =2 dx 2 x2 + 1 x2 + 1 q Z p =2 du = 2u = 2 x + x2 + 1 + C Moralen er ikke vær redd for a˚ benytte v˚ agale substitusjoner. For kompletthet s˚ a g˚ ar en derivasjonen av u nærmere i sømmene. Ved bruk av kjerneregelen s˚ a er 0 0 g(x)1/2 = 1/2g(x)1/2−1 · g 0 (x) = 12 √g (x) s˚ a g(x)

d dx

q p 1 x + x2 + 1 = 2 =

1 2

=

1 2

=

1 2

 p d  2x x + x2 + 1 1 1 + 2√x2 +1 dxp p = √ √ 2 x + x2 + 1 x + x2 + 1 √ p 2 √ x+ √ x +1 x + x2 + 1 x2 +1 p p · √ √ x + x2 + 1 x + x2 + 1 p √ √  x + x2 + 1 x + x2 + 1 √ √  x2 + 1 x + x2 + 1 p √ x + x2 + 1 √ x2 + 1

˚ vise. Som var det en ønsket a Løsning 2 Vi benytter seg av den litt mindre v˚ agale substitusjonen √ p du 2x x + x2 + 1 du dx =1+ √ = √ ⇒ = √ u = x + x2 + 1 , 2 2 dx u 2 x +1 x +1 x2 + 1 S˚ a integralet v˚ art kan bli omformet som følger √ Z p Z Z √ du x + x2 + 1 1 du √ √ I= dx = u =2 2 u 2 u x +1 q p √ = 2 u + C = 2 x + x2 + 1 + C

V

14.1 Oppgaver

267

Her ble det implisitt brukt at √ √ √ √ 0 1 1 u u u 1 √ = √ og √ = x = u u 2 x u u

15. Integralet kan bli løst p˚ a flere ulike metoder, den antakeligvis mest kreative metoden er vist under. Z Z dx dx  I= = 7 7 x −x x 1 − x16 1 du 6 6 u=1− 6 , = 7 ⇒ du = 7 dx dx x Zx Z x 1 1 1 1 1 6 I= du = log |u| + C · 7 dx = 6 x 6 u 6 1 − x16 1 1 I= log 1 − 6 + C 6 x

˚ først bestemme integralet av cos4 (x) for 16. Den slaviske metoden blir a ˚ bruke delvis integrasjon. deretter a

her vil vi heller benytte proposisjon (2.8.1) Z π Z π π xR(sin x, cos2 x) dx = R(sin x, cos2 x) dx . 2 0 0 Heldigvis kan integralet v˚ art skrives p˚ a denne formen siden cos4 (x) = (cos2 (x))2 . Dermed s˚ a er Z π Z Z π/2 π π I= x cos4 (x) dx = cos4 (x) dx = π cos4 (x) dx 2 0 0 0 Hvor en kan se siste overgang enten fra symmetri, eller ved a˚ dele integralet ved ˚ beregne det siste integralet p˚ π/2. Det finnes uttallige m˚ ater a a. Det raskeste er ˚ gjennskjenne det som et av Wallis’ integral nok a Z

π/2



π

4

cos (x) dx =

22n+1

0

2n n



˚ vise integralet p˚ som gir oss svaret direkte. Vi velger dog a a et mer elementært vis. Via symmetri eller substitusjonen x 7→ π/2 − x har vi Z 0

π/2

cos4 (x) dx =

Z 0

π/2

sin4 (x) dx

268

V

30 Langsvar

Dermed s˚ a kan integralet n˚ a skrives som Z Z Z π π π π π π cos4 (x) + sin4 (x) dx = 3 + cos(4x) dx = 3 dx I= 2 0 6 0 6 0 ˚ regne ut. Dette gir dermed Siste integralet er enkelt a Z π Z π π 3π 4 x cos (x) dx = 3 dx = 6 0 16 0 som var det som skulle vises. Vi g˚ ar s˚ a mellomregningene litt nærmere i sømmene. ˚ se den trigonometriske omskrivningen har vi For a 2  2 1 + cos 2x 1 − cos 2x + 2 2   1 1 1 1 + cos 4x 3 + cos 4x 1 + cos2 (2x) = + = = 2 2 2 2 2 4

cos4 (x) + sin4 (x) =



˚ se at det siste integralet er null kan enten sees ved direkte regning eller symA metri. Vi velger dog (som vanlig) en litt annen vri. La f (x) = cos(x) eller f (x) = sin x da er Z

2πn

f (x) dx = 0 ,

n∈N

0

˚ bruke substitusjonen x = 2n s˚ siden vi integrerer over hele perioder. Ved a a har vi Z π f (2nu) du = 0 0

˚ f˚ som ønsket. For a a integralet v˚ art setter vi f (x) = cos x og n = 2. ˚ benytte seg av delvis integrasjon med 17. Løsning: 1 Her velger en a 1 log x 1 , u0 = 2 , v=− x2 x x

u = log(x)2 , v 0 = Dette gir seg at Z 

log x x

2 dx = −

1 log(x)2 + 2 x

Z

log x dx x2

Igjen bruker en delvis integrasjon hvor u = log x , v 0 =

1 1 1 , u0 = , v=− x2 x x

S˚ a dette gir seg at det ubestemte integralet kan skrives som Z 

log x x

2 dx = −

log(x)2 1 − 2 log x − x x

Z −

1 dx x2

V

14.1 Oppgaver

269

Med innsatte grenser gir dette seg  2 ∞ Z ∞ log x log(x)2 log x 2 dx = − −2 − =2 x x x x 1 1 Den siste grensen kan for eksempel vises via rekkeutvikling, eller L‘hˆ optial. La f (x) = log(x)a /x hvor a > 1 da har en log(x)a−1 log(x)a−a a! log(x) x x L = lim = lim = · · · = lim =0 x→∞ x→∞ x→∞ x 1 1 Hvor en benyttet seg av L‘hˆ optial, a antall ganger da en har et ∞/∞ uttrykk hver gang. a

a

Løsning: 2 Uten videre lar en t = log x s˚ a x = et og dt = x1 dx innsetning gir da 2 Z ∞ Z ∞ Z ∞ log x (log x)2 dx e−t t2 dt = 2! dx = = x x x 1 0 0 Den siste likheten kommer fra gammafunksjonen, som er definert som følger. La n˚ a z være et positivt heltall, da er Z ∞ e−t tz dt = z! 0

Denne funksjonen er bevist i Del I, og gjør en sterk tilbakekomst i Del III. 18. Z

˚ uttrykke N˚ a trenger en a = = = = = Z

(x + 2)

2

(x + 7)

5

dx =

(x + 2)

2

Z 

x+2 x+7

2

1 dx 5 (x + 7)3 (x + 7) x+2 du 5 La n˚ au= , = x+7 dx (x + 7)2 2   Z  x+2 1 5 1 dx = 5 x+7 x + 7 (x + 7)2 I =

dx =

1 x+7

5 via u, Legg merke til at 1 − u = x+7 , slik at   Z 1 1−u u2 · du 5 5   1 3 1 1 u − u4 + C 25 3 4 1 1 3 u [4 − 3u] + C 25 12  3    1 x+2 x+2 4−3 +C 300 x + 7 x+7  3   1 x+2 22 + x +C 300 x + 7 x+7

1 (x + 2)3 (22 + x) +C 300 (x + 7)4

270

V

30 Langsvar

20. Legg merke til at π/2 √

Z

s

 2  √ Z π/2 1 cos 1 x dx dx = 2 x 2 2 0 0   π/2 i √ √ h π 1 − sin(0) = 2 = 2 2 sin x = 2 2 sin 2 4 0 π/2

Z

1 + cos x dx =

0



2 cos

Den vanskeligste overgangen er nok den første, den er vist i større detalj under. Vi har  cos(2x) = cos2 x − sin2 x = cos2 x − 1 − cos2 x = 2 cos(x)2 − 1 Legger en til 1 p˚ a begge sider, og setter x = t/2 f˚ ar en  cos t + 1 = 2 cos

1 t 2

2

˚ vise. Den neste tingen som kan virke uklart er hvorfor som var det en ønsket a en fjerner absoluttegnet og dette er enkelt og greit fordi cos(x/2) > 0 n˚ ar x ∈ 0 det kun brukt kjente ting som at (sin x) = [0, π/2]. Videre i utregningen blir √ cos x, sin(0) = 0, sin(π/4) = 1/ 2 osv. 23. Fra tidligere vet en at Z

1

Z f (x) dx =

−1

1

f (x) + f (−x) dx 0

Ved innsetning f˚ ar en da at Z

1

Z

1

f (x) dx = −1

0

Z

1

= 0

Z

1

= 0 1

(−x)2012 x2012 + dx x 1+e 1 + e−x 1006 (−x)2 x2012 ex + dx x −x 1+e 1+e ex x2012 ex · x2012 + dx x 1+e ex + 1

x2012 (1 + ex ) dx 1 + ex 0  1 Z 1 1 1 2012 2013 = x dx = ·x = 2013 2013 0 0 Z

=

24.

(5.20)

V

14.1 Oppgaver

271

Løsning 1 Vi legger merke til at integralet er odde omkring x = π/2, og derfor null. ˚ bruke substitusjonen u = π − x. Vi Løsning 2 En helt tisvarende metode er a har vist tidligere at Z a Z a f (x) dx = f (a − x) dx 0

0

Dette gir seg p˚ a v˚ art integral at Z π I= sin x sin 2x sin 3x dx Z0 π = sin(π − x) sin(2(π − x)) sin(3(π − x)) dx Z0 π = (− sin x)(− sin 2x)(− sin 3x) dx 0 Z π =− sin x sin 2x sin 3x dx 0

Dobbler en integralet v˚ art f˚ ar en at Z π Z 2I = sin x sin 2x sin 3x dx − 0

π

sin x sin 2x sin 3x dx

0

2I = 0 I=0 N˚ a var en svært nøye i utledningen v˚ ar her. Men det er nok a˚ si at dersom I = −I s˚ a er I = 0. 28. Sm˚ akjip oppgave, legg merke til at r r r 1+x 1 1+x 1 1 = · = 1+ x3 x2 x x x

(5.21)

Herfra benytter en seg av substitusjonen 1 1 , x= x u−1 1 1 1 ⇒ x du = − dx ⇒ du = − dx x u−1 x u=1+

du 1 =− 2 dx x

Integralet v˚ ar kan dermed skrives som Z r I=

x+1 dx = − x3

Z Z

=−

r

1 1+ x

! −

1 x

 dx

(5.22)



u du u−1

(5.23)

272

V

30 Langsvar

29. Legg merke til at en kan skrive om integranden som følger 

2

x (log x − 1) =  x4 − (log x)4

2

x log x 4 x log x

log x − 1 log2 x

· −1

Dette hinter kraftig til at substitusjonen γ = x/ log x vil virke. Da f˚ ar en fra kvotientregelen γ=

x log x − 1 dx ⇒ dγ = log x log2 x

Innsatt f˚ ar en da direkte at integralet kan skrives som Z 2 x (log x − 1) dx I= x4 − (log x)4  2 x Z log x log x − 1 = dx ·  4 log2 x x −1 log x

Z = Z = 1 4 1 = 4 =

γ2 dγ −1   1 1 1 1 1 − + 4 γ−1 γ+1 2 1 + γ2   γ−1 1 log + arctan(γ) + C γ+1 2     x − log x x 1 log + arctan +C x + log x 2 log x γ4

Og en er ferdige. Oppspaltingen av brøken kan eksempelvis gjennomføres slik x2 1 (x2 − 1) + (x2 + 1) 1 1 1 1 = = + −1 2 (x2 − 1)(x2 + 1) 2 x2 − 1 2 x2 + 1 1 (x + 1) − (x − 1) 1 1 1 1 1 1 1 1 + = − + = 4 (x − 1)(x + 1) 2 x2 + 1 4 x−1 4 x+1 2 x2 + 1

x4

32. Begynn med substitusjonen z = 2x − 1 s˚ a x = log(1 + z)/2. Integralet blir da Z ∞ Z ∞ x−1 1 log(z + 1) √ x dx = dz I= 2 1/2 (1 + z) log(z) x − 1) z 2 − 1 log(2 log (2) 0 0 Deler n˚ a integralet opp fra (0, 1) til (1, ∞). I det siste integralet blir variabelskifte z → 1/y benyttet s˚ a I=

1 log2 (2)

Z 0

1

log(1 + z) 1 dz + 1/2 z (1 + z) log(z) log2 (2)

Z 0

1

log(1 + 1/y) dy + 1/y)(− log(y)) y 2

y −1/2 (1

V

14.1 Oppgaver

273

Ved a˚ bytte tilbake fra dummy-variablenen y tilbake til z i det siste integralet og forenkle integranden kan uttrykket forenkles til Z 1 log(1 + z) log z − log(1 − z) 1 dz + dz 2 1/2 (1 + z) log(z) log (2) 0 z 1/2 (1 + z) log(z) 0 z Z Z 1 1 dr π dz 1 = = = z 1/2 (1 + z) log2 (2) log2 (2) 0 1 + r2 2 log2 (2)

1 = log2 (2)

Z

1

Hvor i nest siste linje ble substitusjonen r 7→ z 1/2 og z = r2 . 35. Hej Hej 37. Et forholdsvis tøft integral om en ikke ser crux’et. Legg merke til at 4x5 − 1 = x · (x5 + x + 1)0 − x0 · (x5 + x + 1) ved n˚ a og bruke kvotientregelen som en har sett nøyere p˚ a før  u 0 v

=

u0 v − uv 0 uv 0 − u0 v = − v2 v2

baklengs. Kan integralet skrives som Z Z x · (x5 + x + 1)0 − x0 · (x5 + x + 1) 4x5 − 1 dx = dx 5 2 (x + x + 1) (x5 + x + 1)2 0 Z  x =− dx x5 + x + 1 x =− 5 +C x +x+1 og en er ferdige. 38. Vi studerer først det ubestemte integralet og legger merke til at første del kan enkelt integreres. r Z Z r x x−1 x−1 − dx = x − log x − dx x+1 x+1 x+1 Hvor det ble brukt at x/(x + 1) = 1 − 1/(x + 1). Det siste integralet kan enten ˚ først sette u2 = (x − 1)/(x + 1) eller via delvis integrasjon løses ved a r Z r Z x−1 x−1 x+1 √ dx = (x + 1) − dx x+1 x+1 (x + 1) x2 − 1 p Hvor u = (x − 1)/(x + 1) og v = x + 1 ble √brukt. det siste integralet har blitt regnet ut før, og første del kan forenkles til x2 − 1 , oppsumert har en Z r   p p x−1 dx = x2 − 1 − log x + x2 − 1 x+1

274

V

30 Langsvar

Ved a˚ bruke dette resultatet og sette inn grenesene bir det opprinnelige integralet r Z ∞ h  i∞ p p x x−1 − dx = x − log(x + 1) − x2 − 1 + log x + x2 − 1 x+1 x+1 1 1 ˚ sette inn x = 1 gir ingen problemer ovenfor og blir 1 − log 2, mens tilfellet A hvor x → ∞ m˚ a behandles med større omhu. Heldigvis kan grensen deles opp og en kan vise følgende   p p lim x − x2 − 1 = 0 og lim − log(x + 1) + log x + x2 − 1 = log 2 x→∞

x→∞

˚ vise første grenseverdi ganger vi med den konjugerte For a √ p x + x2 − 1 1 2 √ √ lim (x − x − 1 ) = lim =0 2 x→∞ x→∞ x + x2 − 1 x+ x −1 ˚ vise siste hvor konjugatsetningen (a − b)(a + b) = a2 − b2 ble brukt. For a ˚ bruke l’hˆ grenseverdi kombineres logaritmene, for deretter a opital ! ! √ √ x + x2 − 1 x + x2 − 1 lim log = log lim x→∞ x→∞ x+1 x+1 ! √ 1 + x/ x2 − 1 = log lim = log 2 x→∞ 1 p p √ siden x/ x2 − 1 = x2 /(x2 − 1) = 1 + 1/(x2 − 1) ∼ 1 n˚ ar x → ∞. Oppsumert har en alts˚ a at r Z ∞ x x−1 − dx = log 2 − (1 − log 2) = 2 log 2 − 1 x + 1 x +1 1 som var det som skulle beregnes.

˚ dele opp integralet i to 40. Dette er et noe langt integral. Begynner med a deler Z 0 Z π x sin x arctan ex x sin x arctan ex I= dx + dx 1 + cos2 x 1 + cos2 x −π 0 | {z } | {z } A

B

˚ sette x 7→ −t i integralet A f˚ Ved a as Z

0

A= −π 0

Z =

π π

Z =

0

x sin x arctan ex dx 1 + cos2 x (−t) sin(−t) arctan e−t (− dt) 1 + cos2 (−t) t sin t arctan e−t dx 1 + cos2 t

V

14.1 Oppgaver

275

˚ legge sammen A og B og benytte at arctan x + arctan 1/x = π/2 kan det Ved a opprinnelige integraler skrives som Z π t sin t (arctan ex + arctan 1/ex ) I= dt 1 + cos2 t 0 Z π t sin t π = 2 0 1 + cos2 t Rb Rb Husk at a f (x) dx = a f (a + b − x) dx, eksempelvis via x 7→ a + b − u. Slik at Z Z π π t sin t π π (π − t) sin(π − t) I= = 2 0 1 + cos2 t 2 0 1 + cos2 (π − t) Z π Z π (π − t) sin t π π π sin t = = dt − I 2 0 1 + cos2 t 2 0 1 + cos2 t Dermed s˚ a er I=

π 4

Z 0

π

π sin t π2 = 2 1 + cos t 4

Z 0

1

 π 3 dy π3 = = 1 + y2 8 2

Som var det som skulle beregnes. 2.11.1 a at cot x er definert som cot x = 41. Før vi begynner med løsningen minner vi p˚ 1/ tan x. Som vil bli mye brukt fremmover, detteRfører direkte til at R a1 = cot x tan x. a ˚ bruke subtitusjonen x 7→ π/2 − u eller ( 0 f (x) dx = 12 0 f (a) + f (a − Ved a x) dx) f˚ as Z 1 π/2 1 1 1 1 + + + dx . I= 2 0 log tan x 1 − tan x log cot x 1 − cot x sin(π/2−x) x = cos Hvor det kun ble brukt at tan(π/2 − x) = cos(π/2−x) sin x = 1/ tan x = cot x. ˚ kombinere logaritmene Tanken er n˚ a at vi kan forenkle uttrykket ved a

1 1 log cot x + log tan x log(cot x tan x) + = = =0 log tan x log cot x (log tan x)(log cot x) (log tan x)(log cot x) siden cot x tan x = 1 og log 1 = 0. Tilsvarende for de to siste leddene f˚ ar vi 1 2 − cot x − tan x 1 + = =1 1 − tan x 1 − cot x (1 − tan x)(1 − cot x) Siden (1 − tan x)(1 − cot x) = 2 cot x tan x + cot x − tan x = 2 − cot x − tan x. Integralet forenkles dermed ned til Z π/2 1 1 I= + dx log tan x 1 − tan x 0 Z 1 1 1 1 1 π/2 + + + dx = 2 0 log tan x 1 − tan x log cot x 1 − cot x Z 1 π/2 π = 0 + 1 dx = 2 0 4

276

V

30 Langsvar

som var det som skulle vises. ˚ se at telleren danner et perfekt kvadrat 42. Integralet kan forenkles ved a 2

e2x − 2 + e−2x = (ex ) − 2ex e−x + e−x

2

= ex − e−x

2

Integralet kan da skrives som q Z log 2 (ex − e−x )2 Z log 2 x Z log 2 x |e − e−x | e − e−x dx = dx = dx x −x x −x e +e e +e ex + e−x 0 0 0 siden ex − e−x > 0 n˚ ar x > 0. Heretter kan substitusjonen u 7→ ex + e−x brukes. x −x Da er du = [e − e ] dx og grensene blir e0 + e−0 = 2 og elog 2 − e− log 2 = 5/2. Z

log 2

0

ex − e−x dx = ex + e−x

5/2

Z 2

du = log(5/2) − log(2) = log(5/4) u

som ønsket. 43. Integralet minner nesten om enhets identiteten sin2 x + cos2 x = 1. Dessverre har vi sin2 u + cos2 v, u = sin x og v = cos x. Vi har derimot Bruker vi x 7→ π/2 − y f˚ as Z

0

sin2(sin π/2 − y) + cos2(cos π/2 − y) dy

I=− π/2

Z =

π/2

sin2(cos y) + cos2(sin x) dy

0

˚ ta gjennomsnittet av Siden cos π/2 − y = − sin y og sin π/2 − y = cos y. Ved a dette integralet med v˚ art originalet f˚ as 1 I= 2

Z

1 2

Z

=

π/2

sin2(sin x) + cos2(sin x) + sin2(cos x) + cos2(cos x) dx

0 π/2

1 + 1 dx = 0

π 2

Siden argumentene er like følger resultatet fra enhetsformelen. 47. 0 48. Vi definerer de ubestemte integralene Z Z sin x dx ex − cos x J= og I = dx. x x e − sin x − cos x e − sin x − cos x Der valget av I er valgt slik at Z ex − sin x − cos x J −I = − x dx = −x e − sin x − cos x

V

14.1 Oppgaver

277

˚ beregne og heldigvis s˚ a er ogs˚ a J + I rimelig pen a Z (−ex + sin x + cos x)0 J +I = dx = log |−ex + sin x + cos x| −ex + sin x + cos x ˚ løse systemet s˚ Ved a a er alts˚ a Z x 1 sin x dx = − + log |−ex + sin x + cos x| x e − sin x − cos x 2 2 Setter en n˚ a inn grensene kan integralet skrives som 2π Z 2π e −1 sin x dx π = − π + log (eπ − 1) = − + log π x − sin x − cos x e 2 e + 1 2 π som var det som skulle bestemmes. Tilslutt noteres det at nevneren er positiv n˚ ar x > 0 slik at det ikke er noen problem med singulariter. I siste overgang ble det benyttet at log(x2 − 1) = log(x − 1) + log(x + 1). ˚ bruke substitusjonen x2 7→ t s˚ 52. Ved a a er 2x dx = dt slik at integralet forenkles til Z 1 log 3 sin t dt I= 2 log 2 sin t + sin(log 6 − t) ˚ bruke proposisjon (2.3.1), eller substitusjonen y 7→ Ved n˚ a og bruke enten a a + b − x har en at Z b Z b f (x) dx = f (a + b − x) a

a

˚ bruke dette p˚ Hvor a + b − x = log 6 − x. Ved a a v˚ art integral f˚ as Z log 3 1 sin(log 6 − t) dt I= 2 log 2 sin(log 6 − t) + sin t Gjennomsnittet av disse to integralene blir følgelig Z sin t sin(log 6 − t) 1 log 3 I= + dt 4 log 2 sin t + sin(log 6 − t) sin(log 6 − t) + sin t Z 1 log 3 sin t + sin(log 6 − t) = dx 4 log 2 sin t + sin(log 6 − t) p  1 1 log 3 − log 2 = log 3/2 = 4 2 Merk at etter substitusjonen x2 7→ t kunne theorem (2.3.1) blitt benyttet direkte siden f (x) − f (a + b − x) er konstant p˚ a intervalet. 53. La f (x) = x7 −3x5 +7x3 −x+1. Da er f (−x) = −(x7 −3x5 +7x3 −x+1)+2, som er det samme som at f (x) + f (−x) = 2. Interesant! For a˚ forenkle integralet benyttes substitusjonen n˚ a y 7→ −x, s˚ a dx = − dy, x = ±π/4 ⇒ y = ∓π/4. Z π/4 Z −π/4 Z π/4 f (x) f (−y) f (−x) dx = − dy = dx 2 2 2 cos (−y) π/4 −π/4 cos x −π/4 cos x

278

V

30 Langsvar

˚ ta gjennomsnittet av disse to uttrykkene for integralet f˚ Ved a as da I=

Z

1 2

π/4

−π/4

f (x) + f (−x) dx = cos2 x

Z

π/4

−π/4

h iπ/4 dx = 2 tan x =2 cos2 x 0

Som var den en ønsket a˚ bestemme. Hvor det ble brukt at f (x) + f (−x) = 2, og at (tan x)0 = 1/ cos2 x. Pent!

2.11.2

1. Ve

2. d) Vi ønsker a˚ vise at integralet over (−∞, ∞) er null. Ved a˚ bruke at integranden er symmetrisk, og substitusjonen x 7→ yt s˚ a er Z



−∞

x2 − y 2 dx = 2 (y 2 + x2 )2



Z

(yt)2 − y 2 y dt = 2y (y 2 + (yt)2 )2

0

Z



t2 − 1 dt (t2 + 1)2

1

1 − t2 dx (1 + t2 )2

0

Legg n˚ a merke til at integralet over (1, ∞) kan skrives som ∞

Z 0

t2 − 1 dt = ((1/x)2 + 1)2

Z

0

dx t2 − 1 =− ((1/x)2 + 1)2 −x2

1

Z 0

via substitusjonen t 7→ 1/x. Dette medfører at Z



Z

1

f (x, y) dx = 2 −∞

Z



f (x, y) dx + 2 0

Z

f (x) dx 1

1

Z f (x, y) dx − 2

=2 0

1

f (x, y) dx = 0 0

som var det som skulle vises. Her ble forkortelsene f (x, y) = (x2 −y 2 )/(x2 +y 2 )2 og f (x, 1) = f (x) innført. Z 2 1

e)



x2 − y 2 dx = 2 (y 2 + x2 )2

Z 1

0

(1/u)2 − y 2 du = −2 (y 2 + (1/u)2 )2 −u2

Z 0

1

u2 − y 2 du (y 2 + u2 )2

V

14.2 Oppgaver

279

˚ dele opp integralet og bruke inverf) Den første likheten kan etableres ved a sjonen u 7→ 1/x p˚ a siste integral. Vi har Z ∞ Z 0 Z 1 log x log 1/u du u2 dx = = − log u du 1 2 2 2 (1 + x2 )2 (1 + u2 )2 1 1 (1 + [ u ] ) −u 0 via u 7→ 1/x. Deler vi opp integralet og bruker likningen ovenfor f˚ as Z ∞ Z Z 1 ∞ 1 − x2 1 − x2 1 − x2 log x dx = log x dx + log x dx 2 2 (1 + x2 )2 (1 + x2 )2 0 0 (1 + x ) 1 Z 1 Z 1 x2 1 − x2 log x log x dx − dx = 2 2 (1 + x2 )2 0 0 (1 + x ) Z 1 1 − x2 = log x dx 2 2 0 (1 + x ) som var det som skulle vises. ˚ løse integralet som innvolverer logritmer har vi ikke s˚ a mange verktøy b) For a tilgjengelig. Senere vil vi f˚ a flere, men s˚ a langt har vi stort sett bare delvis integrasjon. Vi velger v0 =

1 − x2 (1 + x2 )2

og

u = log x

Husker vi tilbake har en allerede regnet ut v 0 , se lemma (2.7.1) med x = 1. v=

x 1 + x2

og u0 =

1 x

Alternativt s˚ a kan vi beregne integralet som følger Z Z (1/x + x)0 1 − x2 1 x dx = − dx = = 2 (1 + x2 )2 (1/x + x)2 1/x + x x +1 ˚ sette inn har vi n˚ Via u 7→ 1/x + x siden (−1/u2 )0 = 1/u. Ved a a  1 Z 1 Z 1 1 − x2 x x dx I= log x dx = log x − · 2 )2 2 2 (1 + x 1 + x 1 + x x 0 0 0 Første del beregnes til null siden x log x → 0 n˚ ar x → 0. Siste integral er elementert Z ∞ Z 1 Z 1 log x 1 − x2 dx π dx = log x dx = − = 2 )2 2 )2 2 (1 + x (1 + x 1 + x 4 0 0 0 som var det som skulle beregnes. 3. Vi beregner først det innerste integralet relativt enkelt og f˚ ar √ Z h ix+ x2 +a2 K= log t dx 1 Z   p = log x + x2 + a2 dx

280

30 Langsvar

V

˚ bruke delvis integrasjon hvor det ble brukt at log √ 1 = 0. En løsning videre er a med v = 1 og eu = x + x2 + a2 . Slik at   2 0 )   Z p 1 + 2√(x 2 2 √ x +a  dx = x log x + x2 + a2 − x ·  x + x2 + a 2   Z p x 2 2 √ = x log x + x + a − dx 2 x + a2  p  p = x log x + x2 + a2 − x2 + a2 + C som ønsket. Den siste algebra-transformasjonen kan en se eksempelvis slik √ (x2 )0 x2 + a2 x √ 1+ √ +√ 2 2 2 2 2 2√ x + a x + a√ x + a2 = x + x2 + a2 x + x2 + a2 √ 2 x + a2 + x 1 √ = √ · 2 2 x +a x + x2 + a2 1 = √ x2 + a2 og endelig s˚ a ble det siste integralet bergnet via substitusjonen u = x2 + a2 s˚ a du/2 = x dx Z Z p √ x dx 1 √ √ du = u + C = x2 + a2 + C = 2 u x2 + a2 ˚ vise siste likheten bestemmer en først et uttrykk for arcsinh(x). Vi har For a ex − e−x 2 x x 2 2ye = (e ) − 1 y=

0 = u2 − 2yu − 1 p 2y ± (2y)2 − 4(1)(−1) u= 2 p 2 u=y± y +1 Siden u = ex og sinh(x) > 0 ∀ x s˚ a er   p x = log y + y 2 + 1   p arcsinh(x) = log x + x2 + 1 Til slutt legger en merke til at arcsinh

x a

! x 2 + log a = log + 1 + log a a   1 1p 2 2 = log ·x+ x +a + log a a a   p = log x + x2 + a2 + log a−1 + log a x + a

r

V

14.2 Oppgaver

281

√ p p √ Hvor en benyttet seg av at x2 /a2 + 1 = x2 /a2 + a2 /a2 = x2 + a2 / a2 og at log a−1 +log a = − log a+log a = 0 eller log a−1 +log a = log a−1 a = log 1 = 0. Uansett, en n˚ a skrive integralet som K=

ix+√x2 +a2

Z h

log t dx 1  p  p = x log x + x2 + a2 − x2 + a2 + C i p h x + log a − x2 + a2 + C = x arcsinh  ax  p = x · arcsinh + x log a − x2 + a2 + C a

˚ vise. Alternativt kan en benytte omskrivningen før en som var det en ønsket a beregner integralet. Da f˚ ar en nesten tilsvarende K=

Z h

log t

ix+√x2 +a2

dx

1

Z =

  p log x + x2 + a2 dx

Z =

arcsinh

x a

+ log a dx

Første integralet beregnes ved delvis integrasjon hvor u = 1 og sinh v 0 = x/a, og siste integralet er trivielt.  = x log a + x · arcsinh

x a Z

= x · arcsinh + x log a − = x · arcsinh + x log a −

p



x a

Z −

x2

x· q

x a

0

2

 dx

+1

x dx + a2

x2 + a2 dx

som ønsket.

4. Velger a˚ først løse det ubestemte ved a˚ prøve den noe v˚ agale substitusjonen u=

p

r2 + R2 − 2Rr cos θ ⇒

du sin θ = √ dθ 2Rr 2 r2 + R2 − 2Rr cos θ

Dermed s˚ a kan integralet skrives om til Z I=



sin θ dθ = r2 + R2 − 2Rr cos θ

Z

du = Rr



r2 + R2 − 2Rr cos θ Rr

282

V

30 Langsvar

det bestemte integralet kan n˚ a skrives som "√ #π Z π sin θ dθ r2 + R2 − 2Rr cos θ √ = Rr r2 + R2 − 2Rr cos θ 0 0 p p 2 2 (R + r) (R − r) = − Rr Rr R − r R+r = − Rr Rr Hvor fortegnet p˚ a |R − r| avhengier av om R > r eller om R < r. Siden |x| = x n˚ ar x > 0 og |x| < −x n˚ ar x < 0. For R > r f˚ as R − r R+r R+r R−r 2 − = − = Rr Rr Rr Rr r Og for R < r f˚ as R − r R − r R+r 2 R+r − = + = Rr Rr Rr Rr R Integralet blir dermed  Z π sin θ dθ 2/r √ = 2 2 2/R r + R − 2Rr cos θ 0

n˚ ar R ≥ r n˚ ar R < r

5. Ve 6. g)

Uten videre s˚ a har en at Z

1

log 1 = 1

1 dt = 0 , t

˚ beregne. Første del avanalysens fundamentalteorem da det ikke er noe areal a sier at la f : [a, b] → R være en kontinuerlig funksjon. La F være funksjonen definert for x i [a, b] ved Z x F (x) = f (x) dt a

da er F 0 (x) = f (x) ˚ bruke dette s˚ ved a a f˚ ar en direkte at d d log x = dx dx

Z 1

1

1 1 dt = t x

V

14.2 Oppgaver

283

Rimelig standard. Neste integral er ikke like bent frem her har en r

Z

ar

1 dt = t

log a = 1

Z

a

1

1 r du = r u

a

Z 1

1 du = r log a u

Via substitusjonen t 7→ ur . Derivasjon gir dt = r · ur−1 du = r · ur u−1 du dt = r · t du/u dt/t = r du/u For grensene har en at t = ar √ ⇔ ur = ar ⇒ u = a og tilsvarende for nedre r r grense t = 1 ⇔ u = 1 ⇒ u = 1 = 1. For det siste integralet f˚ ar en Z log(ab) = 1

ab

dt = t

a

Z 1

dt + t

Z a

ab

dt = t

Z 1

a

dt + t

Z

b

1

du = log a + log b u

hvor t ∈ [1, ab] = t ∈ [1, a] ∪ [a, ab] og substitusjonen t 7→ au ble benyttet p˚ a siste ˚ dele p˚ integralet. Da er dt = a du og ved a a t f˚ as dt/t = a du/au = du/u. ˚ bruke h) Vi begynner med spesialtilfellet α = 1. En frekk løsning2 blir a l’hopital s˚ a lim

x→0

log(1 + x) 1/(1 + x) = lim =1 x→0 x 1

Alternativt kan vi løse oppgaven som følger. La f (x) = log(x), da er lim

x→0

log(1 + x) log(1 + x) − log(1) = lim x→0 x x = lim

x→0

f (1 + x) − f (1) x

= f 0 (1) = 1

Hvor kun definisjonen av den deriverte ble brukt. En siste metode før vi g˚ ar videre Z Z Z 1 Z 1 1 1+x dt 1 x dt log(1 + x) du = lim = lim = lim = lim du = 1 x→0 x 1 x→0 x 0 t + 1 x→0 0 1 + xu x→0 x t 0 ˚ se p˚ Via t 7→ xu. For α > 1 s˚ a deriveriverer vi begge sider for a a vekstraten d d log(1 + xα ) ≤ (αx) dx dx αxα−1 ≤α 1 + xα xα−1 ≤ xα + 1 xα−1 (1 − x) ≤ 1 2 Begge

˚ definere den metodene vist her bruker nesten sirkulær logikk. Da grensen ofte brukes til a deriverte av logaritmefunksjonen. Vi kan alts˚ a ikke bruke den deriverte til a˚ bestemme grensen. Dog g˚ ar det fint siden v˚ ar definisjonen av logaritmen var gitt som et integral, alts˚ a følger den deriverte direkte fra analysens fundamentalteorem. Dette er en av fordelene med integraldefinisjonen.

284

30 Langsvar

V

˚ ta stegene i Dersom α ≥ 1 og x > 1 s˚ a holder den siste ulikheten trivielt. Ved a motsatt rekkefølge har vi vist d d log(1 + xα ) ≤ (αx) dx dx hvorp˚ a integrasjon gir oss ulikheten v˚ ar. Konstanten bestemmes av spesialtilfellet α = 1. 7. I utgangspunktet et heselig integral, men forenkles raskt ned til noe mer h˚ andterlig. La f (x) betegne integranden da er   1 1 + (1/x)α 1 1 f (1/x) = 2 log 2 x x 2 log 1/x(1 + (1/x)2 )   α 1+x 1 = − log α 2x log x(1 + x2 ) Fra proposisjon (2.5.2) eller ved a˚ dele integralet ved x = 1 og benytte x 7→ 1/u R∞ R1 p˚ a siste integral har vi at 0 f (t) dt = 0 f (t) + f (1/t)/t2 dt. Dermed s˚ a er 1

     1 + xα 1 + xα dx I(α) = log − log α 2 2x log x(1 + x2 ) 0 Z 1 Z 1 α dx π dx = = α = [log xα ] 2 log x(1 + x2 ) 4 0 1+x 0 Z

Siden log

1+xα 2xα

= log

1+xα 2

− log xα . Mycket pent svar!

i) Legg merke til at       1 + xb 1 + xa 1 + xa − log log = log 1 + xb 2 2 Dette medfører at vi kan skrive   Z ∞ log 1+xab 1+x dx π J(a, b) = = I(a) − I(b) = (a − b) 2 1 + x log x 4 0 ˚ sette inn verdiene v˚ Ved a are i likningen over f˚ ar vi √ ! 1 + x4+ 15 √   q Z ∞ log √ √  √ √ 1 + x2+ 3 π  π dx = 2 + 15 − 3 = 2 + 6 3 − 5 . (1 + x2 ) log x 4 4 0 ˚ se den siste algebraovergangen er noe vanskelig, men kan for eksempel føres A r √ √ √ √ 2 15 − 3 = 15 − 3 q q q √ 3 √ √ √ 2 √ √ √ = 15 − 2 3 15 + 3 = 18 − 6 5 = 6 3 − 5 √ √ √ √ √ √ ˚ vise. Siden 3 15 = 3 3 5 = 3 5 . Som var det vi ønsket a

V

14.2 Oppgaver

285

j) Nei, integralet konvergerer ikke. Det bl˚ aser opp ved x = 1, faktoren 1/2 ˚ sikre (eller en annen vilk˚ arlig konstant større enn 1) er alts˚ a helt sentral for a seg konvergens.

˚ først finne cos 2θ. Legg merke til at 8. Begynner med a sin2 θ =

1 − cos 2θ 1 + cos 2θ og cos2 θ = 2 2

˚ dele disse p˚ Ved a a hverandre f˚ as sin2 θ 1 − cos 2θ 1 − tan2 θ = ⇒ cos 2θ = 2 cos θ 1 + cos 2θ 1 + tan2 θ Uttrykket for tan2 θ kan regnes ut eksakt som 2

Z

tan (θ) = a

b

2ab x − dx 3 x ab

!2

 =

ba x2 + x2 2ab

a = b

1 4



b2 a2 − 2 + a2 b2



Ved n˚ a og sette inn verdier kan cos 2θ skrives som 1 − tan2 θ 1 + tan2 θ 1 − (b/a − a/b)2 /4 = 1 + (b/a − a/b)2 /4 4 − (b/a − a/b)2 = (b/a + a/b)2  2 2  4b a (a2 − b2 )2 a2 b2 = − 2 2 2 2 2 b a a b (a + b2 )2 h  i 1 2 2 2 2 2 = 4a (25 − a ) − a − (25 − a ) 252 8 2 8 4 a + a − 1 = −8β 2 + 8β − 1 =− 625 25

cos 2θ =

hvor vi i siste overgang innførte β = a2 /5. Tar først noen hjelperesultater før en finner cos θ uttrykt via. En har tan2 (x) =

sin2 (x) 1 − cos2 (x) 1 = = −1 2 cos (x) cos2 (x) cos2 (x)

og 

b 2a

2

   2  a 2  b 2 b  a   a 2 a b (a2 + b2 )2 +1+ = +2 + = + = 2b 2a 2a 2b 2b 2b 2a 4a2 b2 | {z } 1

286

V

30 Langsvar

Dermed s˚ a kan en skrive om integralet som !2  2 2ab x 1 b a − = tan (θ) = dx − x3 ab 4 a b a  2  a 2 1 b 1 − −1= + 2 cos (θ) 2a 2 2b 2 2 2 1 (a + b ) = cos2 (θ) 4a2 b2 √ 2ab 2ab 2a 25 − a2 cos θ = 2 = = a + b2 25 25 2

Z

b

˚ bestemme. som var det en ønsket a 9. Før en gjør noen antakelser om t prøver en a˚ benytte seg av delvis integrasjon. Med u = ex og v 0 = sin x f˚ as Z t I= ex sin x dx 0 Z t 0 = [ex cos x]t + ex cos x dx 0 Z t t = 1 − e cos t + ex cos x dx 0

Herfra bør en legge merke til at

Rt 0

1/t dx = 1 s˚ a

= −et cos t +

Z 0

t

ex · cos x +

1 dx t

Til slutt s˚ a m˚ a et cos t = 0 for at uttrykkene skal være like. Dette skjer hvis og bare hvis t kan skrives som t = πn − π2 for alle n ∈ Z. Alts˚ a m˚ a t være periodisk.

˚ se første 10.R For a R a likhet kan substitusjonen x 7→ π − u benyttes, eller identia teten 0 f (x) dx = 0 f (a − x) dx. Begge metodene gir at Z π Z π Z π cos2 x dx cos2 (π − u) dx cos2 x dx = = 0 1 + cos x sin x 0 1 + cos(π − u) sin(π − u) 0 1 − cos x sin x Helt tilsvarende kan en gjøre med sinus integralene. Neste likhet kan være noe ˚ se, men det finnes flere fremmgangsm˚ vanskeligere a ater. For eksempel s˚ a kan vi vise at Z π Z π (cos x)2 (sin x)2 cos(2x) J= − dx = dx = 0 1 1 + cos x sin x 0 1 + cos x sin x 0 1 + 2 sin(2x) Via u 7→ 2x og du/2 = dx f˚ ar en Z 2π Z 2π cos u du cos u J == = du 1 2 + sin u 1 + 2 sin u 2 0 0

V

14.2 Oppgaver

287

Herfra kan en enten benytte seg av wiesrtrass substitusjonen t 7→ tan(u/2) (theorem (2.5.3)) eller kompleks analyse for a˚ beregne intgralet.3 Første metode gir da Z



J= −∞

2

1−t2 1+t2 2t + 1+t 2

2 dt = 1 + t2

Z



−∞ ∞

Z =

−∞

(t2

1 − t2 dt + 1 + t)(t2 + 1)

1 + 2t 2t − 2 dt = 0 2 1+t+t t +1

˚ bruke substitusjonen 2t 7→ 2u + 1 p˚ Siste overgang kan vi enkelt se ved a a siste integralet. Alternativt. Siden begge integrande har periode π holder det a˚ vise at identiten holder p˚ a et vilk˚ arlig intervall med lengde π. Via u 7→ x − π/2 kan integralet ˚ betrakte skiftes til [−π/2, π/2]. Siden funksjonene er symmetriske holder det a ˚ vise dette følger via x 7→ π/2 − u. identiteten over [0, π/2]. A ˚ beregne selve integralet er rett frem A I=

1 2

Z 0

π

(cos x)2 (sin x)2 + dx = 1 + cos x sin x 1 + cos x sin x

Z

π

1+

0

dx/2 1 2 sin(2x)

Benytter s˚ a u 7→ 2x og t 7→ tan(u/2) Z I= 0



dx = 2 + sin u

Z



−∞

1 2t dt = 2 2 + 2t/(1 + t ) 1 + t2

Z



−∞

dt 1 + t + t2

En kan fullføre kvadratet og f˚ a t2 + 1 + t = (t + 1/2)2 + 3/4. S˚ a hefra bruker en p 3/4 u 7→ t + 1/2 s˚ a π

Z

(cos x)2 = 1 + cos x sin x

0

Z



−∞ ∞

Z =

−∞

dt (t + 1/2)2 + 3/4 √ Z ∞ 2 du 2π 1 3 du = √ = √ 3 2 2 2 1 + u 3 −∞ 3 4 (u + 1)

og vi er ferdig. Siden verdien av det siste integralet er π.

11. 3 Ved

˚ bruke de komplekse formene (se avsnitt (3.9.1)) kan integralet skrives som a

Z 0



cos u i du = − 2 + sin u 4

I

z + z −1 1+

|z|=1

1 4i

dz = (z − z −1 ) z

I

dz z2 + 1 z z 2 + 4iz − 1

|z|=1



Polenene til integranden befinner seg ved z = 0 og z = −(2 − 3 )i, med henholdsvs residue −1 og 1. Det følger da fra residue theoremet (??) at integralet er null.

288

V

30 Langsvar

˚ benytte den oppgitte substitusjonen, da er Løsning 1 Velger a √ 1 + x4 2x(1 − x2 ) cos θ = ⇒ √ dx = sin θ dθ 2 1+x 1 + x4 (1 + x2 )2 ˚ bruke at sin2 θ + cos2 θ = 1 kan en skrive om sin θ som følger Ved a √ p 1 − x2 2x dθ 2 √ ⇒ sin θ = 1 − cos θ = dx = √ 2 2 4 1+x 2 (1 + x ) 1 + x ˚ bestemme grensene har en at For a √ θ = arccos

1 + x4 1 + x2

!

√ S˚ a θ = arccos(0) = 0 og θ = arccos(1/ 2 ) = π/4. Integralet forenkles dermed dramatisk til Z 1 Z π/4 dθ π 1 − x2 √ = √ √ dx = 2 4 2 4 2 0 0 (1 + x ) 1 + x og dette fullfører regningen. ˚ dele teller og nevner p˚ Løsning 2 Ved a a x2 kan integralet skrives om til Z 0

1

1 − x2 √ dx = (1 + x2 ) 1 + x4

Z 0

1

1 − 1/x2 q dx √ (x + 1/x) (x + 1/x)2 − ( 2 )2

˚ n˚ Ved a a sette y 7→ x + 1/x og dt(1 − 1/x2 ) dx blir integralet Z 0

1

1 − x2 √ dx = (1 + x2 ) 1 + x4

Z

0

1 1 q = √ √ 2 1 y y 2 − ( 2 )2 1 π π π = √ − = √ . 2 4 2 4 2

  0 x + 1/x √ arcsec 2 1

som er det samme som før. 12. Merk at det spiller ingen rolle hvordan en uttrykker vinkelen. Her velger ˚ benytte oss av arccos slik at vi a  x  ϑ(x) = arccos H Alts˚ a hosliggende over hypotenusen. For a˚ finne hyptenusen benyttes pytagoras, men først m˚ a høyden fra x til sirkelenp beregnes. Formelen for øvre halvdel med radius r og sentrum c er gitt som y = r2 − (x − c)2 . Slik at s 2  2 p a−b a+b y= − x− = −x2 + x(a + b) − ab , 2 2

V

14.2 Oppgaver

289

dermed kan hypotenusen uttrykkes som H=

p

x2 + y 2 =

Z

b

p x(a + b) − ab .

Integralet blir n˚ a følgelig I(a, b) :=

!

x

arccos

p

a

x(a + b) − ab

dx .

Men merk helt tilsvarende kunne en vist at s !   (x − a)(b − x) 1p ϑ = arcsin (x − a)(b − x) = arctan x(a + b) − ab x Men vi holder oss som sagt til arccos, samme fremmgangsm˚ ate funger uansett ˚ begynne a ˚ beregne dette ’beistet’ benyttes hvordan en definerer vinkelen. For a delvis integrasjon p med u = arccos(x/ (a + b)x − ab ) og v = x − ab/(a + b) som i hintet. Z

b

h ib Z ϑ(x) dx = v · ϑ(x) + a

a

a

b

(a + b)x − 2ab p dx 2(a + b) (b − x)(x − a)

Fra figur s˚ a er vinkelen null n˚ ar x = a og x = b. Dette medfører at første del ˚ kollapser til null, som en og kan se via innsetning siden arccos(1) = 0. For a beregne det gjennst˚ aende integralet brukes følgende substitusjon 2x − (a + b) b−a p √ S˚ a x = a → w = −1 og x = b → w = 1, og dx/ (b − x)(x − a) = dw/ 1 − w2 . Substitusjonen mapper [a, b] til [−1, 1] og er generelt en nyttig greie. Integralet blir dermed Z 1 Z 1 Z 1 dw w dw (b2 − a2 )w + (b − a)2 √ √ √ dw = A +B 2 2 4(a + b) 1 − w 1−w 1 − w2 −1 −1 −1 w=

2 2 Hvor A = (b − a)2 /4(a + b) og B = (b √ − a )/4(a + b) er konstanter. Legg merke til at siste integralet er null da w/ 1 − w2 er en odde funksjon. Det første integralet kan eksempelvis √ √løses ved a˚ sette w = sin x slik at dw = cos x dx. Med 1 − w2 = cos x. S˚ a dw/ 1 − w2 = dx.

Z

1

A −1



dw = 2A 1 − w2

π/2

Z

dx = Aπ . 0

Da integranden ar problemet med √ er symmetrisk omkring origo. Dette unng˚ fortegnet av 1 − w2 . Ved og sette inn f˚ ar en endelig at 1 b−a

Z

b

ϑ(x) dx = a

1 1 (b − a)2 b−a 4 a+b

Z

1

−1



dw π b−a = · 2 4 b+a 1−w

290

V

30 Langsvar

Som var det som skulle vises. Omformingen av integranden kan eksempelvis gjøres slik   (a + b)x − 2ab x ab 1 p p = − 2 a+b 2(a + b) (b − x)(x − a) (b − x)(x − a)   1 ab 1 p = (b + a − w(b − a)) − 4 a+b (1 − w2   (b + a)2 − 4ab − w(b2 − a2 ) 1 p = 4(a + b) (1 − w2 =

(b − a)2 − w(b2 − a2 ) p 4(a + b) (1 − w2

I første overgang ble det brukt at x = [b + a − w(b − a)]/2, i andre at at (b + a)(b − a er (b + a)2 − 4ab = b2 − 2ab + a2 = (b − a)2 . a) = b2 − a2 . I aller siste overgang s˚ La oss avslutningsvis se nærmere p˚ a den den delvise integrasjonen.   g 0 (x) x(a + b) − ab ϑ0 (x) · v = p . (5.24) a+b 1 − g(x)2 p √ Hvor g(x) = x/ (a + b)x − ab . Hvor det ble brukt at [arccos u]0 = u0 / 1 − u2 . Herfra har en " # p 1 x(a + b) 0 g (x) = (a + b)x − ab − p (a + b)x − ab 2 (a + b)x − ab =

x(a + b) − 2ab p 2(a + b)x − ab) (a + b)x − ab

(5.25)

Nevneren kan skrives om som følger 1 p 1 − g(x)2

 = 1−

x2 (a + b)x − ab

−1/2

p (a + b)x − ab = p (x − a)(b − x)

(5.26)

˚ sette inn likning (5.26) og (5.25) inn i (5.24) kan ϑ0 (x)v uttrykes som Ved a ! p !  (a + b)x − ab (a + b)x − ab x(a + b) − 2ab p  p a+b 2 (a + b)x − ab (a + b)x − ab (x − a)(b − x) Etter en serie dramatiske forkortningen forenkles utrrykket ned til ϑ0 (x) · v =

(a + b)x − 2ab p 2(a + b) (b − x)(x − a)

Dette fullfører sagaen om det delvis vanskelige integralet. 13. Viser at funksjonen er en konstant, slik at integralet blir trivielt. Velger a˚ bestemme den deriverte av y = arcsin x først y = arcsin x sin y = x

V

14.2 Oppgaver

291

Deriverer begge sider implisitt med tanke p˚ a y, og f˚ ar dx dy 1 1 dy = = dx cos y cos(arcsin y)

cos y = 1 ·

Herfra har en fra enhetssirkelen og pytagoras at cos2 x + sin2 x = 1, bruker  2  2 en x = arcsin y f˚ as cos arcsin y + sin arcsin y = 1 slik at cos(arcsin y) = p 1/ 1 − y 2 . Dermed s˚ a blir den deriverte y = arcsin x ⇒ y 0 = √

1 1 − x2

Tilsvarende kan det vises at y = arccos x ⇒ y 0 = − √

1 1 − x2

følgelig s˚ a er f 0 (x) = 0. Siden den deriverte er null, betyr dette at f (x) er en konstant. Eksempelvis s˚ a er f (0) = π/2. Dermed s˚ a er f (x) = π/2 for alle x. Videre s˚ a er −1 ≤ cos x ≤ 1 og tilsvarende for sin x, slik at definisjonsmenden til f er D = [−1, 1]. Integralet v˚ art er dermed høyde gange bredde eller J = h · b = (π/2) · 2 = π. Noe mer formelt kan det og føres som Z

1

J=

Z

1

π π π dx = + =π 2 2 2

f (x) dx = −1

−1

som ønsket. ˚ først se at p˚ a intervalet (0, π/2) er b˚ ade sin x og cos x 14. Det enkleste blir a ˚ dele opp integranden. strengt positive. Vi st˚ ar alts˚ a fritt til a π/2

Z

(1 + cos x) log (1 + sin x) − log(1 + cos x) dx ,

I= 0

˚ bruke substitusjonen x 7→ π2 − x eller symmetrien mellom sin x og cos x Ved a har en ogs˚ a at I kan skrives p˚ a formen π/2

Z

(1 + sin x) log(1 + cos x) − log(1 + sin x) dx ,

I= 0

˚ ta gjennomsnittet av disse to integralene f˚ Ved a ar en 1 I= 2

Z

π/2

sin x log(1 + cos x) + cos x log(1 + sin x) dx , 0

Dermed s˚ a har en Z π/2 Z I= log(1 + cos x) sin x dx = 0

0

1

h i2 log(x + 1) dx = (y log y) − y 1

292

V

30 Langsvar

I regningen ovenfor ble først symmetrien mellom sin x og cos x brukt se . Begge ˚ beregne ett av dem. Dette kan og integralene er like store, og vi trenger bare a ˚ dele integralet i to og bruke substitusjonen v 7→ 1 + cos x p˚ sees ved a a første integral og w 7→ 1 + sin x p˚ a andre. I andre overgang Rble substitusjonen u 7→ cos x brukt og i siste ble y 7→ x + 1 brukt. Sammen med log x dx = x log x − x + C. Alts˚ a har en π/2

Z

 log

0

(1 + sin x)1+cos x 1 + cos x



Z

1

log(x + 1) dx = −1 + 2 log 2

dx = 0

som var det som skulle bestemmes.

16. a) Legg merke til x6 + 1 og x2 + 1 deler samme nullpunkt, nemlig x = ±i. Polynomdivisjon gir da x4 − x2 + 1 2

x +1



6

x +1 − x6 − x4 − x4 x4 + x2 x2 + 1 − x2 − 1 0

Dermed s˚ a er x6 + 1 = (1 + x2 )(x4 − x2 + 1). Ved innsetning ser en da at Z 0

1

x4 + 1 dx = x6 + 1

1

Z 0

(x4 − x2 + 1) + x2 dx = (1 + x2 )(x4 − x2 + 1)

Z 0

1

dx + 1 + x2

Z

1

x2 +1

x6

0

Første integralet er trivielt, og siste kan løses ved a˚ sette u 7→ x3 ⇒ du = 3x2 du. Videre s˚ a blir x = 0 7→ u = 0 og x = 1 7→ u = 1 s˚ a grensene blir som før. Z 0

1

dx 1 + 1 + x2 3

1

Z 0

du 4 = u2 + 1 3

Z 0

1

dx = 1 + x2

Z

1

1 + x2 dx

0

 Z

1

0

dx 1 + x2



˚ se omskrivningen av 4/3 kan en eksempelvis tenke p˚ For a a følgende m˚ ate  1 Z 1 1 x3 4 =1+ = x+ = 1 + x2 dx 3 3 3 0 0 ˚ beregne selve integralet blir n˚ Der cruxet er a˚ se andre overgang. A a enkelt siden Z 0

1

x4 + 1 4 dx = x6 + 1 3

Z 0

1

i1 dx 4h 4 π π = arctan x = · = 1 + x2 3 3 4 3 0

V

14.2 Oppgaver

293

b) Integralene er like store som følger fra lemma (2.6.1) med n = 6. Alternativt ˚ vise via substitusjonen x 7→ 1/u. er det like enkelt a Z 0



x4 dx = − 1 + x6

Z

0



(1/u)4 du = 1 + (1/u)6 u2

Z



0

dx 1 + x6

som ønsket. Her en i siste overgang byttet tilbake til x som integrasjonsvariabel. Legger en sammen integralene eller bruker lemma (2.6.1) har en Z 0



x4 dx = 1 + x6



Z 0

Z

dx 1 = 1 + x6 2



0

1 + x4 dx = 1 + x6

Z 0

1

1 + x4 dx 1 + x6

I den siste overgangen ble likning (2.74) fra theorem (2.10.1) brukt. At integranden tilfredstiller likning (2.73) kan sees ved direkte utregning   1 + (1/x)4 1 1 + x4 1 1 = = = R(x) R 2 6 2 x x 1 + (1/x) x 1 + x6 eller ved a˚ bruke at 1 + x4 er symmetrisk med indeks 4 og 1 + x6 er symmetrisk ˚ bruke a) har vi da med indeks 6 (proposisjon (2.10.3)). Ved a Z   Z 1 Z ∞ Z ∞ 1 π x4 dx dx 2 dx = = = 1 + x dx 6 6 2 1 + x 1 + x 1 + x 3 0 0 0 0 som var det som skulle vises. 17. Dette integralet faller direkte ut fra Riemann-Lebesgue lemmaet (2.4.2). Lemmaet kan benyttes siden 1/(1 + cos2 x) er en periodisk, kontinuerlig funksjon og funksjonen sin x er og kontinuerlig, og deriverbar.  Z π  Z π  Z π sin t 1 dt lim dt = sin tdt x→∞ 0 1 + cos2 (tx) π 0 1 + cos2 t 0 Dette integralet kan for eksempel løses ved Weiestrass substitusjon, settes u 7→ tan(t/2) s˚ a er Z π Z ∞ Z ∞ dt dt 2 du 1 + u2 = = du  2 2 2 2 1+u 1 + u4 0 1 + cos t 0 0 1 + 1−u 1+u2 som kommer i fra theorem (2.5.3). Dette integralet ble studert i eksempel (2.6.7), s˚ a Z ∞ Z π 1 + u2 π dt = du = √ 2 4 1+u 2 0 0 1 + cos t Dette integralet kan løses noe mer intuitivt via substitusjonen u = tan t. Siden tan x g˚ ar mot uendelig n˚ ar x → π/2 benytter en at funksjonen er like omkring x = π/2, s˚ a Z 0

π

dt = 1 + cos2 t

Z 0

π/2

2 dt 1 + cos2 t

294

V

30 Langsvar

˚ bruke substitusjonen s˚ ved a a er cos2 t = 1/(1 + u2 ) og dt = du/(1 + u2 ) slik at Z ∞ Z ∞ √ Z ∞ dy du π 2 du √ = = √ = 2 2 2 2 2 1 + 1/(1 + u ) 1 + u 1+y 1 + (u/ 2 ) 2 0 0 0 √ √ I andre overgang ble substitusjonen y = u/ 2 brukt, og deretter at 2 /2 = √ 1/ 2 . Det neste integralet er mye enklere Z π h i0 sin t dt = cos t = cos 0 − cos π = 2 π

0

Slik at Z lim

x→∞

0

π

sin t 1 dt = 2 1 + cos (tx) π



π √ 2



√ 2= 2

18. Siden gjennomsnittsverdien av f og g er 1, la oss for enkelhetensskyld sette h ≡ g ≡ f et øyeblikk s˚ a Z a Z a 1 h(x) dx ⇒ h(x) dx = 8 1= 8−0 0 0 Dette vil komme til nytte snarlig, forenkling av integralet gir Z 4 Z 4 I= 8f (x) − 4g(x) dx + 4g(x) − 8f (x) dx −8

˚ bruke at Ved a

Rb a

8

f (x) dx = Z

Ra b

−f (x) dx bytter grensene i siste integral plass.

4

Z 8 · f (x) − 4g(x) dx +

=

8

8f (x) − 4g(x) dx ,

−8

4

Integralene kan n˚ a sl˚ as sammen. Siden g(x) er odde, er integralet over (−a, a) null Z 8 Z 8 Z 8 I= 8f (x) − 4g(x) dx = 8f (x) dx = 16 f (x) dx = 128 −8

−8

0

Slik at I = 27 . Dermed s˚ a er a = 7 som var det som skulle bestemmes. 19. Benytt substitusjonen f −1 (x) = y slik at f (y) = x. Da funksjonen er strengt voksende, kontinuerlig og deriverbar s˚ a er dx = f 0 (y). Dermed s˚ a er Z 10 Z 5 f −1 (x) dx = yf 0 (y) dt 0

1

Benyttes n˚ a delvis integrasjon med u = y og v = f (y) blir integralet h  Z 5 Z 5 5  0 yf (y) dt = yf (y) 1 − f (y) dy = 5f (5) − f (1) − 7 = 47 . 1

Tislutt s˚ a er f (5) = 10 og f (1) = 0.

1

V

14.2 Oppgaver

295

˚ kalle integralene for henholdsvis J og K. 20. La oss begynne med a π/2

Z

(sin x)a+1 dx og K =

J=

π/2

Z

0

(sin x)a−1 dx ,

0

√ Hvor a = 2 . M˚ alet blir n˚ a a˚ finne en sammenheng mellom integralene, noe som ˚ se p˚ kan gjøres ved eksempelvis delvis integrasjon. Ved a a J med u = (sin x)a og v = − cos x har en π/2

h iπ/2 Z J = − cos x sin(x)a + 0

a cos2 x sin(x)a−1 dx

0

π/2

Z

a 1 − sin2 x sin(x)a−1 dx 

= 0

Z

π/2

=a

sin(x)a−1 dx − a

0

Z

π/2

sin(x)a+1 dx

0

Hvor en kan gjennkjenne de to siste integralene som henholdsvis K og J. Lik˚ finne slik ningen J = aK − aJ kan løses for J/K , det er jo dette vi ønske a at J a 2a − a2 = = K 2+a 4 − a2 ˚ sette inn har en alts˚ Der teller og nevner ble ganget med 2 − a. Ved a a √ (sin x) 2 +1 0 √ R π/2 (sin x) 2 −1 0

R π/2

dx

=

dx

√ √ √ 2 2 − ( 2 )2 √ =2− 2 4 − ( 2 )2

og dette fullfører utregningene.

21. En kan først forenkle integralet noe via y 7→ x2 . Da er dy = x dx og Z a

b

Z

2x dx p

(x2



a2 )(b2



x2 )

b2

= a2

Z

dy p

(y −

a2 )(b2

− y)

v

= u

dy p

(y − u)(v − y)

˚ vise at Hvor i siste overgang variablene u = a2 og v = b2 ble innført. For a integralet er uavhengig av a og b, beregnes Z K= a

b

dx p

(x − a)(b − x)

eksplisitt. Her kan en for eksempel benytte substitusjonen x 7→ a cos2 φ + b sin2 φ ,

296

30 Langsvar

V

med dx = 2(b − a) cos φ sin φ. N˚ ar x = a blir φ = 0 og n˚ ar x = b blir φ = π/2. Innsetning gir da Z π/2 2(b − a) cos φ sin φ dφ q K=   0 a cos2 φ + b sin2 φ − a b − a cos2 φ − b sin2 φ Z π/2 2(b − a) cos φ sin φ dφ q =   0 b sin2 φ − a(1 − cos2 φ) b(1 − sin2 φ) − a cos2 φ Z π/2 2(b − a) cos φ sin φ dφ p = sin2 φ cos2 φ (b − a) (b − a) 0 Hvor det ble brukt at sin2 x + cos2 x = 1, to ganger. Legg merke til at b˚ ade sin φ og cos φ er positive p˚ a intervalet s˚ a fortegnet blr positivt n˚ ar vi tar roten. Tilsvarende er selvsagt (b − a)2 positivt siden b > a. Integralet forenkles da dramatisk til Z b Z π/2 Z π/2 dx 2(b − a) cos φ sin φ dφ p = = 2 dx = π (b − a) sin φ cos φ (x − a)(b − x) a 0 0 Oppsumert s˚ a er alts˚ a Z b Z b 2x dx dx p p = =π 2 2 2 2 (x − a )(b − x ) (x − a)(x − y) a a som var det som skulle vises. Merk integralet kunne og vært vist ved en litt ˚ gange ut teller f˚ mindre vill substitusjon. Ved a ar en Z b Z b dx dx p q K= = 2 2 − (x − (a + b)x + ab) a a (a − b)2 /4 − x − (b + a)/2 ˚ bruke substitusjonen u 7→ Hvor kvadratet ble fullført i andre overgang. Ved a x − (b + a)/2 n˚ a blir integralet Z b Z (b−a)/2 dx 2 du p p = (x − a)(b − x) (a − b)2 − 4u2 a −(b−a)/2 Utregningen fullføres ved en siste substitusjon u 7→ (a − b) sin(v)/2 som gir Z b Z −π/2 π π (a − b) cos v dv dx p p = = + =π 2 2 2 2 (x − a)(b − x) (b − a) cos v π/2 a som før. 22. Legg først merke til at integralet er positivt siden 1 + x2 > 0 n˚ ar x > 0. Videre s˚ a er teller positiv da x4 > 0. Tislutt er (1 − x)4 > 0 om enn synkende p˚ a x ∈ [0, 1]. Vi noterer først at x4 − 1 = (x2 − 1)(x2 + 1) og skriver ut teller x4 (1 − x)4 = x4 − 4x5 + 6x6 − 4x7 + x8 = (x6 − 4x5 + 5x4 ) + (5x6 − 4x7 + x8 ) − 4x4 + 4 − 4 = x4 (x2 − 4x + 5) + x4 · x2 (x2 − 4x + 5) − 4(x4 − 1) − 4 = x4 (x2 − 4x + 5)(1 + x2 ) − 4(x2 + 1)(x2 − 1) − 4

V

14.2 Oppgaver

297

˚ n˚ Ved a a dele begge sider av likningen p˚ a x2 + 1 f˚ as endelig x4 (1 − x)4 4 = x4 (x2 − 4x + 5) − 4(x2 − 1) − 2 x +1 1 + x2 Alternativt s˚ a gir tidenes lengste polynomdivisjon x6 − 4x5 + 5x4 − 4x2 + 4 x2 + 1



x8 − 4x7 + 6x6 − 4x5 + x4 − x8 − x6 − 4x7 + 5x6 − 4x5 4x7 + 4x5 5x6 − 5x6

+ x4 − 5x4 − 4x4 4x4 + 4x2 4x2 − 4x2 − 4 −4

Uansett s˚ a kan integralet n˚ a skrives som Z

1

x4 (1 − x)4 dx 1 + x2

1

x4 − 4x5 + 6x6 − 4x7 + x8 dx 1 + x2

I= 0

Z = 0

Z =

1

x6 − 4x5 + 5x4 − 4x2 + 4 −

0

4 dx 1 + x2

x7 2x6 4x3 − + x5 − + 4x − 4 arctan x 7 3 3 1 2 4 = − +1− +4−π 7 3 3 

1

=

0

Siden arctan(1) = π/4 og arctan(0) = 0 =

22 − π. 7

Siden integralet v˚ art er positivt følger det at 22 22 −π >0 ⇒ >π 7 7

I>0 ⇒

som ønsket. Siden 2 < 1 + x2 < 1 n˚ ar x ∈ (0, 1) har en at Z 0

1

x4 (1 − x)4 dx < 1

Z 0

1

x4 (1 − x)4 dx < 1 + x2

Z 0

1

x4 (1 − x)4 dx 2

298

V

30 Langsvar

˚ opphøye ulikheten i −1. Integralet kan beregnes forholdsvis enkelt Ved a Z

1

x4 (1 − x)4 dx

S= 0

Z =

1

x4 (1 − 4x + 6x2 − 4x3 + x4 ) dx

0

1 2 6 1 1 − + − + 5 3 7 2 9 1 = 630 =

Dermed s˚ a er 1 630

22 1 − 7 630

<

22 −π 7

<

<

π

<

1 1 2 630 22 1 − 7 1260

23. Siden φ er en løsning av 1 = x2 − x medfører dette at φ − 1 = 1/φ. Dermed s˚ a kan en skrive Z ∞ √ Z ∞ φ φ x arctan x x arctan x dx I= dx = 2 2 φ x (1 + x ) (1 + xφ ) 0 0 ˚ substituere u = 1/x f˚ Ved a as Z

0

(1/u)φ arctan 1/u (1 + (1/u)φ )



2

Z (−u) du = 0



xφ arctan 1/x dx 2 x (1 + xφ )

Ved a˚ ta gjennomsnittet av disse integralene og bruke at arctan x + arctan 1/x = π/2 forenkles integralet betraktelig. 1 I= 2

Z 0



π xφ (arctan x + arctan 1/x) dx = 2 φ x 4 (1 + x )

Z 0



xφ (1 +

2 xφ )

dx x

˚ benytte seg av substitusjonen y 7→ xφ som gir Neste steg blir a  ∞ Z ∞ dy π 1 π y π I= = − = 4φ 0 (1 + y)2 y 4φ 1+y 0 4φ ˚ g˚ For a a den siste substitusjonen litt nærmere i sømmene. Derivasjon gir dy = φxφ−1 dx ⇒

1 dx 1 dy dx dy = xφ ⇒ = φ x φ y x

Hvor det ble benyttet at xa+b = xa xb og i siste overgang at xφ = y. Dermed s˚ a er Z ∞ √ φ x arctan x π I= dx = 2 φ 4φ (1 + x ) 0

V

14.2 Oppgaver

299

˚ vise at 2 < 6I < 3 eller Som var det som skulle vises. N˚ a gjennst˚ ar det bare a 1 3

<

√ 2 (1 + 5 ) < 3

2 π √ 4 1+ 5

<

π

<

1 2 √ 1+ 5

√ ˚ vise ulikheten til venstre, alts˚ ˚ bruke at Velger a at 2(1 + 5 ) < 3π. Ved a √ først a 1 + 5 = 4 sin(3π/10) f˚ as direkte at. √ 2(1 + 5 ) = 8 sin(3π/10) < 8 sin(5π/10) = 8 ˚ se første ulikheten legg merke til Siden 8 < 3 · 3 < 3π stemmer ulikheten. For a at sin x stiger p˚ a x ∈ (0, π/2) s˚ a er sin(3π/10) < sin(5π/10). For a˚ vise den neste ulikheten krever litt mer spissfindighet. La oss først benytte oss av at 22/7 > π, som ble vist i forrige oppgave. Fra taylorutvikling s˚ a er √

1+x ≈1+

1 x2 x− + ··· 2 8

S˚ a lenge |x| < 1. Siden rekka konvergerer s˚ a er første ledd x/2 et overslag andre ledd x2 /8 et underslag for den egentlige verdien osv. Dermed s˚ a er r   √ 1 1 1 1 1 15 5 +1=1+2 1+ >1+2 1+ − =3+ 4 2 41 8 42 64 Da er en egentlig ferdig siden √ 15 22 5 +1>3+ > >π 64 7 ˚ førskt skrive om 22/7 til 3 + 1/7 Den midterste ulikheten kan en enkelt se ved a ˚ trekke fra 3 fra begge sider av ulikheten f˚ . Ved a as 15/64 > 1/7 ⇒ 105 > 64. Via kryssmultiplikasjon. Oppsumert har en alts˚ a vist at 1 π 2 1 √ < < , 3 4 1+ 5 2 som ønsket. 24. Vi noen raske omskrivninger kan integralet skrives p˚ a formen   2  Z  x −1 (1 − x2 ) ln(1 + x2 ) + (1 + x2 ) − (1 − x2 ) ln(1 − x2 ) I= xe x2 +1 dx 4 2 (1 − x )(1 + x )     1 − x2 2 2   Z (1 − x ) ln − (1 + x )   1 − x2 1 + x2   =−   x exp − 1 + x2 dx (1 − x2 )(1 + x2 )(1 + x2 )   1 − x2 2   Z (1 − x ) ln − (1 + x )   2x 1 1 − x2 2 dx 1 + x2   =− exp −   2 2 2 4 (1 − x ) 1+x 1+x 1 + x2 

=−

1 4

2



    Z   1 − x2 1 + x2 2x 1 − x2 2 dx ln − exp − (5.27) 1 + x2 1 − x2 1 + x2 1 + x2 1 + x2

300

V

30 Langsvar

Videre s˚ a benyttes weierstrass-substitusjonen fra theorem (2.5.3). x = tan

t 1 − x2 2 dx 2x , cos t = , og dt = . , sin t = 2 2 1+x 1 + x2 1 + x2

Dette gjør at likning (5.27) skrives om til  Z  1 1 I=− ln (cos t) − sin t e− cos t dt. 4 cos t Videre s˚ a kan vi la y = cos t ⇒ dy = − sin t dt slik at  Z  Z  Z −y 1 −y 1 e 1 ln y − e dy = e−y ln y dy − dy . I= 4 y 4 y

(5.28)

(5.29)

Dette integralet kan løses via delvis kanselering, avsnitt (2.7.1). Med u = e−y ⇒ du = −e−y dy og Z −y Z e dy = e−y ln y + e−y ln y dy (5.30) y Setter vi n˚ a likning (5.30) inn i (5.29) kan integralet skrives som Z   Z 1 1 I= e−y ln y dy − e−y ln y + e−y ln y dy = − e−y ln y + C. 4 4 Oppsumert vil integralet v˚ art dermed bli   2  Z t x −1 (1 − x2 ) ln(1 + x2 ) + (1 + x2 ) − (1 − x2 ) ln(1 − x2 ) x exp dx (1 − x4 )(1 + x2 ) x2 + 1 0   1 − t2 1 1 − t2 = − exp − ln 1 + t2 + C 4 1 + t2 ˚ vise. som var det vi ønsket a 25. Begynner med a˚ legge merke til at omr˚ adet en integrerer over kan skrives om som følger x2 + y 2 − 4x ≤ 0 ⇒ (x − 2)2 + y 2 ≤ 22

(5.31)

Alts˚ a beskriver D en disk med sentrum i (2, 0) og radius 2. Dette kan en og se ˚ tegne x2 + y 2 − 4x = 0. fra a

2

y

1 x 1 −1 −2

2

3

4

V

14.2 Oppgaver

301

Vi kan da beskrive omr˚ adet via dobbeltintegralet Z

4



Z

0

4x−x2

√ − 4x−x2

arctan exy dy dx

Vi integrerer alts˚ a først langs y-aksen fra bunnen av sirkelen til toppen. Deretter integrer vi langs x-aksen som g˚ ar fra 0 til 4. Integralet kan n˚ a deles opp i to deler Z 4Z 0 Z 4 Z √4x−x2 xy arctan e dy dx + arctan exy dy dx √ − 4x−x2

0

0

0

˚ bruke substitusjonen y 7→ −y i det første integralet f˚ Ved a ar en 4

Z



Z

4x−x2

arctan e 0

−xy

4

Z



Z

4x−x2

arctan exy dy dx

dy dx +

0

0

0

Dette forenkler integranden betraktelig da en kan sl˚ a sammen integralene og benytte ?? Z 0

4



Z

4x−x2

0

π arctan e−xy + arctan exy dy dx = 2

Z

4



Z

4x−x2

dy dx 0

0

Alts˚ a at arctan x + arctan 1/x = π/2. Integralet beskriver n˚ a bare arealet av en halvsirkel med radius 2 slik at arealet blir   ZZ π π · 22 arctan exy dy dx = = π2 2 2 S Som ønsket. Her ble det brukt at arealet av en halvsirkel er πr2 /2. Merk en kunne og ha beregnet integralet direkte, med noe mer grisete regning. ZZ

π arctan exy dy dx = 2 S

Z

4



Z

4x−x2

dy dx 0

0

Z π 4p 4 − (x − 2)2 dx = 2 0 Z π π/2 p = 2 4 − 4 sin2 u cos u du 2 −π/2 Z π/2 =π cos2 u du = π 2 −π/2

samme som før. I andre overgang ble substitusjonen x−2 7→ 2 sin u benyttet med √ dx = 2 cos u du. Videre s˚ a er cos2 x positiv p˚ a x ∈ (−π/2, π/2) slik at cos2 x = cos x. Integralet over cos2 x er som før lik halve omr˚ adet s˚ a 1 · (π/2 + π/2)/2. Alternativt Z π/2 Z π/2 Z π/2 π π 2 cos2 u du = cos2 u du + sin2 u du = + =π 2 2 −π/2 −π/2 −π/2 Siden omr˚ adet over cos2 x og sin2 x er like store. Dette st˚ ar det mer om p˚ a slutten av avsnitt (2.3).

302

V

30 Langsvar

28. For enklere notasjon byttes √ ϑ til θ, og vi kaller integralet for I. Vi har at 2 cos(π/3) = 1 og 2 sin(π/2) = 3 slik at  π √ −θ 3 cos θ − sin θ = 2 sin(π/2) cos(θ) − 2 cos(π/3) sin(θ) = 2 sin 3 (5.32) Hvor identiteten 2 sin(A + B) = cos A sin B + cos B + sin A ble brukt. Via flere identiter s˚ a har vi π  √  3 cos θ − sin θ sin θ = 2 sin − θ cos θ 3   π π π 1 − 2θ − cos = − 2 sin2 θ − = cos 3 3 2 6 ˚ bruke disse frekke omskrivingene f˚ Ved a ar vi Z π/3 √ 1/2 I= ( 3 cos θ − sin θ) sin θ cos θ dθ 0

Z

π/3 

= 0

1 = √ 2

Z

1 − 2 sin2 θ 2

1/2 cos θ dθ

π/6

1 − 4 sin2 θ

1/2

−π/6

 π cos θ + dθ 6

Hvor likning (5.32) ble brukt i første overgang og substitusjonen θ → θ + π/6 i siste overgang. Tanken er videre at vi kan bruke cos(A + B) = cos A cos B − sin A sin B p˚ a siste leddet. √  Z π/6 1/2 1 3 1 2 I= √ 1 − 4 sin (θ) cos(θ) − sin(θ) dθ 2 2 2 −π/6 r Z π/6 1/2 1 3 = 1 − 4 sin2 (θ) cos(θ) dθ (5.33) 2 2 −π/6 ˚ bruke 1 − sin2 θ = cos2 θ, da dette vil kanselere Tanken er n˚ a at vi ønsker a ˚ gjøre dette benyttes enkelt og greit sin x 7→ 21 sin u. Selve bort rottegnet. For a integralet blir da Z π/6 Z π/2 1/2 1/2 2 1 − 4 sin (θ) cos(θ) dθ = 1 − sin2 (θ) cos u du −π/6

−π/2

1 = 2

Z

π/2

1 cos (u) du = 4 −π/2 2

Z

π/2

du = −π/2

π 4

˚ sette inn π/4 i likning (5.33) f˚ Ved a ar en som ønsket r r Z π/3 √ 1/2 π 3 1 3 π · = ( 3 cos ϑ − sin ϑ) sin ϑ cos ϑ dϑ = 2 2 4 8 2 0 Dette fullfører beviset. 29. En frekk løsning her blir a˚ benytte to trigonometriske substitusjoner. som var det som skulle vises.

V

14.2 Oppgaver

303

˚ benytte to trigonometriske substitusjoner. 30. En frekk løsning her blir a Z a dx √ I= 2 2 0 x+ a −x ˚ la x = a cos u, og dx = −a sin u du. Grensene blir s˚ Ved a a x = a → u = 0 og x = 0 → u = π/2. Dermed s˚ a er 0

Z I=

π/2

1 −a sin u du √ = 2 2 2 a a cos u + a − a cos u

π/2

Z 0

a sin u du cos u + sin u

Hvor det blant annnet ble benyttet at a2 − a2 cos u = a2 (1 − cos2 u) = a2 sin2 u. Videre s˚ a ble grensene snudd og sin u er positiv fra 0 til π/2. Derimot om substitusjonen x = a sin u f˚ as Z

π/2

I= 0

a cos u du 1 p = 2 2 2 a a sin u + a − a sin u

Z

π/2

0

a cos u du cos u + sin u

Hvor dx = a cos u du. Grensene blir s˚ a x = a → u = π/2 og x = 0 → u = 0. Ved samme argument som før, velger den positive roten siden cos u er positv for alle ˚ ta gjennomsnittet av disse to integralene f˚ u ∈ (0, π/2). Ved a as 1 2a

Z 0

π/2

a cos u a sin u 1 + du = cos u + sin u cos u + sin u 2

Z

π/2

0

cos u + sin u π du = cos u + sin u 4

som var det som skulle vises. 31. a)

Vi har at for alle a, b s˚ a er Z b Z f (x) dx = a

b

f (a + b − x) dx

a

˚ bruke dette s˚ Ved a a er Z π/2 Z π/2 Z π/2 sin x dx sin(π/2 − x) dx cos x dx = = 3 3 3 3 3 sin x + cos x sin (π/2 − x) + cos (π/2 − x) cos x + sin3 x 0 0 0 Slik at K(1, 3) = K ∗ (1, 3). Integralene kan dermed skrives som K(1, 3) =

K(1, 3) + K ∗ (1, 3) 1 = 2 2

Z 0

π/2

sin x + cos x dx sin3 x + cos3 x

Merk at enten fra pascals trekant eller binomialformelen s˚ a er an + bn = an−1 − ban−2 + . . . − bn−2 a + bn−1 a+b Slik at sin3 x + cos3 x 1 = sin2 x − sin x cos x + cos2 x = 1 − sin 2x sinx + cos x 2

(5.34)

304

V

30 Langsvar

˚ sette dette inn i likning (5.34) f˚ Ved a as Z Z 1 π/2 dx 1 π dx K(3, 1) = = 2 0 2 0 2 − sin x 1 − 21 sin 2x Der substitusjonen u 7→ 2x ble brukt i siste overgang4 . Dette integralet kan løses for eksempel via Weiestrass-subtitusjonen t = tan(x/2) s˚ a Z π Z ∞ Z ∞ 1 dx dy 2 dy 1 = = 2 2 0 2 − sin x 2 0 y −y+1 (2y − 1)2 + 3 0 I andre overgang ble theorem (2.5.3) benyttet og litt opprydning i algebra. ˚ benytte seg av substitusjonen 2y − 1 = Tilslutt s˚ a fullføres kvadratet. Ved a √ 3 tan t blir s˚ a integralet Z π/2 √ 3 sec t dt π 1 π √ = + = 2 2 6 3 −π/6 3 tan t + 3 √ Her ble det blant annet brukt at sec x = 1 + tan2 x, arctan(1/ 3 ) = π/6 og arctan x → π/2 n˚ ar x → ∞. Z 0

π/2

sin x dx = sin3 x + cos3 x

π/2

Z 0

b) Vi har at for alle a, b s˚ a er Z b Z f (x) dx = a

cos x dx 2π = √ sin3 x + cos3 x 3 3

b

f (a + b − x) dx

a

slik at Z K(n, m) = 0

π/2

sinm (π/2 − x) dx = K ∗ (n, m) sinn (π/2 − x) + cosn (π/2 − x)

Dermed s˚ a er K(n, m) = K ∗ (n, m) og K(n, m) + K ∗ (n, m) 1 K(n, m) = = 2 2

Z

π/2

0

sinm x + cosm x dx sinn x + cosn x

for alle n, m. Dersom n = m har en at Z π 1 π/2 sinn x + cosn x dx = K ∗ (n, n) = K(n, n) = 2 0 sinn x + cosn x 4 c) Dette integralet kan først forenkles ved a˚ benytte seg av proposisjon (2.3.2) eller dette spørsm˚ alet. Siden K(2014, 2014) er en likefunksjon. Slik at La n˚ a for enkehletensskyld f (x) = 4 Det

sin2n x sin x + cos2n x 2n

er ikke noe problem a˚ benytte seg av Weiestrass-substitusjonen direkte, men regningen blir noe ˚ se hva jeg mener. mer grisete. Prøv selv for a

V

14.2 Oppgaver

da er Z

π/2

f (x) dx = 1 + αx

−π/2

Z

π/2

0

305

f (x) f (−x) αx dx + dx = x 1+α 1 + α−x αx

Z

π/2

f (x) dx 0

Siden f (x) = f (−x) da f er en likefunksjon. Dermed s˚ a er Z

π/2

−π/2

f (x) dx = 1 + αx

Z

π/2

f (x) dx = K(2n, 2n) = 0

π 4

Spesielt s˚ a er Z

π/2

−π/2

dx π (sin x)2014 = 2014 x + (cos x) 1+α 4

(sin x)2014

˚ numerere ekvivalensene som følger 32. Velger først a (1) Z

na

ma

log(x − a) dx x2 + a2

a/n

(2) m  log 2a2 arctan n − arctan m 2a

a/m

Z ⇔

⇔ (3)

log(x + a) dx x2 + a2

m (4) log 2a2 1 1 arctan − arctan 2a m n ,

˚ vise (1). Ved a ˚ skrive Da dette vil gjøre utregningen noe ryddigere. Begynner a om nm = n + m + 1 f˚ as n = (m + 1)/(m − 1). I første integral, benyttes substitusjonen u = x/a. S˚ a du = dx/a. Z n Z Z na log(ax − a) 1 n log a + log(u − 1) log(x − a) dx = a du = du I1 = 2 2 x2 + a2 a m 1 + u2 m (ax) + a ma For v˚ art andre integral benyttes substitusjonen u = a/x, s˚ a du = −a dx/x2 . 2 2 Dette kan skrives om til dx = −(x /a) du = −a du/u . Z

a/m

I2 = a/n

=

1 a2

Z

log(x + a) dx x2 + a2 a/m

a/n

log a + log(1 + x/a) 2

(x/a) + 12

dx =

1 a

Z

n

m

log a + log(1 + 1/u) du u2 + 1

˚ vise at I1 − I2 = 0, addisjon gir Vi ønsker n˚ aa Z 1 n log(u + 1) − log u − log(u − 1) I1 − I2 = du a m u2 + 1   u+1 m+1 Z m−1 log u−1 − log u 1 = du 2 a m u +1

306

V

30 Langsvar

Skriver om uttrykket via substitusjonen y = (u + 1)/(u − 1), videre s˚ a er dy = −2 du/(u − 1)2 = −(y − 1)2 dy/2. Mer regning gir da   y+1  Z − log y  log y−1 1 m dy −2 = −  2 m+1 a m−1 (y − 1)2 y+1 + 1 y−1   y+1 m+1 Z m−1 log y−1 − log y 1 =− dy 2 a m y +1 = −(I1 − I2 ) Og n˚ a siden I1 − I2 = −(I1 − I2 ) s˚ a er I1 − I2 = 0 som ønsket. ˚ vise ekvivalens (2) n˚ Velger a a, alts˚ a Z a/n log(2a2 )  1 1 log(x + a) = arctan + arctan I2 = 2 2 2 x +a a m n a/m ˚ faktorisere litt s˚ Ved a a kan en se at substitusjonen x=a

t+a t−a

og

dx =

−2a2 dt (a − t)2

˚ utføre litt mer mellomregninger s˚ Ved a a er  2  t + a2 2 2 2 x + a = 2a (a − t)2 Som gjør at uttrykket dx/(x2 + a2 ) = − dt/(t2 + a2 ). En annen ting som gjør at denne substitusjonen ble forsøkt, er at den er sin egen invers. Ved a˚ sette inn f˚ as  2  Z ma Z na Z ma log 2a /(t − a) log(x − a) dx log(2a2 ) I1 = = dt = dt − I1 2 2 2 2 x +a t +a t2 + a2 na ma na Legg merke til at vi ender opp med v˚ art opprinnelige integral p˚ a høyresiden og ˚ løse for I1 forenkles integralet til ved a Z Z log(2a2 ) na dt log(2a2 ) n du I1 = = 2 2 2 2 2a ma (t + a ) m u +1 Der substitusjonen t 7→ au ble brukt. Siste integralet er bare den antideriverte av arctan x, s˚ a en f˚ ar Z ma  log 2a2 log(x − a) dx = arctan n − arctan m 2 2 x +a 2a na ˚ gange begge sider med som var det som skulle vises. Vi viser s˚ a (3), ved a 2a/ log(2a2 ) f˚ as arctan n − arctan m = arctan

1 1 − arctan m n

som selvsagt holder da en kan skrive den om som arctan n + arctan

1 1 = arctan + arctan m n m

V

14.2 Oppgaver

307

og b˚ ade høyre og venstre side er n˚ a lik π/2 fra ??. Alternativt kan en og trekkke ˚ bruke sammen begge sidene ved a   x+y arctan x + arctan y = arctan 1 − xy som er sum formelen for tangens, og deretter bruke at nm = n + m + 1 men det ˚ vise (4) g˚ overlates til lese. For a ar en frem p˚ a samm m˚ ate som i (1) Z

a/m

I2 = a/n

log(x + a) dx x2 + a2

˚ løse dette integralet s˚ For a a kan følgende substitusjon benyttes x=a

a−t a+t

˚ dele integralet og f˚ Tanken er igjen a a −I2 p˚ a høyre side. Som før s˚ a er a2

dx dt =− 2 2 +t a + t2

og n˚ ar x = a/m s˚ a er t = a(−1 + m)/(1 + m) = a/n og tilsvarende n˚ ar x = a/n s˚ a er t = a(−1 + n)/(1 + n) = 1/m. Endelig s˚ a kan integralet skrives som Z

a/m

I2 = a/n

Z

log(x + a) dx x2 + a2

a/n

=− a/m

log[2a2 /(a + t)] dt = t2 + a2

Z

a/m

a/n

log(2a2 ) dt − I2 t2 + a 2

Dette er en likning som kan løses med hensyn p˚ a I2 slik at I2 =

log(2a2 ) 2

Z

a/m

a/n

dt log(2a2 ) = 2 2 t +a 2a

Z

1/m

1/n

du 1 + u2

Via den kjære substitusjonen t 7→ au. Integranden er den deriverte av tangens, slik at en f˚ ar Z a/m log(x + a) log(2a2 )  1 1 dx = arctan + arctan 2 2 x +a 2a m n a/n som var det som skulle vises. Dette fullfører den noe lange oppgaven. Merk at en ogs˚ a kunne ha brukt theorem (2.9.1) fra begynnelsen, men dette ville kanskje ikke gitt den samme insikten.

33. Noe problematisk oppgave om en ikke holder tungen bent i munnen. Ved ˚ gange med 4/4 og s˚ a a fullføre kvadratet f˚ as 1 4 4 = = x2 + bx + c (2x + b)2 + 4c − b2 (2x + b)2 + (π/10)2

308

30 Langsvar

V

˚ sette inn i Hvor det i siste overgang ble benyttet at 4c − b2 = π 2 /100. Ved a integralet kan en n˚ a benytte substitusjonen u 7→ 2x + b s˚ a du = 2 dx. Z a/2 dx I= 2 + bx + c x −b/2 Z a/2 2 dx =2 2 2 −b/2 (2x + b) + (π/10) Z π/10 du =2 2 u + (π/10)2 0   π/10 2 x = arctan π/10 π/10 0 20 [arctan(1) − arctan(0)] = π i 20 h π = −0 =5 π 4 Hvor det ble benyttet at Z

x 1 dx arctan = x2 + α 2 α α

Som er et kjent integral og kan eksempelvis vises via x = arctan u. 34. Oppgaven er egentlig algebra og elementær faktorisering i forkledning. P˚ a samme m˚ ate som en kan se at q q √ √ √ √ 3 − 2 2 = 12 − 2 2 + ( 2 )2 = 1 − 2 , kan en og forenkle røttene i integranden. Faktorisering av kvadratrøttene gir som følger √ √ √ 2 √ √ √ x + 2 2x − 4 = ( x − 2 )2 + 2 2x − 4 + ( 2 )2 = x−2 + 2 √ √ √ 2 √ √ √ x − 2 2x − 4 = ( x − 2 )2 − 2 2x − 4 + ( 2 )2 = x−2 − 2 . ˚ sette dette inn i det ubestemte integralet f˚ Ved a as da r Z q q √ √ x + 2 2x − 4 + x − 2 2x − 4 dx I= s r Z r √ 2 √ 2 √ √ = x−2 + 2 + x−2 − 2 dx Z r √  √ √ √ = x − 2 + 2 + x − 2 − 2 dx . √ √ Herfra ser en at x − 2 − 2 < 0 n˚ ar x ∈ [2, 4] og positiv n˚ ar x > 4. S˚ a √ √ √ √ 2 − x− n˚ ar x ∈ [2, 4] √2 x − 2 − 2 = √ x−2 − 2 n˚ ar x > 4

V

14.2 Oppgaver

309

Siden |x| = x n˚ ar x > 0 og |x| = −x n˚ ar x < 0 naturligvis. Ser først p˚ a tilfellet n˚ ar x ∈ [2, 4] da er Z r √

Z q√  √  √ √ x−2 + 2 + 2 − x − 2 dx = 2 2 dx = 23/4 x + C1

N˚ ar x > 4 s˚ a kan integralet skrivet som r Z  Z √ √  √ √  √ 4√ x−2 + 2 + x − 2 − 2 dx = 2 (x − 2)1/4 dx = 2 (x − 2)5/4 + C2 5 For at funksjonen skal være kontinerlig s˚ a m˚ a konstantene være like i punktet x = 4 s˚ a √  √ 8 3/4 8 2 √ 4 8 2 + C1 = 2 + C2 ⇒ C2 = 2 − 5 + C1 5 5 ˚ legge sammen alt dette f˚ Ved a as endelig r Z q q √ √ x + 2 2x − 4 + x − 2 2x − 4 dx I= √  4√2 (x√− 2)5/4 ar x ≥ 4  /5 + C1 n˚ = 23/4 x + 8 2 4 2 − 5 /5 + C1 n˚ ar x ∈ (2, 4) ˚ beregne. Det bestemte integralet blir s˚ som var det en ønsket a a Z 4 rq Z 4 q √ √ x + 2 2x − 4 + x − 2 2x − 4 dx = 23/4 x dx = 27/4 2

2

Som er et søtt lite svar p˚ a et stort stygt beist.

35. k)

 √ √ √ Legg merke til at vi har x2 − x = x ( x )3 − 1 . Ved a˚ bruke at a3 − 1 =

(a − 1)(a2 + a + 1) s˚ a kan vi gjøre følgende omskrivning √

√ .  √ √ 3 x +1 1 x +1 √ √ √ √ √ = · x ( x ) − 1 x x +x+ x x2 − x x x +x+ x √  √ √ √ x +1 √ · ( x − 1) x x + x + 1 = √ x x +x+ x  √ √ √ √ Ved a˚ bruke at x x + x + 1 = x x + x + x forenkles n˚ a integralet ned til Z



Z .  √  √ x +1 1 √ √ √ dx = x +1 x − 1 dx x x +x+ x x2 − x Z 1 x(x − 2) = x − 1 dx = x2 − x = +C 2 2

310

V

30 Langsvar

l) Helt tilsvarende som i forrige deloppgave f˚ ar vi Z a

b

√ Z b .  √ √ x 1 √ √ √ x x − 1 dx dx = x x +x+ x x2 − x a   b Z b √ 2√ 1 = x − x dx = x x− x 2 3 a a

Vi ser at uttrykket i parentesen er null n˚ ar x = 0, en mulighet er alts˚ a at a = 0 eller b = 0. Den andre muligheten for at uttrykket i klammene er null, da er 2√ 4 16 1 1 2 x− x − x=0 ⇒ x− =0 x =0 ⇒ 2 3 4 9 9 s˚ a x = 16/9. Alts˚ a er Z

16/9

0

√ . x 1 √ √ √ dx = 0 2 x x +x+ x x − x

og a = 0, b = 16/9 siden a > b.

36. Dette er en øvelse i delvis integrasjon. Telleren hinter om at delvis ˚ velge integrasjon vil hjelpe med a v=

1 . x − arctan x

(5.35)

For at dette skal g˚ a m˚ a det finnes to funksjoner u og dv slik at  u dv =

arctan x x − arctan x

2 (5.36)

˚ finne. Ved a ˚ derivere likning (5.35) f˚ Disse er heldigvis ikke vanskelige a as en mulig dv, og ved a˚ dele denne funksjonen p˚ a begge sider av likning (5.36) f˚ as u dv = −

x2 1 1 + x2 og u = − (arctan x)2 . 1 + x2 (x − arctan x)2 x2

Overraskende nok s˚ a blir ogs˚ a u0 relativt pen u0 =

2 arctan x )(arctan x − x) . x3

˚ bruke den kjære formelen for delvis integrasjon Ved a Z Z u dv = uv − u0 v , kan integralet I skrives som Z 

arctan x x − arctan x

2 dx = −

(1 + x2 )(arctan x)2 + x2 (x − arctan x)

Z

2 arctan x dx . (5.37) x3

V

14.2 Oppgaver

311

˚ sette u = arctan x og dv = 2x−3 s˚ Det siste integralet kan igjen løses ved a a Z Z arctan x 2 arctan x dx dx = − + 3 2 2 x x x (1 + x2 ) Z Z arctan x dx dx =− + − x2 x2 1 + x2 arctan x 1 =− − − arctan x + C . 2 x x Delbrøksoppspaltingen kan for eksempel bli sett ved a˚ legge til og trekke fra x2 i teller. For a˚ f˚ a det endelige svaret er resten bare faktorisering. La oss først skrive ˚ gange med x − arctan x i teller og nevner om det siste integralet, ved a   Z arctan x 1 2 arctan x x − arctan x dx = − − − arctan x x3 x2 x x − arctan x .  arctan x2 2 + (arctan x) − 1 − x arctan x (x − arctan x) = x2 (1 + x2 )(arctan x)2 1 + x arctan x = − . 2 x (x − arctan x) x − arctan x ˚ sette dette uttrykket Hvor konstanten ble droppet av praktiske grunner. Ved a for integralet inn i likning (5.37) f˚ ar en endelig at 2 Z  arctan x 1 + x arctan x +C. (5.38) dx = x − arctan x arctan x − x ˚ vise. Ved a ˚ snu likning (2.81) p˚ ˚ som er nesten det vi ønsket a a hodet, alts˚ aa opphøye begge sider i −1 s˚ a er sumformlene for tangens er gitt som 1 1 + tan a tan b = . tan(a − b) tan a − tan b For at høyresiden i denne likningen skal være lik høyresiden av likning (5.38) s˚ a m˚ a vi ha at tan a = x og at tan b = arctan x. Dette gir som ønsket 2 Z  1 arctan x dx = , x − arctan x tan(β − tan β) hvor x = tan tan β eller β = arctan(arctan x). 37. Teknikken en bruker her blir selvsagt delvis integrasjon. Vi velger her u=x u0 = 1 Slik at en f˚ ar Z

1

 x

0

v 0 = f 00 (2x) 1 v = f 0 (2x) 2

0 hx i1 Z 1 1 1 0 0 f (2x) dx = f (2x) − f 0 (2x) dx 2 2 0 0 2  1 1 1 = f 0 (2) − f (2x) 2 4 0  1 0 1 = f (2) − f (2) − f (0) = 2 2 4

312

V

30 Langsvar

I siste linje ble det brukt at f (0) = 1 , f (2) = 3 og f 0 (2) = 5. 39. At f /g har et kritisk punkt betyr at den deriverte er null, alts˚ a   d f f 0 g − g0 f = 0, = dx g g2 for x = 5 og x = 7. Siden g 2 > 0 medfører dette at f 0 g − g0 f = 0 ,

for x = 5, 7 .

(5.39)

Dette blir lagt litt i bakhodet, mens en tar fatt p˚ a selve integralet. For enkelhetens skyld blir det videre skrevet f (x) = f og g 00 (x) = g. Først benyttes delvis integrasjon to ganger som gir Z

7

h i7 Z f g 00 dx = f g 0 − 5

5

7

h i7 Z f 0 g 0 dx = − f 0 g − g 0 f + 5

5

7

f 00 g dx

5

˚ benytte (iii) og (iv) s˚ En kan skrive om f 00 g ved a a (iv)

z }| { z(iii) }| {   2 f 00 g = 2g 00 − f · g 00 · 1/g 00 = 2 − f g 00 

Innsetning gir n˚ a at Z

7

h i7 Z f g 00 dx = − f 0 g − g 0 f + 5

5

Z

7

f g 00 dx = 0 +

Z

5

2x − f g 00 dx

5 7 00

Z

f g dx = 2

2 5

Z

f 00 g dx

7

Z

5

7

7

x dx 5

7

f g 00 dx = 2

5

I første overgang ble likning (5.39) benyttet og at f 00 g = 2 − f g 00 . 40. √ Legg merke til at arccos(x) bare er definert for x ∈ [−1, 1] og siden en ˚ har x , s˚ a m˚ a 0 ≤ a ≤ b ≤ 1. Da integralet v˚ art er positivt, og en ønsker a maksimere I, m˚ a en la integralet v˚ art g˚ a over det maksimale intervallet. Alts˚ a er √ a = 0 og b = 1. Vi har at sin(u)2 + cos(u)2 = 1, la n˚ a u = arccos( x ) da har en  √ 2  √ 2 sin arccos x + cos arccos x =1  √ 2 sin arccos x =1−x Alts˚ a er Z I= 0

1

 √ 2 sin arccos x dx =

Z 0

1

 1 1 1 1 − x = x − x2 = 2 2 0

V

14.2 Oppgaver

313

˚ se at 41. Det er ikke videre vanskelig a  2 g(x)2 = sin(log x) + cos(log x) = 1 + sin(2 log x) Hvor en i siste uttrykk benyttet seg av sin2 x+cos2 x = 1 og sin(2x) = 2 cos x sin x. ˚ vise at Vi ønsker alts˚ aa Z 1 Z 1 cos2 (log x) dx = 1 + sin(2 log x) dx 0

0

˚ stadfeste denne likheten benytter en seg av delvis integrasjon med u = For a cos2 (log x) og v 0 = 1 1

Z

f (x)2 dx =

0

1

Z

cos2 (log x) dx

0

h i1 Z = x cos(log x) − 0

Z

1



0

−1 sin(2 log x) dx x

1

sin(2 log x) dx

=1+ 0 1

Z

1 + sin(2 log x) dx

= 0 1

Z

g(x)2 dx

= 0

˚ vise. Legg for ordens skyld merke til at det ikke var Som var det en ønsket a ˚ vise likheten. For a ˚ fylle p˚ nødvendig og beregne integralene for a a litt mer detaljer: I den delvise integrasjonen ble det benyttet at v 0 = 1, v = x og u0 =

2 d  2 2 1 cos (log x) = cos(log x)[− sin(log x)] = − log(2 sin x) dx x x

og grenseverdien kan eksempelvis vises slik h i1 L = lim x cos(log x) = 1 − lim n cos(log n) n→0

n

n→0

˚ vise den siste grensen p˚ Siden 1 cos(log 1) = cos 0 = 1. Det er flere m˚ ater a a. En ˚ legge merke til at limx→0 xf (x) = 0, dersom det eksisterer en K metode er a ˚ tenke p˚ slik at |f (x)| ≤ K. En m˚ ate a a dette som er at om f ikke vokser mot uendelig, s˚ a vil x før eller siden dra grensen mot null. I dette tilfellet er K = 1, siden cos x oscillerer mellom −1 og 1 for alle x. Alternativt s˚ a kan en definere L = x sin(log x) og R = x cos(log x). Da er L2 ≥ 0 og R2 ≥ 0 for alle x. Videre s˚ a er L2 + R2 = x2 , slik at n˚ ar x g˚ ar mot null s˚ a m˚ a L og R og g˚ a mot null. ˚ bruke substitisjonen x 7→ 1 − y, alts˚ a at R a 42. Det første R a steget vi gjør blir a f (x) dx = f (a − x) 0 0 Z

1

 (1 − y) log

J= 0

  √ √  Z 1 2+ 1−y 2+ x √ √ x log dx = dx 2− x 2− 1−y 0

314

V

30 Langsvar

Vi kan s˚ a dele J inn i følgende to integral 1

Z J=

√  x log 2 + x dx −

0

Z

1

√  x log 2 − x dx

0



√ Vi kan bruke substitusjonen y 7→ 2 + x p˚ a første integralet og y 7→ 2 + x p˚ a andre integralet slik at Z

3 3

Z

2(y − 2) log y dy −

J= 2

1

2(y − 2)3 log y dy

2

dette medfører som ønsket at   √ Z 1 Z 3 2+ 1−y √ (1 − y) dy = 2 (y − 2)3 log y dy 2 − 1 − y 0 1 ˚ beregne integralet n˚ A a er ikke spesielt vanskelig. 44. Løsning 1 Det fine med dette integralet er at det kan løses ved hjelp av en ˚ først bruke serie mer eller mindre normale substitusjoner. Vi velger her a u = x2 ⇒ du = 2x dx Z ∞ x29 I= 17 dx (5x2 + 49) 0 14 Z x2 1 ∞ = 2x dx 2 0 (5x2 + 49)17 Z 1 ∞ u14 = du 2 0 (5u + 49)17 ˚ forenkle nevner, og bruker derfor substitusjonen Videre ønsker en a 49 49 t ⇒ du = dt 5 5    Z ∞ 14 49 1 49 5 t = dt  17 2 0 5 5 49 5 t + 49  14  Z ∞ 1 49 49 t14 = dt 17 2 5 5 49 (t + 1)17 0 Z ∞ 1 t14 = dt 15 2 2 · 5 · 49 0 (t + 1)17

u=

˚ ta det igjen. Vi skriver Det siste integralet har en vært borte i før, men velger a om  14 t14 t 1 = (t + 1)17 t+1 (t + 1)3

V

14.2 Oppgaver

Vi lar y =

315

t 1 og legger merke til at 1 − y = s˚ a t+1 1+t t t+1

1 dy 1 ⇒ dy = dt = dt (t + 1)2 (t + 1)2 Z ∞ t14 1 dt = 15 2 2 · 5 · 49 0 (t + 1)17

y=

y

= =

1 2·

515



0



x29 (5x2 + 49)

17

0

·

1

492

1−y dy }| { }| { 14 z }| { z 1 1 t dt t+1 t+1 (t + 1)2

1

Z

1 515

=

Z

· 492

z 



Z

y 14 (1 − y) dy

0



1 15 1 16 y − y 15 16

2 · 515 · 492 1 = 2 · 515 · 492 · 15 · 16 14! dx = 15 2 · 5 · 492 · 16!

1 dy 0

˚ vise =) Som var det en ønsket a ˚ beregne T (1) først, da har en 45. Det enkleste er a T (1) =

 h f (a) + f (b) f (a) + f (b) + 0 = (b − a) . 2 2

Siden f er kontinuerlig og integralet er begrenset betyr dette at T (n) konvergerer mot A. S˚ a Z b f (x) dx = lim T (`) `→∞

a

La oss regne ut høyresiden eksplisitt. Ved a˚ sette inn uttrykket for trapesmetoden f˚ as ! n−1 X  h lim T (n) = lim f (a) + f (b) + h f (xk ) n→∞ n→∞ 2 k=1

= lim

n→∞

= lim

n→∞

b−a n

n−1 X

f (xk )

k=1

n−1 b−a X f (xk ) + f (xn−k ) 2n k=1

I siste overgang legger vi sammen siste elementet med første, nest siste med andre osv. Virker dette ulogisk, s˚ a bare skriv ut et par ledd og sammenlign. En har ogs˚ a at   a+b f (x) + f (a + b − x) = f (a) + f (b) = 2f (5.40) 2

316

30 Langsvar

V

˚ sette x = a eller Siden f (x) + f (a + b − x) er konstant for alle x ∈ [a, b]. Ved a ˚ sette x = (a + b)/2 f˚ ˚ bruke x = b f˚ as første likhet, og ved a as andre. M˚ alet er a ˚ forenkle summen, dette kan bli gjort ved a ˚ skrive om dette for a

b−a · (n − k) n  b−a =a+b− a+ k n

xn−k = a +

= a + b − xk

˚ bruke dette i summen v˚ Ved a ar f˚ ar en n˚ a at

lim T (n) = lim

n→∞

n→∞

= lim

n→∞

= lim

n→∞

n−1 b−a X f (xk ) + f (xn−k ) 2n k=1

b−a 2n

n−1 X

f (xk ) + f (a + b − xk )

k=1

n−1 b−a X f (a) + f (b) 2n k=1

n−1 1 X b−a (f (a) + f (b)) lim 1 n→∞ n 2 k=1   f (a) + f (b) = (b − a) 2

=

Der første del av likning (5.40) ble benyttet. Videre ble f (a) + f (b) satt utenfor summen siden dette er en konstant. Tilslutt s˚ a ble grensen beregnet som følger

  n−1  1 1 X 1 1 lim 1 = lim 1 + · · · + 1 = lim (n − 1) = lim 1 + {z } n→∞ n n→∞ n | n→∞ n n→∞ n k=1

n−1 ganger

som g˚ ar mot 1 n˚ ar n → ∞. Dette viser punkt 1 og punkt 3 siden

Z

b

f (x) dx = lim T (`) = T (1) a

`→∞

da gjennst˚ ar det bare a˚ vise punkt 2. Merk at selv om T (n) konvergerer mot T (1) kan det være at T (10) > T (1), T (15) < T (1) osv. For a˚ vise det for alle k kan en

V

14.2 Oppgaver

317

enten benytte seg av induksjon, eller eksplisitt regning n−1

T (n) =

X  h f (a) + f (b) + h f (xk ) 2 k=1

=

n−1  h X h f (a) + f (b) + f (xk ) + f (x2n−1−k ) 2 2 k=1

n−1  h h X = f (a) + f (b) + f (xk ) + f (a + b − xk ) 2 2 k=1

 h h = f (a) + f (b) + (n − 1)[f (a) + f (b)] 2 2    hn  f (a) + f (b) = f (a) + f (b) = (b − a) 2 2 Siden h = (b − a)/n s˚ a hn = b − a. Videre ble de fleste triksene fra forrige del benyttet. ˚ bruke antagelsene kan en skrive om som følger 46. Ved a dp ∆S n · lf /T = = dt ∆V Vg Deler en n˚ a begge sider av likningen p˚ a p f˚ as n · lf /T 1 dp = p dT pVg  n · lf /T Lf d  log p = = 2 dT nRT T R som ønsket. b) Ved innsetning f˚ as n˚ a

Z

p

p0

 d  α − βT log p = dT T 2R Z T   α β d 1 − dT log p dT = 2 dT R T0 T T      p 1 T 1 1 log =− β log +α − p0 R T0 T T0 (  β/R )    T 1 1 p = p0 exp − log · exp α − T0 T0 T  β/R    T0 α 1 1 p(T ) = p0 exp − − T R T T0

˚ se at b˚ 47. a) Første metode er a ade x og ex er strengt voksende funksjoner, x slik at xe ogs˚ a m˚ a være strengt voksende. Alts˚ a er f (x) = xex injektiv. Siden

318

V

30 Langsvar

x og ex kontinuerlige, s˚ a er f kontinuerlig. Tilslutt legges det merke til at f (0) = 0e0 = 0, og f (1) = e > 2. S˚ a dermed finnes det en unik konstant 0 < Ω < 1 slik at f (Ω) = 1. P˚ a bakgrunn av at funksjonen er kontinuerlig, strengt voksende. Alternativt anta at likningen xex = 1 har to løsninger Ω1 og Ω2 . Siden n˚ a e = 1/x s˚ a er x

eΩ1 = eΩ2

og

1 1 = Ω1 Ω2

og begge likningene gir at Ω1 = Ω2 , s˚ a igjen s˚ a er Ω unikt definert. ˚ løse likningen f = g, da f˚ b) Ønsker a as x = x2 log x ⇔ 0 = x(1 − x log x) Slik at enten s˚ a er x = 0 eller s˚ a er x log x = 1, en ser videre p˚ a siste løsning. 1 = x log x 1/x = log x e1/x = elog x 1 1/x e =1 x Sammenlikner en n˚ a med definisjonen av Ω konstanten ser en at x=

1 x

= Ω s˚ a

1 = eΩ Ω

Hvor den siste likheten ble tatt fra definisjonen av Omega, alts˚ a ΩeΩ = 1 ⇒ Ω e = 1/Ω. ˚ beregne integralet c) Her ønsker en a Z

exp(Ω)

Z

A=

g

1 dy dx f

0

Z

exp(Ω)

x − x2 log x dx )  exp(Ω) ( exp(Ω) Z exp(Ω) 1 2 1 3 1 3 1 = x − x log x − x · dx 2 3 3 x 0 0 0 =

0

1 2Ω 1 e − e3Ω log eΩ + 2 3 1 2Ω 1 = e − e2Ω · ΩeΩ + 2 3 1 2Ω 1 = e + e3Ω 6 9 1 1 = + 2 6Ω 9Ω3 =

1 3Ω e 9 1 3Ω e 9

V

14.2 Oppgaver

319

˚ vise. I siste overgang ble igjen egenskapen at ΩeΩ = 1 som var det en ønsket a b+c benyttet, og at a = ab · ac . 1+0 1+1

d) Første iterasjon med Ω0 = 0 gir seg Ω1 = gir Ω2 =

1+1/2 1+e1/2

=

1 2

. Den neste iterasjonen

. Slik at √  2 1 = 1+ e Ω2 3

Innsatt i nederste uttrykk for arealet f˚ as da 1 1 + 2 6Ω 9Ω3  2   1 1 1 1 = + Ω 6 9 Ω  2   √  √  1 2 2 1 1+ e 1+ e + · ≈ 3 6 9 3   √ 2 13 √ 8 1+ e + e ≈ 243 4

A=

Dette er bare en halvgod tilnærming da Ω2 eΩ2 6= 1 alts˚ a stemmer ikke tilnærmingen som ble benyttet i c). Ω konstanten lengre. En kan bruke et tidligere uttrykk for arealet, for høyere nøyaktighet. Men det f˚ ar være opp til leser. Benytter i stedet initialverdien Ω0 = log 2 f˚ as Ω1 =

1 (1 + log 2) 3

 og

exp (Ω1 ) = exp

1 log 2e 3

 =

√ 3

2e

Setter en inn dette i det andre uttrykket en har for arealet f˚ as   1 1 1 1 1 + A = e2Ω + e3Ω = e3Ω 6 9 6 eΩ 9     1 1 1 1 −1/3 √ = 2e (2e) + + = 2e 9 6 9 6 3 2e Benytter en seg n˚ a av tilnærmingen e ∼ 8/3 f˚ as 23 1 = · 3 3



24 3

1/3

8 1 + · = 3 9



3 2

5/3 +

16 27

For en endelig tilnærming kan en se at (3/2)5 = 32/243 ∼ 30/240 = 1/8 s˚ a (3/2)5/3 ≈ 1/2 og da blir A = 1 + 5/54.

48. x2 + yey = 1 ,

y > −1

(5.41)

320

30 Langsvar

V

Velger først benytte delvis integrasjon slik at integralet kan skrives som  b Z b Z 1 2 1 b 2 0 xf (x) dx = x f (x) dx x f (x) − 2 2 a a a ˚ sette inn henholdsvisx = a og x = b i likning (5.41) og løse med hensyn Ved a p˚ a yey , kan b˚ ade f (a) og f (b) bestemmes. Dette gir 1 1 log . 2 2 ˚ bruke Slik at f (a) = 0 og f (b) = log 1/2, som kan sjekkes via innsetning. Ved a den anbefalte substitusjonen u = ef (x) s˚ a er f (x) = log u, og f (a)ef (a) = 0 og

f (b)ef (b) =

du = f 0 (x)ef (x) dx ⇒

du = f 0 (x) dx . u

˚ bruke Tilslutt s˚ a kan x2 skrives som x2 = 1 − f (x)ef (x) = 1 − u log u, ved a likning (5.41). Integralet kan n˚ a skrives som  b Z 1 2 1 b 2 0 I= x f (x) − x f (x) dx 2 2 a a Z 1 1 exp(f (b)) du = b2 f (b) − (1 − u log u) 2 2 exp(f (a)) u h i 1/2 1 1 (u − 1) log(u) − u = b2 f (b) + 2  2   1  1 1 1 1 1 =− 1 + log 2 log 2 − − log 2 − (1) 2 2 2 2 2 1 1 = − log(2) · (1 + log 2) + 4 4 Som var det som skulle vises. Her er B = 1/4 og A = −B. 49. a) Første del s˚ a har en at V (r) =

2Ze2 2Ze2 r2 r2 = = E 4π0 r 4π0 r2 r r

som ønsket. Videre s˚ a vil integralet kunne skrives som   Z q   2 r2 T ∼ exp − 2m V (r) − E dr = } r1 Z r2 r i hr 2 2 log T ∼ E − E dr 2m =− } r1 r Z r2 r √ r2 ∼ − 2 2mE log T = − 1 dr } r r1 Herfra benyttes substitusjonen r = r2 x s˚ a dr = r2 dx. r Z 1 2√ 1 log T ∼ − 2mE r − 1 dx = 2 } x r1 /r2 som ønsket.

V

17.2 Oppgaver

321

3.3.2 5. Via substitusjonen y 7→ x + t kan integralet skrives p˚ a formen Z t+1 Z 1 log Γ(y) dy log Γ(x + t) dx = t

0

˚ bruke For enkelhet byttes det n˚ a tilbake til x som integrasjonsvariabel. Ved a likning (3.19) kan den deriverte med hensyn p˚ a t skrives som Z t+1 ∂ d d log Γ(y) dy = log Γ(1 + t) (t + 1) − log Γ(t) t ∂t t dt dt Ved n˚ a og anvende xΓ(x) = Γ(x + 1) kan den deriverte forenkles til Z t+1  ∂ log Γ(y) dy = log Γ(t + 1)/Γ(t) = log t ∂t t ˚ derivere høyresiden f˚ Tilsvarende ved a as Z t i ∂ h ∂ log x dx = t log t − 1 = log t ∂t 0 ∂t Siden de deriverte er like m˚ a f og g avike med høyst en konstant. Anta at f og g aviker med mer en enn konstant s˚ a f kan skrives som f (x) = g(x) + h(x) hvor h0 (x) 6= 0. Derivasjon gir at f 0 (x) = g 0 (x) + h0 (x), som er en motsigelse siden f 0 (x) = g 0 (x), dermed s˚ a m˚ a h0 (x) = 0 og h = C. ˚ bestemme konstanten kan vi se p˚ For a a grensetilfellet n˚ ar t → 0. Da er Z 1 Z t lim log Γ(x + t) dx = C + lim log x dx t→0

t→0

0

0

Integralet p˚ a høyre side g˚ ar mot null, og p˚ a venstre side st˚ ar en igjen med det klassiske√log Γ(x) integralet som ble vist i proposisjon (3.3.9). Dermed s˚ a er C = log 2π og Z 1 Z t √ log Γ(x + t) dx = log 2π + log x dx 0

0

som ønsket. La oss n˚ a ta utgangspunkt i Raabes formel, en liten omskrivning gir Z u+1 √ log Γ(x) dx = u log u − u + log 2π

(5.42)

u

som er formen som vil bli vist her. Ved a˚ dele opp integralet kan det skrives som Z u+1 Z u+1 Z u log Γ(x) dx = log Γ(x) dx − log Γ(x) dx u

0

Z

1

Z

0 u+1

Z

u

log Γ(x) dx + log Γ(x) dx − log Γ(x) dx 0 1 0 Z Z u u √ = log 2π + log Γ(x + 1) dx − log Γ(x) dx.

=

0

0

322

V

30 Langsvar

Hvor det i siste overgang ble brukt proposisjon (3.3.9) og i midterste integralet ble substitusjonen x 7→ y + 1 brukt. Legg n˚ a merke til at siden xΓ(x) = Γ(x + 1) s˚ a er x = Γ(x + 1)/Γ(x). Ved innsetning s˚ a er Z

u+1



Z



Z0 u

u

log Γ(x) dx = log 2π + u

= log 2π +

 log Γ(x + 1)/Γ(x) dx √ log x dx = u log u − u + log 2π

0

˚ trekke fra logaritmen p˚ Ved n˚ aa a begge sider f˚ ar en som ønsket Raabes formel.

3.3.3

˚ benytte seg av substitusjonen som før. Her ble korol1. Det enkleste blir a lar (3.3.3) brukt i andre overgang. i andre overgang. ˚ benytte seg av substitusjonen t 7→ ex da f˚ 2. Det enkleste blir a as Z Z ∞ ∞ n−1 t π enx dx = dx = B(n, 1 − n) = Γ(n)Γ(1 − n) = x 1 + e 1 + t sin πn −∞ 0 Her ble Eulers refleksjonsformel theorem (3.3.4) brukt i siste overgang. Alternativt kan og substitusjonenu 7→ ex benyttes, da er Z ∞ Z ∞ 1 1 π/(1/n) π enx du dx = = = x 1/n 1 + e n n sin πn sin πn 1 + u −∞ 0 som før. Her ble korollar (3.3.3) brukt i andre overgang. i andre overgang. ˚ definere følgende funksjon 3. Vi velger a ϕ(x) :=

B(x, y)Γ(x + y) Γ(y)

˚ vise at ϕ(x) = Γ(x), ved a ˚ benytte seg av Bohrog m˚ alet med beviset blir a mullerup teoremet. Vi g˚ ar igjennom punkt for punkt 1. ϕ(1) = 1. Her har vi fra gammafunksjonen at Γ(1 + y) = yΓ(y). For betafunksjonen f˚ as Z B(1, y) =

1

t1−1 (1 − t)y−1 dt =

0

Z

0

uy−1 du =

1



uy y

Ved innsetning f˚ as n˚ a at ϕ(1) =

B(1, y)Γ(1 + y) = Γ(y)

1 y

· yΓ(y) Γ(y)

=1

1 = 0

1 y

V

17.3 Oppgaver

323

som ønsket. Videre s˚ a m˚ a vi vise at 2. ϕ(x + 1) = xϕ(x). Ved a˚ ta utgangspunkt i B(x + 1, y) og delvis integrasjon f˚ as Z 1 B(x + 1, y) = tx (1 − t)y−1 dt 0 x Z 1 t = (1 − t)x+y−1 dt 1 − t 0 0 Z 1  x (1 − t)x+y xtx−1 t (1 − t)y − − · = x+y x+y (1 − t)x+1 0 1 Z 1 x = tx−1 (1 − t)y−1 dt x+y 0 x = B(x, y) x+y Ved innsetning ser vi n˚ a at ϕ(x + 1) =

(x + y)Γ(x + y) x Γ(x + y + 1) B(x + 1, y) = · B(x, y) = xϕ(x) Γ(y) Γ(y) x+y

˚ vise at log ϕ er konveks. Ved a ˚ bruke blant annet lemVi mangler n˚ a bare a ma (3.3.1) som før. La p ∈ (0, ∞) og velg q slik at p1 + 1q = 1. Da har vi at       Γ up + vq u v u v ϕ + = B + ,y p q Γ(y) p q For betafunksjonen har vi at   Z 1 u v B + ,y = t(u/p+v/q−1 (1 − t)y−1 dx p q 0 Z 1 u 1/p  v 1/q = t (1 − t)y−1 t (1 − t)y−1 dt 0

Z ≤

1/p Z

1 u

t (1 − t)

y−1

v

y−1

t (1 − t)

dt

0

1/q

1

dt

0

= B(u, y)1/p B(v, y)1/q ˚ gjennomføre akkuratt den samme utregningen f˚ Ved a as ulikheten   v u + + y ≤ Γ(u + y)1/p Γ(v + y)1/p Γ p q Dermed f˚ ar vi at følgende ulikhet gjelder   1/p 1/q v {Γ(u + y)B(u, y)} {Γ(v + y)B(v, y)} u + ≤ ϕ p q Γ(y)  1/p  1/q Γ(u + v) Γ(v + v) = B(u, y) B(v, y) Γ(y) Γ(y)  1/p  1/q = ϕ(u) ϕ(v)

324

V

30 Langsvar

Dersom vi n˚ a lar λ = 1/p og dermed 1 − λ = 1/q. Hvor λ ∈ (0, 1) f˚ ar vi  ϕ λu + (1 − λ)v ≤ ϕ(u)λ ϕ(v)1−λ  log ϕ λu + (1 − λ)v ≤ λ log ϕ(u) + (1 − λ) log ϕ(v) for alle par u, v ∈ (0, ∞). Dermed s˚ a er log ϕ er konveks og da følger det fra Bohr–Mollerup theoremet at ϕ(x) = Γ(x), følgelig s˚ a er ϕ(x) =

B(x, y)Γ(x + y) ϕ(x)Γ(y) Γ(x)Γ(y) ⇒ B(x, y) = = Γ(y) Γ(x + y) Γ(x + y)

˚ vise. som var det vi ønsket a 3.3.4 3.3.9 1. Bevis. Gjennom hele beviset antas det at vi deriverer med hensyn p˚ a x uten tap av generalitet. og et identitsk bevis kan føres hvor en deriverer med hensyn p˚ a y. Her har vi først at 1−x−y 1−x−y 1−x−y 1−u= , 1−v = , 1 − uv = 1−y 1−x (1 − x)(1 − y) ˚ derivere polylogaritmen f˚ Ved a as [Li2 (f )]0 = L02 (f ) · f 0 = −

log(1 − f ) · f 0 = − log(1 − f ) · log0 (f ) f

der det ble brukt at f var en kontinuerlig funksjon. Den deriverte av polylogaritmen fikk vi fra analysens fundamentalteorem. N˚ a regner vi først ut de enkle logaritmisk deriverte s˚ a 1 1 , log0 (v) = , og x 1−x For klarhet regner vi ut `en og `en derivert da er log0 (u) =

∂ ∂x ∂ ∂x ∂ ∂x ∂ ∂x ∂ ∂x

log0 (u · v) =

1 1 + x 1−x

log(1 − x) x log(y − x) = Li2 (y) = − ·0=0 y = Li2 (x) = −

log(1 − x − y) − log(1 − y) x log(1 − x − y) − log(1 − x) = Li2 (u) = − log(1 − u) log(u)x = − 1−x = Li2 (u) = − log(1 − u) log(u)x =

= Li2 (uv) = log(1 − uv)(log u + log v)0 

1 1 + x 1−x ∂ ∂ ∂ log(1 − y) = Li2 (u) + Li2 (v) − Li2 (x) − . ∂x ∂x ∂x 1−x   = log(1 − x − y) − log(1 − x) − log(1 − y)



V

18.1 Oppgaver

325

Addisjon gir oss n˚ a endelig at den deriverte kan skrives som = =−

∂ (Li2 (u) + Li2 (v) − Li2 (uv) − Li2 (x) − Li2 (y)) ∂x

log(1 − y) 1−x

Deriverer vi n˚ a venstresiden ser vi at ∂ log(1 − y) log(1 − x) log(1 − y) = − ∂x 1−x slik at høyre og venstre side i likning (3.35) er ulike med høyst en konstant C. Alts˚ a log(1 − x) log(1 − y) = Li2 (u) + Li2 (v) − Li2 (x) − Li2 (y) − Li2 (uv) + C , settes x = 0, y = 0 s˚ a er u = 0 og v = 0 slik at log2 (1) = Li2 (0) + Li2 (0) − Li2 (0 · 0) − Li2 (0) − Li2 (0) + C som medfører at C = 0. Dette fullfører beviset.

2. Velger a˚ vise først Landen’s identitet. Derivasjon med hensyn p˚ a Li2 [z/(z − 1)] gir d Li2 dz



z z−1



z/(z−1)

log(1 − t) dt t 0    z log 1 − z−1 1   = log(1 − z) + log(1 − z) = (z − 1)2 z/(z − 1) 1−z z d =− dz

Z

Integrasjon gir n˚ a direkte at   1 z Li2 = − log2 (1 − z) − Li2 (z) + C z−1 2 ˚ sette z = 0 s˚ ˚ la z = −z f˚ konstanten bestemmes ved a a C = 0. Ved a as   1 z = − log2 (z + 1) Li2 (−z) + Li2 1+z 2 som vi kjenner igjen som Landen’s identitet.

1. Det vi ønsker er a˚ dele inn funksjonen i perioder p˚ a T siden f (x+T ) = f (x) for alle x. Vi har alts˚ a Z ∞ ∞ Z (n+1)T X −st Lf (s) = e f (t) dt = e−st f (t) dt 0

n=0

nT

326

V

30 Langsvar

˚ bruke substitusjonen t 7→ nT + w har en Ved a Lf (s) =

∞ Z X n=0

T

e

−s(nT +w)

f (w + nT ) dw =

0

∞ X

−nT s

Z

e

n=0

T

e−sw f (w) du

0

˚ se at f (x + nT ) = f (x) kan en for eksempel bruke samme induksjonsarFor a ˚ gument som i beviset for theorem (2.4.1) p˚ a side (32). Beviset fullføres ved a P∞ bruke sumformelen for den geometriske rekken n=0 xn = 1/(1 − x) !Z Z T ∞ T X  1 n −T s −sw f (t)e−st dt Lf (s) = e e f (w) du = 1 − e−T s 0 0 n=0 som var det som skulle vises.

˚ sette inn definisjonen av L -transformasjonen f˚ 2. Ved a as Z t  Z ∞ Z t  Lf (s) = L f (w) dw = f (w) dw est dt 0

0

0

˚ bruke delvis integrasjon, med Tanken er n˚ aa Z du = f (t) dt

u=

t

f (w) dw 0

dv = e−st dt

v=−

1 −st e s

Dette fører til at integralet blir  0 Z t Z ∞ e−st e−st Lf (s) = f (t) dt f (w) dw − − s s 0 0 ∞   Z Z t 1 1 ∞ −st =− lim e−st f (w) dw + e f (t) dt s t→∞ s 0 0 

R0 Siden 0 f (w) dw = 0 s˚ a forsvinner første del. N˚ ar t → ∞ s˚ a vil e−st dominere integralet s˚ afremt s > 0. Dermed vil grensen g˚ a mot null for alle s hvor L transformasjonen av f er definert. Siste integralet kjenner vi igjenn og vi f˚ ar Z t  1 Lf (s) = L f (w) dw = −0 + F (s) s 0 som var det som skulle vises. 3.6

3.7

V

24.1 Oppgaver

327

3.8

3.10.1

1. La I betegne integralet og benytt substitusjonen x = 1 − u, s˚ a dx = − du s˚ a 1/2

Z I := 0

log(x) log(1 − x) dx = x(1 − x)

Z

1

1/2

log(1 − u) log(u) du . (1 − u)u

Ved n˚ a og ta gjennomsnittet av disse integralene f˚ as I=

1 2

1

Z 0

log(x) log(1 − x) dx . x(1 − x)

En kan n˚ a eksemeplvis legge merke til at Z 0

1

log(1 − x) log x dx = x

Z 0

1

log(1 − x) log x dx , 1−x

(5.43)

Rb Rb ˚ bruke at a f (x) dx = a f (a + b − x) dx, eller substitusjonen ved enten a 1 − x = u. Uansett vender vi tilbake til v˚ art opprinnelige integral, og bruker delbrøkoppspalting f˚ as   Z Z 1 1 log(x) log(1 − x) 1 1 1 1 dx = + log(x) log(1 − x) dx 2 0 x(1 − x) 2 0 x 1−x Z 1 log(x) log(1 − x) = dx x 0

I=

˚ løse hvor vi benyttet oss av likning (5.43). N˚ a er integralet relativt simpelt a Enten kan vi huske p˚ a definisjonen av dilogaritmen Li2 (z) og benytte oss av delvis integrasjon Dette gir at h i1 Z I = − log x Li2 (x) + 0

0

1

∞ X Li2 (t) 1 dt = Li3 (1) = = ζ(3) t k3 n=1

˚ finne. I den delvise integrasjonen ble v = Li2 (z) og Som var det vi ønsket a u = log(x) brukt videre s˚ a ble den rekursive definisjonen av polylogaritmen benyttet og at Lin (1) = ζ(n). Alternativt kan en og løse siste integral uten bruk

328

V

30 Langsvar

av dilogaritmer 1

log(x) log(1 − x) dx x 0   Z 1 log x x2 x3 x+ =− + + · · · dx x 2 x 0   Z 1 x x2 log x 1 + =− + + · · · dx 2 3 0  Z 1 ∞  X 1 log(x)xn dx =− n+1 0 n=0    ∞ X 1 1 =− · − n+1 (n + 1)2 n=0 Z

I=

=

∞ X n=0

1 (n + 1)3

hvor vi kjenner igjen siste sum som Apierys konstant ζ(3).

2. Vi bruker substitusjonen x = t−1 slik at Z

1/2

Z

2

1 − e−1/x dx =

0



0

2

1 − e−t dt t2

Herfra defineres følgende funksjon Z



I(a) = 0

2

1 − e−ax dx x2

hvor vi legger merke til at I(1) er integralet vi ønsker. Derivasjon under integraltegnet gir I 0 (a) =

d da Z ∞

= Z0 ∞

Z 0



2

1 − e−ax dx x2 2

∂ e−ax dx ∂a x2 2

e−ax dx 0 Z ∞ 2 1 e−y dy = √ a r 0 1 π = 2 a =

Hvor i siste overgang ble det klassiske gaussiske integralet benyttet. Et bevis for dette finnes flere steder i heftet, blant annet ??. Integrasjon gir n˚ a at √ I(a) = aπ + C

V

24.1 Oppgaver

329

siden I(0) = 0 er konstanten null, og vi f˚ ar dermed at 1/2

Z

√ 2 1 − e−1/x dx = I(1) = π

0

Og dette fullfører oppgaven. 3. Integralet her kan regnes ut utelkukkende ved hjelp av en serie elemen˚ komme p˚ tære substitusjoner og omskrivninger. Likevell er det a a disse s˚ apass krevende at integralet havner i denne delen. Første steg er a˚ dele opp intervalet (0, ∞) til (0, 1) og (1, ∞). Deretter mappes (1, ∞) p˚ a (0, 1) via x 7→ 1/x, som er blitt gjort mange ganger før ∞

Z

log x dx = (1 + x2 )2

0

Z

1

0

log x dx + (1 + x2 )2



Z 1

1

Z

log x dx = (1 + x2 )2

1 − x2 log x dx (1 + x2 )2

0

Legg n˚ a merke til at Z Z 1 − x2 (1/x + x)0 1 x dx = − dx = = 2 2 2 2 (1 + x ) (1/x + x) 1/x + x x +1 Via u 7→ 1/x + x siden (−1/u2 )0 = 1/u. Siden første del er integrerbar virker det ˚ benytte seg av delvis integrasjon. Dette gir svært fristende a Z 0

1

h x log x i∞ 1 − x2 log x dx = − 2 2 (1 + x ) x2 + 1 0



Z 0

dx π x =0− x2 + 1 x 4

Siste integralet er bare arctan x, og første leddet g˚ ar mot null. Nevner g˚ ar mot null og teller er begrenset. Alternativt via L’hoptial s˚ a er lim

x→0

log x h ∞ i 1/x ⇒ lim 0 = −x x→0 1/x2 1/x ∞

Der [∞/∞] betyr at b˚ ade teller og nevner g˚ ar mot og en kan derivere. Merk at den deriverte av teller vokser proposjonalt med log x, og teller synker proposjonalt med x3 s˚ a n˚ ar x → ∞ vil uttrykke g˚ a mot null. 4. Begynn med substitusjonen 5. En kan legge merke til at vi kan skrive log(1 − x) om til et integral som følger −x

Z log(1 − x) = 0

du = 1+u

Z 0

x

dm m−1

via substitusjonen u 7→ −m. Integralet kan n˚ a skrives om som følger Z I= 0

1

log(1 − x) dx = 1+x

Z 0

1

Z 0

x

dm dx = (m − 1)(1 + x)

Z 0

1

Z 0

1

x du dx (ux − 1)(1 + x)

330

V

30 Langsvar

der substitusjonen m 7→ ux ble benyttet i siste overgang. Siden x = (x + 1) − 1 skrives integralet om til Z 1Z 1 Z 1Z 1 Z 1Z 1 du dx du dx x du dx − = (ux − 1) (ux − 1)(1 + x) 0 0 (ux − 1)(1 + x) } |0 0 {z } | 0 0 {z A

B

Siden vi har følgende delbrøkoppspalting 1 u 1 = − (ux − 1)(1 + x) (ux − 1)(u + 1) (x + 1)(u + 1) s˚ a kan B skrives som Z 1Z B=

1

Z 1Z 1 u du dx du dx − 0 (ux − 1)(1 + u) 0 (1 + u)(1 + x) 0 0 Z 1Z 1 du dx =I− 0 (1 + u)(1 + x) 0 Z 1  Z 1  du dx =I− 0 1+u 0 1+x = I − log2 (2)

For A s˚ a har en Z 1Z 1 Z 1 Z 1 ∞ X du dx log(1 − u) log(1 + u) 1 A= = du = − du = − u u n2 0 0 (ux − 1) 0 0 n=1 ˚ skrive ut maclaurinrekka til log(x + 1) som konSiste overgang kan sees ved a vergerer da x ∈ (0, 1). Siste sum har vært beregnet gjentatte ganger og er ζ(2) = π 2 /6. Eventuelt kan en legge merke til at integralet er polylogaritmen Li2 (1), som en og har sett og regnet p˚ a før. Legger vi sammen alt dette f˚ as I =A−B =−

 π2 π2 − log2 (2) − I ⇒ 2I = log2 (2) − 6 6

og dette fullføret beviset da Z 1 log(1 − x) 1 π dx = log2 (2) − 1 + x 2 12 0 som var det som skulle beregnes. 3.10.2 1. Vi beregner integralet begge veier Z ∞Z ∞ Z ∞ h i∞ ds π −st I1 = e sin t dt ds = = arctan s = 2 1 + s 2 0 Z0 ∞ Z0 ∞ Z0 ∞ Z ∞ Z ∞ sin t I2 = e−st sin t ds dt = sin t e−st ds dt = dt t 0 0 0 0 0

V

24.2 Oppgaver

331

˚ vise. Hvor da selvsagt som var det vi ønsket a Z ∞ e−st sin t dt = L {sin t} (s) =

1 1 + s2

0

˚ skrive om integralet via t 7→ −x + (u + v)/2 s˚ 2. En begynner med a a Z v Z a (x − u)n (v − x)n dx = − (a − t)n (a + t) dt −a

u

Hvor forenklingen a = (u + v)/2 ble innført. Benyttes n˚ a substitusjonen t 7→ (1 − 2y)a s˚ a blir integralet Z v Z 1 (x − u)n (v − x)n dx = (2a)n y n (2a)n (1 − y)n 2a dy u

0

= (u + v)2n+1

Z

1

y n (1 − y)n dy

0

Dette medfører at vi kan skrive om summen som følger Z v ∞ X 1 S= (x − u)n (v − x)n dx 2n+1 (v − u) u n=1 ∞ Z 1 X = y n (1 − y)n dy n=1

0

Siden y ∈ [0, 1) s˚ a vil og y(1 − y) ∈ [0, 1) og en kan benytte seg av summen av den geometriske serien. Z 1 y(1 − y) = dy 1 − y(1 − y) 0 Z 1 1 dt − 1 = 1 − y(1 − y) 0 Z 1 1 = 2 dy − 1 0 3/4 + (y − 1/2) Z 1/2 1 = dy − 1 2 + 3/4 t −1/2   2 1 = √ · 2 tan−1 √ −1 3 3 r π 4 −1 = 3 3 Hvor det ble brukt blant annet at   Z b dx 2 b = arctan 2 2 a a −b a + x

4.

og

∞ X n=1

xn =

x ∀ x ∈ [0, 1) 1−x

332

V

30 Langsvar

m) Legg merke til at for x ∈ [0, 1] s˚ a er Z Z 1 1 xn 1 xn dx ≤ dx ≤ x n 0 0 x+1 Siden 1/n → 0 n˚ ar n → ∞ har vi fra skviseteoremet at Z 1 xn lim dx = 0 n→∞ 0 x + 1 som var det som skulle vises. Den første ulikheten kan for eksempel vised ved ˚ kryssdele f˚ at xn · x ≤ xn (x + 1) og ved a ar en xn /(x + 1) ≤ xn /x. n)

Elementært Z In+1 + In =

1

xn+1 dx + x+1

0

Z

1

0

xn dx = x+1

Z

1

0

xn (x + 1) 1 dx = x+1 n+1

o) Ved en smart omskriving har vi n X (−1)k 1 1 1 1 = − + − ··· + k+1 0+1 1+1 2+1 n+1

k=0

= (I0 + I1 ) − (I1 + I2 ) + (I2 + I3 ) − · · · + (In + In+1 ) = I0 + In+1 ˚ la n → ∞ har vi Hvor resultatet fra b) ble brukt In + In+1 = 1/(n + 1). Ved a n X (−1)k lim = lim I0 + In+1 = I0 + I∞ = I0 n→∞ n→∞ k+1 k=0

Hvor resultatet fra a) ble brukt. Siden I0 = ∞ X (−1)k = k+1

k=0

Z

1

0

R1 0

x0 dx + lim n→∞ x+1

dx/(x + 1) = log 2 har vi Z

1

0

xn dx = log 2 x+1

som var det som skulle vises. Dette fullfører oppgaven. 5. Begynn med substitusjonen 6. Tanken er a˚ bruke en fiffig substitusjon for a˚ vise identiten. La y 7→ da er 1 − y =

(1−x)2 2(1+x2

og dy =

1

1−x2 (1+x2 )2

= 2y 1/2

(1+x)2 2(1+x2 )

dx 1+x2

(1 + x)2m−1 (1 − x)2n−1 dx (1 + x2 )m+n −1 Z 1 n− 21 1 1 1 1 −1 1 2y 2 (1 − y) 2 · dy = (2y)m− 2 · 2(1 − y) · 2 1+x 1 + x2 0 Z 1 = 2m+n−2 · y m−1 · (1 − y)n−1 dy Z

I=

0

= 2m+n−2 · B(m, n)

V

24.2 Oppgaver

333

Begynner med substitusjonen x = tan u og husker p˚ a at 1 + tan2 u = sec2 u. Z 1 (1 + x)2m−1 (1 − x)2n−1 dx 2m+n−2 B(m, n) = (1 + x2 )m+n −1 Z π/4 (1 + tan u)2m−1 (1 − tan u)2n−1 = sec2 (u) du sec2(m+n) (u) −π/4 Z π/4 (cos u + sin u)2m−1 (cos u − sin u)2n−1 du = −π/4

Z

π/4



= −π/4

cos x + sin x cos x − sin x

cos α

α α Hvor vi i siste overgang satt m = 1+cos og n = 1−cos . Dermed s˚ a er s˚ a kan 2 2 integralet skrives som   cos α Z π/4  1+cos α α Γ 1−cos cos x + sin x −1 Γ 2 2 dx =2 J= cos x − sin x Γ(1) −π/4     1 + cos α 1 + cos α = 2−1 Γ Γ 1− 2 2 π −1  =2 α sin π 1+cos 2 π  = 2 sin π cos2 α2

som var det som skulle vises. ˚ bruke substitusjonen x = (2n + 1)t s˚ 7. Ved a a kan In skrives som Z

(2n+1)π/2

In = 0

sin x dx = x

Z 0

π/2

sin(2n + 1)t dt t

˚ skrive om Tanken er n˚ a at vi betrakter π/2 − In , og bruker likning (3.76) til a π/2. Alts˚ a har vi  Z π/2  π 1 1 Hn = − In = − sin(2n + 1)x dx 2 sin x x 0 Videre bruker vi at f (x) = 1/ sin x − 1/x, g˚ ar mot null n˚ ar x → 0, slik at f (0) ∼ 0. Funksjonen f har alts˚ a en kontinuerlig begrenset derivert p˚ a intervalet (0, π/2) og vi kan trygt bruke delvis integrasjon    Z π/2 1 1 1 Hn = − cos(2n + 1)x + f 0 (x) cos(2n + 1)x dx sin x x 2n + 1 0 Første del av uttrykket g˚ ar mot null siden limx→0 f (x) = 0 og at cos(2n+1)π/2 = ˚ la n → ∞ s˚ a ser en at Hn → 0, siden integralet er − sin πn = 0, n ∈ N. Ved a begrenset. Dette medfører at In → π/2 og Z lim In = lim

n→∞

n→∞

0

(2n+1)π/2

sin x π dx = x 2

334

V

30 Langsvar

˚ vise at Hn faktisk konvergerer kan vi for som var det som skulle vises. For a eksempel se at Z π/2 1 0 |Hn | = f (x) cos(2n + 1)x dx 2n + 1 0 Z π/2 1 ≤ |f 0 (x)| |cos(2n + 1)x| dx 2n + 1 0  Z π/2  1 2 ≤ 1− · 1 dx 2n + 1 0 π π/2 − 1 ≤ 2n + 1 som g˚ ar som O(1/n) n˚ ar n → ∞. ˚ bruke substitusjonen x = (2n + 1)t s˚ 9. Ved a a kan In skrives som 10. Integralet kan løses p˚ a flere metoder, den enkle b’en hinter mot at derivasjon under integraltegnet kan benyttes. Vi definerer Z

π/2

I(b) = −π/2

log(1 + b sin x) dx . sin x

Derivererer en begge sider av denne likningen med hensyn p˚ a b f˚ as ∂ I (b) = ∂b 0

Z

π/2

log(1 + b sin x) dx = sin x

−π/2

Z

π/2

−π/2

dx . 1 + b sin x

(5.44)

Det siste integralet er langt enklere a˚ beregne, og kan løses med a˚ bruke weierstrasssubstitusjonen t = tan x/2. Da f˚ as Z ∞ Z ∞ 1 2 dt 2 dt I 0 (b) = = . 2 2 2 2 −∞ 1 + 2bt/(1 + t ) 1 + t −∞ (t + b) + 1 − b √ ˚ bruke substitusjonen t + b = 1 − b2 tan y s˚ Ved a a blir integralet Z π/2 √ Z π/2 1 − b2 sec2 y dt 1 π 0 I (b) = = √ dy = √ . 2 2 2 1−b 1 − b2 pi/2 (1 − b )(tan y + 1) −π/2 ˚ integrerer begge sider av likning (5.44) fra 0 til b Neste steg blir a Z

b 0

Z

I (b) db = 0

0

b



π db , 1 − b2

og siden I(b) = 0 kan en skrive venstresiden som I(b) − I(0) = I(b). Høyresiden blir s˚ a Z b π √ I(b) = db = π arcsin b , 1 − b2 0

V

24.2 Oppgaver

335

siden arcsin 0 = 0. Dette var det som skulle vises, men vi har n˚ a vist integralet for |b| < 1, siden substitusjonenene vi benyttet bare er gyldige for |b| < 1. Det ˚ vise at resultatet stemmer i grensetilfellet b = 1, dersom b = 1 som gjenst˚ ar er a s˚ a er ˚ beEn kunne og ha brukt kompleks integrasjon og proposisjon (3.9.2) til a regne integralet over 1/(1 + b sin x). Siden vi integrerer over [−π/2, π/2] og ikke (−π, π), s˚ a blir integralet halvparten av integralet over enhetssirkelen5 . I(b) =

  m X dx dz 2 = πi , z Res k 1 + b(z − z −1 )/2i iz bz 2 + 2iz − b

I

1 2

k=0

|z|=1

√   Nevner har to singulariter, men bare z0 = −i + b2 − 1 /b ligger i enhetsdisken. I(b) = πi Res(z0 ) = πi lim

z→z0

2 iπ π = = √ (bz 2 + 2iz − b)0 i + bz0 1 − b2

som vist før. Den men følger direkte fra √ siste overgangen√virker kanskje noe rar,√ at 2bz0 = −i + b2 − 1 = i(−1 + 1 − b2 ), s˚ a i + bz0 = i 1 − b2 .

˚ regne ut de partiellderiverte til In f˚ 11. Ved a as d In = − dα

Z

d In = − dβ

Z



n cos2 x (α cos2 x + β sin2 x)n+1



n sin2 x (α cos2 x + β sin2 x)n+1

0

0

Addisjon av de partiellderiverte gir da 

∂ ∂ + ∂α ∂α



Z



I1 = ∇In = −n 0

(sin2 x + cos2 x)dx = −nIn+1 (α cos2 x + β sin2 x)2

(5.45)

hvor ∇ operatoren ble brukt. Dette gir ∇In−1 = (1 − n)In , ˚ skrive om likning (5.46) f˚ som var det som skulle vises. Ved a as In = − 5A ˚

1 ∇In−1 n−1

vise at

Z

π/2

−π/2

dx 1 = 1 + sin x 2

Z 0



dx 1 + b sin x

˚ spare plass a ˚ gi leser en meningsfyllt fritidssyssel. overlates til leser, b˚ ade for a

(5.46)

336

V

30 Langsvar

√ Bruker vi denne likningen n ganger med initalverdien I1 = π/(2 ab ) f˚ ar en følgende relasjon   ∂ ∂ 1 √ I2 = −∇I1 = −2π + ∂α ∂β ab 1 I3 = − ∇I2 2   ∂2 ∂2 1 2π ∂2 √ + 2 + = 1 · 2 ∂α2 ∂αβ 2 ∂β 2 ab  2 ∂ ∂ 1 2π √ = + 1 · 2 ∂α ∂β ab 1 I4 = − ∇I3 3   3 ∂3 ∂3 1 ∂ ∂3 2π √ +3 + + +3 =− 3 2 2 3 1 · 2 · 3 ∂α ∂α ∂β ∂α∂β ∂β ab  3 ∂ ∂ 2π 1 √ + =− 1 · 2 · 3 ∂α ∂α ab .. .  n  ∂ ∂n ∂n 1 2π √ + n + · · · + In = n n−1 n (−1)2(−3)(4) · · · ∂α ∂α ∂β ∂β ab  n 2π ∂ ∂ 1 √ = (−1)n + n! ∂α ∂β ab for teste om dette faktisk er en løsning, kan en se om uttrykket oppfyller likning (5.46), men dette overlates til leser. En kan og uttrykke differensialoperatorene som en sum  n X  k  X n   n   ∂ ∂ n ∂ n−k ∂ n ∂n + = = ∂α ∂β k ∂αn−k ∂β k k ∂αn−k ∂β k k=0

k=0

uten at dette gjør regningen noe enklere. For alle praktiske hensyn er det enklere ˚ benytte seg av likning (5.46) for a ˚ regne ut In . Men litt algebramagi har aldri a skadet noen. 13. Legg merke til at integralet er et frullani integral og fra theorem (3.8.2) har en at Z ∞ f (ax) − f (bx) b dx = f (0) log x a 0 Dermed s˚ a kan A skrives som   Z ∞ −ϕx e − e(ψ−1)x 1−ψ A= dx = log x ϕ 0 ˚ forenkle Med f (x) = exp(−x) og f (0) = 1. Før vi setter inn kan det være lurt a uttrykket for A. Siden x2 + x = 1 s˚ a er 1/x = x + 1. Dermed s˚ a er 1/φ = 1 + φ.      √  1−ψ ψ A = log = log 1 + ϕ − = log (2 + ϕ − ψ) = log 2 + 5 ϕ ϕ

V

30 Langsvar

337

√ Der det ble benyttet at ψ/ϕ = ψ − 1 og at ϕ − ψ = 5 . Første likhet kommer fra ˚ dele p˚ ψ + ϕ = ψϕ, ved a a ϕ og isolere ψ/ϕ p˚ a høyre side f˚ as likheten. Husk at de hyperbolske funksjonene er definert som sinh x =

ex + e−x 2

og

cosh x =

ex − e−x 2

˚ legge sammen og trekke fra likningene og sette inn x = A f˚ Ved a as følgende likningsett √ sinh A + cosh A = eA = 5 + 2 √ sinh A − cosh A = e−A = 5 − 2 ˚ ta gjennomsnittet av likningene f˚ Ved a ar en et uttrykk for sinh A, subtraksjon gir cosh A s˚ a √ sinh A = 2 og cosh A = 5 Hvor exp(−A) ble regnet ut p˚ a følgende m˚ ate √

−1

exp(−A) = [exp(log 2 + 5 )]

√ √ 1 5 −2 5 −2 √ = √ = 5−4 5 +2 5 −2

˚ sette rett inn i de hyberbolske funksjonene, men Det er selvsagt fullt mulig a undertegnede er dessverre allergisk mot brøker. ˚ regne ut I er ikke spesielt vanskelig, og kan gjøres via substitusjonen 15. A x = sin u. Vi viser først at   Γ 21 Γ 12 π  π= = sin(π/2) Γ 12 + 12 ˚ omforme integralet bruker vi u = 1 − x2 s˚ Dette kan vi sette inn i I, for a a du = −2x dx som gir Z

1

1 π dx = 2 2 1−x 0   Z 0 1 Γ 12 Γ 21 1  =− du = √ √ 2 Γ 12 + 12 1 2 u 1−u   Z 1 Γ 21 Γ 12 1−1/2 1−1/2  = u (1 − u) du = Γ 12 + 12 0

I=



˚ anta at p˚ a bakgrunn av dette s˚ a kan det være rimelig a Z B(x, y) = 0

1

tx−1 (1 − t)y−1 dt =

Γ(x)Γ(y) . Γ(x + y)

˚ gi leser en pustepause og for a ˚ la Denne siden er med hensikt blank, for a forfatter sl˚ ass mot dinosaurer.

V

BIBLIOGRAFI

339

Bibliografi [] Zafar Ahmed, Knut Dale, and George Lamb. Definitely an integral: 10884. The American Mathematical Monthly, 109(7):670–671, 2002. [] T. Ambderberhan, M. L. Glasser, V. H. Moll M. C. Jones, R. Posey, and D. Varela. The cauchy-schl¨ omilch transformation. http://129.81.170. 14/˜vhm/papers_html/schlomilch-sub.pdf, 1999. [] James Bonnar. The Gamma Function. CreateSpace Independent Publishing Platform, 2 edition, Nov 2013. [] J. W. Brown. An extension of integration by parts. The American Mathematical Monthly, 67(4):327, 1960. [] Philip J. Davis. Leonhard euler’s integral: A historical profile of the gamma function. The American Mathematical Monthly, 66(10):849–869, 1959. The development of the laplace transform, [] Michael A. B. Deakin. 1737–1937 i. euler to spitzer, 1737-1880. Archive for History of Exact Sciences, 25(4):343–390, jan 1981. The development of the laplace transform, [] Michael A. B. Deakin. 1737–1937 ii. poincar to doetsch, 1880–1937. Archive for History of Exact Sciences, 26(4):351–381, 1982. [] Richard P. Feynman. Company, Inc, 1985.

Surely You’re Joking, Mr.Feynman! WW.Norton

[] I. S. Gradshteyn and I. M. Ryzhik. Table of Integrals, Series, and Products. Academic Press, 7 edition, 2007. [] Victor. H. Moll. The integrals in gradshteyn and ryzhik. part 2: elementary logarithmic integrals. Scientia, (14):7–15, 2007. [] Roger Nelsen. Symmetry and integration. The College Mathematics Journal, 26(1):39–41, jan 1995. [] Erin Pearse. Math 209d - lecture notes. http://www.math.cornell.edu/ ˜erin/analysis/lectures.pdf, 1999. [] J.L. Raabe. Angen¨ aherte bestimmung der factorenfolge 1.2.3.4.5.... n=...(1+n)=...xne-xdx, wenn n eine sehr grosse zahl ist. Journal f¨ ur die reine und angewandte Mathematik, 25:146–159, 1843. URL http: //eudml.org/doc/147170.

340

BIBLIOGRAFI

V

[] Walter Rudin. Principles of Mathematical Analysis. International Series in Pure and Applied Mathematics. McGraw-Hill Higher Education, 3 edition, Sep 1976. [] Walter Rudin. Real and Complex Analysis. International Series in Pure and Applied Mathematics. McGraw-Hill Science/Engineering/Math, 3 edition, May 1986. [] David Sirajuddin. Fresnel integrals. http://www.itcanbeshown.com/ integrals/Fresnel%20Integrals/fresnel_integrals.pdf, 2008. [] Steve Smith. Integration by parts, the tabular method, ii. http: //chelseas-roost.co.uk/resources/maths/DIS_II.pdf, 1999. [] Daniel Zwillinger. The Handbook of Integration. A K Peters/CRC Pres, 1 edition, Nov 1992.

View more...

Comments

Copyright ©2017 KUPDF Inc.
SUPPORT KUPDF